PREP 2015

Ace your homework & exams now with Quizwiz!

D. ornithine transcarbamylase deficiency The male neonate in the vignette has a classic presentation of a urea cycle disorder of which ornithine transcarbamylase (OTC) deficiency is one type. These disorders are caused by mutations resulting in the absence or partial functioning of 1 of the first 4 enzymes in the urea cycle biochemical pathway responsible for the breakdown of nitrogen, a waste byproduct of protein catabolism in the body. In a normal individual, nitrogen is broken down into urea and excreted through the urine. If nitrogen levels build up in the body because of inefficient breakdown, it accumulates quickly in the form of ammonia. Hyperammonemia is quite toxic to the brain and can cause irreversible damage without immediate intervention. Neonates will appear normal at birth, but within 48 to 72 hours they will develop cerebral edema. This typically manifests as poor feeding, obtundation, hypothermia, seizures, hyperventilation, hyporeflexia, unusual posturing, and ultimately, coma. Classic laboratory findings include elevated ammonia levels (> 210 μg/dL [150 μmol/L]) with a normal anion gap and normal glucose level in the presence of respiratory alkalosis on blood gas measurement. To distinguish between the specific types of urea cycle defects, one must order a plasma amino acid analysis and a urine orotic acid. Ornithine transcarbamylase deficiency is associated with extremely high urinary orotic acid levels. Carbamoyl phosphate synthetase I (CPS I) deficiency is associated with low to undetectable urine orotic acid levels, thus excluding CPS I deficiency. Definitive diagnosis is dependent on either enzymatic analysis or molecular genetic testing of the genes involved. All of the urea cycle disorders are autosomal recessive in inheritance except OTC, which is X-linked recessive. The patient in the vignette has classic OTC deficiency inherited from his mother. The mothers often complain of protein or meat intolerance. Of carrier females, 15% can develop hyperammonemia in times of stress, such as pregnancy and delivery, and could potentially require chronic lifetime management for periods of hyperammonemia. A family history of early infant death, presumably because of hyperammonemia, may also be seen. Treatment of urea cycle disorders is a metabolic emergency necessitating immediate recognition to avoid irreversible brain damage. Severe hyperammonemia is treated with dialysis and hemofiltration to rapidly reduce the plasma ammonia concentration, along with the intravenous administration of arginine hydrochloride and nitrogen scavenger drugs to promote the excretion of excess nitrogen through alternative pathways. Restriction of protein for 12 to 24 hours is essential, with calories provided through carbohydrates and fat. Care must be taken to stabilize the patient with intravenous fluids and inotropic drugs if necessary. Long-term management mandates the use of specialized formulas, oral nitrogen-scavenging drugs, dietary restriction of protein, and avoidance of hyperammonemic episodes. Patients are at high risk of decompensation, necessitating hospitalization for close observation of clinical status and ammonia levels if they have gastrointestinal and respiratory illnesses. Most patients are routinely treated on a long-term basis by biochemical or metabolic geneticists, in addition to their primary care provider. They should have emergency protocols in place at home, at the primary care provider's office, and at the local hospital. Methylmalonic acidemia is an organic acid disorder that commonly presents with elevated ammonia levels, high ketone levels, and a high anion gap metabolic acidosis (> 20). Elevated plasma and urine concentrations of methylmalonic acid in the absence of hyperhomocysteinemia or homocystinuria are diagnostic. Other common laboratory findings are neutropenia and thrombocytopenia. During infancy or in the neonatal period, patients can present with lethargy, low tone, hypothermia, respiratory impairment, vomiting, dehydration, and obtundation. This is another metabolic emergency. Tyrosinemia type 1 presents in infancy with significant liver involvement and eventually renal tubular dysfunction, growth failure, and rickets. Untreated children may present with repeated neurologic crises involving a change in mental status, peripheral neuropathy, abdominal pain, and occasionally respiratory failure. If untreated, many die before the age of 10 years. Laboratory abnormalities include increased succinylacetone concentration in the blood and urine; elevated tyrosine, methionine, and phenylalanine on serum amino acids; and elevated tyrosine metabolites on urine organic acids. Galactosemia presents in the neonatal period with jaundice, hypotonia, scleral icterus, bruising, bleeding, and cataracts in the face of rapidly progressive liver failure. Classic laboratory abnormalities of galactosemia include positive urine-reducing substances, abnormal liver function studies, coagulation abnormalities suggestive of a progressive bleeding diathesis, and a culture workup for sepsis, especially due to Escherichia coli. Specific urea cycle disorders include N-acetyl glutamate synthetase deficiency, CPS I deficiency, OTC deficiency, citrullinemia type I, argininosuccinic aciduria, and arginase deficiency. PREP Pearls - Urea cycle defects are caused by the body's inability to metabolize waste nitrogen byproducts created by the breakdown of protein and other nitrogen-containing molecules, resulting in severe hyperammonemia that is quickly toxic to the brain and necessitating immediate recognition of this metabolic emergency. - Infants with urea cycle defects will appear normal at birth, but will develop cerebral edema within 48 to 72 hours. This typically manifests as poor feeding, obtundation, hypothermia, seizures, hyperventilation, hyporeflexia, unusual posturing, and ultimately, coma. - Classic laboratory findings of urea cycle defects include elevated ammonia levels (> 210 µg/dL [150 µmol/L]) with a normal anion gap and normal glucose level in the presence of respiratory alkalosis on blood gas measurement. - Severe hyperammonemia is treated with dialysis and hemofiltration to rapidly reduce the plasma ammonia concentration, intravenous administration of arginine hydrochloride and nitrogen scavenger drugs to promote the excretion of excess nitrogen through alternative pathways, and protein restriction with cardiovascular stabilization. ABP Content Specifications(s) - Recognize the clinical features associated with urea cycle defects - Plan the appropriate immediate and long-term management of urea cycle defects, while considering the long-term prognosis

A 2-day-old neonate is transferred to your neonatal intensive care unit (NICU) with altered mental status and seizures. He was delivered at 39 weeks' gestation via spontaneous vaginal delivery without complications to a 37-year-old mother—gravida 4, now para 2—with unremarkable prenatal laboratory test results. His growth parameters are appropriate for gestational age. At 28 hours of age, the patient began to develop deep, rapid breathing that progressed within 8 hours to respiratory distress with central nervous system excitation (patient couldn't fall asleep). A sepsis workup was initiated at the outside hospital with a hazy chest radiograph, normal complete blood cell count, liver function tests, and C-reactive protein. Blood, urine, cerebrospinal fluid cultures, and viral studies were sent, and the child has been started on a regimen of ampicillin, gentamicin, and acyclovir. Arterial blood gases show a respiratory alkalosis. One hour later at the outside hospital, he developed seizure-like activity with tonic-clonic movement of all 4 extremities and desaturations as low as 80%. A loading phenobarbital dose was given. An ammonia level was obtained and was 1,613 μg/dL (1,152 µmol/L), with worsening of his neurologic status. He was intubated due to obtundation and is now hypotonic and hyporeflexic. Transfer to a tertiary NICU was initiated. Upon arrival at the NICU, the initial laboratory results show the following: Electrolytes: - Sodium,145 mEq/L (145 mmol/L) - Potassium, 4.2 mEq/L (4.2 mmol/L) - Chloride, 112 mEq/L (112 mmol/L) - Bicarbonate, 20 mEq/L (20 mmol/L) - Blood urea nitrogen, 6 mg/dL (2.1 mmol/L) - Creatinine, 0.9 mg/dL (80 µmol/L) - Glucose, 112 mg/dL (6.2 mmol/L) - Anion gap, 13 mEq/L (13 mmol/L) - Total bilirubin, 6.0 mg/dL (102.6 µmol/L) Serum amino acids: - Glutamine, 49.3 mg/dL (3,376.0 µmol/L); reference range, 474.0 to 737.0 µmol/L - Alanine, 4.6 mg/dL (516 µmol/L); reference range, 182.0 to 398.0 µmol/L - Citrulline, undetectable; reference range, 14.0 to 32.0 µmol/L - Ornithine, 0.3 mg/dL (23 µmol/L); reference range, 25.0 to 103.0 µmol/L - Proline, 4.0 mg/dL (347 µmol/L); reference range, 93.0-264.0 µmol/L - Urine orotic acid, 24 mmol/mol creatinine; reference range, 1.4 to 5.3 mmol/mol creatinine Upon arrival at the NICU, the mother reveals that there is a positive family history for a maternal uncle with early neonate death at 2 weeks of age from respiratory distress. The family's ethnicity is European Caucasian. Consanguinity is denied. The mother also has avoided meat and milk products for the majority of her life. Of the following, the MOST likely diagnosis is A. carbamoyl phosphate synthetase I deficiency B. galactosemia C. methylmalonic acidemia D. ornithine transcarbamylase deficiency E. tyrosinemia type 1

E. mitochondrial disease The patient in the vignette has a mitochondrial disorder, most likely MELAS (mitochondrial encephalomyopathy with lactic acidosis and stroke-like episodes). He presents with early normal development followed by the development of childhood-onset myopathy, seizures, migraines, early signs of diabetes mellitus, stroke-like episodes associated with periods of regression, laboratory evidence of lactic acidosis in both serum and cerebrospinal fluid (CSF), and elevated CSF protein. In addition, neuroimaging is suggestive of bilateral globus pallidus involvement in the context of progressive white matter changes. This is a classic presentation for MELAS. This patient's family history of symptoms in his mother also suggests a potential maternal inheritance pattern that is common among mitochondrial disorders arising from mutations in the mitochondrial DNA. Mitochondrial disorders are a heterogeneous group of diseases arising from functional abnormalities in oxidative phosphorylation, also known as the mitochondrial respiratory chain, which is the final common biochemical pathway for aerobic metabolism. Tissues and organ systems that are extremely dependent on aerobic metabolism are typically involved to the greatest degree, including the heart, muscles, and the nervous system. Mitochondrial disorders can result from gene mutations or deletions in the nuclear DNA or the mitochondrial DNA (mtDNA), thus they can be inherited in an autosomal-recessive, autosomal-dominant, X-linked, or maternal inheritance pattern. Mitochondrial DNA is transmitted exclusively by maternal inheritance, as women give their mitochondrial DNA to all of their offspring and men do not transmit mtDNA to any of their offspring. A woman can transmit a variable amount of mutated mtDNA to each of her offspring, thus yielding significant clinical variability among her children. If a defect is in the nuclear DNA that encodes components of the respiratory chain, it can be transmitted by the mother or father to their offspring in a dominant or recessive pattern. Mitochondrial disorders can present at any age and may initially affect only a single organ or multiple organ systems; however, the predominant features tend to be neurologic and exhibit myopathic symptoms in most individuals. One of the cardinal features is progressive organ system involvement over the course of the disease. Significant clinical variability is common and presenting symptoms may include short stature, cardiomyopathy, diabetes mellitus, proximal myopathy, intolerance to exercise, pancytopenia, sensorineural deafness, optic atrophy, pigmentary retinopathy, ptosis, and external ophthalmoplegia. Neurologic manifestations are typical and include developmental delay, seizures, dementia, migraine headaches, spasticity, hypotonia, neuropathy, ataxia, encephalopathy, and stroke-like episodes. Many affected patients will manifest a particular cluster of features that will delineate a specific mitochondrial syndrome. Some of the more common syndromes include myoclonic epilepsy with ragged-red fibers, Leigh syndrome, Kearns-Sayre syndrome, MELAS, and chronic progressive external ophthalmoplegia. Diagnosis is sometimes made by recognizing a characteristic clinical presentation of a specific disorder involving the mitochondria; however, it is frequently necessary to take a good family history, check blood or cerebrospinal fluid (CSF) lactate or pyruvate levels, obtain neuroimaging, order cardiac evaluation, order molecular genetic testing, and obtain a muscle biopsy specimen for histologic evidence, such as ragged red fibers and respiratory chain enzyme analysis, supporting a mitochondrial diagnosis. Neuroimaging findings are diagnostic and may illustrate basal ganglia calcification, diffuse atrophy, focal atrophy of the cortex or cerebellum, generalized leukoencephalopathy (white matter changes), or cerebellar atrophy. Laboratory findings commonly feature lactic acidosis in the blood or CSF. Treatment of mitochondrial disorders is mainly supportive from a systemic standpoint because there is no defined cure at this time. Certain mitochondrial disorders can benefit from supplementation of vitamins and cofactors, such as coenzyme Q10, riboflavin, idebenone, or carnitine depending on the disorder, but a systematic review of the Cochrane database finds no evidence of the global benefit of these supplements for mitochondrial disorders. Biotinidase deficiency, if untreated, presents with seizures, hypotonia, developmental delay, vision abnormalities, hearing loss, ataxia, hair loss, and skin rashes in young children. As the children age, they develop spastic paresis and motor weakness. It is commonly diagnosed on newborn screening because it is a highly treatable disorder that is amenable to oral supplementation of biotin with excellent outcomes. Once many of the manifestations described have occurred, they are typically irreversible; thus the importance of early recognition and treatment. Congenital disorders of glycosylation are a group of disorders caused by altered glycosylation of N-linked oligosaccharides. These disorders present in infancy with clinical variability, ranging from severe developmental delay, hypotonia, and systemic involvement to coagulopathy with stroke-like episodes, to normal development with associated recurrent hypoglycemia and failure to thrive. Patients often have inverted nipples, abnormal subcutaneous fat distribution, strabismus, gastroesophageal reflux, hypoproteinemia because of protein-losing enteropathy, and a hypoplastic cerebellum with ataxia. It is diagnosed with a carbohydrate-deficient transferrin isoform analysis, which is a biochemical screening test. Homocystinuria patients have a "marfanoid" appearance. This disorder is caused by cystathionine β-synthase deficiency resulting in intellectual delays, ectopia lentis, severe myopia, tall stature, and thromboembolism that can lead to an early death. Biochemical features include significantly elevated concentrations of plasma homocystine, total homocysteine, and methionine. Treatment is focused on correction of the biochemical abnormalities involving plasma homocystine and homocysteine concentrations that can help prevent thrombosis. Patients are typically placed on protein- and methionine-restricted diets, as well as betaine, folate, and vitamin B12 supplementation. Lysosomal storage disorders are a heterogeneous group of disorders that present with accumulation of undigested or partially digested macromolecules in varying organs, causing cellular dysfunction and systemic pathology. They are classified by the type of the accumulated substrate: mucopolysaccharidoses, mucolipidoses, oligosaccharidoses, etc. Patients can present with coarse facies, macroglossia, dysostosis multiplex, cardiomegaly, hepatosplenomegaly, unusual ophthalmologic findings (corneal clouding, cherry-red spot in the macula), developmental delay, regression, hypotonia, seizures, and intellectual disability. PREP Pearls - MELAS (mitochondrial encephalomyopathy with lactic acidosis and stroke-like episodes) is a mitochondrial disorder that presents with childhood onset of myopathy, seizures, migraines, early signs of diabetes mellitus, stroke-like episodes associated with periods of regression, and laboratory evidence of lactic acidosis in both serum and cerebrospinal fluid. - Mitochondrial disorders can present at any age, occur with any inheritance pattern, and may affect only a single organ or multiple organ systems; however, the predominant features tend to be neurologic and myopathic in most individuals with progressive organ system involvement over time. - Tissues and organ systems that are extremely dependent on aerobic metabolism are typically involved to the greatest degree, including the heart, muscles, and the nervous system. ABP Content Specifications(s) - Recognize the clinical features associated with mitochondrial disorders

A 5-year-old boy is brought to your office as a new patient with a complicated history of weakness in his thighs and upper arms with exercise intolerance, recurrent migraine headaches, recurrent vomiting, loss of appetite, and seizures. The symptoms started at the age of 2.5 years. The seizures are described as altered consciousness, with paralysis of one side of his body that progresses to a generalized rhythmic jerking of all 4 extremities. The mother has noted that following his seizures, he experiences some loss of skills without re-attainment of the skills. The seizures had initially occurred every 3 to 4 months, but are now monthly. She has also noted some gradual hearing loss. She reports some polydipsia and polyuria. His early development was normal, but his development slowed with some periods of regression following the onset of his seizures. In fact, he now seems delayed (at about the level of a 3-year-old) when compared to his peers. He also has problems with attention. His mother has a history of migraines, an unexplained cardiomyopathy, and exercise intolerance. His height and weight are at less than the fifth percentile. Other than myopathic facies, he is nondysmorphic in appearance. No hepatosplenomegaly is noted. His musculature is thin, with noted hypotonia and weakness. His skin is pale. Laboratory investigations show lactic acidosis both in blood and in the cerebrospinal fluid (> 2.5 mmol) and an elevated lactate-to-pyruvate ratio (> 20 mmol). Cerebrospinal fluid protein was elevated at 80 mg/dL. Brain magnetic resonance imaging shows white matter change in the frontal and parietal white matter, with decreased T1 and increased T2 signal. There is abnormal diffusion within the globus pallidus bilaterally. There is a mild degree of ventriculomegaly, but no obstructive hydrocephalus. There is no intracranial mass lesion. Of the following, the MOST likely diagnosis is A. biotinidase deficiency B. congenital disorder of glycosylation C. homocystinuria D. lysosomal storage disease E. mitochondrial disease

B. abdominal ultrasonography The child in the vignette presents with hematemesis and splenomegaly, with a medical history that includes premature delivery and instrumentation of the umbilical vein during the neonatal period. These findings strongly suggest a diagnosis of portal hypertension (PH) and bleeding either from esophageal or gastric varices. After initial hemodynamic stabilization, a nasogastric aspirate returned coffee-ground material, indicating cessation of active bleeding. In this setting, esophagogastroduodenoscopy (EGD) is the preferred modality for identifying the source of upper gastrointestinal bleeding. This procedure should be conducted within the first 24 hours of presentation and after the patient is stabilized. Before endoscopy, and especially in the patient who presents with signs of PH and who has not experienced a bleeding episode, imaging studies will help define the cause of portal venous obstruction. The most appropriate of these tests is abdominal ultrasonography with Doppler assessment of blood flow. Portal hypertension is defined either as a portal venous pressure of more than 5 mm Hg, or a portal to hepatic vein pressure gradient of more than 10 mm Hg. In most cases, PH is suspected on the basis of combined clinical and laboratory data. As the result of portal venous congestion, collateral circulation develops at the junction of the high pressure and the low pressure venous system. The result is the formation of varices that, in children, develop predominantly in the esophagus and stomach. The child's medical history is of paramount importance in raising an index of suspicion for PH. Any patient with a history of chronic liver disease is at risk. Intrahepatic causes include disorders leading to presinusoidal, sinusoidal, or postsinusoidal venous obstruction. In children, liver disorders leading to cirrhosis comprise the most prevalent causes of PH, with biliary atresia being the most common. Extrahepatic PH may develop as the result of posthepatic and prehepatic causes of portal venous obstruction. A common prehepatic etiology is portal vein thrombosis, arising from instrumentation of the umbilical vein during the neonatal period. Other etiologies of portal vein thrombosis include neonatal omphalitis (and other intra-abdominal infections), severe dehydration, and blunt abdominal trauma. Posthepatic etiologies of portal venous obstruction include Budd-Chiari syndrome and veno-occlusive and cardiac diseases. A family history of metabolic disorders or a hypercoagulable state are additional risk factors. Physical examination in the majority of children with PH will demonstrate splenomegaly. A prominent exception is the child with asplenia or polysplenia, as in the fetal form of biliary atresia. Splenomegaly is the most sensitive among clinical and laboratory indicators (excluding imaging or endoscopic studies) for the presence of esophageal varices. Depending on the etiology of PH, the liver size may be enlarged (metabolic disease, Budd-Chiari syndrome), normal, or small (cirrhosis). The presence of ascites indicates underlying liver disease. Aside from splenomegaly, other physical evidence of varices (caput medusae, rectal hemorrhoids) is uncommon in children. Laboratory studies will suggest a diagnosis of PH if the portal venous pressure gradient is sufficient enough to cause splenomegaly and consequent hypersplenism. Leukopenia and thrombocytopenia are common. Splenic sequestration of red blood cells may also occur. Anemia in the patient with PH may be the result of hypersplenism, variceal bleeding, or portal gastropathy. In patients with PH secondary to intrinsic liver disease, particularly hepatitis B and C, hypoalbuminemia has been reported to be a nonspecific factor associated with the presence of varices. Elevations in aminotransferases or coagulation abnormalities indicate hepatocellular injury and dysfunction, but these tests do not specifically suggest PH. Following ultrasonographic documentation of portal venous obstruction and hepatofugal (reverse) portal venous flow, endoscopic intervention will allow for visualization and appropriate management of esophageal and gastric varices (eg, endoscopic ligation of esophageal varices). Other diagnostic studies are not indicated. Computed tomography may demonstrate varices, but is associated with significant radiation exposure. Furthermore, the advantage of ultrasonography includes the ability to use Doppler technology to assess blood flow. Tagged red blood cell scans may be useful in evaluating gastrointestinal blood loss in cases in which the site of bleeding is unknown, but is only valuable in the actively bleeding patient. Hepatobiliary scintigraphy may be used in the evaluation of cholestasis and percutaneous liver biopsy remains an essential part of the workup for patients with chronic liver disease. However, in the child in the vignette, the clinical and laboratory findings indicate normal hepatobiliary function and suggest PH secondary to a prehepatic cause. PREP Pearls - Abdominal ultrasonography with Doppler is the best noninvasive test for assessing portal vein status. - Splenomegaly is the most sensitive indicator of portal hypertension and varices. - In the patient with upper gastrointestinal bleeding, a coffee-ground nasogastric aspirate indicates cessation of active bleeding. ABP Content Specifications(s) - Identify the physical and laboratory features associated with portal hypertension

A 9-year-old girl presents to the emergency department because of an episode of hematemesis. Her past medical history is unremarkable except for her birth at 30 weeks' gestational age, followed by a 4-week stay in the neonatal intensive care unit. During her neonatal hospitalization, she was on a ventilator for 10 days because of respiratory distress syndrome and was also treated for suspected sepsis. Her hospital records indicate that both umbilical artery and vein catheterizations were employed for management. On physical examination, she is afebrile, pale, and diaphoretic. Vital signs include a resting heart rate of 100 beats/min and a blood pressure of 90/60 mm Hg. The lungs are clear to auscultation, and a grade 2/6 holosystolic murmur is heard throughout the precordium. Abdominal examination demonstrates no hepatomegaly. The spleen tip is palpated 4 cm below the left costal margin. Rectal examination shows dark, guaiac test positive stool. Initial laboratory data demonstrate the following: - Hematocrit, 24% (0.24) - White blood cell, 2,800/µL (2.8 × 109/L) - Platelets, 80 × 103/µL (80 × 109/L) - Blood urea nitrogen, 32 mg/dL (11.4 mmol/L) - Creatinine, 1.0 mg/dL (88.4 µmol/L) - Prothrombin time, 12 s - Partial thromboplastin time, 33 s - International normalized ratio, 0.9 - Aspartate aminotransferase, 60 U/L (reference range, 20-50 U/L) - Alanine aminotransferase 30 U/L (reference range, 20-60 U/L) - Albumin, 4.0 g/dL (40.0 g/L) After the intravenous administration of 1,000 mL of normal saline, her vital signs are improved and she demonstrates no orthostatic changes. A nasogastric tube is passed, and it returns fluid that contains coffee-ground-appearing particles that clears after lavage with room temperature saline. Of the following, the MOST appropriate test to demonstrate the etiology of this child's bleeding is A. abdominal computed tomography B. abdominal ultrasonography C. hepatobiliary scintigraphy D. percutaneous liver biopsy E. tagged red blood cell scan

C. medium-chain acyl-CoA dehydrogenase deficiency The patient in the vignette, has medium-chain acyl-CoA dehydrogenase deficiency (also known as MCAD), which is the most common defect in the fatty acid oxidation pathway. This enzyme's role is to fuel hepatic ketogenesis. It is an autosomal recessive inborn error of metabolism that primarily affects white patients of northern European origin. Medium-chain acyl-CoA dehydrogenase deficiency is a hepatic fatty acid oxidation disease that frequently presents with episodic hypoketotic hypoglycemia with hyperammonemia and elevated liver function tests. It is frequently noted with fasting or illness and usually manifests in the first 2 years after birth. A child may present with vomiting and lethargy after a period of fasting, or a history of a viral infection accompanied by poor oral intake. Often, the patient has hepatomegaly or even seizures during the acute episode. It has been found to be a cause of a small percentage of sudden infant death syndrome. Roughly 20% of previously unrecognized patients with MCAD die with the first episode if not treated promptly. However, since its inclusion in newborn screening, most patients are diagnosed in the neonatal period and treated appropriately, thus decreasing mortality and eliminating many sudden death events. Medium-chain acyl-CoA dehydrogenase deficiency can also be present with a Reye-like syndrome or a comatose presentation, thus illustrating its varying presentation in families. Therapy is aimed at avoidance of fasting, which is crucial in infancy, and immediate treatment of acute illness episodes with intravenous infusion of 10% glucose and oral L-carnitine (100-200 mg/kg per day). In conjunction with the avoidance of fasting, most infants and toddlers are treated with daily carnitine supplementation and a mild reduction of dietary fat to about 20% of total calories. Most acute episodes occur in early childhood, with fewer events noted after age 4 years. It is imperative that siblings of affected children be screened as well. Heterozygous carriers are not symptomatic. Although MCAD is part of the newborn screening evaluation, a biochemical diagnosis requires urine acylglycines, urine organic acids, and an acylcarnitine profile that typically shows elevation of the C6:0, C8:0, and C10:0 acylcarnitine species. The acylcarnitine profile is diagnostic whether the child is sick or well. Definitive diagnosis is via MCAD enzymatic activity measurement in fibroblasts, fatty acid β-oxidation determination in fibroblasts, or molecular sequencing of the ACADM gene. Mitochondrial β-oxidation has a major role in the body's energy production, especially during fasting. This pathway is quite complicated, with more than 20 individual steps and up to 11 inherited defects in proteins affecting this process in humans. Disorders include carnitine transporter deficiency; carnitine palmitoyltransferase (CPT) I and II deficiency; carnitine/acylcarnitine translocase deficiency; very long-chain, medium-chain, and short-chain acyl-CoA dehydrogenase deficiencies; and long-chain 3-hydroxyacyl-CoA dehydrogenase (LCHAD) and short-chain 3-hydroxyacyl-CoA dehydrogenase deficiencies. Other disorders in the fatty acid oxidation pathway are manifest by skeletal or cardiac muscle weakness, such as very long-chain acyl-CoA dehydrogenase deficiency, LCHAD, CPT II deficiency, short-chain acyl-Co-A dehydrogenase deficiency, carnitine transporter defects, and carnitine/acylcarnitine translocase deficiency. Very long-chain acyl-CoA dehydrogenase deficiency has 2 phenotypic clinical presentations: hypertrophic cardiomyopathy and a less severe form with recurrent hypoglycemia. Carnitine deficiency is the primary feature of the carnitine transporter defect, but is a secondary manifestation of all the other fatty acid oxidation disorders, with the exception of CPT I deficiency, which is associated with elevated carnitine levels. Long-chain 3-hydroxyacyl-CoA dehydrogenase in a fetus has been shown to be strongly associated with HELLP (hemolysis; elevated liver enzymes; low platelet count) syndrome and acute fatty liver in the mother during pregnancy, at times necessitating emergency delivery in the final trimester. Classic maple syrup urine disease (MSUD) in untreated infants presents with an unusual maple syrup smell in the cerumen and urine, with progressive evolution of ketonuria, irritability, decreased feeding, and encephalopathy, quickly evolving to coma and respiratory deterioration if it is not immediately recognized. By the end of the first day after birth, elevated branched-chain amino acids (leucine, isoleucine, and valine) and allo-isoleucine are noted in the plasma, as well as elevated branched-chain hydroxyacids and ketoacids in the urine. This is identified on newborn screening via tandem mass spectrometry. Hereditary tyrosinemia type 1, if untreated, presents in the first year with progressive liver involvement and renal tubular acidosis in association with poor growth, rickets, and episodic neurologic crises. It is treated with nitisinone and a low-tyrosine diet that greatly affects overall survival. If untreated, many patients die by 10 years of age from liver failure, liver cancer, or neurologic crises. Peroxisomal biogenesis disorders are a continuum of disorders that include Zellweger syndrome, neonatal adrenoleukodystrophy, and infantile Refsum disease. Patients with Zellweger syndrome present in the neonatal period with low tone, poor feeding, distinctive facies, liver dysfunction, and seizures, progressing to death by the end of the first year. Neonatal adrenoleukodystrophy and infantile Refsum disease present with hearing loss, retinal dystrophy, developmental delay, low tone, and liver dysfunction at later ages with slow progression. Elevated plasma very-long-chain fatty acids are classic on laboratory analysis. Phenylalanine hydroxylase deficiency is caused by the inability to metabolize dietary phenylalanine, which can lead to irreversible brain damage over time. It is asymptomatic in the neonatal period, but as time progresses, the patient develops microcephaly, eczema, seizures, a musty body odor, reduced skin and hair pigmentation, behavioral issues, and severe intellectual deficits. It is identified on newborn screening and treated with strict adherence to a low protein diet and specialized medical formulas that are phenylalanine-free. Errata: 3/2015: Content updated for clarity. PREP Pearls - Medium-chain acyl-CoA dehydrogenase deficiency (MCAD) is a hepatic fatty acid oxidation disorder presenting with episodic, hypoketotic (low ketones in blood and urine) hypoglycemia along with hyperammonemia, and elevated liver function tests in the face of a prolonged fasting episode or intercurrent illness. - Medium-chain acyl-CoA dehydrogenase deficiency is mandatorily screened for on state newborn screening profiles because it is a treatable disorder that can be fatal if not immediately treated and proactively recognized. - Treatment involves intravenous infusion of 10% dextrose and carnitine supplementation. Do NOT delay treatment while awaiting diagnostic confirmation if MCAD is suspected because it could have fatal consequences for the patient. - Fatty acid oxidation disorders can affect heart, liver, or skeletal muscle, depending on the specific type of fatty acid defect. ABP Content Specifications(s) - Recognize the laboratory features associated with disorders of fatty acid and carnitine metabolism - Recognize the clinical features associated with disorders of fatty acid and carnitine metabolism

A 6-month-old infant is brought into the emergency department in the morning by his mother due to progressive lethargy, vomiting, and diarrhea. She reports he developed a low-grade fever about 18 hours to 24 hours prior to arrival. He has been unable to take his normal feeds during this time, so she has been supplementing with oral electrolyte solution with limited success. When she put him to bed last night, he was tired but interactive. This morning, he is difficult to arouse and not making good eye contact. You are immediately concerned about dehydration. You order laboratory tests to be drawn as the intravenous line is placed. Vital signs show a temperature 37.6°C, heart rate of 145 beats/min, respiratory rate of 32 breaths/min, and blood pressure of 88/53 mm Hg. On physical examination, you note an ill-appearing, lethargic infant with a sunken fontanelle, decreased tearing, and dry skin with tenting. Liver margin is palpable 2 cm below the costal margin. Mild hypotonia is present. He is also difficult to arouse. Blood glucose level at time of intravenous placement is 30 mg/dL (1.7 mmol/L). Urinalysis shows no ketones or signs of infection. Complete blood cell count and C-reactive protein tests are unremarkable. Serum ketones are 0.2 mg/dL (0.02 mmol/L, reference range, 0.5-3.0 mg/dL [0.05-0.30 mmol/L]). Serum bilirubin is 1.0 mg/dL (17.1 µmol/L). Aspartate aminotransferase and alanine aminotransferase levels are 180 U/L and 192 U/L, respectively. Ammonia level is 175 µg/dL (125 µmol/L). Blood, urine, and cerebrospinal fluid cultures are pending. Family history is remarkable for a prior sibling that died of sudden infant death syndrome at 5 months of age after a viral illness. Past medical history is unremarkable. The infant is begun on broad spectrum antibiotics. Based on the examination and studies obtained thus far, you highly suspect an inborn error of metabolism. You immediately begin a 10% dextrose solution at an infusion rate of 10 mg/kg per min, with close monitoring of the blood glucose level. Metabolic studies are pending. Of the following, the MOST likely diagnosis is A. hereditary tyrosinemia type 1 B. maple syrup urine disease C. medium-chain acyl-CoA dehydrogenase deficiency D. peroxisomal biogenesis disorder E. phenylalanine hydroxylase deficiency

A. absence of melanocytes The child in the vignette demonstrates the typical appearance of vitiligo: lesions with chalk white skin color and sharply demarcated borders. In this condition, melanocytes undergo destruction, seen microscopically as loss or absence of melanocytes, and decreased melanin in the few remaining melanocytes. Vitiligo is a fairly common cause of depigmentation with peak onset between 10 and 30 years of age; more than 50% of patients have onset before 20 years of age. Initial lesions commonly involve the hands, forearms, feet, and face, although eventually the condition can affect any body part, including mucous membranes. Lesions are usually asymptomatic. Affected areas are more easily seen in people of darker complexion, although detection of subtle lesions may be enhanced by using an ultraviolet light at 365 nm wavelength. In nearly half of patients, hair within the lesion is affected as well-producing leukotrichia (poliosis, loss of pigment); however, follicular disruption is not noted. Lesions occurring in a dermatomal distribution are called segmental vitiligo and may have a worse prognosis for repigmentation. Lesions are extremely sensitive to sun injury. The cause of vitiligo is unclear, although it is likely multifactorial and many cases involve a genetic predisposition. Twelve percent to 35% of affected pediatric patients have a family history of vitiligo. There is an observed association between the condition and autoimmune disorders, most notably autoimmune thyroiditis, though the association is better defined for adult patients than for children. Because vitiligo may precede the onset of autoimmune thyroiditis, some experts recommend thyroid antibody screening as part of the initial evaluation for vitiligo, though treatment of thyroid disease will not affect the vitiligo. Lesions often develop at sites of previous minor trauma, and new injury can lead to extension or development of additional disease areas in affected patients (Koebner phenomenon). Patients with vitiligo are observed to have greater rates of anxiety and depression than the general population, and stress (physical, psychological) has been proposed as an inciting event for the condition. Vitiligo is not associated with fungal infection, atopic disease, or infectious etiologies; therefore, skin biopsy is unlikely to show fungal hyphae, immunoglobulin E deposition, or neutrophilic infiltration. Treatment of vitiligo is usually observation because up to 25% of patients undergo spontaneous repigmentation. However, the psychological stress of these very noticeable lesions should be considered, and various treatments have been tried to ameliorate them. Camouflage cosmetic products can be helpful for many patients. High-potency topical steroids and calcineurin antagonists have shown some benefit, particularly for children. Phototherapy, sometimes combined with topical steroids, is commonly used for patients with widespread disease. Vitamin-D derivatives such as calcipotriol have been effective in some cases. Treatment with a 308-nm excimer laser has recently shown promise for localized lesions. There are many other causes of depigmentation in children including postinflammatory hypopigmentation, tinea versicolor, hypopigmented nevi, lichen sclerosus, and morphea. Among the most common is pityriasis alba, an asymptomatic condition primarily affecting children between 3 and 16 years of age. Pityriasis alba occurs most commonly on the face and appears as indistinct, irregular, slightly scaly, faintly hypopigmented areas. Lesions are more easily seen in children with darker skin tone and after sun exposure. The cause of the condition is unknown, but many experts feel it is a low-grade eczematous dermatitis. An infectious cause has been proposed, but there is no confirmed relationship between pityriasis alba and a specific organism. Skin biopsy is rarely indicated, but if done, it may show a variety of nonspecific histopathologic findings. However, there is usually a decreased amount of melanin in a normal or increased number of epidermal melanocytes. Treatment is primarily aimed at careful sun protection and adequate lubrication; low potency topical corticosteroids or calcineurin inhibitors may be used when there is concern about the cosmetic appearance. The condition usually resolves by late adolescence. PREP Pearls - Vitiligo, characterized by the absence or destruction of melanocytes, is a common cause of skin hypopigmentation in children. characterized by absence or destruction of melanocytes. - Vitiligo presents as sharply demarcated, markedly hypopigmented areas, often involving hands, forearms, feet, and face; eventually any body area may be involved, including hair and mucous membranes. - Vitiligo is sometimes associated with autoimmune diseases, particularly autoimmune thyroiditis. - Pityriasis alba is a form of low- grade eczema leading to indistinct areas of hypopigmentation, particularly involving the face. - Pityriasis alba, is treated with careful sun protection and lubrication, typically resolves by late adolescence. ABP Content Specifications(s) - Recognize the clinical features of pityriasis alba - Recognize the clinical manifestations of vitiligo

An otherwise healthy child visits your office for evaluation of hypopigmented areas on both knees that are nonpruritic, sharply demarcated, and nonscaly (vitiligo). The mother reports that his father has similar lesions. The family requests a skin biopsy to confirm the diagnosis. Of the following, a skin biopsy on this child would MOST likely demonstrate A. absence of melanocytes B. follicular disruption C. fungal hyphae D. immunoglobulin E deposition E. neutrophilic infiltration

A. body mass index percentile The risk of an individual child becoming overweight is influenced by several genetic and environmental risk factors. Those with a family history of obesity or those born with low birth weight from maternal smoking (or any cause) are at increased risk for obesity. An important message is that fad diets or excessive activity alone are to be avoided. Rather, adopting a healthy lifestyle that includes regular exercise and healthy eating, and matching the calories consumed to the amount expended with exercise to maintain a healthy weight and growth pattern is key to avoiding obesity. PREP Pearls - Documenting height, weight, and body mass index frequently will allow for early detection and intervention for children who are moving in the direction of overweight status. - Prevention of obesity is important, as most lifestyle interventions later on in adolescence are less successful. ABP Content Specifications(s) - Know which interventions have been effective and ineffective in caring for patients of various ages who are obese - Identify the genetic and environmental risk factors for obesity, including lifestyle choices

In order to enhance the preventive care in your office's practices, you decide to use your new electronic health record (EHR) to identify patients with risk factors that lead to obesity and adult diseases. You make changes to the EHR template that should help the physicians in your practice capture the required information. When reviewing records a few months later, you notice that many details are not recorded, despite the template prompts. In order to develop a continuous quality improvement action plan, you need to consider the desired information in order of importance. Of the following, the MOST important item to record is A. body mass index percentile B. detailed dietary intake C. family history of obesity D. gestational age and birth weight E. maternal smoking during pregnancy

E. supine sleep position Since the introduction of the "Back to Sleep" campaign in 1994, the incidence of sudden infant death syndrome (SIDS) has decreased by half. The American Academy of Pediatrics (AAP) policy statement on SIDS and other sleep-related infant deaths provides guidance on appropriate safe sleep recommendations. Evidence strongly supports that the supine sleep position is the safest, and is recommended in all but a very limited number of cases. In the past, the side sleep position was advocated to prevent aspiration, but no data support this as protective, but evidence shows that the SIDS rate is higher for children sleeping in this position. Infants with gastroesophageal reflux disease should be placed supine to sleep, unless they have an upper airway disorder that impairs airway protective mechanisms, in particular, certain types of laryngeal clefts. The association between prone sleep position and SIDS is at least as strong, and possibly stronger for preterm infants than for term infants. Therefore, preterm infants should sleep in the supine position as soon as clinical status is stable and at least by 32 weeks postmenstrual age. Although SIDS has shown a dramatic decrease, the number of deaths attributed to other sleep-related causes, including suffocation and strangulation, quadrupled from 1984 to 2004. Many experts use the term sudden unexpected infant death syndrome (SUIDS) rather than SIDS to describe deaths of young infants not attributed to a known condition or injury, particularly those occurring in a sleep environment. In recognition of other risks in the sleep environment, "Back to Sleep" campaign messages have expanded to include a more comprehensive promotion of safe sleep. All infants should sleep on a firm, properly fitted mattress in a crib, bassinet, or approved play yard. The sleep surface should be free of soft objects or loose bedding (including quilts, blankets, bumpers, sheepskins, toys, etc) that can obstruct the airway. Wedges and positioners should not be used. Sitting devices such as car seats are not recommended for routine sleep and can increase the occurrence of gastroesophageal reflux and positional plagiocephaly. Although room sharing with the parents for at least the first several months of life decreases the risk of SIDS by as much as 50% and is therefore recommended, bed sharing is more controversial. At present, the AAP recommends against bed sharing, citing a summary odds ratio for SIDS of 2.88 with bed sharing (95% confidence interval: 1.99-4.18). One of the new recommendations that pediatricians should relay to parents and to nursery staff is that bed sharing by twins and higher order multiples leads to several risk factors for SUIDS including overheating, rebreathing, side positioning, and accidental suffocation, and therefore increases their risk of sleep-related death. Data that show the benefits of bed sharing by twins are limited; therefore, they should be placed on separate surfaces for sleep, starting in the newborn nursery. In-bed devices marketed to make bed sharing safe are not recommended. Breastfeeding has been shown to be protective against SIDS and should be encouraged. Tobacco, alcohol, and drug use, both during pregnancy and after birth, increase SIDS risk and are discouraged. Epidemiologic evidence indicates that pacifier use may be protective against SIDS, and their use should be considered. Home apnea alarms do not protect against SIDS and are discouraged. There are limited data to allow for reliable recommendations regarding other factors that have been suggested as having an impact on sudden death, including ventilation, swaddling, and environmental toxins. PREP Pearls - Except in a very few instances, supine is the safest sleep position for an infant including those with gastroesophageal reflux. - The safest place for an infant to sleep is on a firm, well-fitted mattress in an approved crib, bassinet, or play yard free of other objects and located in the parents' room. - Current American Academy of Pediatrics recommendations extend beyond "Back to Sleep" to address other sleep environment factors. - Bed sharing, while controversial, is not recommended, and even twins should sleep on separate sleep surfaces. ABP Content Specifications(s) - Counsel parents regarding prevention of sudden infant death syndrome

Question 77 You are the primary care physician for newborn twins born at 33 weeks' gestation and discharged from the neonatal intensive care unit after a 4-week stay. They required oxygen for 3 days after birth, but little other respiratory support. They were diagnosed with gastroesophageal reflux disease. They have occasional episodes of coughing and gagging without color change. They are otherwise well. Of the following, the BEST advice to give the parents is to place the twins in A. prone sleep position B. side sleep position C. sitting devices, such as car seats, during sleep D. sleep positioners to keep their heads elevated E. supine sleep position

D. not appropriate for a child with a suspected concussion The soccer player in the vignette has sustained a concussion. Same day return-to-play is never appropriate for children and adolescent athletes following concussion. The Consensus Statement on Concussion in Sport delineates the criteria for diagnosis of sports concussion. An athlete must sustain a direct blow to the head or experience linear or rotational acceleration of the skull as a result of a transmitted force. The direct or transmitted force must result in symptom development, generally within several hours following injury. Symptoms fall into 1 of 4 groups: somatic (eg, headache, vision changes), cognitive (eg, memory disturbance, difficulty concentrating), mood changes (eg, irritability, depression), and sleep disturbance. This girl's injury meets the criteria for concussion diagnosis; she experienced a direct blow to her head and became symptomatic immediately after her injury. The treatment of concussion includes physical rest from all exertional activities, including physical activities with minimal risk of contact. Cognitive rest should also be implemented for a child with concussion symptoms. Cognitive rest may include time off from school, school accommodations (eg, limited tests and homework), and curtailing screen time (eg, watching television, using computers or smart boards, and texting). Initiating a step-wise gradual return to physical activities is indicated once individuals with concussion have been asymptomatic for 24 hours, as long as the neurologic examination is unremarkable. Additional evaluation, such as computerized neuropsychological testing, may be used as an adjunct to determine when return to play is appropriate. Both the Clinical Report on Sports-Related Concussion from the American Academy of Pediatrics and the Consensus Statement on Concussion in Sport detail the appropriate progression for return to physical activity following sports concussion. The first 3 responses are incorrect because same day return-to-play following concussion is contraindicated, even for individuals with short symptom duration and normal physical examination. Loss-of-consciousness is not a criterion for the diagnosis of concussion. Headache is the most common concussion symptom, though it is not exhibited in about 25% of individuals with concussion. The presence or absence of headache should not be used as a marker of concussion severity. PREP Pearls - Same-day return to play is never appropriate for children and adolescent athletes following a suspected concussion. - Loss of consciousness is not required for the diagnosis of concussion. - Up to 25% of children with concussion do not report headache. ABP Content Specifications(s) - Understand the criteria for return to play in sports after a head injury

You are volunteering as a team physician for a high school soccer tournament. During a game, a 14-year-old adolescent girl is elbowed in the head by another player. You evaluate the injured teenager on the sideline; the girl reports dizziness, nausea, and headache, but physical examination findings are unremarkable. You hold the girl out of the remainder of the game. After the game, the girl reports that all symptoms resolved within 15 minutes after the injury. She and her parents would like to know if she could participate in a tournament game later that day. Of the following, the MOST appropriate recommendation for this family is that same-day return to play is A. appropriate for this girl because physical examination was completely normal B. appropriate for this girl because she did not lose consciousness C. appropriate for this girl because symptoms resolved in less than 30 minutes D. not appropriate for a child with a suspected concussion E. not appropriate for this girl because headache was one of her symptoms

E. there were a family history of febrile seizures The girl in the vignette had a simple febrile seizure. Febrile seizures are generalized tonic clonic seizures that occur in children, typically between 6 months and 6 years of age, associated with a fever. They are divided into 2 types: simple and complex. Simple febrile seizures last less than 15 minutes and occur only once in 24 hours. Complex febrile seizures last longer than 15 minutes, have focal features, or present with more than 1 seizure in 24 hours. Complex febrile seizures suggest an underlying disorder causing seizures, such as a brain lesion or an epileptic syndrome, and may require further diagnostic evaluation. Simple febrile seizures often have no underlying cause and resolve over time. After a first simple febrile seizure, approximately 30% of children will have a recurrent febrile seizure. The factors that increase the risk of recurrent simple febrile seizure are younger age at the first febrile seizure, family history of febrile seizures, lower degree of fever at the time of seizure, and shorter time interval between the onset of fever and the first seizure. The features of complex febrile seizures, such as duration of the first seizure or increased numbers of seizures in the first episode, do not increase recurrence risk. The finding of a bacterial pathogen does not increase the risk of recurrent febrile seizures. For estimating recurrence risk, all the seizures that occur in one 24-hour period count as just 1 seizure episode. Even though recurrence risk may be low, children who have had a flurry of seizures or prolonged seizures may benefit from a "rescue" benzodiazepine medication at home, in case they do recur. PREP Pearls - Simple febrile seizure recurrence risk is approximately 30%. - Risk factors for recurrent febrile seizure include younger age at the first febrile seizure, family history of febrile seizures, lower degree of fever at the time of seizure, and shorter time interval between the onset of fever and the first seizure. ABP Content Specifications(s) - Understand the risk factors associated with febrile seizures - Understand the difference between simple and complex febrile seizures

You saw a 15-month-old girl for a health supervision visit last week. At that time, she had no medical problems and was developing typically. After leaving your office, she developed a temperature of 39°C and had 1 generalized tonic-clonic seizure about 2 minutes long. She was evaluated in the emergency department, diagnosed with acute otitis media and discharged home. Today, the girl and her mother return for follow-up. The mother expresses concern about the chance of another febrile seizure. You tell her that her risk of recurrent febrile seizure is about 30%. Of the following, the MOST correct statement is that this girl's risk of recurrent febrile seizure would be higher if A. a bacterial pathogen had been identified B. her temperature had been 40°C or higher C. her seizure had lasted 30 minutes or longer D. she were 24 months old or older E. there were a family history of febrile seizures

C. iridocyclitis Uveitis, also known as iridocyclitis, is a serious complication of juvenile idiopathic arthritis (JIA) resulting from chronic nongranulomatous inflammation of the anterior eye chamber that affects the iris and ciliary body of the eye. The uveitis associated with JIA is usually clinically silent with an insidious onset. Risk factors for developing uveitis in JIA patients include JIA subtype, age at onset of disease, and antinuclear antibody (ANA) status. The highest risk group is female patients with pauciarticular JIA (≤4 joints in the first 6 months of diagnosis) that are ANA positive and diagnosed before 4 years of age. Screening guidelines have been developed for uveitis based on risk. The severity of uveitis does not correlate well with arthritis activity; therefore, the status of joint disease should not affect the frequency of screening. Although ANA is often positive in patients with JIA who have uveitis, it is important to remember that ANA is positive about 57% of the time in all patients with JIA. No laboratory studies are diagnostic of JIA. Some laboratory studies can support the diagnosis of JIA such as anemia, leukocytosis, elevated acute phase reactants, and hypergammaglobulinemias. However, these laboratory values are not consistently abnormal. Cataracts are a late complication of chronic uveitis or chronic ophthalmic steroid use and are not associated with specific laboratory findings. A young patient recently diagnosed with JIA would be unlikely to have this complication. A positive rheumatoid factor (RF) is uncommon in children younger than 7 years, and is only positive in 10% to 15% of patients with JIA. Patients with JIA who have bone erosions are more likely to be RF positive. Osteoporosis is a complication seen with chronic steroid use in patients with difficult-to-control JIA. Osteoporosis is not associated with any antibody study abnormalities. Vitamin D and calcium should be supplemented in patients with chronic steroid use to help prevent osteoporosis. Leg length discrepancy, resulting from increased inflammation in affected joints that increases blood flow and growth factors, is more common in patients with JIA who have prolonged poor disease control. Some of these patients may have increased acute phase reactants, such as erythrocyte sedimentation rate, C-reactive protein, and thrombocytosis; however, there is no association of leg length discrepancy with antibody studies. PREP Pearls - No laboratory studies are diagnostic of juvenile idiopathic arthritis (JIA). Laboratory results suggestive of JIA include anemia, leukocytosis, thrombocytosis, and elevated inflammatory markers (erythrocyte sedimentation rate and C-reactive protein). - Antinuclear antibody (ANA) titers are present in just over half of patients with JIA. - A positive ANA test is associated with a higher risk of uveitis in young, female patients. - Rheumatoid factor is positive in a small number of patients with JIA and is associated with bone erosions. ABP Content Specifications(s) - Recognize the laboratory findings associated with juvenile rheumatoid (idiopathic) arthritis and its complications

A 3-year-old girl presents to your practice with knee swelling. She was referred to rheumatology after the initial workup ruled out septic arthritis. The pediatric rheumatologist informs you that the child has juvenile idiopathic arthritis. The patient is antinuclear antibody positive and rheumatoid factor negative. Of the following, the patient is at the MOST risk for A. bone erosions B. cataracts C. iridocyclitis D. leg length discrepancy E. osteoporosis

C. karyotype Girls with Turner syndrome, such as the girl in the vignette, have characteristic physical features coupled with complete or partial absence of the second sex chromosome. Guidelines from the Turner Syndrome Study Group recommend considering the diagnosis of Turner syndrome in any girl with unexplained growth failure, pubertal delay, or any constellation of the following clinical findings. The girl in this vignette has a high arched palate, short metacarpal bones, and poor linear growth. The most common clinical feature seen in Turner syndrome is poor linear growth (short stature). A karyotype should be considered in a girl with abnormal linear growth and otherwise normal screening test results, even with unremarkable physical examination findings and no other phenotypic findings of Turner syndrome. Given the child's poor linear growth, growth hormone deficiency is also a possible diagnosis. A random or AM growth hormone level, however, is not useful, as growth hormone levels peak overnight during deep sleep. During the day, they are often low. That is why growth hormone stimulation testing is sometimes performed by a pediatric endocrinologist. A better test to screen the growth hormone axis is insulin-like growth factor 1 or insulin-like growth factor binding protein 3. Concentrations of these hormones are much more stable in the circulation and can be measured during clinic visits. However, the child in this vignette has classic physical findings associated with Turner syndrome, and a karyotype (chromosomal analysis) is the best test to make this diagnosis. PTPN11 testing for Noonan syndrome would not be a first-line test without specific findings of this disorder present on examination. Noonan syndrome is characterized by proportionate postnatal short stature, dysmorphic facial features, chest deformities, and congenital heart disease (most commonly pulmonary valve stenosis and hypertrophic cardiomyopathy). Developmental delay, cryptorchidism, and clotting disorders are also common. This condition is autosomal dominant. Both males and females can have Noonan syndrome. Hypothyroidism is a very common cause of poor growth, and all children who are growing poorly deserve consideration of thyroid function testing. However, the child in the vignette very clearly has physical examination signs consistent with a syndromic cause of her poor growth, and a karyotype will confirm the diagnosis of Turner syndrome in this child. PREP Pearls - The most common clinical feature seen in Turner syndrome is poor linear growth (short stature). - Turner syndrome can present with normal physical examination findings other than poor growth or delayed puberty. - Girls with unexplained short stature should have karyotyping performed. ABP Content Specifications(s) - Recognize the clinical features associated with primary gonadal dysgenesis (Turner syndrome) in patients of various ages - Plan the appropriate clinical evaluation of a patient with primary gonadal dysgenesis (Turner syndrome)

An 8-year-old girl is referred for evaluation of poor linear growth. Her height was at the third percentile at 6 years of age, but her linear growth velocity then declined over the next 2 years. Her weight continues to track at the third percentile. Her history is significant for a mild speech impairment thought to be related to her high arched palate. On physical examination, you also appreciate a short fifth metacarpal bone in both hands (Item Q114). The remainder of her physical examination is unremarkable. Of the following, the BEST test to confirm this patient's underlying diagnosis is A. AM growth hormone level B. insulin-like growth factor 1 C. karyotype D. PTPN11 testing E. thyroid-stimulating hormone

C. obtain an occupational therapy assessment Occupational therapists (OT) play a vital role in evaluating and treating dysfunctional feeding practices and food aversions, and should be involved early in the evaluation of a child with failure to thrive. Although previously children were often admitted to the hospital to observe weight gain in a controlled environment, cost issues and the recognition that the child is usually better served in his own home environment now make hospitalization a less useful approach to the management of failure to thrive in all but the most severe cases. While many practitioners obtain screening laboratory tests for children with failure to thrive, studies have shown that laboratory evaluation rarely detects problems that were not already indicated by the history and physical examination. PREP Pearls - Failure to thrive is a physical sign of undernutrition that is usually multifactorial. -A multidisciplinary team is useful in the evaluation and management of failure to thrive. -Hospitalization for evaluation and treatment of failure to thrive is reserved for severely affected children or cases in which social factors place the child at significant risk. -Evaluation and treatment of failure to thrive is aimed at improving nutrient quantity and quality, optimizing feeding practices, and overcoming food aversions. ABP Content Specifications(s) - Plan the management of an infant with failure to thrive

At his health supervision visit, you note that a 15-month-old child's weight has fallen to the third percentile, while his height and head circumference remain at the 25th to 50th percentile. He was born at full term with an appropriate weight for gestational age. The child has had no illnesses other than mild upper respiratory infections; he has no history of vomiting, diarrhea, or constipation; and he has had no wheezing or breathing problems. His mother reports he is a picky eater who often refuses food of various textures. There is no family history of growth problems or chronic diseases. He has lived alone with his mother since his father was deployed with the military 2 months ago. Physical examination findings are unremarkable, except for the low weight. Of the following, the MOST appropriate initial management for this child is to A. begin supplemental nasogastric feedings B. hospitalize him to monitor weight gain C. obtain an occupational therapy assessment D. order thyroid-stimulating hormone and insulin-like growth factor 1 levels E. refer him to a child psychiatrist for separation issues

D. a history of having a renal transplant' Melanoma is a rare cancer in children and youth despite a 2% per year increase in incidence over the last 30 years. Risk factors for melanoma include white race, female gender, adolescent age group, family history of melanoma (but not other skin cancers), and immunodeficiency. Prior organ transplant confers a 4-fold increased risk of melanoma compared with the general pediatric population, and survivors of childhood cancer, especially leukemia and retinoblastoma, are at even higher risk. Other aspects of the history and physical examination that are found more frequently in children and youth with melanoma include xeroderma pigmentosum, presence of dysplastic nevi, and a large number of melanocytic nevi. Few, small congenital nevi (less than a few centimeters in diameter) are not associated with melanoma development among children. Dysplastic nevi may transform into melanoma, but they also indicate an increased risk of developing melanoma elsewhere on the body. Although blistering sunburns during childhood are risk factors for onset of melanoma during adulthood and sometimes adolescence, they are not often associated with childhood-onset melanoma. Recent evidence however suggests UV-A exposure from tanning beds is a risk factor in the increasing rate of melanoma in both adolescents and adults. Clinical features of melanoma are well defined for adults, but are less consistent for children. For adults, the "ABCD" (Asymmetry, Border irregularity, uneven Color and Diameter > 6 mm) approach has increased awareness and recognition of melanoma. In pediatric patients, melanoma often is recognized only because of a change in a pre-existing nevus. These changes include rapid growth, bleeding, color change, or itch. Additional findings can include unexplained lymph node enlargement, subcutaneous mass, and pain from distant metastases. Unfortunately in children, melanoma may be diagnosed late because the lesions are often asymptomatic or even amelanotic. Vigilance for changes in pre-existing nevi, especially in the presence of the above risk factors, is important for early diagnosis of melanoma in children. PREP Pearls - The incidence of melanoma among adults and children has been increasing over the last 30 years, but it still remains uncommon in children. - Risk factors for melanoma development in pediatric patients include white race; female gender; adolescent age group; immunodeficiency; solid organ transplant; childhood malignancies; the presence of xeroderma pigmentosum, dysplastic nevi, or many melanocytic nevi; and a family history of melanoma. - Melanoma in children is most often detected because of a change in the appearance of a pre-existing nevus, although lesions are often asymptomatic. - The "ABCD" approach to diagnosing melanoma is less reliable in children than in adults. ABP Content Specifications(s) - Recognize the clinical findings associated with melanoma

During a 9-year-old boy's health supervision visit, his mother tells you that she recently heard a news report about melanoma in children. She asks you if her child is at risk for this disease. Of the following, the factor MOST likely to increase his risk is A. a 7-mm congenital pigmented nevus B. a family history of basal cell skin cancer C. a history of a blistering sunburn D. a history of having a renal transplant E. male sex

C. incomplete starch digestion Until complete exocrine pancreatic function is achieved (between 4 and 6 months of age), and certainly in infants younger than 4 months of age, dietary starches may be hydrolyzed incompletely by salivary amylase alone. No available evidence supports the hypothesis that the introduction of solid foods either accelerates development of oral-motor skills or helps infants to sleep through the night. Although cereals are generally the first solids introduced, no studies support any particular sequence of solid food progression. However, common sense dictates that new foods should be offered one at a time to confirm tolerance. The absence of atopic signs, including vomiting, eczema, and urticaria, or the more common presentation of gastrointestinal bleeding, make the diagnosis of cow's milk protein allergy unlikely. Congenital lactase deficiency is rare and generally associated with severe diarrhea and inanition during the newborn period. Late-onset primary lactase deficiency is extremely common worldwide, with an onset usually towards the end of the first decade of life or later. Secondary lactase deficiency, as a consequence of intestinal mucosal injury, may accompany a wide range of gastrointestinal insults. Sucrase-isomaltase deficiency (CSID), the most common congenital disaccharidase deficiency, may present after the introduction of cereals or fruits to the infant diet. In CSID, the presenting symptom is most frequently a watery, acidic (pH <6.0) diarrhea. PREP Pearls - Symptoms of colic, including flatulence and bloating, are often the result of bacterial hydrolysis of unabsorbed carbohydrate. - In infants younger than 4 months of age, dietary starches may be hydrolyzed incompletely. - Lactase deficiency is extremely rare during the first year after birth, unless it occurs as the consequence of intestinal mucosal injury. ABP Content Specifications(s) - Identify the age-related changes in the ability to absorb and digest different nutrients relevant to infant feeding - Understand the consequences of introducing solid food prematurely - Know the appropriate age and sequence for introducing solid food into an infant's diet

During a routine health supervision visit, the mother of a 10-week-old infant tells you that her baby has been experiencing bloating and flatulence for the past week. His diet consists of 5 oz to 6 oz of a cow milk-based formula given 5 times per 24 hours. Because of frequent episodes of emesis after feedings, rice cereal was added to each bottle at 2 months of age. The baby has 4 to 5 loose to seedy stools per day. On physical examination, the baby is alert and vigorous. His length and weight are tracking between the 50th and 75th percentiles. The infant's mother asks you whether switching to a soy protein-based formula will help her baby's "gassiness." Of the following, the MOST likely cause of this infant's symptoms is A. cow-milk protein allergy B. excessive energy intake C. incomplete starch digestion D. lactose malabsorption E. sucrase-isomaltase deficiency

D. stop all egg and egg products The child in the vignette has atopic dermatitis and possible food allergy to egg. She has a history of developing allergic urticaria and worsening of her eczema with ingestion of egg. She should therefore avoid exposure to egg and egg products, and undergo allergy testing to further evaluate the suspicion of food allergy to egg. Recently published National Institute of Allergy and Infectious Diseases Food Allergy guidelines suggest that children younger than 5 years of age with moderate to severe atopic dermatitis (AD) be considered for evaluation for a food allergy trigger if at least 1 of the following conditions is met: persistent AD, despite optimized management and topical therapy, or reliable history of an immediate reaction after ingestion of a specific food. Based on the history, it is possible that this child has an egg allergy, exacerbating the eczema. Many children with AD have sensitive skin that reacts on contact with foods that can cause irritation. These foods include fresh fruit juices such as strawberry, citrus fruits, ketchup, pasta sauce, soy sauce, and similarly strong flavored and acidic substances. These contact reactions can be distinguished from true food allergy in that the suspected food is tolerated if a barrier moisturizer is applied to prevent direct contact, or if the food can be tolerated in other preparations. The child in the vignette can tolerate strawberry-flavored yogurt, but has a contact reaction when strawberry touches the skin around the mouth. She does not have an allergic reaction to potatoes because she can eat mashed potatoes without any reaction. The ketchup, along with the French fries, is causing her irritant contact reaction around her mouth. Many children with a similar history of having local reactions to strongly flavored tomato sauces are able to tolerate tomato in cooked dishes, thereby distinguishing it from a true immunoglobulin E-mediated food allergy. As the AD improves and the child's skin gets less sensitive with age, they are able to tolerate these sauces. While diphenhydramine is helpful in mitigating itching in AD, its use prior to ingestion of suspected food allergy trigger is not advisable because it could potentially mask the symptoms of allergy. Use of low potency steroid cream on the face would lessen the amount of inflammation around the mouth, and might decrease the likelihood and severity of contact urticaria or dermatitis, but will not alter the course of allergic sensitization to the food. Mastocytosis refers to a group of rare disorders of excessive mast cell proliferation and accumulation limited to the skin (cutaneous mastocytosis), or involving bone marrow and other extracutaneous tissues (systemic mastocytosis). Children are usually affected by the cutaneous form of mastocytosis, with most improving or resolving by adolescence. An evaluation for a mast cell disorder is appropriate in a patient with features of mast cell activation, such as flushing, tachycardia, diarrhea, fatigue or musculoskeletal pain, hypotensive syncope, or near syncope affecting at least 2 organ systems. Patients may also present with recurrent and unexplained episodes of anaphylaxis. The physical examination must include a careful skin inspection to look for lesions of urticaria pigmentosa (small yellow-tan to reddish-brown macules, slightly raised papules or nodules, or plaque-like lesions that urticate or flush when rubbed, Darier sign), or mastocytomas (nodules and bullae). The presence of Darier sign is pathognomonic for the presence of mast cells within the lesion. However, lesions consistent with mastocytomas (localized accumulations of mast cells) should not be rubbed or scratched, as this may lead to generalized flushing and hives. Instead, historical verification of the occurrence of these symptoms, if the lesion is disturbed, should be attempted. This child does not have symptoms or signs (absence of skin findings) to suggest further evaluation of mastocytosis. PREP Pearls - Children younger than 5 years of age with moderate to severe atopic dermatitis (AD) should be considered for evaluation of food allergy if there is a history of an immediate reaction after ingestion of a specific food, or if the AD is hard to control despite optimized management. - Triggers of itching in patients with AD can include foods acting as contact irritants, vasodilators, or allergens. - Mastocytosis should be suspected in children with yellow-brown macules that urticate on stroking, or if there are unexplained, recurrent episodes of spontaneous urticarial or allergic symptoms associated with symptoms involving multiple organ systems. ABP Content Specifications(s) - Recognize the relationship between eczema and food allergies, and how to evaluate a patient for both

During a wellness visit, the parents of a 1-year-old girl with eczema report that their daughter develops a facial rash after eating different foods. She developed hives around her mouth and worsening of her eczema after eating scrambled eggs at 10 months of age. She was noted to develop a blotchy rash around her mouth when she ate fresh strawberries, but tolerated strawberry-flavored yogurt. She also gets a similar blotchy rash when she eats French fries and ketchup, but not with mashed potatoes. Her physical examination is unremarkable, except for dry skin around her mouth and on her cheeks, and a pea-sized, dry patch on her chin. Of the following, the MOST appropriate recommendation to the parent is to have the child A. be evaluated for mastocytosis B. begin a trial of diphenhydramine before suspected triggers C. begin low potency corticosteroid cream on the face D. stop all egg and egg products E. stop egg, tomato, strawberry, and potato

PREP Pearls Symptomatic heat illness occurs with a core temperature of 40°C (104°F) or higher. With exertional heatstroke, affected individuals have a temperature above 40.5°C (104.9°F) and mental status changes such as confusion and disorientation. Hypotension, tachypnea, sustained tachycardia, vomiting, and decreased sweating with dry skin are also frequently seen. Heat-related illness should be treated with rapid cooling to a body temperature of 38°C (100°F). At mass participation sporting events, ice water bath immersion is the preferred method of rapid cooling. Exercise-associated collapse generally occurs at the end of a long athletic event after a participant voluntarily stops the activity. Venous pooling in the lower extremities leads to transient hypotension and subsequent collapse. Affected individuals report feeling dizzy or faint prior to collapse. Treatment of exercise-associated collapse consists of placing the athlete in a supine position with the legs elevated and offering oral fluids. Hyponatremia is another possible cause of confusion and loss of consciousness. During a long endurance event, excess consumption of water can lead to hyponatremia. Clinicians should also consider arrhythmia, hypoglycemia, and seizure as possible causes of collapse after ruling out heat illness. - Rectal temperature is a quick and accurate method of assessing for the presence of heat-related illness. - A patient with heat-related illness should undergo rapid cooling to reduce body temperature to 38°C (100°F). ABP Content Specifications(s) - Understand the mechanisms of heat-related illness, including age-related factors - Plan the appropriate evaluation of heat illness, and manage appropriately - Recognize the clinical findings associated with heat illness, including complications

During the month of August, while you are acting as one of the medical providers for a marathon, volunteers carry a 16-year-old male adolescent into the medical tent. The volunteers report that he collapsed, unwitnessed, right before the finish line. On initial examination, the patient is mumbling and appears disoriented. His heart rate is 120 beats/min, and his respiratory rate is 20 breaths/min. His skin is cool and dry to the touch, and he is shivering. Of the following, the BEST next step would be to A. begin immediate rapid cooling B. obtain a blood glucose reading C. obtain a rectal temperature reading D. place an intravenous line and deliver a normal saline bolus E. place the patient in a supine position with his legs elevated

A. a calm, caring home environment with consistent discipline is an effective treatment for children with early neglect and maltreatment The boy in the vignette suffered neglect and abuse for the first 3 years of his life. As a result he has several risk factors for aggression. The fact that his mother was addicted to drugs may have been a risk factor for his current behavior problems, but not all substances have the same risks. Many randomized, controlled trials have provided evidence that various methods of behavior management training are clinically effective for oppositional defiant disorder, conduct disorder, and aggression. This approach was previously labeled as "parent management training." But the term was changed because it implies that parents are at fault for their child's behavior, which does not help build a therapeutic alliance. The core element of all behavior management training programs is instruction about the creation of a calm, caring home environment with consistent discipline. There is significant reason to hope for behavioral improvement for the boy in the vignette, but it may take years of corrective parenting and "proving" to him that someone really is there for him and has his best interests in mind. Medications like alpha-agonists and antipsychotics are sometimes given to children with significant oppositionality and aggression and can have short-term benefits, but they have not been demonstrated to yield any long-term improvements in functional outcome. Approximately 30% of children with oppositional defiant disorder (which may be an appropriate diagnosis for the child in the vignette) will progress to having conduct disorder, and approximately 50% of children with early-onset conduct disorder will progress to having antisocial personality disorder as adults. Therefore, the chance of this child becoming an adult with antisocial personality disorder is at most 15%. This is a high-risk period for the boy in the vignette. Continued unresponsive or punitive parenting at this age will make recovery from his early childhood difficulties much less likely. As such, one cannot reasonably state that regardless of what is done now most children will outgrow this problem within a few years. Intrauterine drug exposure is clearly linked to neurobehavioral outcomes in children. However, the outcome depends on many factors including the specific substance abused by the mother, the amount and frequency of use, and whether multiple drugs were abused. PREP Pearls - Children who have been abused, neglected, and/or prenatally exposed to drugs of abuse are at significant risk for aggression and acting out problems. - Young children with aggression problems of multifactorial origin are best treated with behavior management training, and with such care less than 15% will exhibit antisocial behaviors as adults. ABP Content Specifications(s) - Recognize the various environmental and biological contributors to the development and maintenance of aggressive behaviors

Search Unanswered Questions Question View: Mode: Learning Exam 1 of 108 (260 total)Print Add Bookmark ASSESSMENT PROGRESS:Total Questions: 260 Questions Answered: 153 Correct Answers: 92 Question 125 You are seeing a 5-year-old boy born to a drug-addicted mother. He was placed in a foster home at 3 years of age because of neglect and suspected abuse and was adopted into his current home 1 year ago. The adoptive parents report that at least twice a day, when he does not get his way, he will have a significant tantrum lasting at least 15 to 30 minutes. He is usually aggressive during these tantrums, throwing things at his parents, kicking walls, and destroying his toys and books. You have previously diagnosed him with oppositional defiant disorder and have not seen evidence of attention-deficit/hyperactivity disorder. After you provided the adoptive mom with tantrum management techniques and a referral to mental health services, she privately asks you, given his early life experiences, if there is any hope for her son's problem behaviors to improve. Of the following, the MOST appropriate statement to make to this parent is that A. a calm, caring home environment with consistent discipline is an effective treatment for children with early neglect and maltreatment B. medications for severe aggressive behavior will usually create lasting improvements in aggression even after medication discontinuation C. most children with aggressive oppositional defiant disorder at this age will develop antisocial personality disorder by adulthood D. regardless of what we do now, most children in this situation will grow out of their aggression problems within a few years E. research has shown that intrauterine drug exposure is not linked to aggressive or conduct disordered behaviors in adulthood

D. water at the start, midway, and finish of the race In the policy statement Climatic Heat Stress and The Exercising Child and Adolescent, the American Academy of Pediatrics recommends that children 9 to 12 years of age drink 3 to 8 oz of fluids every 20 minutes during sports participation. If most participants will finish in 40 minutes, race organizers should provide water at the start, halfway point, and end of the race. Sports drinks generally contain sugar and electrolytes. These sport drinks have a sweet taste, therefore children may hydrate more when given access to them. However, for exercise lasting 1 hour or less, water is sufficient for hydration. The 5k race described in the vignette will take place during the summer. Exercise during hot weather can put athletes at risk for heat illness. Children and teenagers can regulate body temperature as effectively as adults during exercise as long they hydrate appropriately. To reduce the risk of heat illness, children should be encouraged to incrementally increase activity when starting a new exercise program. Parents and coaches should make sure that training schedules for activities during hot weather months take into account the need for gradual heat acclimatization. Community physicians can help provide education to organizers of youth athletic events regarding the need to modify or cancel activities with extreme heat and humidity. Mass participation event organizers should understand the importance of staffing the event with personnel familiar with the signs, symptoms, and treatment of heat illness. If only 1 station with water or sports drinks were provided at the start or finish of the race, athletes would not have the opportunity to hydrate every 20 minutes as recommended for children and adolescents. While electrolyte-containing sports drinks may encourage hydration, hydration stations with water at the beginning, middle, and end of the race is sufficient for this 5 km race. PREP Pearls - Children and teenagers can regulate body temperature as effectively as adults during exercise, as long they hydrate appropriately. - Children should be provided with opportunities for hydration every 20 minutes during physical activity. - For exercise lasting 1 hour or less, water is sufficient for hydration. ABP Content Specifications(s) - Plan optimal age-appropriate replacement for fluid losses associated with athletic activity

The director of a local community organization is planning a 5-km (3.1-mile) race that is the culmination of a summer running program for children aged 9 years to 12 years. She asks for your advice about the best way to plan for adequate hydration of race participants. She estimates that most runners will finish the race in 40 minutes or less. Of the following, the MOST appropriate recommendation for hydration of runners in this race is that participants should be provided with A. drinks supplemented with electrolytes at every mile on the course B. drinks supplemented with electrolytes at the finish of the race C. water at the finish of the race D. water at the start, midway, and finish of the race E. water at the start of the race

E. vasovagal reaction The most likely cause of the reaction for the child in the vignette is a vasovagal reaction. Vasovagal syncope refers to syncope triggered by emotional or orthostatic stress such as venipuncture, painful or noxious stimuli, fear of bodily injury, prolonged standing, heat exposure, or exertion. This "classical" presentation commonly occurs in young and otherwise healthy individuals. There is often a prodrome of nausea, pallor, and diaphoresis consistent with increased vagal tone. Syncope is typically of short duration and occurs in the standing position. The supine position restores adequate blood flow to the brain and reverses the temporary loss of consciousness. This course may help distinguish vasovagal syncope from syncope associated with arrhythmia, which is typically of abrupt onset and of short duration. Anaphylaxis is typically associated with circulatory changes, such as tachycardia (initially) and hypotension, respiratory findings, such as wheeze, and cutaneous symptoms, such as urticarial rashes. The history, physical examination, and objective clinical and laboratory tests are important components in the evaluation and diagnosis of drug hypersensitivity. The history should focus on previous and current drug use, the toxicity and allergenicity of previously and currently used drugs, and the temporal sequence of events between initiation of therapy and onset of symptoms. Physical examination should include all systems that could possibly account for the clinical presentation. Immediate hypersensitivity type I reactions are immunoglobulin E (IgE)-mediated and result in immediate reactions, such as anaphylaxis. These are exemplified by symptoms of urticaria, laryngeal edema, wheezing, and cardiorespiratory collapse, which typically occur within min of exposure to the drug. Cutaneous manifestations are the most common presentation for drug allergic reactions. Although drug allergic reactions may present with noncutaneous physical findings, these are generally nonspecific and are not nearly as helpful in diagnosis and management decisions. A retrospective diagnosis of anaphylaxis may be determined by detecting an increase in serum total tryptase levels above baseline. The history of the boy in this vignette having tolerated the amoxicillin-clavulanic acid for 9 days rules out immediate hypersensitivity to this agent as a cause of this child's symptoms. Additionally, there were no cutaneous findings of urticarial rash, which is often seen in IgE-mediated reactions. The rate of penicillin-induced anaphylaxis after parenteral administration is about 1 to 2 per 10,000 treated patients. Approximately 10% of patients report a history of reacting to a penicillin-class antibiotic. However, when evaluated for penicillin allergy, up to 90% of these individuals are able to tolerate penicillins and therefore carry an erroneous label of being "penicillin allergic." The child is not latex allergic because he can tolerate exposure to other rubber products including balloons and rubber gloves. The history of tolerating ibuprofen for 3 days or more and the absence of symptoms of IgE-mediated allergy make allergy to ibuprofen and nonsteroidal anti-inflammatory drugs less likely. Possible systemic allergy to local anesthetics is often of concern to patients and their dentists or physicians. Documentation of IgE-mediated reactions is extremely rare. Most adverse reactions to local anesthetics are caused by nonallergic factors that include vasovagal responses, toxic or idiosyncratic reactions because of inadvertent intravenous epinephrine, or anxiety. Local anesthetics are either group 1 benzoic acid esters (eg, procaine, benzocaine) or group 2 amides (eg, lidocaine, mepivacaine). On the basis of patch testing, the benzoic acid esters cross-react with each other, but they do not cross-react with the group 2 amide drugs. It is not known what, if any, relevance this has on immediate-type reactions to local anesthetics, though it has been speculated that a person allergic to a benzoic acid ester may be able to tolerate an amide, and vice versa. PREP Pearls - Immediate hypersensitivity to a drug would typically manifest with symptoms within minutes to hours of being exposed to the medication with cutaneous symptoms of urticarial rash. Systemic reactions such as laryngeal edema, wheezing, and cardiorespiratory collapse would signal the onset of a severe systemic allergic reaction or anaphylaxis. - While anaphylactic reactions would present as tachycardia and hypotension, vasovagal reactions would have bradycardia and hypotension. ABP Content Specifications(s) - Recognize the clinical features associated with a drug allergy or hypersensitivity, and manage appropriately

The father of a 6-year-old boy calls your office to report that the boy had an allergic reaction while undergoing a dental filling for carious teeth. The boy had been very anxious about the procedure. Within 2 min of receiving the injection of local anesthetic lidocaine, he complained of feeling dizzy. His face turned pale and he was nauseated, according to what the father had been told. No hives or wheezing was reported. The procedure was immediately stopped and the child was given diphenhydramine and monitored. His symptoms improved and he was sent home after spending an hour under observation. He had been on day 9 of a 10-day course of amoxicillin-clavulanic acid for the tooth infection, and had also been taking ibuprofen for pain as needed for the prior 3 days. He has had dental procedures done before without problems and has never had any reactions with balloons or rubber gloves. Of the following, the MOST likely cause of the child's reaction is A. amoxicillin-clavulanic acid B. ibuprofen C. latex D. lidocaine E. vasovagal reaction

A. eczema herpeticum The child in the vignette most likely has eczema herpeticum as a complication of atopic dermatitis (AD). Atopic dermatitis can be complicated by viral skin infections, such as herpes simplex, warts, and Molluscum contagiosum, implicating local defects in T-cell function. The presence of punched-out erosions, vesicles, or infected skin lesions, as seen in this child, that do not respond to oral antibiotics should initiate a search for a potentially serious form of herpes simplex infection termed Kaposi varicelliform eruption or eczema herpeticum. Herpes simplex infection can be diagnosed by a Giemsa-stained Tzanck smear of cells scraped from the vesicle base, commercial immunofluorescence assays, or viral culture. Prompt institution of antiviral treatment, administered intravenously, is of critical importance in the patient with widespread AD because life-threatening dissemination has been reported. Notably, in patients with AD, smallpox vaccination or even exposure to recently vaccinated subjects can cause a severe, widespread, life-threatening dermatitis called eczema vaccinatum, which is similar in appearance to eczema herpeticum. Molluscum contagiosum commonly presents as single or multiple small, flesh-colored papules with central umbilication that typically resolves spontaneously in most immunocompetent individuals. Verrucae, or warts, can occur singly or in coalescing lesions, and can be flat, mosaic, or filiform in appearance. The diagnosis is usually made based on clinical observation. Spontaneous regression occurs in as many as two-thirds of warts within 2 years. The absence of fever, preceding illness, and oral vesicles on the buccal mucosa and tongue make hand, foot, and mouth disease a less likely diagnosis in this child. Skin infections, particularly with Staphylococcus aureus, can be a common and recurrent problem in patients with AD. Impetigo is a superficial bacterial infection most commonly caused by S aureus or β-hemolytic streptococci. It may be classified as primary impetigo (direct bacterial invasion of previously normal skin) or secondary impetigo (infection at sites of minor skin trauma, such as abrasions, or underlying conditions such as eczema). Nonbullous impetigo is the most common clinical form of impetigo with papular lesions that progress to vesicles and pustules that enlarge and break down to form thick, adherent crusts with a characteristic golden appearance. Patients with moderate-to-severe AD have been found to make immunoglobulin E antibodies against staphylococcal toxins present on their skin. Recent studies have demonstrated that TH2 immune responses increase binding of S aureus to inflamed skin lesions, contributing to the underlying inflammation present in patients with AD. This is supported by the observation that treatment with topical glucocorticoids or tacrolimus reduces S aureus counts on atopic skin. Hand washing to minimize the spread of staphylococci and preventive measures to decrease the amount of skin colonization may be helpful. Topical therapy is recommended in the management of impetigo with a small number of nonbullous lesions. Systemic antibiotics are indicated only in patients with extensive infection. PREP Pearls - Children with atopic dermatitis (AD) can have skin infections, such as herpes simplex, warts, impetigo, and Molluscum contagiosum, complicating their course and management. - Herpes simplex infection of the skin manifesting as eczema herpeticum or Kaposi varicelliform eruption is a complication of AD that mandates urgent diagnosis and prompt institution of appropriate antiviral therapy. - The presence of AD is a contraindication to receiving the small pox vaccine because patients can potentially get eczema vaccinatum, a severe and life-threatening dermatitis. ABP Content Specifications(s) - Recognize the clinical findings associated with atopic dermatitis - Recognize complications associated with atopic dermatitis - Plan the appropriate management of atopic dermatitis

The mother of a 2-year-old child with severe atopic dermatitis emails you a photograph of the child's left hand. The mother reports that the child has developed new bumps that look different from the typical red, dry, itchy eczematous rash. She describes the rash as a mixture of little blisters and sores. The child has been running a fever of 38.6°C for the past 2 days and has been tired and irritable. You view the photograph (some vesicles, many punched out erythematous ulcerations) and request the mother to bring the child in to see you that day for an urgent visit. Of the following, your MOST likely diagnosis is A. eczema herpeticum B. hand-foot-mouth disease C. impetigo D. molluscum contagiosum E. verruca vulgaris

E. pass a nasal suction catheter bilaterally Newborns who present with nasal stuffiness should have a suction catheter passed through both nares to verify patency. In the newborn period, nasal obstruction presents with a spectrum of clinical findings ranging from nasal stuffiness to respiratory failure. Severe nasal obstruction can be life threatening because newborns are obligate nasal breathers. Rapid identification and stabilization are essential. Nasal obstruction is most commonly caused by mucosal inflammation. In the immediate neonatal period, suctioning is often the underlying cause and the resultant swelling responds to supportive measures including humidity and saline drops. The initial evaluation will often demonstrate transient improvement with suctioning and passage of a 5-French catheter. If evidence of nasal obstruction persists, further evaluation with a fiberoptic scope, computed tomography (CT) scan, or magnetic resonance imaging (MRI) may be needed to rule out less common causes such as encephalocele and glioma. Choanal atresia or stenosis should be suspected if a 5-French catheter is unable to pass through one or both nares. Bilateral choanal atresia often presents in the delivery room in a newborn who is pink only with crying and turns blue when the crying stops. Attempts at ventilation using a bag mask are often unsuccessful, with prompt placement of an endotracheal tube improving oxygenation and ventilation. Neonates with unilateral choanal atresia or mixed choanal atresia/stenosis may present in the nursery with mild to moderate respiratory distress including nasal stuffiness, increased work of breathing with retractions, decreased aeration, and cyanosis. Affected neonates may worsen over time, as continued attempts to decrease their congestion by bulb suctioning only cause further mucosal swelling and increased obstruction. Newborns suspected to have choanal atresia or stenosis should be immediately transferred to a tertiary center for further evaluation, including flexible nasendoscopy and CT imaging of the paranasal sinuses and skull base. Several surgical techniques including transpalatal and endoscopic resection have been developed to establish patency of the nares. Restenosis may occur in up to 20% of cases. Newborns with choanal atresia should undergo a complete multisystem evaluation because up to 75% of cases have other anomalies. CHARGE (coloboma, heart defects, choanal atresia, retardation of growth and development, genitourinary anomalies, ear anomalies and hearing loss) association is the most common condition seen with choanal atresia. The newborn in the vignette has findings suggestive of unilateral choanal atresia or stenosis and should have a nasal suction catheter passed bilaterally. If the catheter passes through both nares, intranasal normal saline drops may be considered to decrease potential underlying mucosal swelling related to bulb suctioning while the newborn is closely monitored. Continued increased work of breathing, in spite of the demonstration of bilateral patent nares, may prompt the clinician to obtain a radiograph of the chest. Further imaging with CT or MRI may be done in consultation with a subspecialist. PREP Pearls - Newborns who present with nasal stuffiness should have a suction catheter passed through both nares to verify patency. - Choanal atresia or stenosis should be suspected if a 5-French catheter is unable to pass through one or both nares. - CHARGE (coloboma, heart defects, choanal atresia, retardation of growth and development, genitourinary anomalies, ear anomalies and hearing loss) association is the most common condition seen with choanal atresia. ABP Content Specifications(s) - Recognize the clinical findings associated with both unilateral and bilateral choanal atresia - Understand the association of choanal atresia with other congenital anomalies - Plan the appropriate evaluation of choanal atresia

While in the normal nursery, the nurse asks you to assess a newborn who has nasal stuffiness. The newborn was delivered 18 hours earlier by spontaneous vaginal delivery complicated by meconium-stained amniotic fluid. The newborn emerged vigorous and required only bulb suctioning in the delivery room. The newborn breastfed poorly overnight and had nasal stuffiness that briefly improved with bulb suctioning. Physical examination reveals a pink, well-perfused newborn with audible nasal congestion, mild supraclavicular retractions, and minimally decreased aeration of the lungs bilaterally. Of the following, the BEST next management step for this newborn is to A. administer intranasal normal saline drops B. continue to closely monitor C. obtain a radiograph of the chest D. order a computed tomography image of the neck E. pass a nasal suction catheter bilaterally

A. blood glucose level A blood glucose value should be obtained immediately, as the newborn in the vignette is demonstrating both symptomatic hypoglycemia and polycythemia. Although the hypoglycemia may be secondary to polycythemia, glucose homeostasis must be achieved without delay. It is essential to monitor blood glucose values in neonates with polycythemia, because up to 40% may have hypoglycemia. If required, an intravenous glucose infusion may also assist in the management of polycythemia through hydration and dilution. Polycythemia, defined as a venous hematocrit value greater than 65% or a hemoglobin value greater than 22 g/dL (220 g/L), has been associated with delayed cord clamping. Recent studies have centered on the timing of cord clamping after birth because of the perceived neonatal benefits of increased blood volume and iron stores, which reduce the risk of physiologic anemia. A committee opinion of the American College of Obstetricians and Gynecologists does not provide the ideal timing for cord clamping because of the lack of evidence in full-term newborns. Some proponents advocate for delayed clamping up to 2 min after delivery due to the existing studies, acknowledging the increased risk of polycythemia associated with this practice. The most common features associated with polycythemia in the newborn include cyanosis, poor feeding, and hypoglycemia. The increased blood viscosity may lead to decreased microcirculatory flow. This diminished flow compromises organ function, leading to neurologic, metabolic, cardiac, renal, and respiratory complications. A newborn with polycythemia may look cyanotic because the increased amount of hemoglobin coupled with normal oxygen extraction allows the deoxyhemoglobin level to exceed 5 g/dL (50 g/L). Although this level of deoxyhemoglobin gives the clinical appearance of cyanosis, the neonate is not hypoxic. The treatment of polycythemia remains controversial. Asymptomatic newborns with hematocrit values between 60% and 70% may be hydrated aggressively with enteral feedings or intravenous fluid, with careful monitoring for 24 to 48 hours. In symptomatic newborns with polycythemia, partial volume exchange transfusion may be performed to decrease the blood viscosity by replacing a calculated blood volume with normal saline. Necrotizing enterocolitis is a known complication of partial volume exchange transfusion. The newborn in the vignette is manifesting symptomatic hypoglycemia and his blood glucose should be checked immediately to determine if parenteral glucose is required. Further evaluation should then be pursued because of the clinical suspicion of polycythemia to guide further management, including obtaining a venous hematocrit and serum calcium level. Chest radiography may be performed at this time because of the tachypnea, as other causes should always be considered. Echocardiography may be considered if the clinical suspicion of congenital heart disease exists. PREP Pearls - Polycythemia is most commonly associated with cyanosis, poor feeding, and hypoglycemia. - Delayed cord clamping is associated with polycythemia. ABP Content Specifications(s) - Recognize the risks associated with polycythemia in newborn infants, and manage appropriately

You are asked to assess a newborn who appears dusky 4 hours after birth. The mother underwent a natural childbirth at term, including immediate skin to skin bonding and delayed clamping of the umbilical cord. The pregnancy was uncomplicated. Physical examination reveals a ruddy, appropriate for gestational age newborn with perioral cyanosis, unlabored tachypnea, and mild jitters. The temperature is 37°C, the heart rate is 160 beats/min, the respiratory rate is 70 breaths/min, and the oxygenation saturation in room air is 95%. Further evaluation reveals a soft I/VI murmur at the left sternal border, clear breath sounds bilaterally, and no abdominal masses. The mother reports that her baby has not latched well onto the breast since birth. Of the following, the MOST appropriate next step in management is to obtain a(n) A. blood glucose level B. chest radiograph C. echocardiogram D. serum calcium E. venous hematocrit

A. depressed and has some fresh cut marks on her arms If the girl in the vignette is depressed and exhibiting self-harm behavior, it may be necessary to intervene and notify her family. In seeking autonomy from parents and exploring the world around them, adolescents may engage in risky behaviors. Every visit with an adolescent is an opportunity to screen for these behaviors and intervene if necessary. It is important to remember that if you don't ask, they won't tell. Ensuring time alone and confidentiality with the adolescent becomes crucial in eliciting sensitive information. Confidentiality refers to an agreement between the patient and provider that information discussed during or after the encounter will not be shared with other parties without the adolescent's explicit permission. Laws regarding notification of parents vary by state; if none exist, then the mature minor doctrine is usually used. A mature minor is a minor who is emotionally and intellectually mature enough to provide informed consent. The minor may live under the supervision of a parent or guardian. Most states allow minors to consent to treatment for sexually transmitted infections, contraception, alcohol and substance abuse, and mental health care services. It is important to inform an adolescent at the first encounter that confidentiality will need to be breached if they are at risk of harming themselves (suicidal), or others (homicidal), or if they are being abused. If the parents need to be involved, that decision should be made with the knowledge of the adolescent and a plan should be made together as to how best to let the parents know. Guidance on dealing with an individual patient is influenced by the relationship with their parents, legal constraints, nature of the behavior, and severity of the consequences. In addition, the history may be helpful (ie, have they exhibited poor decision making with serious consequences to life and limb? Exhibited impulsiveness? Shown an ability to learn from past mistakes?). This information might lead to the decision to override the patient's preference, ie, justified paternalism. However, it is very important that this decision be documented in the medical record. PREP Pearls - Ensuring time alone and confidentiality with an adolescent is crucial in eliciting sensitive information. - Confidentiality must be breached if the adolescent is at risk for harming themselves (suicidal), others (homicidal), or if they are being abused. - The plan for how confidential information will be shared with the parent should be made with the adolescent, before disclosure. ABP Content Specifications(s) - Understand when a parent must be notified about an adolescent's medical condition

A 15-year-old adolescent girl presents to your office for a routine physical examination. She is now in high school and maintaining her grades. You speak with her alone as part of the visit, and after discussing the limits of confidentiality, you screen her for high-risk behaviors. She reveals a number of behaviors that put her health at risk but asks that you keep the information confidential. Of the following, you are MOST likely to deny her request if she is A. depressed and has some fresh cut marks on her arms B. experiencing frequent recurrences of herpes genital ulcers C. requesting an intrauterine device for contraception D. sexually active with inconsistent protection against pregnancy E. using marijuana frequently

D. polyuria Recurrent episodes of hypernatremic dehydration are indicative of increased free water loss. Such patients may have polyuria characterized by an increased total urine volume that results from an underlying defect in water balance. This presents with the excretion of large volumes of dilute urine, as seen in diabetes mellitus (osmotic diuresis), diabetes insipidus (antidiuretic hormone disorders), and psychogenic polydipsia. Both nocturia and an increased frequency of micturition may or may not be associated with polyuria. Episodes of recurrent hypernatremic dehydration, positive family history, and the patient's age are indicative of diabetes insipidus (DI), as seen in the patient in the vignette. Patients with DI may have a positive family history of polyuria indicative of increased free water losses. Disorders with positive family history for glucosuria, hematuria, renal failure, or vesicoureteral reflux have a different clinical presentation. Glucosuria may be seen in neonatal diabetes; however, serum chemistry in such patients reveals hyperglycemia with or without ketoacidosis and ketonuria. Familial hematuria is seen in patients with Alport syndrome (X-linked inheritance) and benign familial hematuria. Patients who have Alport syndrome present with asymptomatic microscopic hematuria and a history of renal failure in the male members of the family. Vesicoureteral reflux presents with recurrent urinary tract infections. Diabetes insipidus occurs secondary to either decreased secretion of antidiuretic hormone (ADH), known as central DI, or renal resistance to ADH effects, known as nephrogenic DI. Patients with DI present with polyuria, polydipsia, and increased thirst. While they may present with varying degrees of dehydration, laboratory evaluation is consistent with hypernatremia in association with a dilute urine (urine osmolality < plasma osmolality). Nephrogenic DI in children is secondary to a mutation in either the ADH-receptor (AVPR 2) or the aquaporin 2 channels. AVPR 2 mutations have an X-linked inheritance and account for 90% of cases; males are more severely affected than females. Neonates and infants with nephrogenic DI present with irritability, failure to thrive, and a preference for water, along with clinical findings of dehydration. Central DI is characterized by decreased ADH secretion. It can be idiopathic (most common) or secondary to central nervous system tumors, infiltrative lesions (histiocytosis), or trauma (surgical or nonsurgical). Water restriction, used to differentiate nephrogenic DI from central DI, is not indicated in neonates and infants. This differentiation is made through evaluation of the response to desmopressin (1 µg subcutaneously or intravenously infused over 20 minutes; maximum dose, 0.4 µg/kg of body weight) by measuring urine osmolality at baseline and at 30-minute intervals over 2 hours. If the urine osmolality remains at less than 100 mosmol/kg over baseline, the diagnosis of nephrogenic DI is confirmed. Psychogenic polydipsia presents with hyponatremia associated with a low urine osmolality that is consistent with water overload. Maximal urine concentration is usually impaired (500 to 600 mosmol/kg) compared to that in normal patients (800 mosmol/kg or more). Water deprivation in these patients will increase the urine osmolality (> 500 mosmol/kg), and there will be no response to desmopressin since endogenous production and release of ADH is normal. PREP Pearls - Polyuria is characterized by an increased total urine volume that results from an underlying defect in water balance. - Polyuria can be seen in diabetes mellitus (osmotic diuresis), diabetes insipidus (antidiuretic hormone disorders), and psychogenic polydipsia. - Patients with diabetes insipidus present with polyuria, polydipsia, increased thirst, varying degrees of dehydration, and laboratory evaluation consistent with hypernatremic dehydration associated with dilute urine (urine osmolality < plasma osmolality). - Neonates and infants with nephrogenic DI present with dehydration, irritability, failure to thrive, and a preference for water. ABP Content Specifications(s) - Interpret the laboratory findings associated with diabetes insipidus - Recognize the signs and symptoms of diabetes insipidus in patients of various ages

A 10-day-old neonate presents to the emergency department with severe dehydration. His blood test results are as follows: - Sodium, 150 mEq/L (150 mmol/L) - Potassium, 4.9 mEq/L (4.9 mmol/L) - Chloride, 115 mEq/L (115 mmol/L) - Blood urea nitrogen, 44 mg/dL (15.7 mmol/L) - Creatinine, 0.7 mg/dL (61.9 µmol/L) The patient was seen in the emergency department 7 days ago with similar concerns and serum chemistry panels showing elevated serum sodium and creatinine concentrations. He was born at full term to a 30-year-old gravida 1, para 1 mother via normal vaginal delivery. The neonate was appropriate for gestational age with no prenatal or postnatal complications. You suspect an inherited disorder because there is a history of similar episodes in males on the maternal side of the family. Of the following, the finding MOST likely to support your diagnosis is A. glucosuria B. hematuria C. hydronephrosis D. polyuria E. vesicoureteral reflux

C. patients with a longer initial remission have a more favorable prognosis than those who relapse within 24 months of diagnosis Children with acute lymphoblastic leukemia (ALL) who experience remission for a longer period before a relapse have a more favorable prognosis after relapse than those who experience a relapse during or soon after therapy. Multidrug chemotherapy regimens and standardized care through large multicenter cancer cooperative groups have drastically increased the rates of initial remission and long-term survival for children with leukemia over the past 5 decades. Furthermore, with risk-stratified chemotherapy and improved supportive care, the cure rate for newly diagnosed pediatric ALL is approximately 85% in the United States. Despite this progress, the Children's Oncology Group data show no significant improvement in outcomes for children with a relapse of ALL from the 1988-1994 era to the 1995-2002 era. Increased drug resistance of leukemic blasts at the time of relapse compared with blasts at initial diagnosis is thought to contribute to the difficulty in cure of a relapse in ALL. Genomic studies show that in most cases, the cells responsible for relapse were present as a dormant subpopulation at the time of diagnosis and then expand during therapy and eventually lead to relapse. Less frequently, in some cases of ALL relapse, the blast cells that have undergone a relapse have genetic mutations that are different from those of the primary leukemia cells, thus suggesting a secondary malignancy. The first relapse of ALL has several risk stratification schemes that are based on time of relapse, immunophenotype (eg, B-lineage, T-lineage), site of relapse (bone marrow, extra medullary [central nervous system and testes] or combined), and minimal residual disease after induction therapy. The Berlin-Frankfurt-Munster Group (Germany), Children's Oncology Group (United States), and St. Jude Children's Research Hospital (United States) have different definitions of early relapse: less than 18 months from diagnosis, less than 36 months from diagnosis, and less than 6 months from completion of therapy, respectively. Various studies have shown that early relapse is associated with aggressive disease and a poor prognosis, with less than a third of these patients surviving, compared with a 50% survival rate for patients with late relapse. Older age (≥ 10 years of age) is considered a poor prognostic indicator for patients with ALL that is newly diagnosed or in relapse. The bone marrow is the most common site of relapse. Isolated bone marrow relapse has a worse prognosis than a combined or isolated extramedullary relapse. Allogeneic stem cell transplantation (SCT) can be performed with a matched sibling or unrelated matched donor, though the former is still the preferred donor type and may have a higher likelihood of success. Emerging data suggest that in some patients experiencing a second remission with late relapse or isolated extramedullary relapse, continued chemotherapy alone may be just as effective as SCT. PREP Pearls - Various studies have shown that early relapse of acute lymphocytic leukemia (ALL) is associated with aggressive disease and a poor prognosis, with less than a third of these patients surviving, compared with a 50% survival rate for patients with late relapse. - Older age (= 10 years of age) is considered a poor prognostic indicator for patients with ALL that is newly diagnosed or in relapse. - The bone marrow is the most common site of relapse in pediatric ALL. Isolated bone marrow relapse has a worse prognosis than a combined or isolated extramedullary relapse. Sites of extramedullary relapse are the central nervous system and testicles. -Allogeneic stem cell transplantation can be performed with a matched sibling donor or unrelated matched donor, though the former is still the preferred donor type. ABP Content Specifications(s) - Recognize clinical findings associated with leukemia, including sites of relapse

A 10-year-old boy is brought to your office for pallor and easy bruising over the past month. He was diagnosed with acute lymphoblastic leukemia 3 years ago and has been in remission after receiving standard-dose chemotherapy. The mother is very concerned that his leukemia has recurred and asks you about this. Of the following, the MOST appropriate statement would be that A. allogeneic stem cell transplantation for relapsed acute lymphoblastic leukemia requires a human leukocyte antigen-matched sibling donor B. patients who are older in age at the time of relapse have a more favorable prognosis C. patients with a longer initial remission have a more favorable prognosis than those who relapse within 24 months of diagnosis D. patients with relapsed acute lymphoblastic leukemia achieve second remission less than 20% of the time E. relapsed acute lymphoblastic leukemia occurs more frequently in extramedullary sites, such as the central nervous system and testicles, than in the bone marrow

A. cerebral angiogram The boy in the vignette has posterior headaches that worsen with Valsalva maneuver, which suggests a symptomatic Chiari I malformation. Acquired hypotonia is rare and requires prompt evaluation. In infants, botulism should be considered. In addition to new onset poor head control and hypotonia, botulism can present with bilateral ptosis that can be mistaken for encephalopathy. In myasthenia gravis, there is usually double vision, dysphagia or choking, and a waxing and waning course, in addition to mild limb weakness or hypotonia. His bilateral upper extremity weakness, hypotonia, and areflexia suggest a cervical spinal cord problem. Lumbar puncture is the diagnostic test for idiopathic intracranial hypertension, and he does have headaches that worsen with a Valsalva maneuver, but he does not have other signs of increased intracranial pressure such as papillitis or cranial nerve VI palsy. PREP Pearls - Occipital headache that worsens with Valsalva maneuver, with a normal fundus examination, suggests a symptomatic Chiari I malformation. - Bilateral arm weakness or hypotonia with preserved leg strength and tone suggests a lesion in the cervical spine. ABP Content Specifications(s) - Plan the appropriate evaluation of hypotonia in patients of various ages

A 10-year-old boy presents to his pediatrician with a complaint of frequent headaches over the past 2 months. The headaches are occipital, dull, and pounding, lasting 1 to 2 hours, and he sometimes feels dizzy. He has not had nausea, vomiting, or visual changes. The headaches worsen when he sneezes or laughs. Ibuprofen and acetaminophen have not helped. His physical examination findings are unremarkable. He has full range of motion of his neck and no tenderness to palpation of the spinous processes. On neurologic examination, his fundi are normal, there is no blurring of the optic disc margins, and his extraocular movements are full and conjugate. Motor examination shows hypotonia of his arms with mildly decreased grip strength and areflexia bilaterally. His lower extremities have normal tone and strength. Of the following, the test MOST likely to yield the correct diagnosis is A. cerebral angiogram B. electroencephalogram C. electromyogram and nerve conduction study D. lumbar puncture E. magnetic resonance imaging of the neck

B. have the boy use padded heel cups and allow return to activity as tolerated The child in the vignette likely has Sever disease, an irritation of the calcaneal apophysis. The use of padded heel cups in the shoes appears to help the symptoms of Sever disease by decreasing impact to the heel. Ice, over-the-counter analgesics, and stretching exercises may also help alleviate symptoms. Children with Sever disease tend to have a waxing and waning course while the calcaneal apophysis is active, generally for 1 to 2 years. Sever disease does not appear to lead to bone or joint damage, therefore these children should be allowed to participate in physical activities provided they do not exhibit severe pain, limp, or alterations in gait or physical activity mechanics that may increase susceptibility to other injuries. Physes are growth areas of the long bones that contribute to bone lengthening. Apophyses are growth centers that are present at locations where tendons attach to bone. The physes and apophyses are cartilaginous, therefore these areas are particularly vulnerable to overuse injuries as a result of traction and direct pressure. Osgood-Schlatter disease, an apophysitis involving the tibial tubercle, and little league shoulder, a separation at the proximal humerus growth plate, are other common examples of overuse injuries involving the growth centers that can occur in skeletally immature children and teens. Adults, on the other hand, are more likely to experience overuse injuries involving the tendons and ligaments, as these structures are the weakest points of the adult skeleton. The Achilles tendon attaches to the superior aspect of the calcaneal apophysis, therefore contraction of the calf muscles applies tension there. In addition, direct force applied to the plantar surface of the calcaneus occurs with high-impact activities and can irritate the apophysis. Children with Sever disease typically report activity-related pain over the medial, lateral, or posterior aspect of the heel. Affected individuals are generally between 7 and 14 years of age; girls with Sever disease tend to be younger than boys with the condition. Some children exhibit intermittent antalgic gait. A history of swelling, mechanical symptoms involving the ankle (eg, catching or locking), or constitutional symptoms is not consistent with Sever disease and should prompt investigation into other causes of heel pain. The hallmark physical examination sign of Sever disease is pain when the examiner applies compressive force to the medial and lateral sides of the heel. Flexible flatfoot and decreased calf muscle flexibility are common associated findings. Radiographs may show sclerosis and fragmentation of the calcaneal epiphysis; however, Sever disease is a clinical diagnosis and radiologic studies are not needed in the absence of signs or symptoms suggesting another diagnosis. This patient has pain after activity and does not have any limp. He should be allowed to participate in sports and recreation activities as tolerated. Immobilization may be indicated for the rare patient with persistent pain, despite rest from activities and symptomatic treatment; however, immobilization can lead to stiffness and weakness and should not be the first step in management. Surgical management is not indicated for Sever disease. This child does not have joint swelling, morning stiffness, pain in other joints, or constitutional symptoms to suggest a rheumatologic cause of his pain, therefore he does not require referral to rheumatology. PREP Pearls - Sever disease (calcaneal apophysitis) is a common cause of heel pain in young athletes between 7 and 14 years of age. - Children with Sever disease can participate in physical activities as long as they are not experiencing gait alterations or severe pain. - The cartilage growth centers of the immature skeleton are particularly vulnerable to overuse injuries. ABP Content Specifications(s) - Plan the appropriate management of an athlete with an overuse injury - Identify the common overuse injuries in athletes

A 10-year-old boy presents to your clinic for evaluation of right heel pain. He reports pain on both sides of the heel that generally occurs after basketball practice. He denies swelling, redness, bruising, or limping. On physical examination, the boy reports pain when you squeeze the right heel; the remainder of the examination is unremarkable. Of the following, the BEST next step is to A. have the boy rest from sports for 1 month, then return to activity B. have the boy use padded heel cups and allow return to activity as tolerated C. have the boy use a walker boot for 4 to 6 weeks before returning to activity D. refer to orthopedics for possible surgical management E. refer to rheumatology for evaluation of possible juvenile idiopathic arthritis

E. referral to otolaryngology Although excision of the tonsils and adenoids of children and adolescents has been a controversial subject for many years, there are some absolute indications for surgery. The girl in the vignette is an example of one of those indications. Significant asymmetry without signs of acute infection of the tonsils on examination should raise suspicion for malignancy, especially if the asymmetry develops rapidly. An immediate referral to otolaryngology is needed for further evaluation. Other absolute indications for surgical excision include extreme obstruction of the airway, interference with swallowing, and uncontrollable hemorrhage from tonsillar blood vessels. Other conditions that lead to the consideration of tonsillectomy or adenoidectomy include recurrent acute or chronic tonsillitis or sinusitis, recurrent acute otitis media or chronic otitis media with effusion, alteration of voice quality because of adenotonsillar hypertrophy, refractory halitosis, peritonsillar abscess, or syndrome of periodic fever, aphthous stomatitis, pharyngitis, and cervical adenitis (PFAPA syndrome). The decision to perform elective tonsillectomy or adenoidectomy should be individualized according to potential risks and benefits for each patient. The use of daily nasal corticosteroids is considered first-line therapy for several mucosal etiologies of nasal obstruction (eg, rhinitis, nasal polyposis) and may be effective in cases of adenoidal hypertrophy. Those who respond inadequately to pharmacologic therapy should be referred to otolaryngology for evaluation and possible surgical management. Oral antihistamines can also be helpful in patients with persistent allergic rhinitis as the cause of their nasal obstruction. Adenotonsillar hypertrophy is most pronounced between 3 and 8 years of age and is usually associated with acute or chronic infections. These structures tend to involute in most children after about 10 years of age because the immunologic activity of this lymphoid tissue decreases after puberty. Therefore, snoring and sleep disordered breathing should resolve with time, rather than worsen as in this patient. Plus, one should not simply reassure the family, given the asymmetry of the tonsillar tissue in the girl in this vignette. Adenotonsillectomy is suggested for healthy children who have adenotonsillar hypertrophy and obstructive sleep apnea. It is not recommended for patients who have hypertrophy without symptoms. Although uncommon, those with chronic airway obstruction from enlarged tonsils and adenoids may develop cor pulmonale or adenoidal facies (elongation and flattening of the midface and retrognathic mandible), so evaluation is important. Tonsil and adenoid size is not predictive of the presence or severity of obstructive sleep apnea, so polysomnography should be performed. Polysomnography is considered the gold standard for assessing suspected obstructive sleep apnea, and referral for this examination before otolaryngology consultation would be appropriate if the concern for malignancy (asymmetric tonsils) was not present. PREP Pearls - Significant asymmetry of the tonsils without signs of acute infection should raise suspicion for malignancy and warrants prompt referral to otolaryngology. - Adenotonsillar hypertrophy is most pronounced between 3 and 8 years of age and is usually associated with acute or chronic infections. - Involution of tonsillar and adenoidal lymphoid tissue occurs in most children around the time of puberty. - Polysomnography is the gold standard for assessing suspected obstructive sleep apnea. ABP Content Specifications(s) - Recognize conditions associated with tonsillar and/or adenoidal hypertrophy

A 10-year-old girl presents for her routine health maintenance visit. Upon further questioning, the mother admits that the patient always snores at night and that the snoring has gotten worse this last year. She has been treated twice for throat infections this year and the mother has been told her daughter's tonsils are large. They have tried allergy medication episodically without much improvement. On physical examination, you notice darkening under the eyes, pale and large nasal turbinates, plus adenoidal facies. The tonsils are hypertrophied and the left tonsil is significantly larger than the right. Of the following, the BEST next step in management is A. prescribe a nasal steroid for daily use B. prescribe an oral antihistamine for daily use C. reassure that the tonsils will get smaller with age D. refer for polysomnography E. referral to otolaryngology

C. serum cortisol The child in the vignette has Addison disease, the most common cause of adrenal insufficiency in the industrialized world. Classic chronic symptoms include weakness, fatigue, anorexia, nausea, abdominal pain, and diarrhea. Poor growth can also occur, as seen in this child. Hyperpigmentation of the skin and mucous membranes can develop over time. Acute symptoms can include muscle, joint and abdominal pain, and hypotension. Laboratory evaluation may reveal hyponatremia, hyperkalemia, and hypoglycemia if the child becomes ill and develops an adrenal crisis. Sometimes the complete blood cell count (CBC) will reveal anemia and eosinophilia as well. Serum cortisol will be very low, revealing the diagnosis of adrenal insufficiency. Other supportive testing includes elevated adrenocorticotropic hormone (ACTH) levels (frequently > 100 pg/mL [22 pmol/L]), and ACTH stimulation testing can reveal a low cortisol response (< 18 μg/dL [497 nmol/L]). Mineralocorticoid deficiency is confirmed with a relatively low aldosterone value in the face of hyperreninemia. Adrenal antibodies can also be measured to confirm the diagnosis of Addison disease as the underlying cause of adrenal insufficiency. Other causes of poor growth include growth hormone deficiency, hypothyroidism, and Turner syndrome. Although insulin-like growth factor 1, thyrotropin, and karyotype testing are reasonable in the evaluation of a child with poor growth, the additional symptoms shown by the child in this vignette, along with her hyperpigmentation and laboratory findings, make these diagnoses much less likely. Similarly, tissue transglutaminase testing for celiac disease is warranted in the routine evaluation of a child with poor growth, especially with gastrointestinal symptoms. However, for girl in the vignette, her severe symptoms and classic laboratory findings fit much better with adrenal insufficiency as the most likely diagnosis. PREP Pearls - Most children who have Addison disease experience a constellation of ill-defined symptoms to include fatigue, generalized muscular weakness, loss of appetite, and poor weight gain. A high index of suspicion is needed to diagnose Addison disease. - A low serum cortisol measurement at the time of acute illness will confirm the diagnosis of adrenal insufficiency. ABP Content Specifications(s) - Recognize the clinical features associated with Addison disease - Recognize the clinical and laboratory manifestations of adrenal insufficiency

A 10-year-old girl presents for poor linear growth and slow weight gain. In the last year, her linear growth has stopped completely, and she continues to struggle to gain weight. Review of systems reveals frequent fatigue, anorexia, and a decreased energy level. She has also had occasional episodes of emesis in the morning that are becoming more frequent. Physical examination shows some darkening of her skin and mucous membranes. The remainder of her examination is unremarkable. Initial laboratory testing shows a serum sodium concentration of 131 mEq/L (131 mmol/L), potassium concentration of 5.8 mEq/L (5.8 mmol/L), and a glucose level of 60 mg/dL (3.3 mmol/L). The rest of her chemistry results are unremarkable. Complete blood cell count shows eosinophilia. Of the following, the BEST next test to determine her underlying diagnosis is A. insulin-like growth factor 1 B. karyotype C. serum cortisol D. thyrotropin E. tissue transglutaminase antibody

C. 25-hydroxyvitamin D level The child in this vignette has vitamin D deficiency rickets. Risk factors for vitamin D deficiency and rickets in an infant include breastfeeding without vitamin D supplementation, dark skin pigmentation, and maternal vitamin D deficiency. The American Academy of Pediatrics recommends 400 IU/day of vitamin D to prevent vitamin D deficiency in infants who are exclusively breastfed or consuming less than 1 L formula per day. Measurement of 1,25-dihydroxyvitamin D (active form) is not useful for monitoring the vitamin D status of patients with vitamin D deficiency (low half life, 1000x lower circulating levels than just 25-HO.) Instead, measurement of 25-hydroxyvitamin D is the appropriate test. Alkaline phosphatase: will be elevated in patients with rickets secondary to vitamin D deficiency. However, alkaline phosphatase is often elevated in disorders of high bone turnover and liver disease. Alkaline phosphatase can also be elevated in benign transient hyperphosphatasemia of infancy and childhood, a temporary condition where alkaline phosphatase is often increased secondary to gastrointestinal infection or failure-to-thrive. Parathyroid hormone: is also increased in vitamin D deficiency, but PTH elevation can also be seen in other forms of rickets and in hyperparathyroidism. PREP Pearls - Measurement of 25-hydroxyvitamin D (the storage form of vitamin D) is the definitive test for diagnosing vitamin D deficiency. - The child who is being exclusively breastfed or consuming less than 1 L formula per day should be prescribed 400 IU/day of vitamin D. ABP Content Specifications(s) - Recognize the presenting signs and symptoms of vitamin D-deficient rickets, and manage appropriately - Recognize the laboratory and radiologic features of vitamin D-deficient rickets

A 12-month-old boy was diagnosed with failure to thrive. The child was born at full term and was healthy and active for the first 6 months after birth. However, he has had poor weight gain for the last 6 months, when his mother noted his appetite had decreased and his energy level had progressively worsened. His diet consisted of breast milk exclusively for the first 6 months. He started solids at 6 months of age but continued to breastfeed. Physical examination shows swollen wrists and ankles and bowing of the legs. The remainder of the examination is unremarkable. Of the following, the single BEST test to confirm this patient's diagnosis is A. alkaline phosphatase level B. 1,25-dihydroxyvitamin D level C. 25-hydroxyvitamin D level D. parathyroid hormone level E. 24,25 vitamin D level

D. reassure and treat symptomatically This case shows that not all ear pain stems from the ear itself as the girl in the vignette suffers from ear pain related to her preauricular lymphadenopathy. Ear pain (otalgia) is one of the most common complaints of childhood. The differential diagnosis of ear pain is categorized by the anatomic location from which the pain originates. Primary otalgia is caused by an abnormality of the ear itself: inflammation, infection, or trauma to the auricle, auditory canal, tympanic membrane, middle ear, or inner ear. When the ear appears normal on physical examination, as in the vignette, secondary otalgia should be considered. Secondary pain is referred from another location and the causes include temporomandibular joint (TMJ) dysfunction, parotitis, pre- or postauricular lymphadenopathy, pharyngitis, tonsillitis, dental infection, maxillary sinusitis, facial nerve palsy, or cervical spine injury. A thorough examination would reveal these potential causes of secondary otalgia. Treatment of the underlying cause will lead to resolution of the ear pain. It is not unusual for benign reactive preauricular lymphadenopathy to occur with viral conjunctivitis, especially that caused by adenovirus, which often presents with conjunctivitis, fever, pharyngitis, headache, upper respiratory symptoms, and sometimes, gastrointestinal complaints. This is such a common constellation of symptoms that one should feel comfortable treating symptomatically and giving reassurance. Further workup, including obtaining a complete blood cell count or Epstein-Barr virus titers, is not warranted. Occasionally, adenoviral infections can cause otitis media or pneumonia, but antibiotics should not be prescribed when viral disease is suspected. Enlarged lymph nodes caused by an acute bacterial infection would be more tender, feel warm to touch, have overlying erythema, and may be fluctuant on examination. The preauricular lymphadenopathy in this case is not of unusual size, location, consistency, or duration, so biopsy is not indicated. PREP Pearls - Primary otalgia is caused by an abnormality of the ear itself: inflammation, infection, or trauma to the auricle, external auditory canal, tympanic membrane, middle ear, or inner ear. - When the ear examination findings are normal, consider secondary causes of otalgia or referred pain: temporomandibular joint dysfunction, parotitis, pre- or postauricular lymphadenopathy, pharyngitis, tonsillitis, dental infection, maxillary sinusitis, facial nerve palsy, or cervical spine injury. ABP Content Specifications(s) - Understand the possible etiologies of ear pain, including referred pain from other anatomic sites

A 12-year-old girl comes to your clinic for evaluation of bilateral pink eyes, low grade fever, and ear pain for 2 days. She reports that she had a lot of crusting in the eyelashes when she awoke this morning. On physical examination, she has minimal clear to mucoid eye drainage and conjunctival injection, but no photophobia or visual changes. The patient has diffuse pharyngeal erythema, but no exudate or tonsillar enlargement. Her tympanic membranes are gray with normal landmarks. You notice some tenderness to movement of the tragus and find pea-sized tender nodules in the preauricular area bilaterally. The patient is very concerned about these bumps that she had not previously noticed. Of the following, the BEST recommendation at this time is to A. obtain a complete blood cell count B. obtain Epstein-Barr virus titers C. prescribe amoxicillin orally D. reassure and treat symptomatically E. refer to surgery for biopsy

C. referral to an orthopedic specialist The clinical history and physical examination for the girl in the vignette suggest that she has sustained a meniscal tear. She has mechanical symptoms, clicking and locking, and a very limited range of motion in the knee. The next appropriate steps in management would be to recommend the patient be non-weight-bearing, with the use of crutches, and referral to an orthopaedic specialist for additional evaluation and treatment. Individuals with mechanical symptoms after a knee injury often require surgical intervention. The menisci are kidney bean-shaped, shock-absorbing cartilage cushions in the knee. Meniscal tears are uncommon in children, with rates of tear increasing during midadolescence. Tears tend to occur with a twisting or pivoting motion, particularly if the foot is planted. In teenagers, meniscal tears are often associated with ligament injuries. Following a meniscal tear, patients typically recall acute onset of pain and may report feeling a "pop" at the time of injury. Following the injury, they often experience pain with extreme flexion of the affected knee. On physical examination, clinicians may detect effusion and joint line tenderness. The examiner should perform the McMurray test, holding the patient's knee in flexion, then rotating the tibia and extending the knee; pain or a palpable click may indicate the presence of a meniscal tear. Radiographs should be considered to look for bony injuries, particularly in a skeletally immature patient. Magnetic resonance imaging is the preferred test to confirm the diagnosis of meniscal injury, evaluate morphology of any tears, and to assess for other injuries (eg, osteochondral fractures). Return to physical activity would be inappropriate for this adolescent as she has mechanical symptoms and limited range of motion. Physical therapy is unlikely to improve range of motion because the patient likely has a meniscal tear that obstructs movement of the knee. Because this girl has mechanical symptoms, she should be made non-weight-bearing pending additional evaluation. The use of a knee immobilizer will make use of crutches more difficult. A patellar stabilizing brace would be indicated for a patient with patellofemoral symptoms, but would not be of benefit for this patient. PREP Pearls - Following an acute knee injury, mechanical symptoms, such as catching or locking, suggest intra-articular pathology and a likely need for surgical intervention. - Meniscal tears tend to occur with a twisting or pivoting motion, particularly if the foot is planted. ABP Content Specifications(s) - Understand when orthopedic consultation is required for a sports-related knee injury

A 15-year-old adolescent girl presents to your office for evaluation of a right knee injury that occurred 4 days ago. She was playing lacrosse, and twisted her body with her foot planted. She experienced immediate onset of pain and continues to have pain with weight bearing. She reports a sensation of "something catching" inside her knee. On physical examination, there is a moderate effusion of the right knee. She is unable to flex the knee beyond 90°, nor straighten the knee to within 20° of full extension. She has tenderness over the medial joint line. Radiographs of the knee are unremarkable. Of the following, the MOST appropriate next step in management is A. gradual return to activities as tolerated B. physical therapy to improve range of motion C. referral to an orthopedic specialist D. use of a knee immobilizer for 4 weeks E. use of a patella-stabilizing brace for 6 weeks

C. febrile proteinuria Benign causes of proteinuria include orthostatic proteinuria and conditions associated with transient proteinuria such as fever, stress, exercise, cold, and abdominal surgery. Urine should be rechecked after the underlying condition resolves. Orthostatic proteinuria (OP) occurs when the patient is active and disappears when the patient is supine or asleep for at least 2 hours. To confirm OP, a first morning urine sample is needed. It is important that the patient collect the urine sample immediately upon waking, as even a small amount of activity can lead to proteinuria. PREP Pearls - Benign causes of proteinuria include orthostatic proteinuria and conditions associated with transient proteinuria such as fever, stress, exercise, cold, and abdominal surgery. - When transient proteinuria is suspected, a urinalysis should be performed after the inciting condition resolves. Persistent asymptomatic proteinuria may be associated with underlying chronic kidney disease. ABP Content Specifications(s) - Identify the possible causes of proteinuria

A 12-year-old girl comes to your office with the chief concern of fever for 1 day. She has had mild abdominal pain since this morning and 1 episode of nonbilious vomiting 4 hours ago. She denies any cough, nasal congestion, or painful urination. Her evaluation reveals a temperature of 38.9°C, heart rate of 86 beats/min, respiratory rate of 16 breaths/min, blood pressure of 98/50 mm Hg, and normal growth parameters. Her physical examination is only significant for mild discomfort on abdominal palpation in the periumbilical region, with no organomegaly, flank tenderness, guarding, or rebound tenderness. Her urinalysis demonstrates a specific gravity of 1.025, pH of 6.0, 2+ protein, and no blood, leukocyte esterase, or nitrites. Of the following, the MOST likely cause of proteinuria in this patient is A. acute glomerulonephritis B. chronic renal failure C. febrile proteinuria D. orthostatic proteinuria E. urinary tract infection

A. fine-needle aspiration thyroid biopsy The child in the vignette has a history of radiation exposure and a thyroid nodule. The best test to determine if the nodule is benign or malignant is fine-needle aspiration (FNA) biopsy. Sometimes FNA results are inconclusive, and surgical removal of part or all of the thyroid becomes necessary. Fine-needle aspiration is most often performed under ultrasonographic guidance. All patients with a thyroid nodule require ultrasonography to assess for features of thyroid malignancy to determine if there are nonpalpable lesions that may also warrant FNA and to assess the lymph nodes in the central and lateral neck. Given this patient's history of radiation exposure to the neck and the presence of a palpable nodule, FNA is still needed and is the only test offered that can give a definitive diagnosis. A 123iodine uptake scan can be used to determine if the nodule is hot (hyperactive) or cold (inactive). In this case, 123iodine uptake should only be performed if the child's thyrotropin (TSH) level is at the bottom of the normal range or actually suppressed. Given the patient had no signs of hyperthyroidism, this is unlikely. Radiation exposure can also lead to autoimmune disease (Hashimoto disease), and given the child's thyroid enlargement, weight gain, and fatigue, checking thyroid peroxidase antibodies to look for autoimmune disease is reasonable. In addition, measurement of thyrotropin levels is recommended. It is important to verify that thyrotropin levels are not suppressed. Thyroid nodules and thyroid cancer are also more common in patients with high-normal or elevated thyrotropin, especially in those with a history of prior radiation exposure. However, an abnormal thyrotropin level would not diagnose the underlying cause of this patient's thyroid nodule. For that diagnosis, FNA is needed. For low-risk patients (those with no history of radiation therapy to the neck and no family history of thyroid cancer) in whom a thyroid nodule is found incidentally, ultrasonography should be done first and FNA would be warranted for nodules greater than or equal to 1 cm in size or for any concerning characteristics seen in the nodule on imaging studies. PREP Pearls - A history of radiation exposure increases the risk of thyroid malignancy later in life. - The best test to assess for malignancy in the evaluation of a discrete thyroid nodule in a high-risk patient is fine-needle aspiration. ABP Content Specifications(s) - Plan the appropriate evaluation and management of a thyroid mass/nodule - Recognize the clinical features associated with a thyroid cyst/tumor

A 13-year-old adolescent girl who was treated successfully for leukemia with chemotherapy and cranial irradiation at 2 years of age now presents for evaluation of a "lump" on the side of her neck. She also reports an increase in fatigue over the last few months, as well as a 3.2-kg weight gain. She denies any skin changes, temperature intolerance, or changes in school performance. On physical examination, you note a firm, hard, fixed mass over the left thyroid. Her thyroid gland also appears to be enlarged. Of the following, the BEST test to determine the underlying cause of this girl's abnormal physical finding is A. fine-needle aspiration thyroid biopsy B. 123iodine uptake scan C. thyroid peroxidase antibody D. thyroid ultrasonography E. thyrotropin

B. placement of an intraosseous needle The young boy in the vignette presents with clinical findings consistent with the diagnosis of septic shock. In addition to support of his airway and breathing, he requires emergent fluid resuscitation. Multiple attempts to establish peripheral intravenous access have been unsuccessful, therefore establishing intraosseous access is the best next step in managing his condition. Rapid volume resuscitation is imperative to restoring adequate organ perfusion in children presenting with various forms of circulatory collapse, including septic shock. Obtaining rapid vascular access with at least 2 large bore peripheral intravenous catheters is the ideal first-line measure to restore circulation in such patients. This access, however, can be challenging because most children with shock have cool, poorly perfused extremities. If sufficient intravenous access cannot be obtained quickly, the American Heart Association's Pediatric Advanced Life Saving (PALS) guidelines recommend placing an intraosseous needle as the best next step in management for children and infants past the neonatal period. Intraosseous needle placement is an acceptable alternative as a first attempt at vascular access in children with full cardiopulmonary arrest or severe shock. Intraosseous needle placement may also be necessary in critical situations when venous assess cannot be quickly achieved (such as for children with shock, status epilepticus, extensive burns, and multiple trauma). When placement of an intraosseous needle is necessary for resuscitating a critically ill child, an 18- or 20-gauge manual intraosseous infusion needle, commercial battery-powered driver device (EZ-IO) or bone injection gun (ie, BIG or FAST1) can be used. The site most often frequently used to place an intraosseous needle in children is the flat part of the proximal tibia, just below the growth plate (1-3 cm below and medial to the tibial tuberosity). Other potential placement sites include the distal femur, distal tibia, proximal humerus, and the anterior-superior iliac spine. The site selected should be prepared using an aseptic technique before needle placement; the clinical provider should use universal precautions throughout the procedure. In conscious children, the skin, subcutaneous tissue, and periosteum at the site of needle insertion should be anesthetized with 1% lidocaine solution. When establishing intraosseous access in the proximal tibia, the provider should position the child's leg with slight external rotation and stabilize it on a firm surface. If using a manual intraosseous needle, the needle with stylet in place should be inserted through the skin perpendicular to the tibia, and slightly angled away from the growth plate once the correct insertion site is identified. The needle should be advanced through the subcutaneous tissue and then through the periosteum, using a twisting motion with gentle but firm pressure. The provider should continue inserting the needle through the cortical bone until a sudden decrease in resistance is felt as the needle enters the marrow space. When placed correctly, needles should stand easily without support. Once an intraosseous needle is placed, the stylet should be removed and a syringe can be attached. Aspiration of bone marrow or blood into the needle hub confirms correct placement, though these fluids may not always be aspirated. A small volume of saline should infuse easily when flushed through the needle without infiltration. Once proper placement is confirmed, the intraosseous needle should be stabilized and secured, and the site should be checked frequently for signs of swelling and displacement. Technique for intraosseous needle placement using a commercial driver or injection device will not be reviewed here. Any resuscitation fluid or intravenous drug can be safely administered by the intraosseous route, including crystalloid and colloid fluid solutions, epinephrine, antiepileptics, antibiotics, glucose, and blood products. Intraosseous fluid and medication dosing is equivalent to that used for intravenous administration. Samples for basic laboratory studies such as hemoglobin, serum glucose, electrolytes, and bacterial cultures may also be obtained from an intraosseous line. Although there are few absolute contraindications to intraosseous needle placement, intraosseous needles should not be placed in fractured or previously cannulated bones or in extremities with a vascular interruption. Intraosseous needle placement should also be avoided in children with underlying bone diseases such as osteogenesis imperfecta and osteopetrosis, in those with burns or infections near the access site, and in children with right-to-left congenital cardiac shunts. Obtaining surgical consultation for assistance with establishing reliable vascular access in this child with decompensated shock would not be unreasonable, but immediate fluid resuscitation is required to prevent further clinical deterioration. An intraosseous needle should be placed right away and goal-directed fluid resuscitation should be initiated before surgical consultation. Eventual placement of a femoral venous catheter may also be needed in this critically ill child, but intraosseous needle placement is the preferred next step for rapid intravascular access. Central venous catheter placement requires considerably more time than intraosseous needle placement, even when performed by individuals experienced in central line placement. Administering fluids enterally by a nasogastric tube would not be an adequate method for restoring intravascular volume in any child with decompensated (or even compensated) shock. Although ultrasonography is a useful tool that may aid in the placement of peripheral as well as central intravenous lines, additional attempts to place a peripheral intravenous catheter are unlikely to be successful and would not be the best next step for this critically ill, poorly perfused boy. Intraosseous needle placement is much more likely to be successful than ultrasound-guided intravenous cannulation in this scenario. PREP Pearls - If adequate intravenous access cannot be obtained quickly, placement of an intraosseous needle is the best next step in management for critically ill children and infants past the neonatal period. - Any resuscitation fluid or intravenous drug can be safely administered by the intraosseous route, at doses equivalent to those used for intravenous administration. - Intraosseous needles should not be placed in fractured or previously cannulated bones or in extremities with a vascular interruption. ABP Content Specifications(s) - Plan the appropriate use of intraosseous therapy

A 14-month-old boy is transported to the emergency department by emergency medical services because of high fever and increasing lethargy. His mother reports that he was at his baseline state of good health until he began having fever 2 days ago. This morning, his mother noted that he seemed much more lethargic and looked pale and that his hands and feet seemed "blue and cold." On physical examination, the boy is toxic appearing and responsive only to painful stimuli. He has thready peripheral pulses and cool extremities. His temperature is 40.2°C, heart rate is 190 beats/min, respiratory rate is 60 breaths/min, and blood pressure is 40/18 mm Hg. His airway is intact, and his oxygen saturation by pulse oximetry is 94% on room air. Multiple attempts by paramedics to establish peripheral intravenous access were unsuccessful, and 3 attempts by an experienced emergency department nurse have also been unsuccessful. You order administration of 100% oxygen by a non-rebreather face mask for the boy. Of the following, the BEST next step in the management of this patient is A. emergent surgical consultation for venous cutdown B. placement of an intraosseous needle C. placement of a femoral venous catheter D. placement of a nasogastric tube for enteral hydration E. reattempt peripheral intravenous catheter placement using ultrasonographic guidance

A. ask him about stressors in his life PREP Pearls - Clinical findings associated with stress include headaches, nausea, chest pain, stomach aches, and fatigue. - For patients presenting with recurrent headaches, nausea, chest pain, stomach aches, and fatigue, organic causes are identified only about 15% of the time. - When evaluating conditions that are commonly stress related, it is best to immediately assess for stress and anxiety as contributors rather than waiting until organic causes are ruled out. ABP Content Specifications(s) - Recognize the clinical findings associated with stress among adolescents

A 14-year-old adolescent boy has been having a problem with recurrent headaches over the past 4 months. These headaches are occasionally associated with periumbilical abdominal pain. He describes the headaches as typically occurring across his forehead or affecting his entire head. The pain feels like a pressure, and is not pounding. Acetaminophen helps, but will often not remove the headache entirely. The headaches happen as often as once a day and sometimes cause him to miss school or not participate in other activities. There are cycles of time when the headaches occur more or less frequently, but not an overall pattern of worsening. The patient has otherwise been healthy, with normal development. His physical examination is within normal limits, with a blood pressure of 115/80 mm Hg and a pulse rate of 88 beats/min. Of the following, the BEST next step would be to A. ask him about stressors in his life B. ask him if he has had a history of recurrent sore throats C. request a complete blood cell count and erythrocyte sedimentation rate D. request computed tomography of the head E. request Lyme disease serology

C. the follow-up outpatient visit High urinary concentrations of calcium, cystine, oxalate, and uric acid due to increased renal excretion or low urine volume is associated with crystal formation, which leads to renal stone formation. Increased urinary excretion of citrate (most important), magnesium, and pyrophosphate is associated with decreased risk for renal stone formation. A low level of these inhibitors is associated with an increased risk for nephrolithiasis in children and adults. The patient described in the vignette has recurrent renal stones. Pain control and hydration are important during the acute event, which is not the appropriate time for 24-hour urine collection. The evaluation for an underlying metabolic abnormality is indicated irrespective of the imaging results in this case. PREP Pearls - Children with renal stones should be evaluated for urinary metabolic disorders by a 24-hour urine collection, with analysis for solutes associated with increased risk of stone formation and decreased excretion of inhibitors of stone formation. - A risk factor for stone formation is identified in nearly 80% of the children with renal stones. - A urinary metabolic abnormality is reported in 40% to 50% of children with recurrent renal stones. - The 24-hour urine collection should be performed once the patient is discharged and is at home, doing normal activities, consuming a regular diet, free of infection, and without intravenous fluid therapy. ABP Content Specifications(s) - Plan the appropriate management of urinary tract stones Recognize factors contributing to the development of urinary tract stones

A 14-year-old adolescent boy presents to the emergency department because he has had flank pain radiating to the groin and blood in his urine for the last 8 hours. There is no history of fever, burning on urination, or trauma. He had a similar episode 1 year ago associated with passage of small brown particles in his urine. On physical examination, you notice the patient is in discomfort and has mild dehydration, a temperature of 38.3°C, heart rate of 100 beats/min, respiratory rate of 28 breaths/min, and blood pressure of 134/80 mm Hg. The patient refuses to let you examine his abdomen. You suspect recurrent urinary tract stones as the underlying cause of the patient's symptoms. Of the following, the MOST appropriate time to begin 24-hour urine collection to evaluate this boy's condition is after A. adequate pain control and hydration B. admission to the hospital C. the follow-up outpatient visit D. imaging studies show no stones in the kidney E. the placement of an indwelling urinary catheter

C. first morning urinalysis, urine protein, and urine creatinine when not menstruating Asymptomatic adolescent female patients, such as the girl in the vignette, should not have a urinalysis performed while menstruating. Urinalysis at the time of menstruation will usually be positive for blood, protein, and, occasionally, leukocytes. Therefore, in such patients, urine should be checked several days after menstruation is completed. According to the Kidney Disease Outcome Quality Initiative guidelines from the National Kidney Foundation, 24-hour urine collection is not routinely indicated to evaluate proteinuria in children. Studies demonstrate that the urine protein-creatinine ratio in a first morning urine sample is as sensitive as 24-hour urine collection for detecting pathologic proteinuria. A urine dipstick will become positive for protein when the urine albumin secretion is more than 300 mg/day. Normal urine albumin secretion is less than 30 mg/day. Testing for high albuminuria (formerly called microalbuminuria) detects urine albumin secretion in the range of 30 to 330 mg/day. High albuminuria is the earliest manifestation of diabetic nephropathy and is also noted to be an early marker in cardiovascular disease-associated renal injury. Microalbuminuria testing should be limited to early detection of renal injury secondary to chronic conditions associated with renal failure. A persistent dipstick-positive proteinuria, or a urine protein-creatinine ratio of greater than 0.2, is considered abnormal. Therefore, a first morning urine specimen when the patient is not menstruating should be tested. A urine protein-creatinine ratio of greater than 0.2 in a first morning sample (necessary to rule out orthostatic proteinuria) is abnormal and indicative of renal pathology that requires evaluation by a pediatric nephrologist. In the absence of risk factors for chronic renal disease in this patient, there is no indication for testing for microalbuminuria. PREP Pearls - Asymptomatic adolescent female patients should not have a urinalysis performed while menstruating. - Twenty-four-hour urine collection is not routinely indicated for evaluating proteinuria. - High albuminuria (microalbuminuria) testing should be limited to early detection of renal injury secondary to chronic conditions. - A urine protein-creatinine ratio of greater than 0.2 in a first morning sample is abnormal and requires evaluation by a pediatric nephrologist. ABP Content Specifications(s) - Plan the appropriate clinical and laboratory evaluation of proteinuria

A 15-year-old adolescent girl returns to your office for evaluation of proteinuria noted on two health supervision visits a year apart. Today, her temperature is 37.9°C, heart rate is 76 beats/min, respiratory rate is 17 breaths/min, and blood pressure is 110/60 mm Hg. She has normal growth parameters and normal physical examination findings. She is currently menstruating. Her urinalysis 1 year ago was only significant for 1+ proteinuria. Her urinalysis today demonstrates a specific gravity of 1.035, a pH of 6.0, 3+ protein, 4+ blood, and no leukocyte esterase or nitrites. Her urine microscopy shows greater than 100 red blood cells per high-power field, less than 5 white blood cells per high-power field, and no crystals or bacteria. Of the following, the MOST appropriate next step in evaluation of this patient is A. first morning urinalysis, urine microalbumin, and urine creatinine the next day B. first morning urinalysis, urine protein, and urine creatinine the next day C. first morning urinalysis, urine protein, and urine creatinine when not menstruating D. 24-hour urine collection for microalbumin when not menstruating E. 24-hour urine collection for protein excretion when not menstruating

B. history of type 2 diabetes mellitus The teenager in the vignette has sustained a plantar puncture wound from a rusty nail. Of the factors listed, his medical history of type 2 diabetes mellitus is most likely to increase his risk of developing a wound infection. Puncture wounds are a common type of minor injury sustained by children, accounting for approximately 3% to 5% of all traumatic injuries presenting to pediatric emergency departments. Although most children sustaining puncture wounds have uncomplicated courses, serious complications can arise from puncture wounds, especially those involving the plantar surface of the foot. All pediatric clinicians should understand the principles underlying appropriate evaluation and management of various puncture wounds. Puncture wounds may arise from various circumstances. Although more than half of all puncture wounds involve the plantar surface of the foot, other affected sites include the legs, arms, hands, and, less commonly, the trunk and head. Most plantar puncture wounds are caused by nails. However, glass, wood, plastic, and other metal objects can be involved as well. Infection is the most common complication that arises from puncture wounds. Wound infection is more likely when puncture wounds are deep, when there is more devitalized tissue, and in cases involving retained foreign bodies. Other factors that have been identified as increasing the risk for puncture wound infection include wounds involving the forefoot, punctures occurring through shoes, and an underlying history of diabetes mellitus. Bacterial agents most commonly implicated in puncture wound infections include Staphylococcus aureus, β-hemolytic streptococci, and anaerobic bacteria. Infections from Pasteurella multocida are often seen in puncture wounds arising from animal bites. Pseudomonas aeruginosa has been commonly isolated in patients sustaining plantar puncture wounds while wearing tennis shoes at the time of injury. Aside from infection, complications that may arise (though less commonly) from puncture wounds include retained foreign bodies, injury to neurovascular structures, and tattooing of skin from débris, which can lead to permanent cosmetic deformity. For all children presenting with puncture wounds, a careful history is essential to guide appropriate management and to identify risk factors for complications. Physical examination should include a complete evaluation of the injured area, including assessment of circulation and motor function distal to the wound. The wound must be inspected for retained foreign material and signs of infection. If there is any suspicion about the possibility of a retained foreign body, diagnostic imaging should be performed. Most glass and metal objects will appear on plain radiography. In addition, tetanus immunization status should be reviewed for all children presenting with puncture wounds; tetanus immunization and tetanus toxoid should be administered when indicated. Uninfected puncture wounds should be irrigated with copious amounts of sterile saline, cleansed with an antiseptic solution, and débrided whenever jagged edges are present. Skin should be cleansed of foreign material to avoid permanent scarring. Foreign bodies should be removed to reduce the risk of wound infection, reduce pain, and avoid subsequent damage to underlying neurovascular structures. Children with simple uninfected plantar puncture wounds can generally be treated with rest, elevation of the affected foot, and intermittent warm water soaks. Families should be educated about the signs and symptoms of wound infection and asked to return if these signs and symptoms occur. There is no evidence to support the routine administration of prophylactic antibiotics to prevent infection after puncture wounds. Antibiotics may be prescribed in cases when a patient's risk for infection is particularly high, at the discretion of the treating clinician, such as in punctures resulting from animal or human bites, for highly contaminated wounds, or in immunocompromised patients who have an elevated risk of infection. Inhaled corticosteroid use has not been associated with increased risk of wound infections. The lack of shoes by the patient in the vignette at the time of the injury does not increase his risk for wound infection. In fact, infection with P aeruginosa is associated with plantar puncture wounds that are sustained while wearing sneakers. Sustaining a plantar puncture wound while walking on a paved surface has not been associated with an increased risk of infection or other complications. Puncture wounds affecting the forefoot, as opposed to the heel, have been found to carry a higher risk of wound infection. PREP Pearls - Infection is the most common complication arising from puncture wounds. - Risk factors for the development of puncture wound infections include wounds that are especially deep, have a large amount of devitalized tissue, involve retained foreign bodies, involve the forefoot, occur through shoes, and affect patients with underlying conditions impairing immunity, including diabetes mellitus. - Children with simple uninfected plantar puncture wounds can generally be treated with rest, elevation of the affected foot, and intermittent warm water soaks. Families should be educated about the signs and symptoms of wound infection and asked to return if the signs and symptoms occur. - There is no evidence to support the routine administration of prophylactic antibiotics to prevent infection after puncture wounds. ABP Content Specifications(s) - Plan the appropriate evaluation of various puncture wounds, including a puncture wound through a sneaker, and manage appropriately

A 14-year-old adolescent boy presents to your office for evaluation 4 hours after he sustained a puncture wound to his left heel. He stepped on a rusty nail while walking without shoes across a parking lot. The nail initially lodged in his left heel, but he pulled it out at that time. His heel bled slightly, but the bleeding resolved after only a few minutes. The patient's mother brought him to your office for evaluation because he has been complaining of constant pain in his left heel since his injury. The teenager has a history of obesity, type 2 diabetes mellitus, and asthma. His medications include an inhaled corticosteroid and metformin. The patient's immunizations are up to date; a review of his chart confirms that he received a tetanus booster 2 years ago. The adolescent appears very well and is in no distress. He is afebrile, and his vital signs are normal for his age. His weight is at the 97th percentile for his age. Examination of the boy's left foot reveals a 4-mm puncture wound on his left heel that is surrounded by some jagged skin tissue. The wound is not bleeding, and there is no surrounding erythema or warmth. There is no drainage from the wound. On palpation of the wound, you do not detect any retained foreign body. While you are irrigating and debriding the puncture wound, the teenager's mother asks you about the likelihood that the wound will become infected. Of the following, the factor MOST likely to increase the patient's risk of developing a wound infection is A. daily inhaled corticosteroid use B. history of type 2 diabetes mellitus C. lack of shoes at the time of the injury D. relative location of the wound on the heel rather than forefoot E. setting of the injury (paved surface)

D. offer an endocrinology referral for a 4-month course of testosterone therapy The boy in the vignette has constitutional delayed puberty (CDP). His family history of CDP, normal prepubertal growth velocity (5 cm/year), and normal screening laboratory test results suggest he does not have an underlying pathological disorder. Although he had been evaluated before and was correctly told he was going to be a "late bloomer" with delayed puberty, this answer does not acknowledge the patient's psychosocial concerns or review the relatively simple, safe, and effective treatments that could be offered to alleviate them. This patient should be referred to a pediatric endocrinologist to review treatment options. It is not necessary to refer boys or girls who clearly are delayed, but are showing true evidence of pubertal progression on physical examination. Many late-maturing boys are concerned about how short and underdeveloped they are relative to their peers. Teasing and low self-esteem are frequently reported. Studies demonstrate that many feel that their growth delay negatively affected their success at school, work, or socially, and 80% of those affected would have preferred to have had treatment to advance their growth and pubertal delay. Therefore, for boys approaching 14 years of age and girls who have reached 13 years of age without showing significant physical changes of puberty, endocrinology referral for evaluation and treatment is appropriate. Typical initial evaluation for delayed puberty includes luteinizing hormone/follicle-stimulating hormone, testosterone, and prolactin. Even when there is no underlying medical condition, therapy is effective, safe, and inexpensive. Eighty percent of boys who are offered treatment accept it, suggesting they are significantly concerned about their pubertal delay. For boys with CDP, monthly intramuscular injections of 100 mg testosterone in oil (eg, testosterone enanthate) for 4 months usually causes a significant increase in linear growth, weight, penile length, and pubic hair development. One month after injections are complete, mean linear growth is 3.8 cm from the start of treatment, a significant increase in height for many of these children. A few months later, testicular size begins to increase, and usually no further treatments are needed. Low-dose oral androgens are also an option (oxandrolone of 2.5 mg/day) and are sometimes used when a less dramatic effect on growth and maturation is desired. For boys with permanent hypogonadism, lower doses of testosterone (50 mg intramuscularly once monthly) are usually started and then gradually increased every 6 months until a full adult replacement dose is reached. Counseling the family that the benefits of treatment do not outweigh the risks does not adequately address the child's impatience to start growing, and for a boy such as the child in this vignette with significant underlying psychosocial concerns, treatment can easily be offered. Several studies have shown that despite rapidly advancing growth with androgen treatment, final height is not affected when treating adolescent boys aged 14 years and older. A brain magnetic resonance image to evaluate short stature is unlikely to show any significant findings, given the child has a normal prepubertal growth rate and likely has CDP. Similarly, treatment with growth hormone would not be the best first choice because this patient is clearly not growth hormone deficient and would most likely benefit significantly from a short course of testosterone therapy to advance his pubertal status. A high-calorie diet will likely have minimal impact on his overall growth and fails to address his main concerns. PREP Pearls - For boys with constitutional delayed puberty (CDP) who are impatient to start growing, a short course of androgen therapy can significantly improve growth and maturation and alleviate psychosocial concerns. - After discussing treatment options, most boys with CDP will opt to begin androgen treatment, suggesting many have unaddressed psychosocial concerns. ABP Content Specifications(s) - Plan the appropriate evaluation of an adolescent boy or girl who has no signs of the onset of puberty - Recognize the clinical features associated with a delay in sexual maturation of various causes - Recognize the psychosocial risks associated with delayed puberty - Identify the causes of delayed puberty

A 14-year-old adolescent boy presents to your office seeking a medication to help him grow and develop muscles. He has been working out for 1 year without much effect. On physical examination, his height is 147.3 cm (4 ft, 10 in [adjusted midparental height, 185.4 cm (6 ft, 1 in)]), he weighs 48.1 kg, and his testes are only 3 mL in volume. The child's growth rate is 5 cm (2 in) per year. Results of laboratory testing are unremarkable, including complete blood cell count, erythrocyte sedimentation rate, electrolytes, prolactin, insulin-like growth factor 1, celiac panel, and thyroid function testing. Luteinizing hormone, follicle-stimulating hormone, and testosterone levels are at prepubertal levels. Bone age shows a delay of 2 years from chronological age. Although he appears healthy, the boy is distraught about his young appearance compared with his peers. The boy has been evaluated for this previously, when he was told that he will be "a late bloomer" and that this is normal. The boy's father was very short as a boy, and was given medication to make him taller. The family has come to your office seeking a second opinion. Of the following, the BEST treatment option for this child is to A. counsel the family that treatment should not begin before 16 years of age B. obtain a brain magnetic resonance imaging study to evaluate for central causes of poor growth C. offer an endocrinology referral for a 1-year to 2-year course of growth hormone therapy D. offer an endocrinology referral for a 4-month course of testosterone therapy E. recommend a high-calorie nutrition supplement to enhance growth

C. clindamycin In this case, antibiotic coverage should include both S aureus and streptococci while awaiting culture and sensitivity results. Of the agents listed, clindamycin provides coverage for S aureus (methicillin-sensitive and methicillin-resistant) and group A Streptococcus. The drug is also concentrated in bone. While doxycycline and trimethoprim/sulfamethoxazole have activity against methicillin-resistant S aureus and methicillin-sensitive S aureus, they are not effective against group A Streptococcus or pneumococcus. PREP Pearls - Hematogenous osteomyelitis typically occurs in the metaphysis of long bones and may follow minor trauma to the infected area. - Staphylococcus aureus is the most common cause of hematogenous osteomyelitis with group A Streptococcus and Streptococcus pneumoniae being other potential etiologies. Empiric antibiotic therapy should be directed against these pathogens, including methicillin-resistant S aureus, unless the incidence of methicillin-resistant S aureus is known to be very low. - Group B Streptococcus and enteric gram-negative rods such as Escherichia coli are possible causes of osteomyelitis in neonates. Kingella kingae is a potential cause of osteomyelitis in children younger than 3 years of age. In children with sickle cell disease and other hemoglobinopathies, Salmonella is an important cause of osteomyelitis. ABP Content Specifications(s) - Recognize the clinical findings associated with osteomyelitis in various anatomic locations - Identify the etiology of osteomyelitis in patients of various ages - Plan the appropriate management of osteomyelitis in patients of various ages

A 14-year-old adolescent boy presents to your office with a 6-day history of pain over his left thigh that has been increasing to the point that on the day of evaluation, he refuses to bear weight on the extremity. The day prior to the onset of symptoms, he was hit with a lacrosse stick in his left thigh. He initially complained of pain, and ice was applied to the area several times over the next 24 hours. Over the next 2 days, he described the area as being stiff, but then began to feel increasing pain in this leg. He felt the area was swollen, but did not notice any redness. He reports no fever, chills, or other symptoms. His vital signs show a temperature of 37.4°C, a heart rate of 96 beats/min, a blood pressure of 110/74 mm Hg, and a respiratory rate of 22 breaths/min. His physical examination findings are unremarkable, except for his refusal to bear weight on his left leg. Additionally, there is swelling above the patella on the left without erythema. He complains of pain with flexion at the knee and hip on the left. A complete blood cell count shows the following: -White blood cell count, 11,500/µL (11.50 × 109/L), with 74% neutrophils, 24% lymphocytes, and 2% eosinophils -Hemoglobin, 12.8 g/dL (128 g/L) -Platelets, 380 × 103/µL (380 × 109/L) -Erythrocyte sedimentation rate, 68 mm/h -C-reactive protein, 52 mg/L (495.2 mmol/L) -A radiograph of the left femur demonstrates periosteal elevation in the distal femur. Magnetic resonance imaging demonstrates a lytic lesion in the distal shaft of the left femur. The orthopedic surgeon performs an aspiration of the lesion that shows abundant neutrophils and is sent for culture. Of the following, the MOST appropriate antibiotic agent to treat this infection initially pending culture results is A. ampicillin B. ceftriaxone C. clindamycin D. doxycycline E. trimethoprim-sulfamethoxazole

D. oral acetaminophen The adolescent girl in the vignette presents with pain and tingling in her left calf, a few hours after being stung by a bark scorpion (Centruroides sculpturatus). Administration of oral acetaminophen is the next best step in her care at this time. Scorpion envenomation is a significant clinical problem, with approximately 5,000 deaths reported worldwide each year from scorpion stings. The American Association of Poison Control Centers received more than 17,000 calls related to scorpion stings in 2009. In the United States, most significant systemic reactions arise from stings of C sculpturatus, also known as Centruroides exilicauda, or the bark scorpion. Bark scorpion envenomations are potentially fatal to young children; thus, pediatric providers must be familiar with their clinical manifestations and management. Scorpions are arthropods that envenomate prey by stinging. They generally avoid humans, and sting only when threatened, as a means of self-defense. Scorpions live in dry, warm, desert regions as well as in tropical regions. In the United States, C sculpturatus is responsible for the most clinically significant envenomations. This species is found in the Southwestern states including Arizona, New Mexico, Texas, southern California, and Nevada, as well as in northern Mexico. C sculpturatus is a yellow-brown scorpion that may range from just over 1 cm to more than 7 cm in length. Members of this species can be differentiated from other scorpion species by several characteristics, including the presence of a small protruding tubercle at the base of their stingers, known as subaculear tooths. Envenomation by C sculpturatus can cause severe toxicity, especially in small children, and has resulted in fatalities. In the 1930s, C sculpturatus stings caused 40 reported deaths, which primarily involved infants and children. Advances in supportive care have dramatically reduced deaths related to scorpion stings in recent decades, with only 1 reported death in the United States since 1970. Scorpion venoms are complex mixtures of bioactive substances. The primary toxic substances in C sculpturatus venom are neurotoxins, which act to cause increased and repetitive firing of neuronal axons. Clinical symptoms of scorpion envenomation can vary widely, from localized pain, to skeletal neuromuscular hyperactivity with or without cranial nerve dysfunction, to multisystem organ failure and even death. Severity of clinical symptoms tends to correlate inversely with age, with infants and young children at the highest risk for severe toxicity. Localized pain and paresthesias at the site of the sting are typically the first clinical symptoms, manifesting almost immediately after envenomation. Affected patients may have a positive "tap test," in which tapping over the site of the sting results in severe pain. Lack of inflammatory changes at the sting site is another characteristic of C sculpturatus envenomation. Dysautonomias (most commonly tachycardia and hypertension), hyperthermia, and rhabdomyolysis from muscular hyperactivity, gastrointestinal disturbances, pancreatitis, and eventual multisystem organ failure may arise from C sculpturatus envenomation. The most common presenting symptoms in younger patients include restlessness, writhing, flailing, and roving eye movements. A clinical grading system to describe the severity of scorpion envenomation was developed by Curry and colleagues. Based on this system, children with grade I envenomation are those experiencing pain or paresthesia at the envenomation site only; those with grade II envenomation have pain or paresthesia both local to and remote from the envenomation site; those with grade III envenomation have either cranial nerve or somatic skeletal neuromuscular dysfunction (but not both); and those with the most severe envenomations (grade IV) display both cranial nerve and somatic skeletal neuromuscular dysfunction. Diagnosis of C sculpturatus envenomation is a clinical one in most cases. Children from endemic areas presenting with typical neurotoxic symptoms should be assumed to have been stung. Older children, such as the adolescent girl in the vignette, may be able to give a clear history of being stung by a scorpion, but younger children (who typically experience more severe symptoms) may not be able to do so. Often, a caregiver may note that a scorpion was seen on or near the child around the time of symptom onset. Differential diagnoses for scorpion envenomation with associated neurotoxicity include seizure, black widow spider envenomation, organophosphate toxicity, sympathomimetic toxicity, strychnine poisoning, or intoxication with phenothiazine. In infants presenting with unexplained crying and restlessness, the differential diagnosis is even broader, but will not be discussed in detail here. The mainstay of therapy for scorpion envenomation is good supportive care, especially support of the airway, breathing, and circulation. All patients presenting after scorpion stings should be monitored for progression to higher-grade symptoms. Children with lower-grade (grades I and II) symptoms can be treated conservatively with application of ice, analgesics, and tetanus prophylaxis as indicated. Most of these children can be cared for at home following observation for progression of symptoms for at least 4 to 5 hours from the time of envenomation. For patients with higher-grade (grades III and IV) symptoms arising from scorpion envenomation, opioid analgesics, as well as sedation with short-acting benzodiazepines, such as midazolam, are recommended. The use of C sculpturatus antivenin is controversial but may have benefit in decreasing the duration of severe symptoms. Centruroides antivenin was only recently approved by the US Food and Drug Administration and is not available in all centers. A medical toxicologist or other physicians with expertise in treating children with higher-grade scorpion envenomation should be consulted when the administration of antivenin is being considered. Centruroides antivenin and intramuscular midazolam would not be indicated for the adolescent girl in the vignette given that she has only low-grade (grade I) symptoms. Intravenous calcium gluconate does not have an established role in the treatment of patients with scorpion envenomation, nor does oral diphenhydramine. PREP Pearls - Clinical signs and symptoms of scorpion envenomation may vary widely, from localized pain and paresthesias near the site of the sting, to skeletal neuromuscular hyperactivity with or without cranial nerve dysfunction, to multisystem organ failure. - Infants and young children are at the highest risk for severe toxicity from scorpion envenomation. - The mainstay of therapy for scorpion envenomation is good supportive care. All children presenting after scorpion stings should be closely monitored for at least 4 to 5 hours for progression of symptoms. - Children with lower-grade symptoms can be treated conservatively with application of ice, analgesics, and tetanus prophylaxis as indicated. ABP Content Specifications(s) - Plan the appropriate management of a scorpion sting

A 14-year-old adolescent girl presents to an Arizona urgent care, where you practice, after being stung by a scorpion. Approximately 3 hours before arrival, the teenager—who is on a camping trip with her Girl Scout troop—felt a sharp pain in her left calf just after she crawled into her sleeping bag to rest for the night. Upon feeling the pain, she got out of the sleeping bag, shook the bag, and saw a scorpion (Centruroides exilicauda) crawl out. Her temperature is 36.8°C, heart rate is 84 beats/min, blood pressure is 90/60 mm Hg, respiratory rate is 18 breaths/min, and pulse oximetry is 99% on room air. A medical history form provided to you by one of the trip chaperones indicates that the adolescent has up-to-date immunizations and no significant past medical history or allergies. The patient appears anxious, but is in no distress. She complains of a sharp pain and occasional tingling in her left calf, but she has no other symptoms. A complete physical examination is significant only for a 2-mm puncture wound on the girl's left calf with no surrounding swelling or erythema. A full neurologic examination, including close assessment of the cranial nerves, reveals no abnormalities. You tap gently on the puncture wound on her left calf, which causes her to cry out sharply with pain. You apply a cool compress to the site of the wound. Of the following, the BEST next step in management for this patient's symptoms is administration of A. Centruroides exilicauda antivenin therapy B. intramuscular midazolam C. intravenous calcium gluconate D. oral acetaminophen E. oral diphenhydramine

A. changing her eating behavior Although the exact prevalence of chronic abdominal pain in children and adolescents is not known, a review of available data suggests that this complaint accounts for up to 4% of office visits to the primary care pediatrician. One study in adolescents reported that 8% of high school students sought evaluation for abdominal pain during a 1-year period. In the vignette, a 15-year-old girl presents with dyspeptic symptoms of upper abdominal discomfort, bloating, and intermittent nausea. She does not manifest any alarming signs or symptoms that would suggest an organic cause for her complaints. Considering her irregular and "unbalanced" dietary habits, a reasonable initial therapeutic approach would be to encourage a change in her eating behavior. Children with functional (ie, nonorganic) abdominal pain report symptoms that include isolated abdominal pain without other physical complaints, dyspepsia, irritable bowel syndrome, or abdominal migraine. In other patients, symptom clusters include clinical features of more than 1 type of functional pain disorder (eg, Patients 12 years of age or older with symptoms for at least 12 weeks (not necessarily consecutive) during the previous 12 months. 1.) Persistent pain or discomfort in the upper abdomen. 2.)No evidence that organic disease is likely explaining symptoms. 3.) Dyspepsia is not exclusively relieved by defecation or associated with the onset of a change in stool frequency or character. For the adolescent in the vignette, with her absence of warning signs, no evidence suggests that additional laboratory studies are of value in her evaluation. The predictive value of blood tests in functional abdominal pain (with or without warning signs) has not been adequately studied. Infection with Helicobacter pylori should be considered for patients with a positive family history or for those who have emigrated from an endemic area. The H pylori antibody titer is an unreliable and nonspecific test to evaluate H pylori-associated gastroduodenal disease, and endoscopy with biopsy remains the gold standard for diagnosis in pediatric patients. In the absence of a history (ie, family or immigration) suggesting H pylori infection or the presence of warning signs, little evidence suggests that a radiographic study or an endoscopy with biopsy has a significant yield of mucosal abnormalities. Empiric acid suppression treatment with a proton pump inhibitor has not been adequately studied in children with a diagnosis of functional dyspepsia and no evidence-based guidelines for treatment have been established. PREP Pearls - Dyspepsia is defined as upper abdominal discomfort with or without symptoms of nausea and bloating. It is usually not associated with a significant underlying acid-peptic disorder. - The Helicobacter pylori antibody titer is not recommended as a screening test for children with dyspepsia. - Blood tests have not been shown to be of value in assessing the child with functional abdominal pain. - Patients with functional abdominal pain may present with complaints isolated to the gastrointestinal tract, or they may describe other pain symptoms. ABP Content Specifications(s) - Recognize the significance of dyspepsia in a child with recurrent abdominal pain

A 15-year-old adolescent girl presents to your office with the complaint of periumbilical abdominal pain. This symptom has persisted for the past year, occurring several times per week at any time of day. The patient denies nocturnal symptoms, weight loss, diarrhea, constipation, and vomiting. Over the past several weeks, she feels "bloated" after meals and often experiences postprandial nausea. During the week, she skips breakfast, has an energy bar for lunch, and eats a large dinner at 8:30 in the evening after returning from swimming practice. She has tried her father's antacids, and these have provided minimal relief. Height and weight are at the 40th percentile for age. She is afebrile with other vital signs within the reference range. Physical examination demonstrates a well-developed, well-nourished adolescent with no abnormal physical findings. Of the following, you are MOST likely to recommend A. changing her eating behavior B. a Helicobacter pylori antibody titer C. referral for an upper gastrointestinal tract endoscopy D. a trial with a proton-pump inhibitor E. an upper gastrointestinal tract series

A. aspiration of the swollen area with compression dressing The boy in the vignette has sustained a large auricular hematoma as a result of trauma to the pinna. Acute drainage of the hematoma and application of a compression dressing will decrease his risk of permanent deformity. Auricular hematomas generally result from blunt trauma or shearing injury to the pinna, most often as a result of participation in rugby or wrestling. Auricular hematoma can lead to the formation of excess fibrocartilage beneath the perichondrium, resulting in a deformity known as "cauliflower ear." Treatment consists of aspirating the hematoma using sterile technique with an 18- or 20-gauge needle, followed by application of a compression dressing to decrease the risk of hematoma reaccumulation. To help prevent auricular hematomas, primary care physicians should counsel young rugby players and wrestlers that use of protective headgear can decrease the risk of ear injury. Observation is not appropriate for this patient, because drainage is needed to minimize the risk of deformity. Referral to otolaryngology for surgical incision and drainage is indicated for recurrent auricular hematoma. Oral antibiotic prophylaxis may be indicated in patients who require surgical drainage, but is generally not needed after needle aspiration. Steroid medication will not alleviate the hematoma. PREP Pearls - Auricular hematomas occur because of direct or shearing forces applied to the pinna of the ear, most often resulting from participation in rugby or wrestling. - Prompt drainage of an auricular hematoma with application of a compressive dressing can decrease the risk of permanent deformity. - Use of protective headgear during wrestling and rugby may decrease the risk of auricular hematoma. ABP Content Specifications(s) - Plan the appropriate management of a sports-related ear injury

A 15-year-old adolescent boy presents to your office for evaluation 1 day after experiencing an injury to the ear during rugby practice. He reports that during a scrum, the pinna of the right ear was pulled forward, causing his ear to "fold over." He now has pain and swelling of the auricle. On physical examination, you note a large nodular swelling over the superior aspect of the auricle. Of the following, the MOST appropriate management of this injury is A. aspiration of the swollen area with compression dressing B. observation only C. surgical removal of the nodule D. topical antibiotics to prevent secondary infection E. topical steroid medication to decrease swelling

A. anterior mediastinal mass The patient in the vignette has unremarkable cardiac examination findings and no hepatomegaly or pulmonary crackles on examination, making the diagnosis of acquired cardiomyopathy with congestive heart failure less likely. Bronchiectasis, characterized by abnormal dilation and distortion of the bronchial tree, resulting in chronic obstructive lung disease, is unlikely in this boy. He had no significant medical history before the onset of his current symptoms, and crackles and rhonchi, which are often heard with bronchiectasis, are absent on lung examination. Although the majority of patients with pericarditis report chest pain, which is typically improved by sitting up and leaning forward, the diagnosis of pericarditis would not explain the facial swelling, plethora, persistent cough, or hoarseness noted on his examination. A pericardial friction rub, which is a physical finding highly specific for the diagnosis of pericarditis, was not noted on examination of the adolescent in the vignette. Obstructive sleep apnea, which is characterized by episodes of partial or complete airway obstruction during sleep, is not consistent with the persistent and progressive symptoms in the previously healthy adolescent described in the vignette. PREP Pearls • Superior vena cava (SVC) syndrome is very rare in children, but most commonly results from underlying mediastinal malignancies. • Onset of generalized facial swelling combined with facial plethora should prompt clinicians to consider the diagnosis of SVC syndrome. • Plain radiography of the chest is effective in identifying the vast majority of mediastinal masses, and should be obtained in any child in whom a mediastinal mass is suspected. ABP Content Specifications(s) • Recognize the clinical findings associated with a chest mass

A 15-year-old adolescent boy presents to your office for evaluation of a persistent cough, hoarseness, and chest discomfort. He was diagnosed 2 weeks ago with atypical pneumonia based on the presence of low-grade fever and scattered wheezes on the patient's lung examination. He completed a course of azithromycin; however, his cough has increased in frequency over the past 2 weeks. He now reports intermittent shortness of breath, especially when he lies down. He slept in a recliner last night because of trouble breathing when he attempted to lie flat on his bed. His mother remarks that his face has seemed more "puffy" than usual over the past few days. He has no significant past medical history. His height and weight are at the 75th percentile for his age. His temperature is 37.9°C, heart rate is 92 beats/min, respiratory rate is 26 breaths/min, blood pressure is 128/74 mm Hg, and pulse oximetry is 98% on room air. On physical examination, the patient appears somewhat anxious, but is in no acute distress. His face appears flushed and slightly swollen. Cardiac examination reveals a regular rhythm without a murmur, rub, or gallop. Lung examination reveals scattered inspiratory and expiratory wheezes in the boy's upper lung fields bilaterally. He has shortness of breath worsened by deep breathing. The adolescent's voice sounds hoarse and he is coughing frequently during the examination. He seems most comfortable when sitting up on the examination table and leaning forward. His abdominal examination is benign and without tenderness, masses, or hepatosplenomegaly. His extremities are well perfused with no peripheral edema. You do note left supraclavicular lymphadenopathy. The remainder of your physical examination yields no additional remarkable findings. As soon as you complete your clinical assessment, his mother asks you to explain what you think is causing his symptoms. Of the following, the MOST likely diagnosis is A. anterior mediastinal mass B. bronchiectasis C. cardiomyopathy with congestive heart failure D. obstructive sleep apnea E. pericarditis

E. without surgery, he is likely to experience recurrent dislocation over the next 12 months The boy in the vignette has suffered an acute traumatic shoulder dislocation. Given his age and gender, he is likely to suffer an additional dislocation in the subsequent year. Adolescents are particularly vulnerable to recurrent dislocations, therefore many orthopedic providers recommend surgical stabilization after an initial traumatic shoulder dislocation. Shoulder dislocations can be acute, resulting from trauma, as is the case for the adolescent in the vignette, or can be a result of multidirectional instability, when lax ligaments allow the humeral head to move off the glenoid. Ligamentous laxity of the shoulder can be associated with generalized hypermobility of the joints. In athletes who engage in sports with repetitive overhead shoulder movements, such as swimming or volleyball, repetitive microtrauma can result in localized ligamentous laxity. Acute traumatic dislocations generally occur when a force is aimed at the arm while the shoulder is held in abduction and external rotation. The boy in the vignette likely had his shoulder in this position when rebounding the basketball. Most young patients have an associated Bankart lesion, a tear of the labrum, which is the soft cartilage lining of the glenoid. Individuals with shoulder dislocations nearly always experience immediate pain and describe a sensation that the shoulder is "out of place." On physical examination, the acromion will appear to be very prominent and the proximal humeral head will not be palpable in its normal position. In the acute care setting, radiography should be performed before reduction to document bony injuries. Radiography allows a clinician to evaluate for the presence of a bony Bankart lesion (labrum injury with a fracture of the glenoid) or a Hill-Sachs lesion (an impaction fracture involving the humeral head). Athletes with a shoulder dislocation should be treated acutely using a reduction maneuver to restore the normal position of the humeral head. Experienced clinicians may be able to perform a shoulder reduction maneuver in the field before muscle spasm makes sedation and analgesia necessary. However, in most cases, a health care provider is not present at the time of injury, and reductions are generally performed in the emergency department using procedural sedation. Following reduction, the shoulder should be immobilized for 2 to 3 weeks. Although slings are typically used to immobilize the shoulder, there is some evidence that use of a brace that positions the shoulder in external rotation for the first 24 hours after an injury reduces the risk of recurrence. After a few weeks, patients should undergo physical therapy to improve range-of-motion and strength in the shoulder. For some patients, elective surgical repair of torn ligaments and the labrum may be indicated to stabilize the shoulder. Younger patients and males have high rates of recurrent dislocation. Male adolescents have approximately a 75% chance of redislocation in the first year after initial dislocation. Some providers feel that individuals in this demographic group should be considered surgical candidates after an initial dislocation. With most shoulder dislocations, the humeral head is displaced anteriorly and inferiorly. Posterior dislocations make up only 5% of dislocations. Rotator cuff tears are uncommon in adolescents. PREP Pearls - Shoulder dislocations can be caused by acute trauma or can result from laxity of the shoulder ligaments. - Adolescents and boys have high rates of redislocation after initial traumatic dislocation. ABP Content Specifications(s) - Understand the natural history of shoulder dislocation

A 15-year-old adolescent boy presents to your office for follow-up 3 days after a right shoulder injury. He was playing basketball when another player pulled his arm posteriorly while he reached up to rebound the ball. He had the feeling that his shoulder popped out. The boy was seen in the emergency department after the injury. Radiography confirmed that he had dislocated his shoulder and he underwent reduction under sedation. You counsel the family about this injury. Of the following, the MOST accurate statement is that A. adolescent boys have lower rates of recurrent dislocation than do adolescent girls B. adolescents are at lower risk for recurrent dislocations compared with adults C. in adolescents with shoulder dislocation, the humeral head is generally posteriorly displaced D. rotator cuff tears are often associated with shoulder dislocation in adolescents E. without surgery, he is likely to experience recurrent dislocation over the next 12 months

C. furosemide The patient described in the vignette has symptoms that began several months ago, but have acutely worsened in the last few days. With the additional information that her chest radiograph and her echocardiogram show a large pericardial effusion, it is important to decide if she has tamponade physiology. She has several signs that she is now experiencing tamponade: her neck veins are distended, her blood pressure is low for her level of anxiety and distress, and her heart sounds are soft. These findings suggest, in order, that she is unable to fill the right side of her heart easily, that her cardiac output is decreased, and that her heart is further away from her chest wall than would be expected. In this situation, administration of furosemide would be dangerous. It would lower her circulating blood volume, make compression of her right ventricle more severe, and acutely decrease her systolic output. Of the choices given, this is most likely to worsen the patient's status. Administration of ceftriaxone would be helpful if this was a bacterial pericarditis and the bacteria were sensitive to that antibiotic. If this were bacterial pericarditis, a much more fulminant course would be expected. Antibiotics would not likely cause an adverse effect. Dopamine may be needed if the patient became hypotensive and would not make the effusion worse. A normal saline bolus would be the best intervention while more definitive treatment is being arranged, including draining the effusion. Supplemental oxygen would not hurt the patient in this setting of decreased cardiac output; it would be helpful to keep tissue and myocardial oxygenation as high as possible. The causes of pericardial effusions are many. The major categories are metabolic (uremia, hypothyroid states), infectious (viral, bacterial, fungal) with or without myocarditis, and autoimmune (lupus, juvenile rheumatoid arthritis, acute rheumatic fever). Presentation can be acute or chronic or a combination, as in this case. The most likely cause in this patient is an autoimmune process of several months' duration, which responded to anti-inflammatory medications. Her shoulder pain is referred from diaphragmatic irritation. She now exhibits tamponade physiology because her pericardium cannot stretch any further. As her effusion accumulates, her right atrium and right ventricle are collapsing. Her heart sounds are soft because of fluid encasing the heart. The flow to her lungs, and therefore to her left ventricle and her systemic circulation, is not adequate, thus making her blood pressure low. PREP Pearls - Pericardial effusions can present with positional chest pain, abdominal pain, or shoulder pain. - A slowly accumulating pericardial effusion can suddenly become dangerous when tamponade physiology develops. ABP Content Specifications(s) - Understand the natural history of pericarditis - Recognize pathogens commonly associated with pericarditis - Recognize the clinical findings associated with pericarditis and plan appropriate initial management

A 15-year-old adolescent girl comes for evaluation with complaints of intermittent shoulder pain that has waxed and waned for the past 3 to 4 months. She has been on multiple courses of nonsteroidal anti-inflammatory medications, such as ibuprofen, has had improvement with each course for 1 to 2 weeks, and then the shoulder pain recurs. Over the last few days, she has had worsening shoulder pain and shortness of breath. She was unable to sleep last night. She is able to speak, but is most comfortable sitting upright on the stretcher. Her heart rate is 130 beats/min, respiratory rate is 32 breaths/min, and blood pressure is 90/60 mm Hg. She is anxious and has increased work of breathing. On physical examination, her neck veins are visible. Chest is clear with equal breath sounds. Her cardiac examination is difficult because of her body habitus, but her heart sounds are quite soft. There is no murmur, rub, or gallop. Her abdominal examination is not reliable because she cannot lie flat. You obtain a chest radiograph (cardiomegaly). She is admitted to the hospital and an echocardiogram shows a large pericardial effusion. Of the following, the intervention MOST likely to worsen this patient's clinical status is A. ceftriaxone B. dopamine C. furosemide D. normal saline bolus E. supplemental oxygen

Risk factors for development of atherosclerotic disease and coronary artery disease include personal and family history, as well as hyperlipidemia. It is well known that smoking, obesity, hypertension, and diabetes are all risk factors. Early coronary artery disease in family members is also a risk factor (men younger than 55 years of age and women younger than 65 years of age). A previous history of Kawasaki disease is a risk factor for later onset coronary artery disease, especially if aneurysms were noted at the time of presentation. Using a body mass index above the 85th percentile, the prevalence of obesity is as high as 30% of the general US population, therefore increased attention to this risk factor is warranted. Cholesterol and lipid screening is recommended twice in all children, once between the ages of 9 and 11 years and again at 17 to 21 years of age. For any patient, if the total cholesterol is greater than 200 mg/dL (5.18 mmol/L), high-density lipoprotein is less than 45 mg/dL (1.17 mmol/L), and low-density lipoprotein (LOL) are greater than 130 mg/dL (3.37 mmol/L), then they are at increased risk for development of adult cardiovascular disease. Thus for the adolescent girl in this vignette, of the choices listed, LDL cholesterol level of 145 mg/dL (3.76 mmol/L), high-density lipoprotein cholesterol level of 20 mg/dL (0.52 mmol/L), fasting glucose level of 150 mg/dL (8.3 mmol/L), and blood pressure of 150/95 mm Hg would place her at the highest risk of cardiovascular disease. PREP Pearls - Hypertension, obesity, diabetes, smoking, and elevated cholesterol are major risk factors for coronary artery disease, along with a family history of early cardiovascular disease. - Early screening for elevated cholesterol and lipids is an important part of routine health maintenance. ABP Content Specifications(s) - Know the risk factors associated with coronary artery disease - Understand the recommendations of the National Cholesterol Education Program for Children

A 15-year-old adolescent girl comes to the office for a preparticipation examination before starting her soccer season. During the past year, she has gained 15 kg. Her weight is now 75 kg, height is 165 cm, and body mass index is 27.5. Her blood pressure is taken with an appropriately sized large adult cuff. The remainder of her physical examination is unremarkable. Laboratory studies are obtained. Of the following, the results (Item Q69) MOST likely to place this child at risk for future coronary artery disease are displayed in (too hard to paste) A. Row A B. Row B C. Row C D. Row D - correct E. Row E

A. potassium hydroxide test Distinguishing extensive tinea pedis from atopic dermatitis or other dermatoses can be difficult. Patients with atopic dermatitis can present with pruritic scaling rashes, however, atopic dermatitis typically has a relapsing course, whereas dermatophyte infections persist without treatment. In studies, allylamines (terbinafine) produced slightly better cure rates than azoles for the treatment of tinea pedis. PREP Pearls - Tinea pedis is caused by infection with Trichophyton or Epidermophyton species. - Visualization of septate hyphae on potassium hydroxide (KOH) wet mount examination of cutaneous scrapings from the affected area can confirm the diagnosis of tinea pedis. - Tinea pedis should be treated with topical antifungal agents. - Use of topical steroids in patients with tinea pedis can complicate the treatment course. - Use of topical antifungal-steroid combination therapies can lead to skin atrophy and excess treatment costs. ABP Content Specifications(s) - Differentiate the clinical findings of tinea pedis from those of atopic dermatitis, and manage appropriately

A 15-year-old, female distance runner presents to your office for a sports participation physical examination. She reports no problems or concerns. Her temperature, vital signs, and growth parameters are normal for age. Her physical examination findings are unremarkable, except for a rash on both feet that she reports has "been there for a long time and itches" (Item Q34 - tinea pedis). No other areas of skin are involved. Of the following, the MOST appropriate test to diagnose this patient's condition is A. potassium hydroxide test B. serum immunoglobulin E level C. skin biopsy D. skin swab for bacterial culture E. zinc level

B. outpatient otolaryngology follow-up in 3 to 5 days The adolescent boy in the vignette presents with a minimally displaced fracture of his nasal bones, with no associated septal hematoma, persistent bleeding, signs of nasal obstruction, or findings of associated orbital or ethmoid fractures. The best next step in the management of his condition is reassessment by an otolaryngologist at an outpatient follow-up visit within 3 to 5 days. Nasal bone fractures are the most common facial fractures that occur in children. Although relatively uncommon in young children, the incidence of these fractures increases in the adolescent and young adult years. Childhood nasal injuries typically result from falls, contact sports, weight lifting, and automobile crashes (including those involving bicyclists or pedestrians). Nasal bone fractures most commonly result from a direct blow to the face in the anterior-posterior direction, though they can also occur with a lateral blow. In children, the nose has a more prominent soft cartilaginous portion, which will bend easily, allowing the injury-causing force to dissipate across the midface. This may result in significant edema and ecchymosis of the nasal structures in children, which can make examination of the facial bones and nasal structure challenging. Direct forces to the nose can lead to fractures of the nasal skeleton and result in deviation or depression of the nasal bones and the nasal septum. Also, greenstick-type fractures of the nose are more common in children. Although nasal bone deformities may be detected during the initial clinical evaluation of affected patients, injury-related swelling may prevent their detection in many cases. Management of pediatric nasal bone fractures depends on a number of factors, including the age of the affected child, degree of nasal obstruction, the presence of nasal septal hematomas (which should be incised and drained emergently), and the presence of associated ocular, facial, intracranial, and cervical spine injuries. An optimal cosmetic result is one of the primary long-term concerns for nasal bone fractures. Children with nasal bone fractures who do not have extreme degrees of deviation, associated septal hematomas, airway obstruction, persistent epistaxis, or a concern for other significant associated injuries should be referred for outpatient re-evaluation by an otolaryngologist within 3 to 5 days. This brief period between the initial evaluation and a follow-up examination allows post-injury edema to resolve, which enables improved visualization of the nasal structures. Any deviation of the nasal septum can be reduced by the otolaryngologist at this short-term follow-up visit without a significant impact on the cosmetic outcome. Parents should be advised to bring a recent photograph of their child to the otolaryngology follow-up visit 3 to 5 days after the initial injury, so that the specialist may compare the appearance of the child's nasal contours before and after the injury. If more than 7 to 10 days elapse between the time of a child's nasal bone injury and reassessment by an otolaryngologist, reduction of the nasal fractures may be much more difficult, and may result in permanent displacement. This is because the active growth centers in children's nasal bones promote rapid healing, and fracture fragments begin to form a strong fibrous union in their deviated positions by this time. Therefore, follow-up in either 10 to 14 days or 4 to 6 weeks after injury would be too late for easy mobilization of displaced fracture elements. Emergent otolaryngology consultation in the emergency department is not necessary for the adolescent boy in the vignette, given that the fracture of his nasal bones is minimally displaced, and there is no associated septal hematoma, persistent epistaxis, concern for airway obstruction, or clinical suspicion for serious associated injuries. Finally, because the patient in the vignette has a displaced fracture of his nasal bridge (even though the degree of displacement is minimal), which may require reduction by an otolaryngologist, reassurance only would not be appropriate and could result in a cosmetic deformity. PREP Pearls - Children with known or suspected nasal bone fractures who do not have extreme degrees of deviation, associated septal hematomas, airway obstruction, persistent epistaxis, or a concern for other significant associated injuries should be referred for outpatient re-evaluation by an otolaryngologist within 3 to 5 days. - Although nasal bone deformities may be detected during the initial clinical evaluation of affected patients, injury-related swelling may prevent their detection in many cases. - If greater than 7 to 10 days elapse between the time of a child's nasal bone injury and reassessment by an otolaryngologist, reduction of fractures by the specialist may be much more difficult, and may result in permanent displacement. ABP Content Specifications(s) - Understand the importance of early referral for surgical correction of displacement of the nasal bones

A 15-year-old, previously healthy adolescent boy presents to the emergency department for evaluation of an injury to his nose that he sustained approximately 2 hours ago. He was playing basketball when he was inadvertently struck directly over his nasal bridge by another player's elbow. He experienced no loss of consciousness. Immediately following the injury, he had an episode of bleeding from his nose that lasted approximately 5 min, but resolved with application of direct pressure. His parents decided to bring him to the emergency department because of severe swelling and pain across his nasal bridge. On physical examination, the patient appears well. He is alert and fully oriented, and his vital signs are normal for his age. There is marked swelling, ecchymosis, and tenderness to palpation across his nasal bridge with flattening of the nasal dorsum. Inspection and speculum examination of the intranasal cavity reveals no findings suggestive of a septal hematoma and no epistaxis or rhinorrhea. There is no dental malocclusion or palatal instability on examination. The remainder of physical examination findings, which include a complete neurologic evaluation, are unremarkable. He is having no respiratory difficulty. Computed tomography of the facial bones reveals a minimally displaced fracture of his nasal bridge with no associated septal hematoma. Of the following, the BEST next step in the management of this patient's condition is A. emergent otolaryngology consultation in the emergency department B. outpatient otolaryngology follow-up in 3 to 5 days C. outpatient otolaryngology follow-up in 10 to 14 days D. outpatient otolaryngology follow-up in 4 to 6 weeks E. reassurance with no need for subspecialty follow-up

B. group A Streptococcus The findings of severe pain, exquisite sensitivity, and bluish discoloration at the site of skin injury plus the degree of systemic illness described for the boy in this vignette are highly concerning for a diagnosis of necrotizing fasciitis caused by infection with group A β-hemolytic Streptococcus (GABHS). This infection can occur in otherwise healthy individuals. In addition to antimicrobial therapy, necrotizing fasciitis requires prompt surgical debridement to remove necrotic tissue and decrease toxin production. Co-infection of Staphylococcus aureus with GABHS may also occur in this setting. Prior to widespread varicella immunization, necrotizing fasciitis with GABHS was reported as a complication of chicken pox. Clostridium perfringens has been associated with gas gangrene following trauma. In this entity, subcutaneous emphysema with skin blisters and serosanguinous discharge are seen in association with the muscle necrosis at the site of injury. Necrotizing fasciitis in association with anaerobes such as C perfringens, as part of a mixed infection of anaerobic and aerobic bacteria, is seen in persons with underlying medical conditions such as diabetes, recent surgery, immune compromise, or peripheral vascular disease. Methicillin-resistant Staphylococcus aureus (MRSA) is a leading cause of skin and soft tissue infection, but the lesions typically present with a purulent component. Necrotizing fasciitis with MRSA alone has only been rarely reported. Mixed bacterial infections leading to the degree of local and systemic illness described in this vignette are not commonly described in normal hosts. Pseudomonas aeruginosa infection has been associated with gangrenous lesions and severe disease in immunocompromised hosts or after severe crush injury, neither of which applies in this case. Initial antibiotic therapy in suspected necrotizing fasciitis typically includes clindamycin in addition to penicillin. Clindamycin has been shown to suppress toxin production and modulate cytokine release. Additionally, it would provide coverage against community-acquired MRSA pending culture results. PREP Pearls - Necrotizing fasciitis is clinically characterized by an exquisitely painful, tender lesion with bluish discoloration at the site of infection. - Surgical debridement is a critical component of the management of necrotizing fasciitis. - Group A streptococcus is the leading cause of necrotizing fasciitis in the previously normal host. ABP Content Specifications(s) - Recognize the clinical findings associated with necrotizing fasciitis

A 16-year-old adolescent boy presents to the emergency department for evaluation of a painful skin lesion on his left lower extremity that has been progressing over the last 3 days. Three days ago, he scratched his left lower calf while climbing over a fence in the playground. The next day, he noticed some redness surrounding the scratch site, but he did not seek medical attention. Over the next 24 hours, he noted a marked increase in pain (9 out of 10) at the site. He also began to feel warm, had a decrease in appetite, and complained of diffuse achiness and malaise. He has been previously well and is not taking any medication. His vital signs show a temperature of 39.2°C, a heart rate of 110 beats/min, a blood pressure of 120/76 mm Hg, and a respiratory rate of 22 breaths/min. Physical examination in the emergency department shows an ill-appearing adolescent. The left lower extremity has a 4 × 5 cm erythematous area surrounding a scratch mark with bluish discoloration. The area is exquisitely sensitive to touch. There is no swelling or discharge at the site. The remainder of the examination findings are unremarkable. Of the following, the MOST likely cause of this lesion is infection with A. Clostridium perfringens B. group A Streptococcus C. methicillin-resistant Staphylococcus aureus D. mixed bacterial infection E. Pseudomonas aeruginosa

E. placement of an endotracheal tube The first priority in treating any child who has ingested a toxic substance is to ensure stability of the airway and to take any necessary steps to maintain adequate ventilation and circulation. The adolescent in this vignette is minimally responsive, has already had episodes of vomiting, and is demonstrating an inability to maintain her own airway (drooling.) Although activated charcoal is the gastrointestinal decontamination agent of choice in pediatric patients, it should never be administered in patients with an unprotected airway. NG tube placement can cause vomiting. Flumazenil is useful in known benzo overdoses, but it may lower the seizure threshold and is contraindicated in tricyclic antidepressant overdoses, as well as unknown overdoses. N-AC likely indicated but airway needs secured first. The clinical benefit of gastric lavage has not been confirmed in controlled studies, and its routine use in managing poisoning cases is no longer recommended. Furthermore, ingestion of a hydrocarbon or caustic substance—which would be a contraindication for gastric lavage—cannot yet be excluded PREP Pearls - The first priority in treating any child who has ingested a toxic substance is to ensure stability of the airway. - In contrast to toxic exposures involving young children—which are generally accidental and involve small quantities of a single substance—adolescent exposures are more likely to be intentional, involve more than one substance, and have serious medical implications. - Regional poison control centers are accessible to health care providers 24 hours a day, 365 days per year, and can assist providers with expert advice. ABP Content Specifications(s) - Plan the management of poisoning by an unknown substance or by multiple substances - Understand the data available from poison control centers

A 16-year-old adolescent girl is brought to the emergency department by emergency medical services (EMS) because of decreased responsiveness. Approximately 30 min before arrival, the patient's mother arrived home from work and found her lying on the bathroom floor unresponsive, with vomitus around her mouth and on the front of her shirt. An open bottle of acetaminophen was on the bathroom counter. After a few unsuccessful attempts to "wake up" her daughter, the mother called 911 for help. En route to the emergency department, the transporting EMS team secured intravenous access and administered 2 doses of intravenous naloxone with no change in her clinical status. As you are evaluating the patient, her mother tells you that her daughter has been very depressed over the past 2 months. During that time, she has gone through a breakup with her boyfriend, and her maternal grandmother, who has terminal lung cancer, moved into the family's home and has been receiving palliative care through a local hospice program. When you question the mother about medications available within the family home, she states that the family's medicine cabinet contained only acetaminophen and multivitamins, but she is unsure what medications are stored in the grandmother's bedroom. The patient's temperature is 37.2°C, heart rate is 130 beats/min, respiratory rate is 16 breaths/min, blood pressure is 92/60 mm Hg, and pulse oximetry is 97% on room air. On physical examination, the patient is minimally responsive and does not respond to any commands. She does not consistently localize to painful stimuli. She is drooling and has dried vomitus on her face. Her pupils are 3 mm in diameter bilaterally and equally reactive. A complete physical examination reveals no findings suggestive of traumatic injury. Her skin appears pale and clammy, but there is no rash. Of the following, the BEST next step in the management of this patient is A. administration of activated charcoal by NG Tube B. administration of intravenous flumazenil C. administration of intravenous N-acetylcysteine D. lavage of gastric contents through a NG tube E. placement of an endotracheal tube

B. excess bone formation following an acute hematoma in the quadriceps muscle group Children and adolescents with myositis ossificans traumatica may have tenderness and induration at the injury site, as well as restricted motion in the joints crossed by the affected muscle group. For example, young athletes with heterotopic ossification in the quadriceps muscles often have decreased knee flexion Radiographic changes are typically present by 3 to 6 weeks after the injury and often lag behind the development of characteristic signs and symptoms. Most individuals with myositis ossificans have symptom improvement with use of ice and nonsteroidal anti-inflammatory medications. Patients with restricted joint motion should perform range-of-motion exercises. Individuals with persistent symptoms, or restricted joint motion that interferes with activity, are candidates for surgical treatment. PREP Pearls - Myositis ossificans traumatica refers to ectopic bone formation in a muscle group following trauma to the muscle. - Myositis ossificans traumatica should be considered in a patient with a history of a direct blow to a muscle that exhibits persistent pain or worsening several weeks after injury. ABP Content Specifications(s) - Plan the appropriate management of bruises and hematomas - Identify complications associated with a deep hematoma of the thigh

A 16-year-old boy presents to your office for evaluation of left thigh pain following an injury during soccer. The boy reports that 3 weeks ago, while playing in a soccer match, he was struck in the front of the thigh by the knee of another player. He had pain and decreased ability to bend the left knee after the injury. These symptoms resolved about 10 days after the injury, but over the past few days, his pain has recurred and the thigh now appears swollen. On physical examination, you note induration and mild, diffuse tenderness to palpation over the anterior aspect of the left thigh and limited flexion of the left knee. Strength and sensation in the affected limb are normal. Of the following, the MOST likely cause of this boy's symptoms is A. compartment syndrome due to late hematoma development in the quadriceps muscle group B. excess bone formation following an acute hematoma in the quadriceps muscle group C. neuropraxia due to a direct blow to the femoral nerve D. partial tearing of the rectus femoris muscle E. thrombosis of the femoral vein secondary to trauma

D. post-traumatic stress disorder Adolescents are at high risk for sexual violence. During the high school years, approximately 1 in 5 male and female adolescents may be victims of dating violence. Rape is a legal term, therefore prevalence data are dependent on how information is collected and measured. Unfortunately, sexual violence is an under-reported crime and the unsupported victims often go on to have a range of early symptoms and future negative outcomes. These symptoms are often referred to as the "rape-trauma-syndrome." In the initial phase, victims experience fear, disbelief, anxiety, guilt, and emotional lability, followed days to weeks later by a longer lasting phase of reorganization and recovery. Up to 80% of victims will experience post-traumatic stress disorder, as did the girl in the vignette. These victims are also at risk for substance abuse, eating disorders, self-cutting, truancy, academic problems, risky sexual behavior, and revictimization. Victims who acknowledge and report an assault, and seek professional services often have a better outcome than those who do not. This makes it important for all adolescents to be screened for sexual violence and to get them help early. Even more important is to have children participate in prevention programs. Pediatricians should be knowledgeable about services available in the communities in which they work and be available to provide initial psychological support. The girl in the vignette is already indulging in binge drinking, displaying mood disturbances, and may be developing fibromyalgia, all of which are seen with increased frequency in sexual assault victims. Binge drinking, irritable bowel syndrome, and depression alone would not account for the full range of symptoms she is experiencing. She does not meet criteria for the diagnosis of somatization disorder, which includes the need for 2 pain symptoms, 2 gastrointestinal symptoms, and 1 sexual and 1 pseudoneurologic symptom. PREP Pearls - Adolescents experience trauma, including sexual violence, at higher rates than either children or adults. - Screening all adolescents for a history of violence is important. Victims who acknowledge and report having been assaulted, and seek services often do better. - Victims of sexual violence may experience post-traumatic stress disorder and are at risk for substance abuse, eating disorders, self-cutting, truancy, academic problems, risky sexual behavior, and revictimization. ABP Content Specifications(s) - Recognize the medical and emotional needs of an adolescent victim of sexual assault, and manage appropriately

A 16-year-old girl is brought to your office by her mother for concerns about falling school grades. The girl seems to have lost interest in a number of activities she previously enjoyed and seems irritable. Her mother states that her daughter has been drunk on a few occasions in the last few months and has developed gastrointestinal symptoms with abdominal pain, loose bowel movements, and a weight loss of about 10 pounds. She has had difficulty falling asleep. She was recently seen in an emergency department for back pain; she was advised to get further evaluation for fibromyalgia. In looking through your records, you note that she was treated for chlamydia 4 months ago, and today she admits that this was the result of a sexual assault. Of the following, the MOST likely explanation for her symptoms is A. binge drinking B. irritable bowel syndrome C. new-onset depression D. post-traumatic stress disorder E. somatoform disorder

B. pelvic inflammatory disease Lower abdominal or pelvic pain in an adolescent girl may be acute or chronic and have a number of possible causes. The history and physical examination should help narrow the differential diagnosis. Intermenstrual intervals can range from 3 to 6 weeks and therefore the 5-week interval reported by the teenager in the vignette is within normal limits. Any pelvic symptoms should prompt a pregnancy test and the results are reported as negative for the adolescent girl in the vignette. She has had acute pain over 2 days and notes that her last menstrual period was heavier and more painful than usual. This should raise a concern that this is not just primary dysmenorrhea (painful menstruation). For a sexually active girl with a negative pregnancy test, the most important consideration is a sexually transmitted infection. Her bimanual examination results confirm the diagnosis of pelvic inflammatory disease (PID), meeting the US Centers for Disease Control and Prevention minimal criteria (abdominal tenderness with cervical motion and adnexal tenderness). PID often presents with a number of less specific symptoms including urinary and gastrointestinal systems. A urinary tract infection would not present with menstrual changes. Ovarian cyst rupture or torsion is usually abrupt in onset, and sharp and severe in intensity. Endometriosis is part of the differential diagnosis for chronic pelvic pain and a definitive diagnosis requires a laparoscopic evaluation. It is an important diagnosis to consider in adolescents with severe dysmenorrhea that does not respond to over-the-counter pain medications. PREP Pearls - Understanding variations in a normal menstrual cycle is helpful when educating and evaluating symptoms in an adolescent. - Documenting a last menstrual period on each encounter and performing a pregnancy test if late with menses or in the presence of any pelvic symptoms is very important. - A change in menstrual flow and increased degree of pain may indicate the presence of a sexually transmitted infection. ABP Content Specifications(s) - Recognize the gynecologic etiologies of acute and chronic abdominal pain - Recognize normal variations in the menstrual cycle in adolescent girls

A 16-year-old, sexually active adolescent girl presents to your office with lower abdominal pain that has lasted for 2 days. She has had some dysuria and loose bowel movements over the last few days. Her current menstrual period came after an interval of 5 weeks, and she finished her flow 3 days ago. She notes that her menstrual flow was heavier and more painful than usual. She is afebrile and has normal vital signs. On physical examination, she exhibits diffuse lower abdominal tenderness with no rebound or guarding. Bimanual examination elicits pain on movement of her cervix and palpation of her adnexa, with no palpable masses. A pregnancy test result is negative and complete blood cell count is normal. Of the following, the MOST likely cause of her pain is A. endometriosis B. pelvic inflammatory disease C. primary dysmenorrhea D. ruptured ovarian cyst E. urinary tract infection

D. osteosarcoma The adolescent in this vignette most likely has a malignant bone tumor, such as osteosarcoma (OS), which is the most common malignant bone tumor in children. The incidence is 4.4 per million per year with the peak incidence seen in the second decade of life during the adolescent growth spurt. Some familial cancer syndromes such as hereditary retinoblastoma, Rothmund-Thomson syndrome, and Li-Fraumeni syndrome may predispose affected individuals to OS, but most cases are not associated with known germline abnormalities. Osteosarcoma typically involves the long bones, most commonly around the knee joint, with the distal femur and proximal tibia being the most frequent sites of involvement. In decreasing frequency, the other common sites of OS are the distal humerus, middle and proximal femur, pelvis, and jaw. Approximately 15% to 20% of patients with OS present with observable metastatic disease on initial staging evaluation, most frequently in the lungs and other areas in the bone. However, all patients with OS are presumed to have micrometastatic disease at diagnosis. This is based on the finding that with amputation alone, 80% of patients will have disease recurrence. Therefore, all patients with OS require systemic chemotherapy, in addition to surgical excision of the tumor. Extent of disease at diagnosis is the most important prognostic factor. The primary site of the tumor is also an important variable, because it can affect the resectability of the tumor. Most patients present with pain and swelling in the affected bone. Symptoms can vary in duration before presentation. Respiratory symptoms, fever, and weight loss are rare, but can occur with extensive metastases. Multi-agent chemotherapy and complete surgical resection (most frequently with a limb salvage procedure) are the mainstays of treatment. In adolescents with a suspected bone tumor, plain radiographs of the affected area should be the initial step in evaluation. Malignant bone tumors usually reveal a mixed lytic and sclerotic lesion, a soft tissue mass with ossification, and periosteal new bone formation with lifting of the cortex, known as the Codman triangle. The adolescent in the vignette presents with worsening leg pain that started 1 month ago with radiographic evidence of a large mixed lytic and sclerotic lesion, known as a Codman triangle. Therefore, OS would be the most likely diagnosis. Although pathologic fractures can occur with bone tumors, no old or recent fracture is visible on radiography. The absence of fever makes the diagnosis of osteomyelitis unlikely. A vascular malformation that was present since birth would be unlikely to suddenly cause pain and swelling, and does not typically invade the bone. With venous malformations, discoloration of the overlying skin may be seen. Osteoid osteoma (OO) is the third most common benign bone tumor, after osteochondroma and nonossifying fibroma, and usually occurs in boys between 7 and 25 years of age. It usually presents with pain that is worse at night and relieved with nonsteroidal anti-inflammatory drugs (NSAIDs). The lesion may be tender to palpation and cause swelling. Swelling is common in OO and may be associated with high levels of prostaglandin production in the tumor, which is also the rationale for pain relief with NSAIDs. The tumor rarely exceeds 1.5 cm, but it can be associated with a significant amount of inflammation. The lesions are usually seen in the diaphysis or metaphysis of long bones such as the femur or tibia. A large mixed lytic and sclerotic lesion with a Codman triangle would not be typical of this benign tumor. PREP Pearls - Osteosarcoma (OS) is the most common malignant bone tumor in children. The peak incidence is in the second decade of life during the adolescent growth spurt. - Osteosarcoma typically involves the long bones, most commonly around the knee joint, with the distal femur and proximal tibia being the most frequent sites of involvement. In decreasing frequency, the other common sites of OS are the distal humerus, middle and proximal femur, pelvis, and jaw. - Although all patients with OS are presumed to have micrometastatic disease (thus the rationale for systemic chemotherapy in addition to wide surgical excision of the tumor), approximately 15% to 20% of patients with OS present with macrometastatic disease on initial staging evaluation, most frequently in the lungs and other bony sites. - Osteoid osteoma is the third most common benign bone tumor, after osteochondroma and nonossifying fibroma, and usually occurs in boys between 7 and 25 years of age. It commonly presents with pain that is worse at night and relieved with nonsteroidal anti-inflammatory drugs. ABP Content Specifications(s) - Recognize the clinical findings associated with osteosarcoma - Recognize the clinical and laboratory findings associated with osteoid osteoma - Understand site(s) of metastasis of malignant bone tumors in children of various ages

A 17-year-old adolescent boy presents to your clinic for left leg pain and swelling. One month ago, he began to have intermittent pain. The pain has been continuous for the past 3 days and has limited his ability to walk. He plays on the varsity football team, but does not recall any direct trauma to that area of the leg. In the office, he is afebrile and has normal vital signs. He is alert and in no apparent distress. There is marked swelling of the left distal thigh, but no bruising or discoloration of the skin. His dorsalis pedis pulses are strong and symmetric. His cardiopulmonary examination is unremarkable. You order a radiograph of his leg (periosteal elevation at metaphysis like sunburst). Of the following, the MOST likely diagnosis is A. healing fracture B. osteomyelitis C. osteoid osteoma D. osteosarcoma E. vascular malformation

B. erythrocyte galactose-1-phosphate uridyltransferase Hepatic dysfunction (international normalized ratio of 1.95) with hepatomegaly and cholestasis during the first few days after birth (particularly, as in this case, within the first 24 hours after commencing feeding) suggest an inborn metabolic error. The associated finding of positive reducing substances in the urine indicate a carbohydrate metabolic disorder, the most prevalent being galactosemia. (positive reducing substance suggests sugar other than glucose is present in urine.) Galactosemia is a disorder of galactose metabolism that, untreated, leads to feeding problems, failure to thrive, life-threatening hepatocellular damage, bleeding, and sepsis. A lactose- or galactose-restricted diet initiated within the first 10 days after birth will result in a rapid resolution of symptoms and greatly reduce the risk of complications. The diagnosis of galactosemia is established by measurement of erythrocyte galactose-1-phosphate uridyltransferase (GALT) activity, erythrocyte galactose-1-phosphate concentration, and GALT molecular genetic testing. In classic galactosemia, GALT activity is less than 5% of control values. Tyrosinemia type I usually presents during the neonatal period with liver involvement, marked by elevated transaminases, hypocholesterolemia, and hyperbilirubinemia. In the child in the vignette, the finding of positive urinary reducing substances suggests a carbohydrate, as opposed to an amino acid metabolic disorder. Urine succinylacetone would diagnose this. PREP Pearls - Direct hyperbilirubinemia in the first week after birth suggests a metabolic or infectious etiology. - Hepatic dysfunction within a few days (often < 24 hours) of introducing breast milk or cow milk-based formulas suggests a diagnosis of galactosemia. - Percutaneous liver biopsy remains the most sensitive and specific test for extrahepatic biliary atresia. ABP Content Specifications(s) - Plan the appropriate diagnostic evaluation of conjugated hyperbilirubinemia in a neonate

A 2,000-g, 34-week-gestation female newborn was admitted to the neonatal intensive care unit at 3 days of age because of jaundice, lethargy, and vomiting. The pregnancy was complicated by illicit drug abuse and poor prenatal care. Delivery was by cesarean birth secondary to maternal preeclampsia, and the Apgar scores were 6 and 7 at one and five minutes, respectively. She initially fed well with cow milk-based formula but was noted to develop jaundice at 24 hours after birth; the baby became increasingly difficult to feed, with multiple episodes of postprandial emesis. Because of these symptoms, a sepsis workup was performed and the baby was started on intravenous antibiotics. Physical examination demonstrates a lethargic, jaundiced infant. She has a temperature of 37.0°C, pulse rate of 130 beats/min, respiratory rate of 20 breaths/min, blood pressure of 88/56 mm Hg, and oxygen saturation of 97% on room air. The liver is palpated 5 cm below the right costal margin. The remainder of the examination findings are normal. Laboratory studies are obtained and include the following results: Total bilirubin, 15 mg/dL (256.5 mmol/L) Direct bilirubin, 5.5 mg/dL (94 mmol/L) Alanine aminotransferase, 205 U/L Aspartate aminotransferase, 170 U/L Prothrombin time, 21 s Partial thromboplastin time, 39 s International normalized ratio, 1.95 Urine test strip, positive for reducing substances Of the following, the MOST appropriate next diagnostic test is A. cell culture for cytomegalovirus from urine B. erythrocyte galactose-1-phosphate uridyltransferase C. polymerase chain reaction for herpes simplex virus in cerebrospinal fluid D. serum a-1 antitrypsin measurement with protease inhibitor typing E. urine succinylacetone

B. 140 mL/kg per day: 140 kcal/kg per day; 30 kcal/oz formula The ideal caloric intake for an infant in congestive heart failure secondary to increased pulmonary blood flow, such as the infant in the vignette, is 130 to 140 kcal/kg per day. On the other hand, the overall fluid intake must be low enough to keep from exacerbating the pulmonary congestion. Minimal hydration requirements for any infant would be at least 100 mL/kg per day; the challenge is to provide a formula or enhanced breast milk at a caloric density that can achieve both. PREP Pearls - The ideal caloric intake for infants in heart failure is 130 to 140 kcal/kg per day with the minimal volume needed for hydration. - Consultation with a dietician to help the parents mix and provide this high-calorie feeding or institute nasogastric tube feedings, if necessary, is an important safety measure. ABP Content Specifications(s) Plan the dietary management of cardiac disease in a patient who is receiving a fluid-restricted diet

A 2-month-old infant is brought to your office for a follow-up visit. She was diagnosed with a large perimembranous ventricular septal defects (VSD) at 4 weeks of age when she weighed 4.2 kg. Her mother states she sweats and tires with feedings and that she does not seem to be gaining weight. Her feedings take up to 50 min and she needs to rest every 5 min; one month ago, she took her feedings in 20 min. A physical examination shows a weight of 4.4 kg, a heart rate of 154 beats/min, and a respiratory rate of 60 breaths/min. Her lungs are clear. The point of maximum impulse is displaced to the left; a 4/6 holosystolic murmur at the left midsternal border obscures S1 and radiates to the left upper sternal border. Her liver edge is palpable 3 cm below the right costal margin. Of the following, the fluid and feeding regimen that would BEST allow optimal growth and weight gain for this child is A. 100 mL/kg per day; 100 kcal/kg per day; 30 kcal/oz formula B. 140 mL/kg per day: 140 kcal/kg per day; 30 kcal/oz formula C. 200 mL/kg per day; 145 kcal/kg per day; 22 kcal/oz formula D. 200 mL/kg per day; 170 kcal/kg per day; 26 kcal/oz formula E. 250 mL/kg per day; 215 kcal/kg per day; 26 kcal/oz formula

B. erythema multiforme EM is an immune-mediated hypersensitivity reaction most commonly associated with herpes simplex virus, mycoplasma pneumonia, nonsteroidal anti-inflammatory drugs, sulfonamides, antiepileptics, and antibiotics. EM is characterized by the classic target lesions with or without mucous membrane involvement. he typical course of the skin eruption is appearance over 3 to 5 days with resolution by 2 weeks. EM major is distinguished from SJS in that EM has typical target lesions with 3 concentric rings over the limbs with or without blistering, whereas SJS has purpuric macules and blisters distributed over the trunk and face. Stevens-Johnson syndrome and TEN are a clinical spectrum of immune-mediated hypersensitivity reactions presenting as a febrile, erosive stomatitis, with ocular involvement, and a widely distributed rash of dark macules with a necrotic center. Cases with more than 30% body surface area affected by epidermal detachment are considered TEN, whereas SJS will have less than 10% body surface area involvement. Histologically, both SJS and TEN present with a lymphocytic infiltrate at the dermal epidermal junction and superficial perivascular region. Serum sickness-like reactions present with pruritic urticarial rashes and arthralgias approximately 1 week after drug exposure. These are most often associated with cefaclor, and somewhat less often with penicillins, other cephalosporins, sulfonamides, macrolides, and bupropion. Treatment is discontinuation of the offending agent, and symptomatic treatment with antihistamines and nonsteroidal anti-inflammatory drugs for the pruritus and arthralgias. PREP Pearls - The first step in treating erythema multiforme is to discontinue the offending drug or treat the inciting infection if still present. - The first step in treating both Stevens-Johnson syndrome and toxic epidermal necrolysis is discontinuation of the offending drug. - Steroids and intravenous immunoglobulin have been shown to be effective in treating Stevens-Johnson syndrome. ABP Content Specifications(s) - Recognize the clinical spectrum of erythema multiforme Recognize the clinical features of Stevens-Johnson syndrome and manage appropriately

A 2-year-old boy comes in to your clinic with a new-onset rash. The rash started yesterday on both palms and spread to the arms and the back of the hands bilaterally. The mother denies any scratching and states that the boy has been active and playful. On physical examination, you observe a well-appearing child with dull red, flat macules, with a targetlike appearance of concentric circles and marginated wheals, over the palms and arms (Item Q51 - dull red targetoid lesion). He has some mild lip swelling, crusting, and bullae on his buccal mucosa, with no gingival involvement. You note in his chart that he had an episode of herpes labialis 2 weeks ago. Of the following, the patient's MOST likely diagnosis is A. erythema marginatum B. erythema multiforme C. herpes simplex infection D. serum sickness E. Stevens-Johnson syndrome

A. α-fetoprotein Children with BWS typically do not present with overgrowth in association with growth hormone abnormalities (insulin-like growth factor 1 and insulin-like growth factor binding protein 3). The organomegaly commonly noted is not associated with liver dysfunction or thyroid abnormalities, though it can be associated with renal abnormalities. The hyperinsulinemic-hypoglycemia noted at birth in 50% of patients with BWS resolves spontaneously in early infancy and would not be a continued problem for a 2-year-old patient, such as the girl in this vignette. PREP Pearls - Within the differential diagnosis of a child with hemihyperplasia, a physician should consider Beckwith-Wiedemann syndrome (BWS), Proteus syndrome (PTEN hamartoma tumor syndrome), Klippel-Trenaunay-Weber syndrome, isolated hemihyperplasia, and neurofibromatosis type 1. - Children with isolated hemihyperplasia have a tumor risk of approximately 5.9%. - The risk for embryonal tumors in children with Beckwith-Wiedemann Syndrome is highest during the first 8 years of age, therefore high-risk surveillance is recommended, including abdominal ultrasonography every 3 months until age 8 years, and serum α-fetoprotein concentration every 2 to 3 months in the first 4 years after birth for early detection of hepatoblastoma and Wilms tumor. The estimated tumor risk for children with BWS is 7.5%. ABP Content Specifications(s) - Recognize the clinical features and risks associated with overgrowth syndrome

A 2-year-old girl arrives at your outpatient clinic with her mother. She is concerned that her left leg and arm seem to be larger than the leg and arm on the right. The mother reports that the child was 4.2 kg at birth and has continued to have rapid growth since that time. She also reports low blood glucose levels during the neonatal period. The mother states the child has always had a large tongue. Her current height is 94 cm (95th percentile) and her current weight is 15.9 kg (> 95th percentile). Physical examination is remarkable for a large tongue, anterior ear lobe creases, umbilical hernia, nephromegaly, and hemihypertrophy of the left arm and leg. Of the following, the MOST important laboratory workup that should be followed closely over time in this patient is A. α-fetoprotein B. insulin level C. insulin-like growth factor 1 D. liver function tests E. thyroid function tests

B. observation and reassurance The 2-year-old girl in the vignette has tibial torsion. The current treatment recommendation for tibial torsion is observation and reassurance. In-toeing in children is usually caused by benign conditions. The most common cause in children aged 1 to 3 years is tibial torsion. In cases of tibial torsion, there is rotation of the tibia so that the foot and knee are misaligned. Internal rotation is more common than external rotation. Tibial torsion can be unilateral or bilateral; when unilateral, the left side is more likely to be affected. Children are commonly diagnosed after parents report clumsiness and frequent tripping. Measurement of the thigh-foot angle can confirm the presence of tibial torsion; serial measurements are not indicated for this condition. Previously used treatments such as splinting, shoe modifications, braces, and exercises have been shown to be ineffective and are no longer recommended. The natural history of tibial torsion is resolution by 4 years of age, so observation and reassurance is recommended. Long-term consequences such as disability are rare, and tibial torsion is not a risk factor for degenerative joint disease. Surgical treatment is rarely recommended. External tibial torsion, seen in out-toeing, can be associated with neuromuscular conditions. Children with gait disturbance or pain associated with external tibial torsion should be referred for orthopedic evaluation. Femoral anteversion is the most common cause of in-toeing after 3 years of age. Femoral anteversion is an increase in the angle of the femoral neck greater than 20 degrees relative to the shaft of the femur. It is bilateral, more common in girls, and is associated with the W sitting position. The in-toeing usually increases in severity until 5 years of age, then spontaneously resolves by 10 to 12 years of age. Usually no treatment is required for femoral anteversion. Plain radiography would not be recommended for the routine evaluation of in-toeing. Metatarsus adductus is the medial deviation of the forefoot relative to the hindfoot and is the most common cause of in-toeing between birth and 1 year of age. Most cases of flexible metatarsus adductus resolve spontaneously and do not require braces, shoes, or splinting. In patients with inflexible metatarsus adductus (metatarsus varus), serial casting has been shown to be effective especially if performed early. Referral to an orthopedic surgeon for serial casting would be appropriate for fixed metatarsus adductus. Referral to an orthotist for nighttime splints is not appropriate because this has been shown not to be effective for tibial torsion. Rarely, in-toeing is associated with pathologic conditions such as neuromuscular disorders, clubfoot, skewfoot, and hip disorders. PREP Pearls - In infants, the most common cause of in-toeing is metatarsus adductus. - In toddlers, the most common cause of in-toeing is tibial torsion, which usually resolves spontaneously by 4 years of age without treatment. - In children older than 3 years, femoral anteversion is the most common cause of in-toeing and usually resolves spontaneously by 12 years of age. ABP Content Specifications(s) - Recognize the clinical findings associated with tibial torsion

A 2-year-old girl is brought to your office by her parents. The mother states that her daughter trips much more often than her other children, and she has noticed that the girl's feet are both turned inward—her left foot more than her right. Her daughter often sits on her feet. On physical examination, the child allows you to move all extremities and no pain or swelling are noted. In the prone position, you note that her feet are rotated toward the midline. You observe the child's gait and note the patella faces forward while the toes turn inward. Of the following, the BEST next step for this patient is A. measurement of the thigh-foot angle every 6 to 12 months B. observation and reassurance C. plain radiographs of the lower extremities D. referral to an orthopedic surgeon for serial casting E. referral to an orthotist for nighttime splints

A. allergic sensitization to milk, egg, peanut Asthma Predictive Factors: include the presence of clinician-diagnosed eczema, parental asthma, clinician-diagnosed allergic rhinitis, wheezing apart from colds, and eosinophilia greater than or equal to 4%. Gastroesophageal (GE) reflux is common in patients with asthma and has been identified as a potential trigger. Gastroesophageal reflux is thought to affect asthma through increased vagal tone, heightened bronchial hyperreactivity, or microaspiration. However, GE reflux does not cause asthma. Nonallergic rhinitis with nasal eosinophilia syndrome (NARES) is the most common type of inflammatory nonallergic rhinitis. PREP Pearls - The presence of allergic sensitization to aeroallergens or certain foods (milk, egg, peanuts) is considered a risk factor influencing the development and severity of allergic disorders in children. - Nonenvironmental factors, such as genetics, diet, and infection, also impact on the presence and severity of allergic disorders. ABP Content Specifications(s) - Recognize the non-environmental factors (eg, genetics, diet, infection) that influence the incidence and severity of atopy in infants and children

A 2-year-old girl is brought to your office for follow-up of severe atopic dermatitis. The child has been doing better with regular use of topical triamcinolone, hypoallergenic skin moisturizer, and bedtime diphenhydramine. She had an episode of wheezing illness during each of the past 2 winters that was triggered by a viral infection. She had received albuterol treatments during the illness. The parents felt the albuterol helped the wheezing, though inconsistently. In the course of the visit, you counsel the parents about the possibility of development of other atopic disorders, such as allergic rhinitis and asthma. The parents ask what factors increase the risk of their child developing asthma or other atopic disorders. Of the following, the BEST response to the parents' questions is A. allergic sensitization to milk, egg, peanut B. body mass index greater than 95th percentile C. gastroesophageal reflux D. neutrophilia during wheezing episodes E. nonallergic rhinitis with eosinophilia

C. excisional biopsy of the lesion The child described in the vignette has subacute or chronic cervical lymphadenitis, defined as lymph node enlargement and inflammation that has persisted for several weeks. Cervical lymphadenopathy is used to describe enlarged lymph nodes in the neck. However, the 2 terms (lymphadenitis and lymphadenopathy) are often used interchangeably. In children younger than 5 years of age who have subacute or chronic, nontender, unilateral lymphadenitis with a characteristic appearance of the overlying skin (violaceous and thin) and no constitutional symptoms, the diagnosis of nontuberculous mycobacterial (NTM) infection is nearly certain and excisional biopsy of the lymph node is appropriate. Incision and drainage of the lymph node is contraindicated because of the risk of creating a chronically draining fistula. Although ultrasonography of the neck can demonstrate lymphadenopathy, it is not a necessary step in making the diagnosis. If radiologic imaging is desired before surgical intervention, computed tomography would better reveal the anatomy of the neck and the extent of lymph node involvement. The differential diagnosis of subacute/chronic lymphadenitis in children includes infectious and noninfectious causes. The first step in the evaluation of a child with chronic cervical lymphadenopathy is to perform a careful history including the duration and location of lymph node swelling, associated constitutional symptoms, ill contacts, animal exposures, trauma, dental problems, immunization status, medications, travel history, and whether or not the lymph node swelling is recurrent or persistent. A thorough physical examination is essential to assess the number, size, shape, consistency, mobility, and tenderness of the lymph nodes. Evidence of oropharyngeal abnormalities (periodontal disease, pharyngitis, aphthous ulcers, etc), conjunctivitis, hepatosplenomegaly, bone or joint abnormalities (arthritis, bony tenderness or palpable lesions), or skin rashes can provide clues to the diagnosis. A tuberculin skin test ,should be performed, especially in children older than 5 years of age with risk factors for tuberculosis. The tuberculin skin test in children with NTM cervical lymphadenitis can minimally react or respond with up to 20 mm of induration, but these children are not ill-appearing. In children with severe, rapidly progressive lymphadenopathy, or those who are ill-appearing with symptoms lasting more than 6 to 8 weeks, laboratory and radiology investigations should include a blood culture, complete blood cell count with peripheral smear, appropriate serologies (ie, cytomegalovirus, Epstein-Barr virus, toxoplasmosis) and appropriate imaging to evaluate for evidence of malignancy, collagen vascular disease, or persistent infection. In children with subacute or chronic cervical lymphadenitis caused by NTM in whom complete excision of the lymph node is not possible, or who are not candidates for surgical excision because of the risk of facial nerve damage or other concerns, antimicrobial therapy with a macrolide (clarithromycin or azithromycin) in combination with a rifamycin (rifampin or rifabutin) or ethambutol for 12 weeks to up to 6 months is recommended. This therapy has been shown to result in a cure in two-thirds of patients, but one-third developed chronic draining fistulae and nearly 80% had antibiotic-associated adverse effects. Although trimethoprim-sulfamethoxazole can be useful for the treatment of acute cervical lymphadenitis caused by Staphylococcus aureus (less effective for group A Streptococcus), it is not a first-choice therapy for the treatment of NTM cervical lymphadenitis. PREP Pearls - Excisional biopsy is the optimal therapy for the treatment of subacute or chronic cervical lymphadenitis caused by nontuberculous mycobacteria (NTM). - Incision and drainage of subacute or chronic cervical lymphadenitis caused by NTM risks the development of a chronic draining fistula. - Subacute or chronic lymphadenitis caused by NTM occurs most often in children younger than 5 years of age and manifests as nontender, unilateral lymphadenitis with a characteristic appearance of the overlying skin (violaceous and thin) and no constitutional symptoms. - In children with NTM subacute or chronic cervical lymphadenitis in whom surgical excision of the node is not possible, the recommended treatment is antimicrobial therapy with a macrolide (clarithromycin or azithromycin) in combination with a rifamycin (rifampin or rifabutin) or ethambutol for 12 weeks to up to 6 months. ABP Content Specifications(s) - Plan the appropriate management of chronic cervical lymphadenopathy - Identify the age-related etiology of chronic cervical lymphadenopathy

A 2-year-old girl presents with a 6-week history of slowly progressive left neck swelling with redness. This started as a small "bump" under her left jaw with pink overlying skin. The area of involvement slowly enlarged and the overlying skin became a deeper red. The parents report that she was prescribed a 10-day course of oral clindamycin 3 weeks ago, but they have noticed no improvement in the redness or swelling. She has not had fever or change in appetite or activity. Her vital signs and growth parameters are normal for her age. The only remarkable finding on physical examination is a 1- to 2-cm, firm, red, nontender area of swelling under the left jaw. Of the following, the MOST appropriate next step in the management of this patient's condition is A. a complete blood cell count with peripheral smear B. a 10-day course of trimethoprim-sulfamethoxazole C. excisional biopsy of the lesion D. incision and drainage of the lesion E. ultrasonography of the neck

B. cardiogenic shock The child in this vignette is in shock, as evidenced by lethargy, respiratory distress, tachycardia, and decreased urine output, all of which indicate decreased organ perfusion. The type of shock is likely cardiogenic, as demonstrated by tachycardia, poor skin perfusion, hepatosplenomegaly, rales, jugular venous distention, and worsening condition upon fluid challenge. Shock is a condition in which oxygen delivery is insufficient to meet the metabolic demands of vital organs. It is a life-threatening state that must be distinguished from other less serious conditions so that treatment can be initiated without delay. Oxygen delivery is the product of cardiac output and oxygen content. It can also be represented by the following formula: Oxygen delivery = Stroke volume × Heart rate × Oxygen content As suggested by this equation, an early compensatory mechanism to increase oxygen delivery in shock, especially in children, is tachycardia. Elevated circulating catecholamines serve to increase stroke volume, heart rate, and blood pressure, as well as preferential diversion of cardiac output away from skin, renal, and splanchnic vascular beds toward the heart and brain. The resultant increased systemic vascular resistance leads to maintained, or even increased, blood pressure, resulting in compensated shock. The clinician should not be reassured by a normal or elevated blood pressure when considering shock. Despite representing a compensatory mechanism, catecholamines and increased systemic vascular resistance can be especially deleterious in cardiogenic shock because of increased afterload, diastolic dysfunction, increased myocardial oxygen demands, and impaired coronary artery filling. Cardiogenic shock further characterizes the insufficiency of oxygen delivery as a failure of the heart as a pump. This can be caused by impaired contractility, arrhythmias, outflow tract obstruction, or impaired filling as seen in diastolic dysfunction, restrictive cardiomyopathy, lesions of reduced cavity size, or tamponade physiology. Impaired contractility is most commonly seen in myocarditis, primary neuromuscular disorders, or metabolic derangements. Single ventricle congenital heart disease with ductal-dependent systemic blood flow, such as hypoplastic left heart syndrome, critical aortic stenosis, and interrupted aortic arch can cause shock if right-to-left shunting across the ductus arteriosus is impaired. Ductal-dependent pulmonary blood flow lesions, such as tricuspid atresia or pulmonary atresia can cause severe cyanosis if left-to-right shunting across the ductus arteriosus is impaired. Heart failure can masquerade in children as pneumonia, asthma, or bronchiolitis because clinical signs also include wheezing, grunting, and difficulty breathing. In heart failure, increased left atrial pressure transmits upstream to the pulmonary circulation, and the resulting increased pulmonary capillary hydrostatic pressure in turn leads to pulmonary edema. Increased lung water and decreased pulmonary compliance can cause tachypnea and grunting, and compression of neighboring bronchioles can lead to so-called "cardiac wheezing." It is therefore imperative to recognize shock in this patient. This is especially true because treatment with bronchodilators can worsen heart failure by causing tachycardia, diastolic hypotension, lactic acidosis, and increased myocardial oxygen demand. In this patient, hypovolemic shock is less likely in light of the rales, hepatomegaly, and worsened condition after receiving fluids. Patients with cardiogenic shock may also be dehydrated because of general illness and increased insensible losses from tachypnea. Therefore, very cautious use of fluid boluses can help such a preload-dependent patient experiencing heart failure. However, boluses should be limited to 5 mL/kg at a time, followed by careful reassessment. If preload is adequate and cardiac output is still impaired, a cardiology consultation should be sought, and inotropes should be started. One of the most important initial steps in caring for the acutely ill child is determining whether the patient is in shock. Poor perfusion, altered mental status, and tachycardia distinguish hypovolemic shock from less serious conditions. Shock can further be classified as cardiogenic shock if signs or symptoms of congestive heart failure are present, such as rales, hepatomegaly, or jugular venous distention. Heart failure is one of many nonpulmonary causes of respiratory distress, and one of the most serious. PREP Pearls - The clinician should not be reassured by a normal or elevated blood pressure when considering shock. - Cardiogenic shock may be distinguished from other causes of shock by the presence of rales, hepatomegaly, and jugular venous distention. - Respiratory distress and wheezing are common signs of cardiac disease in children. - Cardiogenic shock can be worsened by aggressive fluid resuscitation. ABP Content Specifications(s) - Recognize that the blood pressure may not be adversely affected by shock

A 20-month-old boy is brought to the emergency department because of difficulty breathing. He was in his state of usual health until 3 days ago, when he developed a fever (up to 39.4°C). Since then, other symptoms have developed, including decreased oral intake, irritability, and vomiting. He has had 1 wet diaper in the last 24 hours. His mother brought him in because of progressive lethargy, cough, and difficulty breathing. Vital signs show a temperature of 38.5°C, heart rate of 190 beats/min, respiratory rate of 50 breaths/min, blood pressure of 85/65 mm Hg, and SpO2 is 90% on room air. Physical examination reveals a generally lethargic child in moderate respiratory distress. Conjunctivae and oral mucous membranes are erythematous and dry. There is no cervical lymphadenopathy. He is tachypneic with use of accessory muscles and grunting. There is wheezing and scattered rales on auscultation. Cardiovascular examination reveals tachycardia, normal S1 and S2 with no rubs, gallops, or murmurs. His extremities are pale and cool, with capillary refill time approximately 4 seconds. Abdomen is soft with palpable spleen and liver edge 2 cm below the costal margin. He receives a 20 mL/kg intravenous fluid bolus of 0.9% saline. Subsequently, his respiratory distress worsens and his pallor increases. Repeat physical examination reveals development of jugular venous distention, and a palpable liver edge of now 4 cm below the costal margin. Of the following, the MOST likely diagnosis is A. bronchiolitis B. cardiogenic shock C. hypovolemic shock D. pneumonia E. status asthmaticus

D. trisomy 13 CHARGE syndrome is an autosomal dominant disorder that is characterized by coloboma, heart defects, choanal atresia, retarded growth and development, genital abnormalities, and ear anomalies. They will often have multiple life-threatening medical conditions. Meckel-Gruber syndrome is typically composed of the triad of an occipital encephalocele, large polycystic kidneys, and postaxial polydactyly. Some of these infants also have orofacial clefting, central nervous system malformations, genitourinary anomalies, and pulmonary hypoplasia, which is a leading cause of death in early infancy. PREP Pearls Of these infants, 82% die in the first month after birth. Most survive less than 15 days. One in 5 will survive for 3 months or more. Five percent to 10% will survive for 1 year or more. - Infants with trisomy 13 typically have polydactyly, cleft lip or palate, holoprosencephaly, scalp defects, and congenital heart defects. - Infants with trisomy 18 typically have clenched hands with overlapping fingers, increased muscle tone, micrognathia, low-set abnormal ears, congenital heart defects, small size for gestational age, single umbilical artery, short palpebral fissures, nail hypoplasia, and short sternum. - Most infants with trisomy 13 and 18 have high infant mortality rates because of their multiple congenital anomalies. ABP Content Specifications(s) - Recognize the clinical features associated with trisomy 13

A 28-year-old woman arrives at your local hospital in active labor. She did not receive any prenatal care. Multiple congenital anomalies are immediately noted upon delivery. Physical examination is remarkable for a female newborn with 2 oval regions of scalp, with noted absence of skin, bilateral cleft lip and palate, hypertelorism, small eyes, cyanosis with mottling, and polydactyly. Cardiac examination reveals a systolic ejection murmur at the upper left sternal border with a loud single S2. Pictures of the newborn (Item Q46A) and her hand (Item Q46B) are shown. Of the following, the MOST likely diagnosis is A. CHARGE syndrome B. holoprosencephaly C. Meckel-Gruber syndrome D. trisomy 13 E. trisomy 18

C. send gene testing for Wiskott-Aldrich Wiskott-Aldrich syndrome is an X-linked recessive disorder caused by mutations in the gene encoding the WAS protein (WASp). The clinical features include thrombocytopenia with small platelets (low mean platelet volume [MPV]), eczema, and immunodeficiency. PREP Pearls - Wiskott-Aldrich syndrome (WAS)-related disorders typically present in infancy with thrombocytopenia, bloody diarrhea, mucosal bleeding, petechiae, purpura, eczema, and recurrent infections, particularly otitis media. - Early diagnosis is important in managing infections and bleeding symptoms and preparing for possible bone marrow transplantation. - For patients with the classic WAS phenotype, bone marrow transplant is the only curative therapy. ABP Content Specifications(s) - Recognize the clinical findings associated with Wiskott-Aldrich syndrome

A 3-month-old boy is brought to your office for a rash and bloody diarrhea. This morning, the mother noticed a pinpoint red rash on his face and body and then saw streaks of blood in his stool. He drinks 24 oz to 32 oz of cow's milk formula per day. The baby is at 50th percentile for height and weight. His temperature is 38.2°C, pulse rate is 120 beats/min, respiratory rate is 24 breaths/min, and blood pressure is 90/55 mm Hg. On examination, the child is fussy but consolable. There are petechiae on his face and waist, and erythematous dry maculopapular patches on his cheeks and trunk. There is fluid behind an erythematous left tympanic membrane. There are no anal fissures visible. The remainder of the physical examination is unremarkable. The following are the results of the infant's laboratory tests: White blood cell count, 5,500/µL (5.5 × 109/L), with 40% polymorphonuclear leukocytes, 52% lymphocytes, 5% monocytes, and 3% eosinophils Hemoglobin, 13 g/dL (130 g/L) Mean corpuscular volume, 85 µm3 (85 fL) Platelet count, 45,000 × 103/µL (45 × 109/L) Mean platelet volume, 5 fL (reference range, 6 fL-10 fL) Of the following, the MOST appropriate next step in evaluating this patient is to A. refer for allergy skin testing B. refer for radioallergosorbent (RAST) testing C. send gene testing for Wiskott-Aldrich syndrome D. send platelet antibodies E. send serum complement levels

C. computed tomography scan of head The infant in this vignette has vomiting, fussiness, a high-pitched cry, a full fontanelle, and hyperventilation. These findings are most likely caused by an inflicted traumatic brain injury (iTBI). The cause is most likely inflicted because other accidental mechanisms for TBI are extremely uncommon in this age group, especially in the absence of a plausible history. The best diagnostic test to reveal the diagnosis is a computed tomography (CT) scan of the head. The most common cause of death from child abuse is iTBI, also known as abusive head trauma, nonaccidental head trauma, or shaken baby-impact syndrome. In the first year after birth, head injuries that do not have a witnessed plausible history are almost universally from abuse because infants in that age group have not attained the developmental level to sustain an accidental injury. Common signs and symptoms of iTBI include lethargy, irritability, vomiting, apnea, and hyperventilation. The hyperventilation is a compensatory mechanism to reduce intracranial pressure. The presentation may include other signs of abuse such as bruises, grab marks, or rib fractures, but isolated head injury is very common. Mechanisms of iTBI in infancy include tearing of the subdural bridging veins from rapid acceleration-deceleration caused by shaking and direct impact to the head. Most of the signs and symptoms of the presentation of a child with iTBI are nonspecific, such as irritability, disordered breathing, vomiting, and disinterest in feeding. The differential diagnosis for fussiness in this age group includes sepsis, meningitis, hydrocephalus, congenital heart disease, corneal abrasion, and hair tourniquet. Colic can also be a cause of fussiness, but generally occurs around 4 to 6 weeks of age, and is a diagnosis of exclusion. A high index of suspicion for iTBI must be maintained because of a nonspecific presentation and a large differential diagnosis. Not only is it a serious condition, but missing the diagnosis can also render the patient and other children in the house vulnerable to subsequent abuse. The radiographic test of choice for acute iTBI is CT (Item C137A, Item C137B). Subdural, epidural, subarachnoid, and intraparenchymal hemorrhages can all occur in iTBI and can be detected on CT. Injuries of various ages can be seen if the abuse has occurred over time. Head ultrasonography can be performed in a child with an open fontanelle to detect bleeding and hydrocephalus, but it is not as sensitive as CT. Magnetic resonance imaging is very sensitive and specific in detecting intracranial pathology. Magnetic resonance imaging requires the patient to be motionless for a long duration and thus requires deep sedation and usually endotracheal intubation, and it is not as practical as CT in the acute setting in this age group. Neurosurgery and critical care consultations should be obtained for management of head injury. Evaluation for other signs of trauma is also necessary. An ophthalmology consultation is warranted to evaluate for retinal hemorrhages (Item C137C). Although retinal hemorrhages can be seen in other conditions, injury to more than 1 layer of the retina and traumatic retinoschisis are pathognomonic of abuse, especially in the context of other signs of abuse. A skeletal survey should be obtained to evaluate for recent and old fractures. Rib fractures are commonly associated with shaking injuries. A trauma and surgery consultation should also be considered if there is suspicion of non-neurologic injury. Abdominal radiography can be part of the workup for vomiting and irritability, but is not likely to contribute to a diagnosis of trauma. Chest radiography can reveal rib fractures, but are sometimes not seen in iTBI. Electrocardiography can be helpful in detecting myocardial infarction in anomalous left coronary artery from pulmonary artery, which is a condition that can cause irritability in this age group, but occurs much less frequently than iTBI. Lumbar puncture can be part of the workup of intracranial pressure, but only after obstructive hydrocephalus has been ruled out with CT. Inflicted traumatic brain injury is a condition that can manifest with nonspecific signs and symptoms in infancy, including fussiness, altered mental status, vomiting, and breathing difficulties. Computed tomography of the head is the initial diagnostic study of choice. Retinal, chest, skeletal, and abdominal trauma should also be evaluated. Police, social work, and child protection medical teams should also be involved to investigate the crime and protect other involved individuals. PREP Pearls - Inflicted traumatic brain injury should be considered in the differential diagnosis of an infant with irritability or altered mental status. Computed tomography of the head is the diagnostic study of choice. - Inflicted traumatic brain injury can be isolated without any other obvious signs of trauma. - Infants younger than 1 year of age rarely have an accidental cause of traumatic brain injury without a plausible history. ABP Content Specifications(s) - Recognize the clinical findings associated with non-accidental traumatic brain injury

A 3-month-old male infant is brought to the emergency department by his mother because of vomiting, difficulty breathing, and fussiness. He was in his usual state of good health in the morning when she left for work. When she returned, the baby was not interested in taking the bottle, and he vomited what little he took. She was also concerned with his breathing pattern, which was faster than usual. Vital signs show a temperature of 37°C, heart rate of 120 beats/min, respiratory rate of 50 breaths/min, and blood pressure of 80/40 mm Hg. Physical examination reveals a fussy infant with a high-pitched cry. He is moving all extremities equally. Pupils are 4 mm, equal, and sluggishly reactive. Anterior fontanelle is full. Cardiovascular examination reveals regular rate and rhythm, and warm and well-perfused extremities, with capillary refill of 2 s. Lung examination shows a breathing pattern that is fast and deep, is clear to auscultation, and has good air movement bilaterally. Abdomen is soft, nontender, nondistended, and with no organomegaly. Skin examination is unremarkable. Of the following, the MOST likely to reveal the diagnosis is A. abdominal radiograph B. chest radiograph C. computed tomography scan of head D. electrocardiogram E. lumbar puncture

B. administer 20 mL/kg 0.9% NaCl IV bolus The infant in the vignette is in compensated hypovolemic shock. It is very important to distinguish this entity from simple dehydration. Evidence of shock in this patient, as opposed to dehydration, includes listlessness, tachycardia, cool extremities, and delayed capillary refill. The patient's blood pressure is normal, which reflects the body's compensatory mechanisms in shock, including elevated endogenous catecholamines, vasoconstriction, and diversion of blood away from some vascular beds such as the skin and splanchnic circulation. (1) Cardiogenic shock is caused by inadequate myocardial function or structural heart disease. This can present with signs and symptoms of right or left ventricular failure, including tachypnea, pulmonary edema, rales, hepatosplenomegaly, ascites, jugular venous distention, and peripheral edema. The murmur noted is likely caused by turbulent flow in a hyperdynamic heart, and has characteristics of an innocent "flow" murmur. Thus, echocardiography is unlikely to benefit the patient. (2) Obstructive shock is caused by a mechanical barrier that impairs cardiac output, for example, right or left ventricular outflow tract obstructions, pericardial tamponade, tension pneumothorax, or pulmonary embolism. (3) Distributive shock occurs because of inadequate intravascular volume caused by increased capillary permeability. (4) Septic shock is caused by an infection that leads to a combination of decreased myocardial function, increased capillary permeability, and decreased intravascular volume. PREP Pearls - Early goal-directed therapy aimed at restoring cardiac output should be immediately initiated in all patients with shock - There are 5 forms of shock: hypovolemic, cardiogenic, distributive, obstructive, and septic. - Oral rehydration therapy should be attempted in most patients with dehydration, unless they are in shock. ABP Content Specifications(s) - Recognize the clinical signs of shock due to fluid loss, and manage appropriately

A 3-month-old previously healthy female infant presents to the emergency department with 2 days of decreased oral intake and nonbloody, nonbilious vomiting. She was in her usual state of health prior to this illness. She usually drinks 6 oz bottles 5 times a day, but during the last 2 days, she has vomited after each feed. She has had only 1 wet diaper in the last 12 hours. She has been increasingly disinterested while feeding, and is becoming more listless. She does not have fever, upper respiratory symptoms, rash, difficulty breathing, or diarrhea. She is a product of a full-term, normal, spontaneous vaginal delivery without complications. She does not have allergies and she has received her 2-month immunizations. Development is normal for age. Vital signs show a temperature of 37°C, pulse rate of 180 beats/min, respiratory rate of 35 breaths/min, blood pressure of 80/40 mm Hg, and oxygen saturation of 99% on room air. Physical examination shows a generally well-developed, but lethargic baby. Her anterior fontanelle and eyes are sunken, and her oral mucous membranes are dry. Tympanic membranes are clear, and her throat is not erythematous. She is breathing without difficulty, and her lungs are clear to auscultation bilaterally. Heart sounds are regular, with a low-pitched, grade 2/6 systolic murmur. Abdomen is soft, nontender, nondistended, and with no organomegaly. Extremities are cool, with capillary refill time approximately 4 seconds. Of the following, the MOST appropriate initial step is to A. administer 4 oz of oral rehydration solution B. administer 20 mL/kg 0.9% NaCl IV bolus C. administer ceftriaxone 50 mg/kg IM D. obtain abdominal ultrasonography E. obtain echocardiogram

B. magnetic resonance imaging of the spine The child in the vignette has signs and symptoms consistent with discitis. Magnetic resonance imaging of the spine is the most sensitive and specific test to confirm the diagnosis. Early in the disease, plain radiographs are typically normal, whereas later in the disease course the disc space may have some narrowing. A technetium bone scan would show signs of inflammation, but would not be specific for this diagnosis. Computed tomography of the spine will show discitis earlier than plain radiography and can augment magnetic resonance imaging by helping to distinguish soft tissue from bone. Discitis, or infection of the intervertebral disc, is thought to be caused by hemotogenous spread of infection; however, no consistent primary infection source has been identified in children. Discitis is rare and usually presents in children younger than 5 years. Most children with discitis will be afebrile or have a low-grade fever. They present with a limp or refusal to walk, back pain, and irritability. The lumbar region is most often affected, and most patients will have localized tenderness. Inflammatory markers and white blood cell counts may be increased, however these test results are not specific. In the majority of cases, blood cultures are negative. Other cultures, such as urine culture, should be considered to help identify possible sources of infection. The most common causative organism is Staphylococcus aureus. Treatment consists of immobilization and 4 to 6 weeks of antibiotics for children with positive cultures, elevated erythrocyte sedimentation rates, or elevated white blood cell counts. For patients with normal laboratory studies, the length and route of treatment is more controversial. Surgery is rarely indicated. Computed tomography of the abdomen would not be appropriate in this case, because the boy in the vignette has findings that localize to the spine. Plain radiography of the pelvis and hips would not be appropriate in this patient with a normal lower extremity examination and no pain with movement or palpation of the lower extremities. Ultrasonography of the abdomen and kidneys is not indicated based on history or physical examination. He has no urinary tract symptoms, a normal urinalysis, and basic metabolic panel. PREP Pearls - Children with discitis present with limping or refusal to walk, back pain, and are often afebrile. - Discitis occurs most often in children younger than 5 years. - Magnetic resonance imaging is the most sensitive and specific test for diagnosing discitis. - Staphylococcus aureus is the most common causative organism in discitis. ABP Content Specifications(s) - Formulate a differential diagnosis of back pain in children of various ages

A 3-year-old boy is brought to the emergency department for evaluation of a limp. The family states that the patient has been crying each of the last 3 evenings. This morning, the mother noticed the patient was limping and has been very irritable. On physical examination, the child's temperature is 38°C. He is fussy and appears ill. The patient withdraws when you palpate along the spine. The child walks with a nonspecific limp, and he flexes his knees, keeping his spine straight, to retrieve a toy he has dropped. His lower extremities are not tender to palpation and appear normal. Plain radiographs of the spine are normal. Laboratory evaluation demonstrates a white blood cell count of 18,000/µL (18 × 109/L), erythrocyte sedimentation rate of 30 mm/h, and a C-reactive protein level of 3.0 mg/L (28.67 nmol/L). Results of a basic metabolic panel and urinalysis are unremarkable. Blood and urine cultures are pending. Of the following, the BEST next test to diagnose this patient's condition is A. computed tomography of the abdomen B. magnetic resonance imaging of the spine C. plain radiography of the pelvis and hips D. technetium bone scan E. ultrasonography of the abdomen and kidneys

A. admit and consult otolaryngology for tympanocentesis with culture The physical examination findings of erythema, swelling, and pain over the mastoid bone with displacement of the pinna outwardly should always raise concern for the possibility of acute mastoiditis with or without acute or chronic otitis media. The diagnosis of mastoiditis warrants prompt clinical and surgical evaluation and the initiation of broad-spectrum antimicrobial therapy; therefore, the girl in the vignette should be admitted to the hospital and otolaryngology consulted. Antimicrobial therapy should be guided by the results of culture and sensitivity obtained via tympanocentesis. A history of recurrent otitis media is a risk factor for mastoiditis. Mastoiditis occurs most often in children younger than 2 years of age. The incidence of mastoiditis has decreased dramatically with the widespread treatment of otitis media with antibiotics. Mastoiditis has several serious potential complications related to direct extension and rarely hematogenous spread of infection, so prompt diagnosis and treatment are imperative. Extracranial complications of mastoiditis (and otitis media) include cholesteatoma, subperiosteal abscess, facial nerve palsy, hearing loss, labyrinthitis, osteomyelitis, and Bezold abscess of the sternocleidomastoid muscle. Intracranial complications of mastoiditis (and otitis media) are uncommon and include meningitis, brain abscess, epidural or subdural empyema, and carotid artery and venous sinus thrombosis. The treatment of acute mastoiditis depends on the extent of disease and presence or absence of complications. The cornerstones of therapy for acute mastoiditis are antimicrobial therapy and drainage of the middle ear and mastoid. Obtaining material for culture and sensitivity from the middle ear is paramount. Applying warm compresses to the postauricular area, treating with ceftriaxone with follow-up in 3 days, and treating with oral levofloxacin are not aggressive enough treatment in the case of probable mastoiditis in an ill-appearing young child. When children present with the characteristic clinical findings of acute mastoiditis, imaging is not always necessary. When imaging is needed, computed tomography, rather than plain skull films, is the study of choice for diagnosis and determining extension of disease. If intracranial extension is a concern, then magnetic resonance imaging should be performed. The bacteria most likely to cause mastoiditis in children include Streptococcus pneumoniae (including multidrug-resistant S pneumoniae), Streptococcus pyogenes, and Staphylococcus aureus (including methicillin-resistant S aureus). Chronic mastoiditis is often polymicrobial, including anaerobes and gram-negative bacilli, particularly Pseudomonas, Escherichia coli, and Klebsiella. Pseudomonas aeruginosa should be considered as a possible pathogen in children with a history of recurrent otitis media and recent antibiotic use, such as the patient described in the vignette. P aeruginosa most commonly causes infection in the setting of tympanic membrane perforation and chronic middle ear infection. Patients with acute mastoiditis should be admitted and parenteral empiric broad-spectrum antibiotics directed against these common pathogens should be initiated. Myringotomy or tympanocentesis with aspiration and drainage of the middle ear for diagnostic and therapeutic purposes is needed. If patients do not respond promptly (within 24 to 48 hours) to this conservative approach, then further surgical intervention is warranted. The culture and sensitivity results of material obtained via tympanocentesis or surgical drainage of the mastoid will further direct antibiotic management. For patients with central nervous system (CNS) complications, it is imperative that bactericidal agents, with adequate CNS penetration, are selected. PREP Pearls - The cornerstones of therapy for acute mastoiditis are directed antimicrobial therapy and drainage of the middle ear and mastoid. - The bacteria most likely to cause acute mastoiditis in children include Streptococcus pneumoniae, Streptococcus pyogenes, and Staphylococcus aureus. Pseudomonas aeruginosa should be considered as a possible pathogen in children with a history of recurrent otitis media and recent antibiotic use. - Several serious potential complications related to direct extension, or rarely, hematogenous spread of infection from mastoiditis can occur. These include extracranial complications such as cholesteatoma, subperiosteal abscess, facial nerve palsy, hearing loss, labyrinthitis, osteomyelitis, and Bezold abscess (involving the sternocleidomastoid muscle) and intracranial complications such as meningitis, brain abscess, epidural or subdural empyema, and carotid artery and venous sinus thromboses. ABP Content Specifications(s) - Plan the appropriate diagnostic evaluation of mastoiditis Recognize the clinical findings associated with mastoiditis - Identify the etiology of mastoiditis - Plan the appropriate management of mastoiditis

A 3-year-old girl comes into your office for evaluation of fever and ear pain for 2 days. When she awoke this morning, her parents noted a "bump" behind her right ear. The patient has a history of recurrent ear infections and she had tympanostomy tubes placed when she was 18 months of age. She has been receiving a "pink antibiotic" for the last several weeks to prevent infections because the tubes have fallen out. On physical examination, her temperature is 39°C, pulse rate is 100 beats/min, respiratory rate is 16 breaths/min, and blood pressure is 94/65 mm Hg. Growth parameters are at the 25th percentile for age. She is ill-appearing, but alert and interactive during the examination. The right tympanic membrane is erythematous and dull appearing. The right postauricular area is swollen, erythematous, and tender to touch. Her right pinna is displaced outwardly. The remainder of the physical examination findings, including neurologic assessment, are unremarkable. Of the following, the MOST appropriate next step in management is to A. admit and consult otolaryngology for tympanocentesis with culture B. apply warm compresses to the postauricular area C. give ceftriaxone and see the patient back in 3 days D. obtain plain skull radiographs E. prescribe levofloxacin orally and see the patient back in 5 days

A. acute lymphoblastic leukemia The child in the vignette who presents with pancytopenia, bone pain, fever, and organomegaly is most likely to have acute lymphocytic leukemia (ALL). The differential diagnoses for new-onset pancytopenia are numerous, including malignancy, bone marrow failure, autoimmune disorders, infection, or peripheral destruction of hematopoietic cells. When more than 1 blood cell type is affected, an examination of the bone marrow by aspiration and biopsy may be warranted. The most common malignancy in children is leukemia; ALL accounts for 80% of all childhood leukemias. There are approximately 3,000 new cases of childhood ALL per year in the United States. The peak age at diagnosis is between 2 and 5 years of age. The presentation can be variable; from an incidental finding on routine laboratory testing in an asymptomatic child to a life-threatening hemorrhage or respiratory distress (because of a mediastinal mass). The most common presenting clinical features are fever, pallor, bruising, petechiae, bone pain, hepatosplenomegaly, and lymphadenopathy. Anemia, thrombocytopenia, and an abnormal leukocyte and differential count are common, but may not be present at the time of diagnosis despite the leukemic infiltration of the bone marrow. Approximately 20% of cases of childhood ALL present with leukocytosis with a white cell count greater than 50,000/μL (50 × 109/L). In patients with leukopenia, however, blasts may not be seen on routine peripheral blood smears. Aplastic anemia is much less common in children, with peaks at 10 to 25 years of age and after 60 years of age. The diagnosis of acquired aplastic anemia (AA) requires the presence of 2 of the following: hemoglobin less than 10 g/dL (100 g/L), platelet count less than 50 × 103/μL (50 ×109/L), and absolute neutrophil count less than 1,500/μL (1.5 × 109/L). Classification of the severity of AA is based on bone marrow cellularity. Most cases of acquired AA are idiopathic. The anemia is usually macrocytic with reticulocytopenia. A detailed history should be obtained and a careful physical examination performed to distinguish between acquired AA and inherited bone marrow failure syndromes, such as Fanconi anemia, dyskeratosis congenita, Shwachman-Diamond syndrome, and congenital amegakaryocytic thrombocytopenia. In contrast to ALL, hepatosplenomegaly and bone pain are not usually seen in patients with AA. Pancytopenia and pain can be seen in juvenile idiopathic arthritis, but hepatosplenomegaly would not be typical. One might also expect to see a higher erythrocyte sedimentation rate in this type of inflammatory disease. Bone diseases such as osteomyelitis and osteosarcoma could present with bone pain and refusal to walk, and possibly pancytopenia. Neither of these conditions would explain the hepatosplenomegaly. Furthermore, the fever would most likely be persistent in untreated osteomyelitis. Osteosarcoma is the most common pediatric bone cancer, with 400 new cases per year in children in the United States. The mean age at diagnosis of osteosarcoma is in adolescence during the second decade of life, which is thought to correlate with the period of most rapid bone growth; therefore, osteosarcoma would be a less likely diagnosis in a 3-year-old child than leukemia. PREP Pearls - The most common presenting clinical features of acute lymphocytic leukemia in children are fever, pallor, bruising, petechiae, bone pain, hepatosplenomegaly, and lymphadenopathy. - Anemia, thrombocytopenia, and an abnormal white blood cell (WBC) and differential count are common, but may not be present at the time of diagnosis despite the leukemic infiltration of the bone marrow. - Approximately 20% of cases of childhood acute lymphocytic leukemia (ALL) present with leukocytosis with WBC counts greater than 50,000/µL (50 × 109/L); patients with ALL who present with leukopenia may not have blast cells on routine peripheral blood smears. - The differential diagnoses for new-onset pancytopenia include malignancy, bone marrow failure, autoimmune disorders, infection, or peripheral destruction of hematopoietic cells. ABP Content Specifications(s) - Distinguish acquired aplastic anemia from childhood leukemia - Plan the appropriate diagnostic evaluation of multiple pancytopenias

A 3-year-old girl presents to your clinic for leg pain. The mother states that 3 weeks ago, her daughter had a fever to touch that resolved spontaneously after a few days; however, the child has been more tired since then. Over the past week, the child has been refusing to walk and asking to be carried. In the office, her oral temperature is 38.3°C, pulse rate is 90 beats/min, respiratory rate is 24 breaths/min, and blood pressure is 100/60 mm Hg. On examination, the child is alert, playful, and in no apparent distress. The edge of her spleen is palpable 4 cm below the left costal margin. She has anicteric sclerae and shotty lymphadenopathy. There is no deformity, swelling, or tenderness of either leg. The remainder of the physical examination is unremarkable. The following are the results of the child's complete blood cell count: - White blood cell count, 3,500/µL (3.5 × 109/L), with 15% polymorphonuclear leukocytes, 75% lymphocytes, 8% monocytes, and 2% eosinophils - Hemoglobin, 9.0 g/dL (90 g/L) - Mean corpuscular volume, 85 µm3 (85 fL) - Platelet count, 45,000 × 103/µL (45 × 109/L) - Reticulocyte count, 0.5% - Erythrocyte sedimentation rate, 25 mm/h (0 mm/h-20mm/h) Of the following, the MOST likely diagnosis in this child is A. acute lymphoblastic leukemia B. aplastic anemia C. juvenile idiopathic arthritis D. osteomyelitis E. osteosarcoma

C. start oral ferrous sulfate Data suggest that untreated iron deficiency, with or without anemia, may have an irreversible detrimental effect on neurocognitive development. In evaluating for the cause of anemia in a child, it can be useful to categorize the differential diagnoses based on the red blood cell (RBC) size. - Anemia with a low mean corpuscular volume (MCV), that is, microcytic anemia, can be seen in iron deficiency anemia, lead poisoning, thalassemia, sideroblastic anemia, or chronic inflammation. - Anemia with a normal MCV can be seen in hemolytic anemia, acute blood loss, or chronic renal disease. - Anemia with an increased MCV is often seen in vitamin B12 deficiency, folic acid deficiency, aplastic anemia, Diamond-Blackfan anemia (pure RBC aplasia), hypothyroidism, liver disease, or bone marrow infiltration. Excessive ingestion of cow's milk, which contains only a minimal amount of iron, can lead to iron deficiency by decreasing the absorption from other foods and by causing small amounts of gastrointestinal blood loss. PREP Pearls - Iron deficiency is the most common cause of anemia worldwide in children younger than 5 years of age, with the highest prevalence in children 2 to 5 years of age. - Treatment of iron deficiency anemia is oral iron supplementation at a dose of 3 to 6 mg/kg per day of elemental iron. It may take 3 to 6 months to replenish iron stores. - Gastrointestinal iron absorption is enhanced by ingestion on an empty stomach, accompanied by ascorbic acid. -- - Concomitant use of prescription or over-the-counter antacid therapies may impair absorption of iron. ABP Content Specifications(s) - Plan the appropriate management of severe anemia of various etiologies, while considering the risks associated with various therapies

A 3-year-old girl presents to your clinic for pallor. The mother reports that for 2 days last week her daughter had a fever and cough, which have now resolved. The child is now tolerating her usual diet, which includes 32 to 40 oz of cow's milk per day. The mother states the child has looked pale over the last several days. Her temperature is 37.0°C, pulse rate is 90 beats/min, respiratory rate is 24 breaths/min, and blood pressure is 100/60 mm Hg. She is at the 95th percentile for weight and 50th percentile for height. On examination, the child is alert, playful, and in no apparent distress. She has anicteric sclerae and no hepatosplenomegaly or lymphadenopathy. The remainder of the physical examination is unremarkable. The following are the results of the child's laboratory tests: Complete blood work: - White blood cell count, 7,500/µL (7.5 × 109/L), with normal differential - Hemoglobin, 6.0 g/dL (60 g/L) - Mean corpuscular volume, 59 µm3 (59 fL) - Platelet count, 495,000 × 103/µL (495 × 109/L) - Reticulocyte count, 0.5% Of the following, the MOST appropriate next step in management is to A. infuse intravenous iron dextran B. reassure that the anemia is postinfectious C. start oral ferrous sulfate D. start oral prednisone E. transfuse packed red blood cells

C. intravenous 5% dextrose and 0.45% sodium chloride at 25 mL/h The progressive nonbilious vomiting and hypochloremic metabolic alkalosis described for the infant in the vignette suggest the presence of a gastric outlet obstruction. Infantile hypertrophic pyloric stenosis (HPS) is the most likely cause of such clinical findings. The initial goals of therapy are assessment of hydration status and institution of appropriate intravenous rehydration, with gradual correction of any electrolyte disturbances. Only when hydration status has normalized and electrolyte abnormalities have been corrected should surgical pyloromyotomy be undertaken. Given this infant's birth weight of 3,250 g and an expected mean weight gain of 20 g/day, the current weight should be approximately 3,750 g. The infant's actual weight of 3,550 g likely indicates mild dehydration (~5%). Based on the serum electrolyte data, fluid therapy should be initiated with 5% dextrose and 0.45% sodium chloride at approximately 1.5 times the calculated maintenance rate (~600 mL per 24 hours, based on the infant's hydrated weight) or 25 mL/h. Oral rehydration therapy, including breast milk and glucose electrolyte solutions, is not recommended in this infant whose serum electrolyte values indicate metabolic derangements consistent with HPS. The infant in the vignette is estimated to be only 5% dehydrated, therefore vigorous fluid management with a bolus infusion is not required. In most cases of HPS, appropriate initial intravenous fluid therapy involves a sodium chloride concentration between 0.45% and 0.9%, depending on the magnitude of the electrolyte deficits. Intravenous therapy with concentrations of less than 0.45% sodium chloride should be avoided, unless the condition is diagnosed early in the disease course and all electrolyte and acid-base values are normal. Potassium should be added to the infusate as soon as urinary output is established to prevent progressive hypokalemia. However, because the serum potassium concentration rises as the serum pH normalizes, potassium levels must be carefully monitored. Bolus infusions with 10 to 20 mL/kg 0.9% sodium chloride are reserved for infants who demonstrate clinically moderate-to-severe intravascular fluid depletion (ie, >7% dehydration). Hypertrophic pyloric stenosis is the most frequent cause of metabolic alkalosis in the first few weeks after birth and the most common indication for abdominal surgery, with an estimated incidence of 1 in 250 live births. The condition occurs more commonly in white males, but greater risk of occurrence in first-born male children has not been proven. Familial clustering has been reported. Other, less common causes of gastric outlet obstruction include duodenal stenosis, gastric duplication, antral web, and annular pancreas. Although hypochloremic, hypokalemic metabolic alkalosis represents the classic electrolyte and acid-base imbalances of HPS, many infants have normal serum electrolyte values, therefore increasing the importance of having a "high index of suspicion" for this diagnosis. The magnitude of any acid-base and electrolyte disturbance is related directly to the duration of vomiting. Persistent emesis results in a progressive loss of gastric fluid and hydrochloric acid. Hypokalemia ensues as the kidneys retain hydrogen ions in favor of potassium. As extracellular fluid volume and hydrogen ions continue to be lost because of continued postprandial emesis and intravascular volume contracts, increasing the relative serum bicarbonate concentration leads to a contraction alkalosis. Serum potassium values fall further as alkalosis stimulates potassium-hydrogen exchange across cell membranes, with potassium driven intracellularly and hydrogen ions driven extracellularly. PREP Pearls - Hypochloremic, hypokalemic metabolic alkalosis represents the classic electrolyte and acid-base imbalances of hypertrophic pyloric stenosis (HPS). In the fluid management of HPS, infused sodium concentrations of less than 0.45% should be avoided. - Serum potassium levels will rise in patients with alkalosis as the blood pH returns to normal. - Bolus intravenous fluid infusions should not be utilized in patients presenting with dehydration levels less than or equal to 5%. ABP Content Specifications(s) - Recognize the acid-base changes associated with pyloric stenosis, and manage appropriately

A 3½-week-old neonate is seen in your office for vomiting. He was born at term and his birth weight was 3,250 g. He is breastfed. For the past 2 to 3 days, he has been exhibiting nonbilious vomiting after each feeding. After vomiting, he seems eager to resume nursing. Over the past 24 hours, his mother has noted fewer wet diapers and less stool than usual. The baby has experienced no fever, diarrhea, or upper respiratory tract symptoms. Physical examination demonstrates an alert infant sucking vigorously on a pacifier. His weight is 3,550 g, temperature is 37°C, pulse rate is 160 beats/min, respiratory rate is 40 breaths/min, and blood pressure is 70/50 mm Hg. His skin turgor is normal. The remainder of the examination findings are unremarkable except for slight abdominal distention. You obtain the following laboratory studies: Sodium, 130 mEq/L (130 mmol/L) Potassium, 3.2 mEq/L (3.2 mmol/L) Chloride, 95 mEq/L (95 mmol/L) Bicarbonate, 30 mEq/L (30 mmol/L) Blood urea nitrogen, 15 mg/dL (5.4 mmol/L) Creatinine, 0.2 mg/dL (18 µmol/L) Of the following, the MOST appropriate initial treatment for this infant is A. continued nursing, pending results of further diagnostic tests B. intravenous 5% dextrose and 0.3% sodium chloride at 15 mL/h C. intravenous 5% dextrose and 0.45% sodium chloride at 25 mL/h D. intravenous 0.9% sodium chloride, 70 mL over 1 hour, followed by infusion with 5% dextrose and 0.3% sodium chloride at 15 mL/h E. oral rehydration with a glucose-electrolyte solution containing 70 mEq to 90 mEq (70-90 mmol/L) of sodium chloride per liter

D. serum lipids The presentation of a child with symptoms of severe abdominal pain, vomiting, and antalgic knee-chest position (assumed to relieve pressure on the pancreas) in association with marked pancreatic enzyme elevations are pathognomonic signs of acute pancreatitis. Her 12-month history of apparently similar, yet less severe episodes, suggests a chronic, relapsing problem. Recurrent pancreatitis is the second most common pancreatic disorder, after cystic fibrosis, in the pediatric age group. The strong family history of premature atherosclerotic coronary artery disease indicates that a familial dyslipidemia may be a potential cause for her illness. Accordingly, the clinician should obtain a serum lipid profile. Symptoms in the child with acute pancreatitis include abdominal pain (90% of patients), vomiting (> 60%), abdominal tenderness (80%), and abdominal distention (20%). These symptoms often increase in severity with meals. Fever, tachycardia, hypotension, jaundice, abdominal guarding, and rebound tenderness occur less frequently and herald a more severe presentation. When a diagnosis of pancreatitis is suspected, serum amylase and lipase levels should be measured and are always increased. However, the magnitude of the enzyme elevation does not always correlate with the severity of pancreatic injury. Abdominal ultrasonography is not required to establish a diagnosis of pancreatitis, though this test may be helpful in assessing the degree of pancreatic edema as well as the presence of peripancreatic fluid. Magnetic resonance cholangiopancreatography and the emerging technique of endoscopic ultrasonography are useful studies in evaluating possible gallstone pancreatitis, but the absence of other signs of cholestasis (eg, jaundice) makes this diagnosis unlikely. Hypercalcemia, particularly in the setting of hyperparathyroidism, is a known cause of acute and recurrent pancreatitis, but the family history and presentation of the girl in the vignette strongly suggest a lipid metabolic disorder. Acute, sporadic pancreatitis is more common than recurrent, chronic pancreatitis during childhood and adolescence. Most cases of acute, sporadic pancreatitis are caused by blunt abdominal trauma or are classified as "idiopathic." Causes of chronic, relapsing pancreatitis include infections, autoimmune disorders, genetic conditions, and drugs. Pancreatitis is a common complication of lipoprotein metabolic disorders that are associated with hypertriglyceridemia. Of the heritable hyperlipidemias, familial lipoprotein lipase (LPL) deficiency and type V hyperlipidemia are the disorders most frequently associated with chronic relapsing pancreatitis. The LPL enzyme is responsible for intravascular hydrolysis of dietary triglycerides carried in chylomicrons. Affected patients have functionally absent LPL and manifest grossly lipemic plasma in the fasting state, with plasma triglyceride measurements in excess of 1,500 mg/dL (17.0 mmol/L) (and as high as 25,000 mg/dL [282.5 mmol/L]). Hypercholesterolemia is another characteristic finding, with total plasma cholesterol concentrations approximating 10% to 20% of the triglyceride value. Familial type V hyperlipidemia also is characterized by extremely high triglyceride concentrations, typically ranging from 500 to 10,000 mg/dL (5.7-113.0 mmol/L), and presents with elevations in both chylomicron and low-density lipoprotein concentrations. The most common cause of recurrent pancreatitis in the United States is hereditary pancreatitis of which 2 types have been described. The molecular defects for both types I and II involve mutations in the gene on chromosome 7q35 that codes for cationic trypsinogen, and both types are transmitted as autosomal dominant disorders with 80% penetrance. Gene mutations allow for autoactivation of trypsinogen in the pancreas instead of in the duodenum, leading to intrapancreatic activation of other enzymes and subsequent pancreatic autodigestion and inflammation. A sweat chloride test may be helpful in the diagnosis of cystic fibrosis (CF). In CF, mutation in the cystic fibrosis transmembrane conductance regulator (CFTR), which serves as a chloride channel, results in reduced chloride and water secretion. The consequence is production of abnormally thick, viscid mucus by all exocrine organs, including the pancreas, where the CFTR is located on the apical membrane of pancreatic ductal epithelium. In addition to causing exocrine pancreatic fibrosis and insufficiency, ductal plugging leads to 1 or more episodes of pancreatitis in a small percentage (< 5%) of those who have CF. PREP Pearls - In the child with recurrent pancreatitis, a careful family history will often suggest a likely cause. - Lipoprotein lipase deficiency and type V hyperlipidemia are commonly associated with recurrent pancreatitis. Recurrent pancreatitis may occur in the setting of cystic fibrosis, with or without a history of pancreatic insufficiency. - Although increased serum amylase and lipase levels are the most characteristic laboratory findings in pancreatitis, the magnitude of these enzyme elevations do not correlate with the degree of pancreatic injury. ABP Content Specifications(s) - Formulate a differential diagnosis for a patient who has chronic or recurrent pancreatitis - Recognize the clinical and laboratory features associated with pancreatitis

A 4 year-old girl has a 12-month history of intermittent, poorly localized abdominal pain. She now presents with a recurrence of crampy abdominal pain. She has vomited after each meal for the past 24 hours. She has no history of fever or diarrhea. The child woke up frequently during the previous night because of the pain. Her parents report that she seems more comfortable lying on her side in a knee-chest position. The family history is negative for gastrointestinal disease. The child's grandfather died of a heart attack at 55 years of age, and her father, age 40, underwent a coronary artery bypass procedure last year. Her height is 97 cm (25th percentile), weight is 14 Kg (10th percentile), temperature is 37.2°C, pulse rate is 130 beats/min, respiratory rate is 25 breaths/min, and blood pressure is 90/60 mm Hg. The girl is difficult to examine because of her abdominal pain. Her lungs are clear to auscultation and her cardiac examination is normal. She complains of severe pain wherever her abdomen is palpated. Initial laboratory data include the following: White blood cell count, 10.4 × 103/µL (10.4 × 109/L) Hemoglobin, 12.5 g/dL (125 g/L) Sodium, 135 mEq/L (135 mmol/L) Chloride, 100 mEq/L (100 mmol/L) Potassium, 4.5 mEq/L (4.5 mmol/L) Amylase, 240 U/L (normal <80 U/L) Lipase, 700 U/L (normal <70 U/L) Aspartate aminotransferase, 60 U/L (reference range, 20-50 U/L) Alanine aminotransferase, 70 U/L (reference range, 20-60 U/L) Of the following, the test MOST likely to demonstrate the underlying cause of this girl's illness is A. abdominal ultrasonography B. magnetic resonance cholangiopancreatography C. serum calcium D. serum lipids E. sweat chloride

C. furosemide The infant in this vignette most likely has congestive heart failure (CHF) and the best next step in management is to administer a diuretic such as furosemide. The presentation of CHF varies with age. Newborns who present with CHF may be very ill, as they frequently will have a systemic circulation that is ductal dependent. An example of this would be a newborn with hypoplastic left heart syndrome. This patient needs a patent ductus arteriosus for systemic circulation. If the pulmonary vascular resistance (PVR) drops for any reason (such as the patient receiving oxygen), then increased flow will flood the infant's lungs and cause respiratory symptoms and worsening CHF. Infants at age 4 to 8 weeks may present with symptoms of CHF when the PVR drops naturally. This may be difficult to distinguish from primary respiratory problems, especially during the bronchiolitis season. Respiratory distress is the typical presentation of an infant with a ventriculoseptal defect (VSD). If the VSD is small, a loud holosystolic murmur will be heard, and if it is very large, only a murmur of increased flow through the pulmonary valve may be appreciated. Infants may also present with poor weight gain and failure to thrive. Of patients with trisomy 21, 40% to 50% will have congenital heart disease. Of those infants, 40% to 50% will have an atrioventricular canal-type defect. Patients with this lesion have a distinctive electrocardiogram, as well as cardiomegaly on chest radiograph. The physical examination in this patient will also show increased work of breathing. Children older than 1 year of age who present with heart failure are less likely to have undiagnosed congenital heart disease if they have had access to regular medical care. In this age group, it is more likely that the patient has acquired heart disease or cardiomyopathy. Myocarditis, often viral, can have an acute presentation with a shock-like syndrome. If a gradual decrease in function is seen, as with inheritable types of dilated cardiomyopathy, children will adapt and may not complain of decreased exercise tolerance. Patients with known congenital heart disease who have had previous surgical procedures and have developed ventricular dysfunction may also present with decreased exercise tolerance at any age. For the infant in the vignette, furosemide is the treatment of choice. It will alleviate the pulmonary congestion caused by circulatory overload. The infant in the vignette has physical examination findings consistent with an atrioventricular canal defect commonly seen with trisomy 21. Her murmur also suggests a patent ductus arteriosus, which would contribute to her symptoms of heart failure. High flow oxygen would make the left-to-right shunt worse in both lesions. Continuous albuterol would increase the heart rate and myocardial oxygen demand, ceftriaxone would not treat the heart failure symptoms, and methylprednisolone would be used to treat reactive airway disease, which is unlikely in this situation. As this infant has CHF, furosemide would be the first intervention. PREP Pearls - Furosemide is the treatment of choice for congestive heart failure (CHF) and will alleviate the pulmonary edema. - Congestive heart failure may be difficult to differentiate from respiratory problems in infancy. - Trisomy 21 is often associated with congenital heart disease (CHD) and may present with CHF. - Echocardiography is crucial in making the diagnosis of CHD or cardiomyopathy. ABP Content Specifications(s) - Recognize the clinical findings associated with congestive heart failure in children of various ages - Plan the appropriate initial diagnostic evaluation of congestive heart failure in children of various ages

A 4-month-old female infant is seen in the emergency department (ED) with respiratory distress. You are in the midst of a respiratory syncytial virus epidemic. She has been ill for 2 weeks, with a 50% decrease in her feedings and history of rapid breathing. The family was so worried about her today that they called an ambulance. She was given an albuterol nebulizer treatment by emergency medical services 1 hour ago, but they did not see any change in her respirations. She does not take any other medicines; she has never been hospitalized or had surgery. Her family moved from out of state when she was 1 month old, and they do not have a pediatrician. On arrival in the ED, she is breathing rapidly at a rate of 70 breaths/min without any cough or stridor. She is alert, but not vigorous, and has typical trisomy 21 facies. Her oxygen saturation is 88% on room air. Her heart rate is 180 beats/min. Her blood pressure is 90/60 mm Hg. On physical examination, her chest is clear with equal breath sounds. Her cardiac examination shows a hyperdynamic precordium, a 2/6 holosystolic murmur at the left midsternal border, and there is also a continuous murmur in the middle of the left scapula that radiates across the back. Her liver is 4 cm below the right costal margin. Her femoral pulses are equal and bounding. Her capillary refill time is 2 seconds. Of the following, the BEST next step in management of this patient is to administer A. ceftriaxone B. continuous albuterol C. furosemide D. high-flow oxygen E. methylprednisolone

E. place the infant in prone knee chest position The patient in this vignette has Tetralogy of Fallot. The best initial intervention for the worsening cyanosis would be to place him in a prone knee chest position. Cyanotic congenital heart disease is a group of lesions that allow desaturated blood to enter the systemic circulation. The lesions include those that have too little pulmonary blood flow or right-to-left shunting. Conditions with decreased pulmonary blood flow include Tetralogy of Fallot, critical pulmonic stenosis, and congenital heart disease that has been palliated with a modified Blalock-Taussig (BT) shunt. Complications can occur from the newborn to early adult period and differ by lesion and type of surgical repair. Patients with each of these conditions will have distinctive physical examination findings that may help differentiate their underlying diagnosis and treatment if they were to present in distress to a new facility or physician. The patient with the BT shunt will have either a median sternotomy or a thoracotomy scar on either side of their chest. He or she will have a loud, continuous murmur on the side of the thoracotomy scar that can be best heard in the midclavicular region. If the murmur is very soft or becomes inaudible in a patient with acute cyanosis, suspect shunt occlusion. This can occur in the setting of dehydration or fever and is life threatening. An infant with an occluded BT shunt will have decreased oxygen saturation from their usual baseline of 75% to 85% that will not change with oxygen delivery. The initial management in this setting is to give fluids to attempt to increase flow through the shunt. Patients with Tetralogy of Fallot are susceptible to hypercyanotic spells or "Tet Spells" when they are volume depleted or dehydrated. They also have intracardiac and specific anatomy and physiology that makes their pathophysiology different from the patients with occluded BT shunt. The components of Tetralogy of Fallot are pulmonic stenosis, aortic override, ventricular septal defect (VSD), and right ventricular hypertrophy. The flow through the VSD is usually left to right because the right ventricular pressure is less than that of the left, and the pulmonary vascular resistance is less than the systemic vascular resistance. The specifics of the right ventricular outflow tract obstruction will determine the likelihood of an infant having a Tet spell, but the initiation will begin with a sudden drop in systemic vascular resistance, such as might be seen with crying or fever. This will cause the intracardiac shunt across the VSD to become right to left (from the right ventricle to the left ventricle). A loud pulmonary stenosis murmur that becomes soft or absent in this patient is a dangerous finding because it means that the flow out of the right ventricle has decreased. As the desaturated blood goes out to the periphery, it will cause acidosis and hyperventilation. This in turn will cause increased rate of blood return to the right heart and exponentially worsen the desaturation as more blood is sent across the VSD. The pulmonic stenosis does not allow much of this returning blood to go to the lungs and get oxygenated. Such infants' saturations will continuously decrease as the spell worsens. The Tet spell physiology can be treated in several ways, but the initial step is to increase the systemic vascular resistance quickly. The knee chest maneuver, as done for the infant in the vignette, will instantly increase the systemic vascular resistance and can be done before any equipment is available. The patient with critical pulmonic stenosis who has no VSD, on the other hand, will have a loud murmur that will be proportional to the obstruction at the pulmonary valve. This patient's murmur will get louder as the obstruction gets worse. This infant's saturations will decrease but not become critically low. Oxygen would not be dangerous for any of the infants described above. They all have decreased pulmonary blood flow and increasing the dissolved oxygen and saturating all the hemoglobin in the blood is desirable. However, 2 L of nasal cannula would not provide enough oxygen to treat a Tet spell. In such situations, 100% oxygen by a non-rebreather mask would be needed. Although there is decreased pulmonary blood flow in the midst of a Tet spell, providing supplemental oxygen helps to increase the dissolved oxygen concentration in the systemic circulation. Increasing the systemic vascular resistance is the first intervention for a Tet spell. It would not be helpful for the infant with a BT shunt obstruction or the patient with critical pulmonic stenosis. Diphenhydramine would sedate the infant slightly and possibly lower the systemic blood pressure if given intravenously. This could worsen the Tet spell, even though sedation would help the hyperventilation. Morphine is used as one of the interventions in a Tet spell to both sedate and decrease hyperventilation. Epinephrine would increase the systemic vascular resistance and the blood pressure. It might help decrease the right to left shunt, but would not be the first intervention. Ice to the face would cause a vagal response and lower the heart rate. It could also increase the infant's agitation and crying, thereby lowering the systemic vascular resistance. This could make the Tet spell worse. The knee chest maneuver is easy to do and can be done at the first sign of a Tet spell. This maneuver can often stop the spell immediately and is the first intervention while oxygen, intravenous fluids, and possibly morphine are being assembled. PREP Pearls - The first intervention in a hypercyanotic spell in patients with Tetralogy of Fallot (TOF) is to place the child in the prone knee chest position. - Loud murmurs that disappear in patients with a Blalock-Taussig Shunt or with TOF are red flags for dangerous, potentially life-threatening physiology. ABP Content Specifications(s) - Recognize complications associated with cyanotic congenital heart disease

A 4-month-old infant with Tetralogy of Fallot is brought to your office for a routine health supervision visit. He has been doing well, growing and thriving, and has not had any episodes of cyanosis. He is scheduled for a cardiology appointment in 1 week, with the plan for surgical repair scheduled for the week after. On arrival at your office, his oxygen saturation is 88% on room air. He is well appearing and interactive. A physical examination reveals that his weight is 6.1 kg, his heart rate is 120 beats/min, and his respiratory rate is 28 breaths/min. His chest is clear. His cardiac examination is significant for a regular rate and rhythm. There is a long 4/6 systolic murmur heard at the left mid to upper sternal border. Immediately after the administration of vaccines, he becomes visibly more cyanotic. Repeat oximetry is now 60% on room air, and the baby is breathing 60 breaths/min. There is a soft 1/6 systolic murmur at the left upper sternal border. Of the following, the BEST initial intervention for this patient is to A. administer diphenhydramine B. administer epinephrine C. administer oxygen at 2 L by nasal cannula D. apply ice bag to face E. place the infant in prone knee chest position

E. increase in phosphorus, potassium, and uric acid The boy in this vignette has newly diagnosed acute leukemia and is at risk for tumor lysis syndrome (TLS), especially after he starts treatment, as a result of his high tumor burden (ie, high peripheral leukocyte count with high percentage of blasts). Tumor lysis syndrome occurs when malignant cells are rapidly lysed, typically after chemotherapy or radiation therapy has been initiated, resulting in the release of cellular components and metabolic abnormalities. Tumor lysis syndrome is more likely to occur in malignancies with a high rate of proliferation such as acute leukemia and lymphoma, or those with a high sensitivity to treatment. Tumor lysis syndrome may lead to renal insufficiency with decreased urine output and elevated serum creatinine. Brisk lysis of tumor cells can cause an increase in serum uric acid, potassium, phosphorus, and lactate dehydrogenase, with a decrease in serum calcium. Unrecognized and untreated TLS can progress to renal failure, arrhythmias, seizures, and death. Various criteria are used to diagnose clinical and laboratory TLS, all of which include elevations in uric acid, potassium, phosphate, and creatinine with hypocalcemia. Empiric management with aggressive hydration promotes the excretion of uric acid and phosphate. The addition of sodium bicarbonate to alkalinize the urine can increase the solubility of uric acid and was historically included in TLS management strategies. However, it does not significantly increase the solubility of xanthine and hypoxanthine, the precursors of uric acid, which can lead to xanthine obstruction in the renal tubules. Therefore, alkalization of the urine is no longer universally recommended. Allopurinol, a xanthine analog, is a competitive inhibitor of xanthine oxidase that blocks the conversion of xanthine and hypoxanthine to uric acid and has been shown to decrease the incidence of obstructive uropathy caused by uric acid precipitation. Rasburicase is a recombinant urate oxidase that rapidly decreases uric acid levels by degrading uric acid to allantoin, a highly soluble substance with no known adverse effects in humans. Of note, rasburicase is contraindicated in patients with glucose-6-phosphate deficiency because of the increased risk of hemolysis. PREP Pearls - Tumor lysis syndrome (TLS) occurs when malignant cells are rapidly lysed, typically after chemotherapy or radiation therapy has been initiated, resulting in the release of cellular components and metabolic abnormalities. Brisk lysis of tumor cells can cause an increase in serum uric acid, potassium, phosphorus, and lactate dehydrogenase, with a decrease in serum calcium. - Tumor lysis syndrome is more likely to occur in malignancies with a high rate of proliferation such as acute leukemia and lymphoma, or those with a high sensitivity to treatment. - Tumor lysis syndrome may lead to renal insufficiency with decreased urine output and elevated serum creatinine. - - Unrecognized and untreated TLS can progress to renal failure, arrhythmias, seizures, and death. ABP Content Specifications(s) - Plan an appropriate diagnostic evaluation to exclude tumor lysis syndrome in patients suspected of having leukemia

A 4-year-old boy is referred to the hospital for the following abnormal complete blood cell count: - White blood cell count, 95,000/µL (95 × 109/L), with 10% polymorphonuclear leukocytes, 25% lymphocytes, 8% monocytes, and 2% eosinophils, 55% blasts - Hemoglobin, 7.0 g/dL (70 g/L) - Mean corpuscular volume, 90 µm3 (90 fL) - Platelet count, 20,000 × 103/µL (20 × 109/L) Bone marrow studies confirm the diagnosis of acute lymphoblastic leukemia. His parents give informed consent to start chemotherapy. On the second day of receiving induction chemotherapy, the child is noted to have markedly decreased urine output with an elevated serum creatinine level. Of the following, the laboratory results that would be MOST likely to be seen in this patient are A. decrease in lactate dehydrogenase, potassium, and uric acid B. decrease in lactate dehydrogenase, uric acid, and white blood cell count C. increase in calcium, phosphorus, and uric acid D. increase in calcium, potassium, and uric acid E. increase in phosphorus, potassium, and uric acid

D. growth hormone deficiency When height is affected more than weight, an endocrine cause is more likely, and of the choices offered, growth hormone deficiency is the most likely cause. Achondroplasia is characterized by short stature, but also megalocephaly, facial dysmorphisms, abnormal vertebrae, disproportionally short tubular bones, and abnormal fingers and hands. It is autosomal dominantly inherited. In familial short stature + constitutional growth delay, linear growth often resets to the bottom of the growth chart in the first 18 months to 2 years after birth and then tracks at a normal growth velocity. Children with familial short stature will continue this growth pattern until their final adult height is reached, and usually have a concordant bone age. They would not reset their growth pattern at 4 years of age, as noted in the child in the vignette. Children with constitutional delay of growth will continue growing at a prepubertal growth rate beyond the expected age, have a delayed bone age, and then have catch-up growth later, consistent with their family history. PREP Pearls - An endocrine cause of poor growth is likely when linear growth velocity declines, but weight gain is either normal or increasing. - In the majority of cases in which poor growth is related to under nutrition, falloff in weight gain will precede falloff in linear growth. ABP Content Specifications(s) - Plan an appropriate diagnostic evaluation to differentiate constitutional growth delay and other conditions causing growth delay - Understand the natural history of constitutional growth delay - Distinguish among constitutional short stature, genetic (familial) short stature, and growth hormone or thyroid deficiencies by growth chart evaluation - Understand the natural history of genetic (familial) short stature

A 4-year-old boy presents for evaluation of short stature. He was growing well until 1 year ago, when he was noted to have a linear growth velocity of 2.5 cm (Item Q38 - drop off of linear growth). His weight is also low, but tracking at the third percentile. His body mass index is normal. History reveals he is sometimes a "picky eater" but has no other gastrointestinal symptoms. Family history shows his mother is 160 cm (5 ft, 3 in) tall, and his father 167.6 cm (5 ft, 6 in). Both parents had delayed puberty. His maternal uncle was diagnosed with a pituitary tumor as a child. On physical examination, you note the child is short, but otherwise well appearing. The child's bone age is 2.5 years at a chronological age of 4 years. Of the following, the MOST likely diagnosis is A. achondroplasia B. constitutional delay of growth C. familial short stature D. growth hormone deficiency E. inadequate caloric intake

C. impaired micellar solubilization Genu varum deformity characteristic is of rickets. In cholestatic syndromes, micellar solubilization is impaired because delivery of bile salts to the intestine is insufficient to permit adequate micelle formation required for lipid solubilization. PREP Pearls - Patients with cholestatic liver disease are at risk particularly for calcium, fat, and fat-soluble vitamin malabsorption as a consequence of reduced intraluminal bile salt concentrations leading to impaired micellar solubilization. - In children with cholestasis, all vitamins should be provided as water-soluble analogues. In particular, vitamin E should be provided as d-a-tocopheryl polyethylene glycol-1000 succinate. - Infants and children with cholestasis often require 150% or greater of recommended energy intake to maintain growth. Dietary fat intake of more than 80% medium-chain triglycerides will lead to essential fatty acid deficiency. ABP Content Specifications(s) - Understand the nutritional causes of growth failure associated with chronic cholestatic disease, including the effects of nutrient digestion and absorption - Know the general dietary recommendations for hepatic disease - Understand the mechanism of rickets associated with hepatic disease

A 4-year-old boy presents to your office because of poor growth. The child was diagnosed with extrahepatic biliary atresia at 6 weeks of age and underwent a hepatoportoenterostomy (Kasai procedure). Since that time, he has remained mildly jaundiced and has experienced 3 episodes of presumed bacterial cholangitis. He is currently awaiting evaluation at a regional liver transplant center. The child consumes a regular diet for age, including a daily multivitamin supplement. His height and weight are at the 10th and 25th percentile, respectively. One year ago, his height and weight were both at the 25th percentile. Physical examination findings show a small, active child in no distress, icteric sclerae, a mildly protuberant abdomen, a firm liver edge palpated 1.5 cm below the right costal margin, and a spleen tip palpated 2 cm below the left costal margin. The lower extremities show genu varum deformity. Laboratory study results include the following: Electrolytes, normal Calcium, 7.8 mg/dL (1.95 mmol/L) Phosphorus, 3.8 mg/dL (1.23 mmol/L) Total bilirubin, 6.5 mg/dL (111.2 mmol/L) Direct bilirubin, 4.0 mg/dL (68.4 mmol/L) Aspartate aminotransferase, 70 U/L (reference range, 20-60 U/L) Alanine aminotransferase, 90 U/L (reference range, 20-50 U/L) Alkaline phosphatase, 650 U/L (reference range, 150-350 U/L) Of the following, the MOST likely cause of this child's poor growth is A. decreased energy intake B. decreased intestinal absorptive function C. impaired micellar solubilization D. impaired nutrient hydrolysis E. impaired re-esterification

D. nonfurry pets, such as a fish PREP Pearls - In a child sensitized to pet dander, avoidance of pet allergen exposure is recommended in order to mitigate severity and worsening of allergic disease. - The most effective measure to minimize allergen exposure in a pet-sensitized patient is removal of the pet in conjunction with allergen abatement interventions at home. - Less-effective, but perhaps more realistic measures include those targeted at reducing exposure to reservoirs and blocking pathways from reservoirs to home occupants. - To date, studies have not validated claims that "hypoallergenic pets" carry a lower allergen load or cause fewer symptoms. - Evidence currently does not support preemptive removal of, or acquiring a pet, in the hope of preventing allergic disease in the unborn child and infant. ABP Content Specifications(s) - Understand the role of the environment (eg, indoor pets, passive exposure to cigarette smoke) in the development and severity of allergic disease

A 4-year-old girl is brought to your office for follow-up of her atopic dermatitis and food allergies to peanuts. The parents report symptoms of sneezing fits and nasal itching that occur mostly during the spring and summer months. Their cat died recently and the girl has been requesting a new cat. The parents did not notice her having any symptoms around the cat, but remarked that she gets worsening symptoms when around the neighbor's cat. They also note that the child's eczema worsens when they visit her grandparents, who have 2 dogs. The parents would like to get her a new pet, but worry it will worsen her allergies and they seek your suggestions. Of the following, your BEST recommendation is to get her A. any pet, as long as they get a high-efficiency particulate air purifier B. a dog, but keep it outdoors in the daytime C. a hypoallergenic, nonshedding dog D. nonfurry pets, such as a fish E. the same breed of cat they had before

C. clindamycin The pustular lesion with surrounding erythema as seen in the girl in the vignette, suggests a skin and soft tissue infection caused by Staphylococcus aureus. The central necrotic component has been associated with community-acquired methicillin-resistant S aureus (CA-MRSA) infection. Early descriptions of CA-MRSA infections indicated that they appeared like a spider bite. Skin and soft tissue infections caused by group A streptococcal infection are associated with cellulitis, but typically without a pustular component. Drainage of purulent cellulitis is recommended with culture performed on the material obtained. In lesions less than 5 cm in a well-appearing child, drainage alone without antibiotic therapy is likely sufficient. In the face of significant surrounding erythema and fever to 39.2°C, as in this patient, antibiotic therapy should be initiated while awaiting culture results. Of the agents listed, clindamycin would be the most appropriate to cover S aureus, including CA-MRSA. Trimethoprim/sulfamethoxazole and doxycycline would also provide appropriate coverage for CA-MRSA in this type of infection, although doxycycline would not be routinely recommended in a child younger than 7 years of age. Ampicillin covers group A streptococcus, but not S aureus. Ceftriaxone may treat some methicillin-sensitive S aureus (MSSA), but not methicillin-resistant S aureus (MRSA) infections. Metronidazole is used for anaerobic bacterial infections. Piperacillin-tazobactam would cover MSSA but not MRSA infections, and is a broader spectrum agent than needed for a skin and soft tissue infection in a normal host. Item C81, shows the algorithm for initial management of skin and soft tissue infections caused by community-acquired S aureus. If the child was more ill and there was concern for invasive disease, empiric therapy with vancomycin, possibly in combination with a semisynthetic penicillin such as nafcillin, would be indicated until culture results are available. PREP Pearls - In normal hosts, skin and soft tissue infections with a pustular component are most often associated with Staphylococcus aureus infections, while cellulitis alone is most often caused by group A streptococcal infection. - Drainage of pustular skin infections is recommended with fluid sent for culture and sensitivity testing. Lesions smaller than 5 cm can be treated with drainage alone. In larger lesions or with the presence of systemic signs or fever, antibiotic therapy, including coverage for methicillin-resistant Staphylococcus aureus (MRSA) is advisable. - Clindamycin, trimethoprim/sulfamethoxazole, or doxycycline provides adequate empiric coverage for MRSA. - In the severely ill child with skin infection, vancomycin, possibly in combination with nafcillin, would be recommended while awaiting culture results. ABP Content Specifications(s) - Plan the appropriate management of cellulitis of the skin of various etiologies

A 4-year-old girl presents to your office for evaluation of a skin lesion over her right knee that has a dark central area and 7 cm of surrounding redness. The redness has increased over the past 2 days. She had a temperature of 39.4°C on the day of evaluation. The parents thought she had been bitten by a spider in the area just before the redness began. She is said to be a bit fussy and complaining of pain at the site. She has no significant past illnesses and has no recent antibiotic usage. Her vital signs show a temperature of 39.2°C, a heart rate of 96 beats/min, a respiratory rate of 16 breaths/min, and a blood pressure of 98/66 mm Hg. Physical examination shows a 2-cm, tender right inguinal lymph node. The lesion is shown in Item Q81 (abscess draining pus.) She cries but is able to bend the knee. The remainder of the examination findings are unremarkable. You take a culture by aspirating the lesion. Of the following pending culture results the BEST initial antibiotic choice for this girl is A. ampicillin B. ceftriaxone C. clindamycin D. metronidazole E. piperacillin-tazobactam

E. require increased quantities of iron because of the decreased bioavailability of iron found in plant sources Iron in plant-based foods has a lower bioavailability than iron found in meat, therefore children who do not eat meat require increased iron intake. Vitamin B12, a cobalamin compound, occurs naturally in animal products including fish, meat, eggs, and dairy. Therefore, individuals following a vegan diet need to consume foods or nutritional products that are supplemented with vitamin B12 to avoid deficiency. Leafy greens are an excellent source of calcium. Soy milk, cereals, and orange juice are examples of products that are commonly available in calcium-fortified forms. Soy is an excellent source of protein for vegetarians. PREP Pearls - Children following a vegan diet require increased iron intake and vitamin B12 supplementation. - A child following a vegan diet would be expected to have normal growth with a body mass index lower than the average for nonvegan peers. ABP Content Specifications(s) - Identify the nutritional complications that can result from vegetarian or vegan diets

A 5 year-old girl presents to your office for a health supervision visit. Her parents have decided to feed the family a strict vegan diet. You counsel the parents about young children who follow a vegan diet. Of the following, the MOST appropriate statement to include is that children on this type of diet A. generally cannot consume the required calories and nutrients to allow for adequate growth B. generally require less protein than omnivores due to the increased digestibility of plant proteins C. require B6 supplementation since B6 is only found in animal products D. require calcium supplementation as leafy greens cannot provide enough calcium for optimal bone health E. require increased quantities of iron because of the decreased bioavailability of iron found in plant sources

A. acute bilirubin encephalopathy The jerking motions and bicycling movements of the newborn in the vignette represent findings associated with acute bilirubin encephalopathy (ABE). The newborn has marked jaundice of his legs and his palms, which suggests his serum bilirubin level is greater than 15 mg/dL (256.5 μmol/L). His clinical findings of lethargy, poor tone, irritability, and poor feeding are typically seen with ABE. The clinical signs of ABE in the neonatal period represent a spectrum. The earliest findings are often subtle and nonspecific. Moderate ABE manifests with arching of the neck and trunk, increasing lethargy, decreased feeding, and irritability with a shrill cry. Intervention at these early phases may prevent the sequelae of kernicterus. Worsening signs suggestive of severe ABE include bicycling movements of the arms and legs, inconsolable crying, inability to eat, fever, seizures, and coma. Active intervention at this point is unlikely to affect the risk of the development of kernicterus. Identification of infants at risk of developing ABE and subsequent kernicterus sequelae is essential. The American Academy of Pediatrics clinical practice guideline for the management of hyperbilirubinemia in the newborn infant of 35 weeks or longer gestation provides a framework that assists in the early identification of at-risk infants. These guidelines recommend that infants showing signs of moderate to severe ABE be given an immediate exchange transfusion, even if the total serum bilirubin is falling, acknowledging that the sequelae of kernicterus may be prevented if severe ABE does not develop. If suspected in the outpatient setting, infants exhibiting symptoms of ABE should be admitted directly to the hospital and not be referred to the emergency department where treatment may be delayed. The newborn in the vignette is manifesting the clinical findings of severe ABE and requires emergent admission for evaluation and treatment. An evaluation for the underlying cause of his hyperbilirubinemia should include a total and direct bilirubin, blood type, direct antiglobulin test (DAT), complete blood cell count, and glucose-6-phosphate dehydrogenase deficiency testing. It is appropriate to check blood glucose and serum calcium levels because of his history of poor feeding, but these are unlikely to be the cause of his jerking motions and bicycling movements. A sepsis evaluation may be performed because of his severe hyperbilirubinemia of unclear origin. Although not likely to be the cause of his clinical findings, herpes simplex should always be considered. The jerking movements of benign neonatal clonus are present only in sleep, thus excluding this as the cause of his abnormal movements. PREP Pearls - The earliest findings of acute bilirubin encephalopathy (ABE) are often subtle and nonspecific. Moderate ABE manifests with arching of the neck and trunk, increasing lethargy, decreased feeding, and irritability with a shrill cry. Worsening signs suggestive of severe ABE include bicycling movements of the arms and legs, inconsolable crying, inability to eat, fever, seizures, and coma. - Infants showing signs of moderate to severe ABE require an immediate exchange transfusion, even if the total serum bilirubin is falling, acknowledging that the sequelae of kernicterus may be prevented if severe ABE does not develop. ABP Content Specifications(s) - Recognize the clinical features and sequelae of acute bilirubin encephalopathy in newborn infants, and manage appropriately

A 5-day-old term newborn is brought to the emergency department for the evaluation of jerking motions of his arms and legs. The newborn was delivered vaginally weighing 3,500 g and discharged to home 36 hours after delivery. He has been exclusively breastfed, with an increasingly poor suck, irritability, and decreased activity noted by the mother over the past 24 hours. Upon admission to the emergency department, his weight is 3,150 g and his vital signs are normal. A physical examination reveals an inconsolable, hypotonic newborn with marked jaundice that involves his legs and the palms of his hands. You note a brief jerking of his right arm, followed by bicycling movements of his upper and lower extremities. Of the following, the MOST likely cause of the jerking is A. acute bilirubin encephalopathy B. benign neonatal myoclonus C. herpes simplex encephalitis D. hypocalcemia E. hypoglycemia

A. galactosemia The male neonate in the vignette has classic galactosemia, which is a disorder of galactose metabolism that results in life-threatening issues such as lethargy, poor growth, vomiting with poor feeding, hepatocellular damage with elevated liver function tests, and clinical jaundice, bleeding diathesis, and sepsis in undiagnosed infants. Urine-reducing substances will be positive in addition to the coagulation and liver function abnormalities. The neonate will be well initially then quickly decompensate after just a few days of age. The neonatal symptoms will resolve immediately if a lactose- or galactose-restricted diet is given in the first 10 days after birth to avert cataracts, imminent liver failure, sepsis, intellectual disability, and potential newborn death. Most patients, even with early treatment, will still be at risk for some developmental delays, including verbal dyspraxia and motor delays. Some patients will have fine motor tremors and abnormalities in coordination and gait. Girls are at high risk for premature ovarian failure. In cases of delayed treatment, the patient will commonly have a long-term impact of intellectual disability and poor growth. Most neonates with galactosemia are diagnosed through newborn screening via a heel prick, which is mandatory in all state newborn screening panels, because it is an easily treated disease that prevents serious sequelae if diagnosed and treated early. Following a positive newborn screen, a measurement of erythrocyte galactose-1-phosphate uridyltransferase (GALT) enzyme activity, erythrocyte galactose-1-phosphate (gal-1-P) concentration, and GALT molecular sequencing can establish the diagnosis. Once the newborn screening finds abnormalities pointing to this disorder, immediate changes should be made to the neonate's diet until the diagnosis is confirmed to avoid potentially serious consequences. A neonate's formula should be switched to a soy-based or elemental formula containing sucrose, fructose, and nongalactose carbohydrates. All milk and formula products containing galactose or casein hydrolysates should be discontinued. Medications containing lactulose also should be avoided. Even after dietary avoidance of galactose, patients should continue to have periodic ophthalmologic examinations for cataracts and screening assessments for developmental delays. If any verbal dyspraxia or motor delays are noted, appropriate interventions and therapies should be put into place. Hereditary hemochromatosis is unlikely in the neonate in the vignette because it would not present in early infancy with such rapid decompensation. Hemochromatosis is a disorder caused by the abnormal storage of iron in joints, liver, heart, pancreas, and lungs. Most people will develop symptoms between the ages of 40 and 60 years. Neonates with hereditary fructose intolerance develop symptoms in infancy after ingesting fruits, juices, or other foods containing fructose, not typically in the neonatal period, but after infant foods and juices begin to be introduced into their diet. Symptoms typically include nausea, diarrhea, vomiting, hypoglycemia, elevated uric acid, and abdominal pain associated with failure to thrive. Continued ingestion can result in kidney and liver damage. Treatment is strict avoidance of fructose ingestion. Tyrosinemia type 1 presents in infancy with significant liver involvement and eventually renal tubular dysfunction, growth failure, and rickets. Untreated children may present with repeated neurologic crises involving a change in mental status, peripheral neuropathy, abdominal pain, and occasionally respiratory failure. If untreated, many die before the age of 10 years. Laboratory abnormalities include increased succinylacetone concentration in the blood and urine; elevated tyrosine, methionine, and phenylalanine on serum amino acids; and elevated tyrosine metabolites on urine organic acids. Urea cycle defects present with a progressive obtundation, severe hyperammonemia, and respiratory alkalosis in the first week after birth, which progresses to death very quickly with irreversible brain damage if not immediately recognized and treated. PREP Pearls - Immediate recognition of galactosemia is important because of the life-threatening consequences that will rapidly unfold if the galactose-containing diet is not discontinued and changed to a soy-based or elemental formula. - Galactosemia is mandatory on newborn screening. Treatment should not be delayed while a definitive diagnosis is in process. - Classic laboratory abnormalities of galactosemia include positive-urine reducing substances, abnormal liver function studies, coagulation abnormalities suggestive of a progressive bleeding diathesis, and a positive blood culture, especially due to Escherichia coli. - Physical findings of galactosemia in the neonatal period could include jaundice, hypotonia, scleral icterus, bruising, bleeding, and cataracts in the face of rapidly progressive liver failure. ABP Content Specifications(s) - Plan the appropriate immediate and long-term management of galactosemia, while considering the long-term prognosis - Recognize the clinical features associated with galactosemia - Recognize the laboratory features associated with galactosemia

A 5-day-old, breastfed, male term neonate presents to your office with a temperature up to 38.9°C, poor feeding, lethargy, vomiting, and diarrhea that have progressively worsened over the last 2 days. The pregnancy and delivery were unremarkable. The birth weight was 2,950 g. The baby did well the first 2 days after birth and was discharged with the mother. Since discharge, his feeding has become more difficult, with vomiting and fussiness. He becomes sleepier after feeding and develops watery diarrhea. His current temperature is 38.6°C, pulse is 140 beats/min, respiratory rate is 40 breaths/min, and blood pressure is 88/54 mm Hg. His weight is 2,700 g. On physical examination, you note an ill-appearing infant with decreased tone, jaundice, scleral icterus, distended abdomen with hepatomegaly, bruising, and mild respiratory distress. His red reflex is abnormal and consistent with cataracts. The child is admitted to the hospital for evaluation for possible sepsis, and intravenous antibiotics are begun. The following are the results of the neonates laboratory tests: - White blood cell count, 18,000/µL (18 × 109/L) - Aspartate aminotransferase, 138 U/L - Alanine aminotransferase, 76 U/L - Total bilirubin, 12 mg/dL (205.3 µmol/L) - Direct bilirubin, 2 mg/dL (34.2 µmol/L) - Prothrombin time, 36.9 s - International normalized ratio, 4.8 - Partial thromboplastin time, 50 s - Urine reducing substances, 3 + - Blood culture, positive for Escherichia coli - Urine and cerebrospinal fluid cultures, negative - TORCH titers, negative Of the following, the MOST likely diagnosis i A. galactosemia B. hereditary fructose intolerance C. hereditary hemochromatosis D. tyrosinemia type 1 E. urea cycle defect

A. heparin given during his hospitalization The child in the vignette shows the characteristic diffuse, nonscaly hair thinning of telogen effluvium. The cause of this phenomenon is likely multifactorial, but medications have been implicated as triggers in many cases. As many as 50% of patients who take anticoagulants, particularly low molecular weight heparins, experience hair thinning a few weeks after the drug exposure. Thus, for the patient in the vignette, exposure to heparin during a hospitalization 2 to 3 months ago is a likely trigger for this child's hair loss. Other medications that may be associated with telogen effluvium include oral contraceptives (particularly when interrupted after long-term use), isotretinoin, anticonvulsants, psychotropics (lithium, sodium valproate, fluoxetine), β-adrenergic blockers (metoprolol and propranolol), angiotensin-converting enzyme inhibitors, isoniazid, and antiretroviral agents (indinavir). Telogen effluvium represents a disruption in the normal hair life cycle. Normally, more than 85% of scalp hairs are in the anagen or growth phase, which lasts approximately 3 years. As the hair follicle begins to involute, the hair enters the 2-week-long catagen or resting phase. The hair then moves to the telogen phase lasting approximately 3 months, during which the hair fiber is shed. Approximately 13% of scalp hairs are in the telogen phase normally and about 50 to 150 hairs are shed daily. During telogen effluvium, the anagen phase is terminated early for many more follicles than usual, pushing them to telogen and leading ultimately to increased daily hair loss. Factors other than medications can also lead to telogen effluvium. Commonly associated triggers include acute, severe illness (particularly with high fever), childbirth, major surgery, and thyroid disease. However, because of the duration of the hair growth cycles, the inciting event for an episode of telogen effluvium usually occurs 2 to 3 months before hair loss is detected. Therefore, an intercurrent upper respiratory infection is an unlikely trigger for this child's hair loss. If hair loss persists for 6 weeks, the child would be considered to have chronic telogen effluvium. Causes for chronic telogen effluvium include thyroid disease, iron-deficiency anemia, malnutrition, chronic illness (eg, lupus), zinc deficiency, or syphilis. The short duration of this child's condition and the absence of other symptoms of thyroid disease make this a less likely trigger than the recent heparin exposure. The primary management of telogen effluvium is to remove or treat the underlying cause. Most patients experience complete hair regrowth within 6 months. Trichotillomania typically presents with irregular frontotemporal patches of hair loss with broken and variable length hairs within the affected region. Therefore, this child's widespread hair loss and absence of broken hairs makes this a less likely diagnosis. Likewise, tinea capitis caused by Trichophyton tonsurans infection presents with patches of scaly hair loss and can be ruled out based on physical appearance. PREP Pearls - Telogen effluvium occurs 2 to 3 months after an inciting event. - The physical findings of telogen effluvium include increased daily hair loss, positive hair pull test, widespread hair loss without scale, and generalized hair thinning. - Triggers for telogen effluvium include stressors (severe illness, high fever, childbirth, trauma), various medications (including heparin, oral contraceptives, isotretinoin, anticonvulsants, psychotropics, ß-adrenergic blockers, angiotensin-converting enzyme inhibitors, isoniazid, and antiretroviral agents), thyroid disease, iron deficiency, malnutrition, and chronic illness. - Treatment for telogen effluvium is to eliminate or address the underlying condition. ABP Content Specifications(s) - Recognize the clinical findings associated with telogen effluvium, and manage appropriately

A 5-year-old boy comes to your office for a mild upper respiratory infection. While there, the parent also mentions concerns about hair loss. The child was hospitalized 2 months ago for a femur fracture due to a motor vehicle collision and the child received heparin prophylaxis. His past medical history is otherwise unremarkable, and he is appropriately grown. The parent denies any skin changes, temperature intolerance, or unusual hair product use. He is not currently taking any medications. On physical examination, there is mild nasal congestion and moderate diffuse hair loss without scaly patches (Item Q134). He has a positive result on hair pull test. The remainder of his examination is unremarkable. Of the following, the MOST likely etiology of this child's hair loss is A. heparin given during his hospitalization B. stress from the upper respiratory infection C. thyroid disease D. Trichophyton tonsurans infection E. trichotillomania

C. has a genetic mutation that can eventually lead to bone marrow failure Congenital infections (eg, cytomegalovirus) can cause cytopenias in the neonatal period, but these usually resolve over time. Leukemia and nutritional deficiencies (eg, vitamin B12 or folate) can cause macrocytosis and cytopenias, but would not explain the dysmorphic features. Wiskott-Aldrich syndrome is an X-linked condition characterized by immunodeficiency, thrombocytopenia, and eczema, but typically does not have any congenital anomalies. PREP Pearls - Fanconi anemia is a rare congenital disorder, usually of autosomal recessive inheritance, associated with high incidence of bone marrow failure, malignancy, congenital anomalies, chromosomal instability, and defective DNA repair. 98% are AR. - The most common physical features in patients with Fanconi anemia, include café-au-lait spots, short stature, abnormal thumbs and radii, and abnormal head, eyes, kidneys, and ears. - The only curative treatment for the bone marrow failure component of Fanconi anemia is hematopoietic stem cell transplantation. ABP Content Specifications(s) - Recognize clinical and laboratory findings associated with Fanconi anemia

A 5-year-old boy is a new patient brought to your office for easy bruisability. His mother noticed an increasing number of bruises and red dots on his trunk and legs over the past few weeks without any witnessed trauma. His weight is at the tenth percentile, but his height and head circumference are below the third percentile. He has low-set ears, 3 café-au-lait spots on his chest, as well as scattered bruises and petechiae on his trunk and legs. You also notice that his thumbs are abnormal in appearance (hypoplastic.) The remainder of the physical examination is unremarkable. His vital signs are stable. The following are the results of the child's complete blood cell count: - White blood cell count, 2,500/µL (2.5 × 109/L), with 25% polymorphonuclear leukocytes, 65% lymphocytes, 8% monocytes, and 2% eosinophils - Hemoglobin, 11 g/dL (11 g/L) - Mean corpuscular volume, 100/µm3 (100 fL) - Platelet count, 25,000 × 103/µL (25 × 109/L) - Prothrombin time, normal - Partial thromboplastin time, normal You refer the patient to a specialist for further evaluation. The mother would like to know the possible diagnoses. Of the following, the MOST appropriate response would be that he likely A. had a congenital infection that led to the dysmorphic features and cytopenias B. has acute lymphocytic leukemia C. has a genetic mutation that can eventually lead to bone marrow failure D. has a nutritional deficiency affecting his growth and bone marrow production E. has an X-linked condition associated with immunodeficiency

A. fetal alcohol syndrome To diagnose FAS, a physician should identify 3 cardinal features: abnormal facial features, growth deficiency, and CNS problems. Heart defects may occur and could include ventricular septal defect or atrial septal defect. Some patients exposed to alcohol in utero can have alcohol-related neurodevelopmental disorder. These patients lack the physical stigmata common with FAS, but have intellectual disabilities and problems with behavior and learning. Fetal anticonvulsant syndrome: typically display behavioral problems including ADHD, autistic features, learning difficulties, speech delay, and gross and fine motor delay. Clinical findings may include intrauterine growth retardation, microcephaly, hypertelorism, epicanthal folds, long philtrum with thin vermilion border, small mouth, flat nasal bridge, low-set ears, hypoplasia of the distal phalanges and nails, cleft palate, meningomyelocele, and congenital heart disease. The prevalence of neural tube defects in neonates exposed to valproic acid in utero is 10 to 20 times higher than in the general population. Mild to moderate intellectual deficits are common. Fragile X syndrome: is an X-linked dominant disorder caused by the FMR1 gene, which is composed of a series of CGG repeats (normal number of repeats < 45). Premutation carriers for fragile X have 55 to 200 CGG repeats. Premutation carrier females can have premature ovarian failure and premutation carrier males can have fragile X tremor ataxia syndrome, which resembles Parkinson disease. Full mutation carriers have greater than 200 CGG repeats and are then clinically affected. Full mutation carriers have intellectual disability and the characteristic physical findings. It can affect boys and girls, but boys are more severely affected. Affected boys typically display mild to moderate intellectual disability (only one-third of girls are intellectually disabled), anxiety, hyperactivity, attention-deficit disorder, autism spectrum disorder, and occasionally seizures. Physical features become more apparent with age and include large ears, a prominent jaw and forehead, pesplanus (flat feet,) long and narrow face, and macro-orchidism after puberty. Only half of girls have the physical characteristics. Rett syndrome: is a neurodevelopmental disorder that predominantly affects girls. Patients with Rett syndrome typically have normal early growth and development followed by a characteristic slowing of development, loss of purposeful hand movements, distinctive hand wringing, decreased brain growth (acquired microcephaly), loss of coordination, developmental regression, autistic-like behaviors, seizures, and intellectual disability. Apraxia and breathing difficulties while awake are quite common. Wil7iams syndrome: is a genetic disorder caused by a deletion on the long arm of chromosome 7. These patients display a hoarse voice, cardiovascular anomaly, mild to moderate intellectual disability, unique personality characteristics, and distinctive facial features. Facial features include a broad forehead, short nose with a broad tip, full cheeks, and a wide mouth with full lips. The most common cardiovascular anomaly is supravalvular aortic stenosis. Affected individuals have engaging, friendly personalities. They do well with language skills and music, but have difficulty with visual-spatial tasks. PREP Pearls - The classic phenotype for fetal alcohol syndrome includes growth deficiency, midfacial hypoplasia, smooth philtrum, thin upper lip, micrognathia, short palpebral fissures, epicanthal folds, small jaw, and microcephaly. - There is no reported "safe" level of alcohol use during pregnancy (first 3 months most risky.) - Neurologic problems in fetal alcohol syndrome include attention-deficit/hyperactivity problems, decreased impulse control, poor coordination, poor memory, learning disabilities (especially in math), speech and language delays, intellectual disability, impaired executive function, and sleep problems. ABP Content Specifications(s) Recognize the clinical and laboratory features associated with fetal alcohol syndrome, and manage appropriately

A 5-year-old boy is brought to your office by his mother for evaluation of behavioral difficulties. The mother reports he has always had a small head, motor and speech delays, and poor growth as compared to his peers. Additionally, he has poor coordination, low tone, impulsivity, hyperactivity, and attention difficulties. He has problems with reciprocal play with his peers in kindergarten. He has had significant problems with learning numbers and their usage. He has a known history of a ventricular septal defect that self-resolved as an infant. His weight is 14.5 kg (5th percentile), his height is 101 cm (5th percentile), and head circumference is 46 cm (less than 3rd percentile). Physical examination is remarkable for a shortened fifth finger, as well as the facies shown in Item Q8. Of the following, the MOST likely diagnosis is A. fetal alcohol syndrome B. fetal anticonvulsant syndrome C. fragile X syndrome D. Rett syndrome E. Williams syndrome

A. familial tall stature Klinefelter syndrome (XXY syndrome) occurs in 1:1,000 live male births and is associated with tall stature, gynecomastia, and decreased upper-to-lower body segment ratio. Cognitive development is usually normal, although speech and language delays are frequently noted. Marfan syndrome is an autosomal dominant connective tissue disorder. Tall stature, increased arm span, and decreased upper-to-lower body segment ratio are present. Height is typically above the 95th percentile after 3 years of age. Additional findings include pectus deformities, arachnodactyly, ocular abnormalities (ectopia lentis), and cardiac abnormalities (aortic root dilatation and mitral valve anomalies). In children with open epiphyses, overproduction of growth hormone results in gigantism. Often, some acromegalic features are seen. In young children, rapid growth of the head may precede linear growth. Coarsening of facial features to include broad nose, enlarged tongue, and excessive mandible growth also can occur secondary to elevated growth hormone levels. gigantism is extremely rare and the presentation is usually dramatic. Sotos syndrome is an autosomal dominant disorder associated with rapid postnatal and early childhood growth with an estimated incidence of 1 in 15,000 newborns. It is caused by haploinsufficiency of the NSD1 protein. Classic features include height or a head circumference greater than or equal to 98th percentile, learning disabilities, craniofacial features that include a triangular shaped face with prominent chin, frontal bossing, apparent hypertelorism, and downslanting palpebral fissures. Neonatal hypotonia, feeding problems, advanced bone age, scoliosis, seizures, and cardiac and genitourinary anomalies are also common. PREP Pearls - Familial tall stature is the most likely diagnosis when there is proportional growth (height, weight, and head circumference) consistent with family history and with no other concerning findings. - Syndromic causes of tall stature are commonly associated with abnormal upper-to-lower body segment ratios, developmental delay, and characteristic facial findings. ABP Content Specifications(s) - Differentiate among the causes of tall stature

A 5-year-old boy presents for evaluation of tall stature (height >3 standard deviations). Review of his growth chart reveals he has always been tall (Item Q57). His head circumference has grown similarly and is significantly above the 97th percentile. He is meeting all of his developmental milestones. The child's mother is 162.6 cm (5 ft, 4 in) tall. His father is 193 cm (6 ft, 4 in) tall and was also very tall for his age as a child. Physical examination shows no other abnormal findings. Of the following, the MOST likely diagnosis is A. familial tall stature B. Klinefelter syndrome C. Marfan syndrome D. pituitary gigantism E. Sotos syndrome (cerebral gigantism)

B. epididymitis The boy described in the vignette presents with epididymitis characterized by left scrotal pain and swelling associated with dysuria, fever, a normal cremasteric reflex, and pain relief with elevation of the testis. Epididymitis is more common in older adolescents, especially those who are sexually active, but can occur in prepubertal males and those without a history of sexual activity. The typical presentation is acute or subacute scrotal pain and swelling with urinary frequency, dysuria, and urethral discharge. Fever may be present. On physical examination, the affected testis is in the appropriate position (vertical lie) in the scrotum, and there may be some erythematous overlying skin. Occasionally, an inflammatory nodule can be palpated along the epididymis. The cremasteric reflex should be present, and this finding (in addition to the position of the testis in the scrotum) can distinguish epididymitis from testicular torsion. Relief with elevation of the testis (Prehn sign) may or may not be present. In epididymitis, urinalysis can be unremarkable or pyuria may be present, but the urine culture often is negative. In sexually active boys, the most common pathogen is Chlamydia followed by Neisseria gonorrhea, Escherichia coli, and then viruses. In younger children, Mycoplasma and viruses are most common. Bacterial urinary tract pathogens, such as E coli, causing epididymitis in prepubertal boys and adolescents who are not sexually active, suggest structural abnormalities of the urinary tract. Treatment of epididymitis in children should be directed at the underlying etiology. In prepubertal and adolescent boys who are not sexually active and lack evidence of urinary tract infection, treatment is supportive and includes rest and nonsteroidal anti-inflammatory agents. Whether or not to give antimicrobial therapy is controversial and should be based on the individual patient's presentation and the clinician's practice. Patients with acute cystitis may have lower abdominal complaints, dysuria, pyuria, a positive urine culture (if caused by a bacterial pathogen), and lack testicular complaints. Urethritis is usually caused by a sexually transmitted infection, and is characterized by urethral discharge, dysuria, itching or burning, and heaviness in the testes without pain. However, patients may be asymptomatic when Chlamydia trachomatis is the cause. Urethritis can accompany epididymitis. A scrotal hernia is an inguinal hernia extending into the scrotum. There typically is an obvious bulge within the scrotum that may or may not be associated with pain. Unlike the swelling caused by epididymitis, the bulge from a hernia can be intermittent and increased with Valsalva maneuvers. An incarcerated hernia would be associated with increasing pain, a tender firm mass in the scrotum, and the overlying skin could be erythematous or discolored. Elevation of the testis would not relieve the pain. PREP Pearls - Epididymitis is characterized by scrotal pain and swelling associated with dysuria, a normal cremasteric reflex, and pain relief with elevation of the testis. - Epididymitis is more common in older adolescents, especially those who are sexually active, but can occur in prepubertal males and those without a history of sexual activity. - In sexually active males, the most common etiologies of epididymitis are Chlamydia and Neisseria gonorrheae, followed by Escherichia coli and viruses. - In prepubertal children and those who are not sexually active, Mycoplasma and viruses are the most common causes of epididymitis. - Routine bacterial urinary tract pathogens, such as E coli, causing epididymitis in prepubertal boys and adolescents who are not sexually active, suggest structural abnormalities of the urinary tract. ABP Content Specifications(s) - Recognize the clinical findings associated with epididymitis

A 7-year-old boy with a history of urinary withholding presents to your office with left scrotal pain and swelling that developed gradually over the last 3 to 4 days. He now complains of pain with urination, as well as subjective fever. He denies nausea, vomiting, or abdominal pain. On physical examination, his temperature is 38.5°C, heart rate is 70 beats/min, respiratory rate is 20 breaths/min, and blood pressure is 100/60 mm Hg. The left scrotal area is swollen and tender to palpation. Pain is relieved with testicular elevation. There is a normal cremasteric reflex. Urinalysis results are unremarkable. Of the following, the MOST likely diagnosis is A. acute cystitis B. epididymitis C. scrotal hernia D. testicular torsion E. urethritis

A. attempt removal with cerumen spoon Irrigation may be used to remove small, inorganic, nonabsorptive objects. Irrigation is not appropriate in this case because the paper might simply absorb the water, swell, and become further wedged into the external ear canal. Insects should be killed before removal by instilling 1% to 2% lidocaine, isopropyl alcohol, or mineral oil into the external auditory canal. One should expect to see erythema or tenderness of the ear canal or pain on manipulation of the pinna if external otitis is present. This is not the case here. PREP Pearls - Most foreign bodies in the external ear canal can be removed by the primary care provider using simple techniques. - Irrigation to remove a foreign body is contraindicated if the tympanic membrane is perforated or if the foreign body is vegetable matter or an alkaline button battery. - An urgent referral to otolaryngology is indicated when the aural foreign body is a battery or sharp object, and any time there is concern for tympanic membrane perforation or injury to the external ear canal or middle or inner ear. ABP Content Specifications(s) - Recognize the clinical findings associated with foreign body in the external ear canal

A 5-year-old boy presents to your office for his annual routine health supervision visit. He has been generally healthy, and his growth and development are normal. The mother reports that she thinks he is ignoring her requests more the past few weeks, but she has no other concerns. On physical examination, you notice white fibrous material in the right external ear canal overlying the tympanic membrane. The patient reports mild pain on examination, but denies any preceding trauma or drainage. Hearing screen reveals a threshold of 35 db at 1,000 Hz and 2,000 Hz on the right side, compared to 25 db on the left side. Of the following, the MOST appropriate next step is to A. attempt removal with cerumen spoon B. irrigate the external auditory canal C. prescribe ciprofloxacin otic drops D. refer to audiology for further assessment E. refer to otolaryngology immediately

E. request preferential seating in the classroom The boy in the vignette exhibits the characteristic clinical findings of persistent middle ear effusion (MEE) following acute otitis media (AOM). Given his failed hearing screen and flat (type B) tympanogram, he should have preferential seating in the classroom until hearing returns to normal. On physical examination, the normal tympanic membrane (TM) should be translucent gray with important landmarks visible and the TM should be in its natural position and mobile. The diagnosis of AOM or otitis media with effusion (OME) is established by documenting the presence of fluid behind the TM. The presence of inflammation differentiates AOM from OME. With OME, the TM may appear opaque or cloudy, an air-fluid level or air bubbles may be evident on examination, and mobility on pneumatoscopy is limited. Pneumatic otoscopy remains the preferred method to diagnose MEE. When pneumatic otoscopy is difficult to perform or when a confirmatory test is desired, tympanometry or acoustic reflectometry are diagnostic adjuncts that may be used. Tympanometry measures compliance of the TM across a pressure range. Normal tympanic membrane compliance leads to a type A curve. A type B curve, as in this vignette, appears flat because of decreased or absent mobility of the TM. Acute otitis media or OME are common causes of limited mobility of the TM. Other causes include tympanosclerosis or TM perforation. The type C curve is shifted to the left, indicating greater negative pressure in the middle ear space, and the curve is often somewhat blunted because of diminished compliance. Type C curves are seen in patients with eustachian tube dysfunction from various causes, including those with resolving or evolving AOM or OME. Acoustic reflectometry is the reflection of sound off the TM. The normal TM absorbs sound, so more sound is reflected when OME or AOM exists. Otitis media with effusion or persistent MEE after an episode of AOM is common and resolves within 3 months in more than 75% of cases. The American Academy of Pediatrics policy on OME recommends watchful waiting for 3 months after diagnosis. Recurrent AOM is defined as 3 or more distinctly documented episodes of AOM within 6 months or 4 or more episodes in a 12-month period. The boy in the vignette shows no signs or symptoms of acute infection nor does he meet criteria for recurrent OM, so treating with amoxicillin-clavulanate is not indicated. Most AOM treatment failures manifest within 48 to 72 hours after initiation of therapy, and if this were the case, then changing the antibiotic coverage would be appropriate. Medical therapies for persistent MEE, such as the use of antihistamines, decongestants, or intranasal corticosteroids, have not proven effective. Patients may experience conductive temporary hearing loss because of the effusion, so providers should make parents aware and ongoing developmental surveillance is important. When OME persists beyond 3 months, or if language delay is a concern, then hearing should be evaluated. Children who are found to have normal hearing despite persistent MEE should be re-evaluated every 3 months. Those who have hearing loss, other symptoms attributable to the effusion (otalgia or vestibular disturbance), or abnormal findings on physical examination beyond simple effusion should be referred to otolaryngology. Referral for allergy testing in a patient who has mild seasonal allergies would not be cost effective as the initial step in care. Pediatric health care providers should advocate for preferential seating in the classroom for patients with temporary or permanent hearing impairment that might affect their school performance. This can be easily achieved immediately and will affect the child's learning by helping him adapt to the environment while he experiences temporary conductive hearing loss because of middle ear disease. Risk factors for developing acute or chronic OME and recurrent AOM include the following: age younger than 2 years, atopy, prolonged bottle use with feeding in the supine position, exposure to smoke, cleft palate or craniofacial anomalies, Down syndrome, child care attendance, compromised immune system, incomplete vaccination, ciliary dysfunction, and history of allergies or chronic sinusitis. These risk factors should be addressed in the case of recurrent AOM or chronic OME. The decision to consider tympanostomy tube placement should be individualized and may be offered for recurrent AOM or if sequelae occur. Sequelae of recurrent AOM or chronic OME include delayed language development, decreased attention, and poor academic performance because of hearing loss—either conductive or sensorineural; TM damage—atrophy, perforation, retraction, atelectasis, scarring; cholesteatoma; and rarely, mastoiditis. PREP Pearls - Primary care providers should advocate for preferential seating in the classroom to optimize learning, until hearing returns to normal, because of the common occurrence of decreased hearing in patients with middle ear effusion. - Tympanic membrane mobility is diminished because of acute or chronic middle ear disease, including infection, effusion, scarring, and perforation. - Risk factors associated with the development of recurrent or chronic middle ear disease should be addressed to reduce the prevalence and severity. - Common sequelae of recurrent or chronic middle ear disease include hearing loss, damage to the tympanic membrane, and rarely, mastoiditis. ABP Content Specifications(s) - Recognize the clinical findings and complications associated with middle ear disease other than otitis media - Recognize the various etiologies of diminished tympanic membrane mobility - Understand risk factors associated with the development of recurrent or chronic middle ear disease

A 5-year-old boy presents to your office for his health supervision visit. Although he is clinically well today, his mother reports that he was treated with amoxicillin 2 months ago for acute otitis media. She is concerned because the school nurse sent a letter home stating that he recently failed his hearing screen. His growth and development are normal. He is up to date on his immunizations. He takes an antihistamine episodically for seasonal allergies. On physical examination, you note that both tympanic membranes are cloudy, grayish white, and have limited mobility on insufflation. His nasal mucosa is pale and boggy. A tympanometry examination reveals a type B curve. Of the following, the BEST initial plan of care is to A. prescribe amoxicillin clavulanate B. prescribe a nasal decongestant C. refer to an allergist for allergy testing D. refer to otolaryngology for tympanostomy tubes E. request preferential seating in the classroom

A. acute respiratory distress syndrome The child in this vignette, who has bilateral infiltrates on chest radiography, a PaO2/FiO2 ratio of 125, and a noncardiogenic cause of respiratory failure in the setting of sepsis, most likely has acute respiratory distress syndrome (ARDS). The pathogenesis of ARDS in this patient is increased pulmonary capillary permeability because of sepsis, septic shock, or systemic inflammatory response syndrome. Acute respiratory distress and failure caused by ARDS occurs in both adults and children, and is a serious cause of morbidity and mortality. According to the 1994 American-European Consensus Conference on ARDS, diagnostic criteria include respiratory failure, a ratio of arterial oxygen tension to fraction of inspired oxygen (PaO2/FiO2) of less than 200, bilateral pulmonary infiltrates on chest radiographs, and a noncardiac cause of pulmonary edema. The term acute lung injury (ALI) indicates patients with a PaO2/FiO2 ratio between 200 and 300. An important event in the pathogenesis of ARDS is the breakdown of the alveolar-pulmonary capillary barrier, which leads to the leakage of proteinaceous fluid into the airspaces. This causes decreased lung compliance, alveolar collapse, and disordered gas exchange marked by ventilation-perfusion mismatch. Patients often breathe rapidly and shallowly and become hypoxic because of decreased compliance and intrapulmonary shunting. Surfactant production and function is also impaired by alveolar type II cell injury, which leads to further alveolar collapse. The repetitive opening and closing of lung units in the face of increased surface tension exacerbates the inflammatory cascade, leading to the secretion of proinflammatory cytokines, such as tumor necrosis factor α and interleukins, which exacerbates the cycle by increasing capillary permeability. Acute respiratory distress syndrome can be caused by direct lung injury from pneumonia, ventilator-induced lung injury, chest trauma, aspiration pneumonitis, acute chest syndrome in sickle cell disease, or indirectly from systemic causes such as sepsis, transfusion-related lung injury, pancreatitis, trauma, or systemic inflammation from other numerous causes. Landmark investigations performed by the ARDS Research Network have led to the widespread acceptance of the "open lung" strategy of ventilator management in ARDS. The aim is to protect the lungs from further injury by maintaining alveolar recruitment with high positive end-expiratory pressure and by limiting inflation pressures and FiO2. The clinician targets lower tidal volumes of 4 to 6 mL/kg and accepts higher level of partial pressure of carbon dioxide (PCO2), which is a strategy known as "permissive hypercapnia." Although congestive heart failure is common in children with leukemia, it is unlikely in this patient because she has received 100 mL/kg in fluid resuscitation and has no palpable spleen or liver. Pulmonary embolism is rare in this age group and is not associated with sepsis, which is the acute condition of this patient. Pulmonary hemorrhage can occur in patients who have insufficient clotting from bone marrow suppression, but bleeding severe enough to cause respiratory failure usually coincides with hemoptysis or the presence of blood in the endotracheal tube. Pulmonary hypertension can cause hypoxia, but is not associated with sepsis, nor is it a common cause of respiratory failure in children with leukemia. Acute respiratory distress syndrome is a condition of respiratory failure in adults and children marked by decreased lung compliance and hypoxia from ventilation-perfusion mismatch and intrapulmonary shunting. It should be treated with the evidence-based guidelines of lung recruitment with positive end-expiratory pressure, and limitation of tidal volume, inflation pressures, and FiO2. PREP Pearls - Acute respiratory distress syndrome can be caused by direct lung injury or indirectly from nonpulmonary inflammatory conditions. - Acute respiratory distress syndrome is associated with pulmonary alveolar-capillary damage, decreased surfactant production and function, and proinflammatory cytokines. - Acute respiratory distress syndrome should be treated to protect the lungs, minimizing inflation pressures and oxygen toxicity, and accepting higher PCO2. ABP Content Specifications(s) - Recognize the clinical features of acute respiratory distress syndrome, including associated sequelae

A 5-year-old girl was admitted from the emergency department because of fever and lethargy. She was recently diagnosed with acute lymphoblastic leukemia and is in the middle of her induction cycle of chemotherapy. She was tachycardic and hypotensive while in the emergency department and received a total of 100 mL/kg of 0.9% saline in boluses. She also was started on a dopamine drip of 5 μg/kg per min. Blood cultures were drawn, and she was started on empiric antimicrobial therapy, including vancomycin and cefepime. Due to respiratory distress and hypoxemia, she required intubation and mechanical ventilation. Within 24 hours, her blood cultures from both peripheral venipuncture and her indwelling central venous catheter have grown gram-negative bacilli. She is now on relatively high ventilator settings, and her arterial blood gas has a pH of 7.30, PCO2 of 50 mm Hg, and PO2 of 80 mm Hg on FIO2 of 0.60. Vital signs show a temperature of 38.5°C, heart rate of 130 beats/min, respiratory rate of 30 breaths/min, blood pressure of 90/30 mm Hg, and SpO2 of 94%. Physical examination shows an intubated and sedated girl that awakens to stimuli, with no focal neurologic findings. Cardiovascular examination shows tachycardia with regular rhythm. She has warm extremities, with capillary refill less than 1 second. Lungs have scattered crackles and decreased air movement bilaterally. Abdomen is soft, nontender, and nondistended with no palpable spleen or liver. A chest radiograph is shown: (bilateral infiltrates.) Of the following, the MOST likely cause of her respiratory failure is A. acute respiratory distress syndrome B. congestive heart failure C. pulmonary embolism D. pulmonary hemorrhage E. pulmonary hypertension

E. oral electrolyte replacement The child described in the vignette has evidence of mild to moderate dehydration, most likely caused by acute viral gastroenteritis, and should be treated with oral electrolyte replacement. This is the preferred first-line therapy for fluid and electrolyte losses in children without severe dehydration, independent of the etiology of dehydration or the patient's age. The desired properties of an oral replacement solution include: - Total osmolarity near 245 mmol/L - Glucose concentration less than 20 g/L - Sodium concentration 75 mEq/L (75 mmol/L) - Equivalent millimolar concentrations of glucose and sodium - Potassium concentration 20 mEq/L (20 mmol/L) - Base (HCO3) concentration near 30 mEq/L (30 mmol/L) Properties of commonly used rehydration solutions versus various beverages are shown. Children with severe dehydration should be treated with an intravenous fluid (IVF) bolus of normal saline, followed by IVF therapy. In cases of severe dehydration, careful calculation of fluid and electrolyte losses to determine the rate of fluid and electrolyte replacement is necessary to avoid serious complications. Children who fail oral electrolyte replacement should be treated with IVF therapy until oral electrolyte replacement therapy is tolerated. Most commonly, acute diarrheal illnesses in children are self-limited, caused by viral pathogens, and do not require pharmacologic therapy. Data do not support the efficacy of antidiarrheal agents in the treatment of acute diarrhea or gastroenteritis in children. Bismuth subsalicylate is an antisecretory agent that can cause toxicity from salicylate absorption. It is rarely used in the management of diarrheal illness in children. Loperamide, an antimotility agent, can be used in children 2 years of age and older for chronic diarrhea associated with inflammatory bowel disease, and for chronic functional diarrhea to decrease the volume of stool from an ileostomy. The side effects of loperamide can be severe, including lethargy, paralytic ileus, toxic megacolon, coma, and even death. Adsorbents such as kaolin-pectin can be used in children 3 years of age and older. They bind mucus and toxins in the gastrointestinal tract and increase the bulk of feces; however, kaolin does not decrease fluid or electrolyte losses or the number of stools passed, and data from high-quality clinical trials fail to demonstrate its efficacy in the treatment of acute pediatric gastroenteritis. PREP Pearls - Oral electrolyte replacement is the preferred first-line therapy for fluid and electrolyte losses in children with mild to moderate dehydration. - Acute diarrheal illnesses in children are usually self-limited, caused by viral pathogens, and do not require pharmacologic therapy. - Bismuth subsalicylate is an antisecretory agent that can cause toxicity from salicylate absorption. - Adverse effects of loperamide can include lethargy, paralytic ileus, toxic megacolon, coma, and even death. - Adsorbents such as kaolin-pectin bind mucus and toxins in the gastrointestinal tract and increase the bulk of feces, but do not decrease fluid or electrolyte losses or the number of stools passed. ABP Content Specifications(s) - Apply age-appropriate guidelines in the use of anti-diarrhea medicines

A 5-year-old previously healthy girl presents to your office with a 1-day history of nonbloody diarrhea. Her mother reports that she developed fever to 38°C, vomited 3 times yesterday, and then began having watery stools without mucus. The mother estimates that there have been 10 episodes of diarrhea in the last 24 hours. She is unable to determine the child's urine output because of the child's frequent diarrheal stools. The girl has not been eating, but she has been drinking water readily. On physical examination, her temperature is 38°C, heart rate is 130 beats/min, respiratory rate is 28 breaths/min, and blood pressure is 90/60 mm Hg. The child is tired, but non-toxic appearing. She is irritable, but consoled by her mother. Mucous membranes are mildly dry and capillary refill is 2 seconds. The remainder of the physical examination is unremarkable. Of the following, the MOST appropriate next step in the management of this patient's condition is to prescribe A. bismuth subsalicylate B. intravenous fluid bolus C. kaolin-pectin D. loperamide E. oral electrolyte replacement

A. oral clindamycin It is classified as bullous (always caused by S aureus) or nonbullous (caused by S aureus or streptococci, including group A β-hemolytic Streptococcus [GAS, Streptococcus pyogenes]) impetigo. The nonbullous form occurs in approximately 70% of cases. Less frequently, ecthyma can occur, which is a form of impetigo characterized by deeper ulcerative lesions extending through the epidermis and surrounded by raised margins. Impetigo can result in poststreptococcal acute glomerulonephritis, but rheumatic fever generally does not occur. - Penecillin: Good for GAS, not staph - Bactrim: Good for staph, not GAS - Bacitracin, Neomycin: Not effective vs MRSA - Mupirocin/Bactroban: good for GAS + staph but rising resistance in some communities PREP Pearls - Children with impetigo typically present with erythematous papules and vesicular lesions on the limbs or the face that rupture, and form brownish or honey-colored crusts over superficial erosions. - Impetigo can be caused by methicillin-susceptible Staphylococcus aureus, methicillin-resistant Staphylococcus aureus (MRSA) or hemolytic streptococci (primarily group A Streptococcus or Streptococcus pyogenes). - Clindamycin has activity against most isolates of staphylococci and streptococci, including MRSA. - Oral antibiotic therapy for the treatment of impetigo should be based on the disease severity and local antimicrobial resistance patterns. - Topical therapy with mupirocin or retapamulin can be considered in treating impetigo depending on the severity of illness. ABP Content Specifications(s) - Recognize the clinical manifestations of impetigo and manage appropriately - Recognize the common pathogens associated with impetigo

A 6-year-old boy arrives at your clinic with a 1-week history of a rash on his face. The parents report that the rash initially started as "red bumps" that became large and seemed to be filled with fluid. Then the lesions "popped" and the rash developed into brownish plaques. His temperature is 37°C and vital signs are normal for his age. On physical examination, there are tender areas of denuded skin with surrounding erythema and overlying brownish crusts on the left cheek and around the mouth, nares, and under the eyes. The conjunctivae are not involved. You note shotty cervical lymphadenopathy. The remainder of the physical examination is normal. Of the following, the MOST appropriate antimicrobial therapy for this patient is A. oral clindamycin B. oral penicillin C. oral trimethoprim-sulfamethoxazole D. topical bacitracin E. topical neomycin

E. tobacco smoke If a parent is unwilling or unable to quit smoking, the option of not smoking in the same room where the children sleep or play should be suggested. If feasible, parents and caregivers should be instructed to restrict their smoking to outside the home. PREP Pearls - Environmental tobacco smoke exposure can trigger worsening of asthma control and be associated with poor asthma outcomes. - Strong odors, aerosol sprays, and ambient particulate matter can trigger acute bronchospasm. - Avoidance of exposure to environmental irritants is recommended to gain optimal asthma control. ABP Content Specifications(s) - Identify the common environmental irritants present in the home that can contribute to respiratory disease in children - Recognize the possible side effects of environmental tobacco smoke exposure

A 6-year-old boy is brought to your office for an urgent visit because of wheezing he has experienced since returning from his father's house this past weekend. Apart from faint end-expiratory wheezes bilaterally, the rest of the physical examination is unremarkable. You order administration of nebulized albuterol in your office. You review his symptoms and medication use with his mother, who reports that he has been using his albuterol almost daily for the past 3 months, ever since the parents separated. The child visits with his father every other weekend. His mother voices her concerns that there are environmental triggers in the father's house that make the child's asthma worse. The mother states that the father's girlfriend wears strong perfumes and likes burning incense and candles. There are no pets or history of visible mold growth in either house. The mother smokes tobacco. She states that she hopes to quit smoking eventually, but with the stress of the separation, she has been smoking more frequently. Of the following, the environmental trigger MOST likely to contribute to this child's symptoms is A. air freshener B. cleaning agents C. incense sticks D. perfume E. tobacco smoke

B. acute disseminated encephalomyelitis The girl in the vignette has acute ataxia, most likely because of acute disseminated encephalomyelitis (ADEM). The viral prodrome a few weeks before the onset of symptoms, and the presence of encephalopathy with new neurologic deficits, differentiates ADEM from other causes of acute ataxia. In ADEM, magnetic resonance imaging of the brain shows asymmetric multifocal abnormalities involving the white matter and often the basal ganglia. Cerebrospinal fluid analysis can show normal or elevated protein and normal or slightly elevated white blood cells, but bacterial and viral cultures and viral polymerase chain reactions studies are always normal. Treatment is with intravenous steroids, or sometimes with immunoglobulin intravenously. Prognosis for recovery is good, although neurocognitive deficits may persist. Acute cerebellar ataxia: presents with acute ataxia a few weeks after a viral infection. However, it does not present with other neurologic deficits such as the right hemiparesis seen in the girl in the vignette. Basilar migraine: can present with ataxia, but headache is prominent, encephalopathy is absent, and the typical clinical course lasts hours, not days or weeks Friedreich ataxia: is a chronic, progressive ataxia and does not present with encephalopathy (low frataxin in mitochondria leads to cellular damage.) Opsoclonus myoclonus ataxia syndrome: typically affects infants and younger toddlers, and does not present with encephalopathy. Opsoclonus: is an erratic conjugate jerking of the eyes in all directions, and should be differentiated from nystagmus: which is a rhythmic conjugate eye jerking, typically in just one direction. Although not one of the responses, a posterior fossa tumor: can present with acute or subacute ataxia. Due to the location in the posterior fossa, these tumors typically also present with signs of increased intracranial pressure such as worsening headaches or lateral gaze palsy. Abnormal gait, due to spasticity or hemiparesis such as found on the examination of the girl in the vignette, can also be seen. PREP Pearls - Acute ataxia with encephalopathy and additional neurological deficits differentiates ADEM from other causes of acute ataxia. - In acute cerebellar ataxia, there are no mental status changes. ABP Content Specifications(s) Recognize the presentation of acute ataxia

A 6-year-old girl has a 1-day history of walking like she is "tipsy." Her parents report that on awakening this morning, she could not sit upright without falling over and that when she walks, she looks like she is drunk. In addition, she has been "sleepy" all day and slept 12 to 16 hours every night for the past week. She had fever and cough 2 weeks previously. Her parents deny any head injuries or exposure to toxins or medications. Her physical examination reveals a temperature of 37.9°C, blood pressure of 92/49 mm Hg, heart rate of 98 beats/min, and respiratory rate of 20 breaths/min. The girl is awake, but does not make eye contact; she follows 1-step commands slowly. She has lateral nystagmus in both eyes when looking to the left; her right arm and leg are slightly weaker than her left arm and leg. When sitting upright, she cannot maintain her balance and when placed standing she wobbles; her gait is wide-based and unsteady. The remainder of the physical examination is unremarkable. Of the following, the MOST likely diagnosis is A. acute cerebellar ataxia B. acute disseminated encephalomyelitis C. basilar migraine D. Friedreich ataxia E. Opsoclonus-myoclonus ataxia syndrome

C. vancomycin and ceftriaxone The redness, swelling, and pain in this boy's eye, including pain on movement as described in this vignette, raise concern for orbital cellulitis. Additional findings in orbital cellulitis may include proptosis, chemosis, ophthalmoplegia, and blurred vision. In view of the inability to open the boy's eye, computed tomography examination with contrast would be indicated to assess the extent of the infection. Orbital cellulitis, especially in this setting with a 9-day prodrome of upper respiratory tract infection, arises as an extension of ethmoidal sinusitis though the lamina papyracea into the muscle and fat of the orbit. Thus, antibiotic management should include agents active against the common pathogens of sinusitis (eg, Streptococcus pneumonia, Haemophilus influenzae, Moraxella catarrhalis), Streptococcus pyogenes, and Staphylococcus aureus (including methicillin-resistant S aureus [MRSA]). Given the deep-seated nature of the infection, hospital admission with parenteral antibiotic therapy is generally indicated. Of the regimens listed, vancomycin and ceftriaxone provide the best coverage for this array of bacteria. The other regimens do not provide adequate coverage for H influenza or M catarrhalis, and cephalexin and gentamicin may not penetrate well into the orbital space. Clindamycin and ceftriaxone would be another appropriate regimen in this setting. If MRSA is not a concern in a given area, ampicillin-sulbactam would be another appropriate therapeutic regimen. Potential complications of orbital cellulits include local abscess formation, optic nerve compression with visual loss, and intracranial extension that can lead to cavernous sinus venous thrombosis, subdural empyema, intracranial abscess, and meningitis. Given the potential severity of this infection, management may include consultations with specialists in pediatric ophthalmology, pediatric otolaryngology, and infectious disease. In addition to antibiotic therapy, surgery may be indicated in the face of these complications or failure to respond to antibiotic therapy over 24 to 48 hours. Orbital cellulitis must be differentiated from periorbital (or preseptal cellulitis). Clinically, periorbital cellulitis, which is an infection limited to the soft tissues anterior to the orbital septum, may also present with fever, unilateral eye swelling with erythema, and warmth and tenderness over the affected eye. However, with the infection limited to the preseptal region proptosis, pain with eye movement and vision impairment are not observed and intracranial complications do not occur. If there is uncertainty, computed tomography with contrast can differentiate the 2 entities. Outpatient management with oral antibiotic therapy and close follow-up may be considered in a nontoxic child with periorbital cellulitis. PREP Pearls - Periorbital cellulitis is a localized infection anterior to the orbital septum that may present with fever, swelling with erythema and tenderness over the affected eye, but without the proptosis and pain with eye movements seen in orbital infection. - Orbital cellulitis is characterized by redness, swelling and pain over the infected eye with proptosis, chemosis, and ophthalmoplegia. Potential complications of orbital cellulitis include local abscess formation, visual impairment secondary to optic nerve compression, and extension of the infection into the central nervous system. - Orbital cellulitis in children typically arises as a complication of ethmoid sinusitis extending through the lamina papyracea of the lateral ethmoid wall into the orbital space. - Computed tomography with contrast defines the extent of orbital cellulitis and can differentiate orbital from periorbital (preseptal) disease. - Antibiotic management should include agents active against the common pathogens of sinusitis. ABP Content Specifications(s) - Recognize pathogens commonly associated with orbital cellulitis - Plan the appropriate management of orbital cellulitis, including associated complications - Recognize and differentiate the clinical and radiologic findings associated with orbital cellulitis and periorbital (preseptal) cellulitis - Plan the appropriate diagnostic evaluation of orbital cellulitis - Understand the natural history of orbital and periorbital (preseptal) cellulitis

A 7-year-old boy is admitted for evaluation and treatment of swelling and redness around his left eye. His illness began with a 9-day history of fever to 38.9°C, cough, congestion, and runny nose. On the evening prior to admission, he complained of mild pain in his left eye, but was able to watch television without difficulty seeing the screen. On the morning of admission, he woke up with marked swelling, redness, and increased pain in his left eye. He is now unable to open the eye. The boy's temperature is 38.7°C, pulse rate is 110 beats/min, blood pressure is 107/75 mm Hg, and respiratory rate is 22 breaths/min. Examination shows swelling and erythema surrounding the left eye. He is unable to open the eye and complains of pain on attempts to move the eye. You are unable to assess extraocular movements because of the swelling. His nose is congested, and his pharynx is unremarkable. His neck is supple and the remainder of the examination findings are unremarkable. Of the following, the BEST initial treatment for this patient is A. cephalexin and clindamycin B. clindamycin and gentamicin C. vancomycin and ceftriaxone D. vancomycin and gentamicin E. vancomycin and rifampin

D. check a von Willebrand panel The child in the vignette with nosebleeds, bruising, and a family history suggestive of autosomal dominant inheritance has a presentation most consistent with von Willebrand disease (vWD), therefore the most appropriate next step in management would be to order a von Willebrand panel. Von Willebrand factor (vWF) promotes platelet adhesion and aggregation, and acts as a carrier protein for factor VIII. A deficiency or defect in vWF impairs platelet function and leads to a more rapid degradation of factor VIII. This can cause mucocutaneous bleeding symptoms, such as menorrhagia, epistaxis, or hemorrhage after tonsillectomy. Von Willebrand disease is the most common inherited bleeding disorder, and is caused by a quantitative or qualitative defect in the vWF protein. Type 1 vWD is an autosomal dominant partial deficiency of vWF. It accounts for 75% of all diagnoses of vWD, with an estimated prevalence of up to 1% of the population. It is associated with a mild to moderate risk of bleeding. In many patients with type 1 vWD, the vWF activity may not be low enough to lower the factor VIII activity below the normal range. In that case, the partial thromboplastin time (PTT) will remain normal, as seen in the boy in the vignette. Because vWF does not have a direct effect on prothrombin time (PT)-based factors (factor 7 and tissue factor), the PT will also be normal in vWD. Type 3 vWD is an autosomal recessive homozygous deficiency (ie, absence) of vWF, and leads to severe disease that can mimic hemophilia. Type 2 vWD has several variants, all of which result from qualitative defects (abnormal functioning) in the vWF protein. Making the diagnosis of vWD can be challenging because many factors can transiently increase vWF levels, such as stress (eg, surgery, anxiety, exercise), inflammation, and estrogen. Samples must be handled properly and processed in a timely manner to avoid proteolysis and platelet contamination. For this reason, the National Heart Lung and Blood Institute (NHLBI) states that "repeated testing for vWD is sometimes needed to identify low levels of vWF." According to the NHLBI vWD expert panel clinical practice guidelines published in 2007, the criteria for diagnosis of vWD are the following: Clinical criteria: Personal or family history of mucocutaneous bleeding or physical evidence of mucocutaneous bleeding. Laboratory criteria: vWF activity assay (eg, as measured by ristocetin cofactor assay) less than 30 IU/dL. These NHLBI criteria do not preclude the diagnosis of vWD in patients with only mildly decreased vWF activity levels of 30 to 50 IU/dL, if there is a strong personal or family history of bleeding symptoms. The treatment options for patients with vWD include therapy to increase vWF levels and antifibrinolytics. Desmopressin 1-desamino-8-D-arginine vasopressin (DDAVP) administered intravenously or intranasally (only Stimate® has the appropriate concentration of DDAVP for use in bleeding disorders) can cause release of vWF from the endothelial cells. Von Willebrand factor levels should be checked before and after administration of this medication, in either form, to document an adequate rise in vWF levels in response to this medication. In patients who do not have a sufficient response to DDAVP, or in cases of acute life-threatening hemorrhage, vWF can be immediately replaced using plasma-derived concentrates containing vWF and factor VIII. The different plasma-derived concentrates vary in their ratios of factor VIII to vWF, and in their levels of high-molecular weight vWF multimers. Antifibrinolytics such as aminocaproic acid and tranexemic acid are given orally or intravenously and can be useful in controlling mucocutaneous bleeding. These agents inhibit fibrinolysis and thereby help stabilize clots once they are formed. Topical agents such as bovine thrombin and fibrin sealant may be used intraoperatively or in the setting of traumatic wounds. Whenever possible, patients should be referred to a pediatric hematologist or the regional Hemophilia Treatment Center for comprehensive care. The patient in the vignette who presents with recurrent, prolonged nosebleeds, normal PT and PTT, and a family history of paternal relatives with bleeding symptoms should raise concern for possible vWD. If the bleeding symptoms were present on the maternal side of the family, factor VIII deficiency (an X-linked recessive disorder) should be a consideration. Factor VIII deficiency would also cause prolongation of the PTT. Factor XII deficiency is a cause of prolonged PTT, but is not associated with any clinically significant bleeding. Bleeding time was used as a screening test for vWD and platelet disorders in the past, but is no longer recommended because of its poor reliability and low sensitivity. PREP Pearls - Von Willebrand disease (vWD) is the most common inherited bleeding disorder, and is caused by a quantitative or qualitative defect in the von Willebrand factor (vWF) protein. - Making the diagnosis of vWD can be challenging because many factors can transiently increase vWF levels, such as stress (eg, surgery, anxiety, exercise), inflammation, and estrogen. Samples must be handled properly and processed in a timely manner to avoid proteolysis and platelet contamination. For this reason, the National Heart Lung and Blood Institute states that "repeated testing for vWD is sometimes needed to identify low levels of vWF." - A deficiency or defect in vWF can cause mucocutaneous bleeding symptoms, such as menorrhagia, epistaxis, or hemorrhage after tonsillectomy. ABP Content Specifications(s) - Recognize the clinical findings associated with von Willebrand disease - Plan the appropriate management of von Willebrand disease

A 7-year-old boy is brought to your office for a preoperative consultation prior to tonsillectomy. The father is concerned because he himself had bleeding 1 week after a tonsillectomy as a child and required cauterization to control the bleeding. His son has a history of recurrent nosebleeds, 2 to 3 times per week for the past month, with the most recent episode lasting 20 min. The nosebleeds occur from either nostril and do not seem to coincide with upper respiratory infections or allergic rhinitis symptoms. He does not have any other bleeding symptoms. The child had a circumcision as a neonate without any bleeding complications, but has not had any other invasive procedures since then. Family history is also significant for a paternal grandfather who was known to be a "free bleeder." There are no bleeding symptoms on the maternal side of the family. On examination, the child has a few bruises of varying ages on both legs. Examination of the nares reveals injected nasal mucosa with no sites of active bleeding. The remainder of the physical examination is unremarkable. His vital signs are stable. The following are the results of the child's complete blood cell count: White blood cell count, 7,500/µL (7.5 × 109/L), with normal differential Hemoglobin, 9.5 g/dL (95 g/L) Mean corpuscular volume, 72 µm3 (72 fL) Platelet count, 245 × 103/µL (245 × 109/L) Prothrombin time, normal Partial thromboplastin time, normal Of the following, the MOST appropriate next step in management would be to A. check a bleeding time B. check a factor VIII activity level C. check a factor XII activity level D. check a von Willebrand panel E. reassure based on the normal coagulation test results

C. hematuria Alport syndrome (AS) is an inherited disorder of basement membrane collagen characterized by involvement of the kidneys (always), ears (often), and eyes (occasionally). Collagen IV, the major structural component of all basement membranes, is composed of heterotrimers (protomers) of α chains. Genes encoding for the chains have been identified on the chromosomes 13q34 (COL4A1, COL4A2), 2q35 (COL4A3, COL4A4), and Xq22 (COL4A5, COL4A6). Gene defects of the α chain lead to abnormal protein assembly, leading to the characteristic abnormalities of AS. Mutations in COL4A5 result in X-linked dominant AS, the most common mode of inheritance characterized by affected male family members and asymptomatic female carriers (Item C139). COL4A3 and COL4A4 mutations lead to autosomal recessive and dominant AS, respectively, with the disease having similar severity in male and female patients. Hematuria is the earliest manifestation and the universal symptom in patients with AS. It is usually detected in childhood or even at birth. Seventy-four percent of patients have hematuria by the age of 6 years. Patients with AS may present with gross/macroscopic hematuria or asymptomatic microscopic hematuria. Male patients fare poorly, with progression to renal failure either by age 20 years (juvenile type) or 40 years or later (nonprogressive or adult type). Progressive renal failure associated with AS is characterized by later onset of increasing proteinuria, elevated serum creatinine, and the development of hypertension and anemia. Important extrarenal manifestations of AS include hearing loss and lenticonus. Bilateral sensorineural hearing loss (initially high frequency and progressively involving lower frequencies) develops over the first decade, with boys being more severely affected. Patients in early stages of AS usually have a normal renal ultrasound with no evidence of hydronephrosis, obstruction, or renal parenchymal abnormality. As renal injury progresses, the renal parenchyma may show increased echogenicity consistent with medicorenal disease (nonspecific changes in acute or chronic renal failure). PREP Pearls - Hematuria is the earliest manifestation and universal symptom in patients with Alport Syndrome (AS). - X-linked dominant is the most common mode of inheritance of AS. Male patients have a worse prognosis with invariable progression to renal failure, whereas female patients are usually asymptomatic carriers and rarely develop renal failure. - Bilateral sensorineural hearing loss in AS develops over the first decade. - Renal ultrasonography is usually normal in patients in the early stages of AS. ABP Content Specifications(s) - Recognize the renal findings associated with Alport syndrome

A 7-year-old boy presents to your office for a health supervision visit. He is in second grade and doing well at school. His family history is significant for several male members who have deafness and renal failure occurring between the ages of 16 and 28 years (mother is carrier, two uncles are positive). His vital signs show a temperature of 37.7°C, heart rate of 80 beats/min, respiratory rate of 16 breaths/min, and blood pressure of 112/60 mm Hg. He has unremarkable findings on physical examination. His mother is worried about potential kidney problems in her son. Of the following, the MOST common early finding in this familial condition is A. anemia B. elevated creatinine level C. hematuria D. hydronephrosis E. hypertension

A. echocardiogram The boy in the vignette presents with progressive weakness and hypertrophy of the proximal muscles beginning in the pelvis and lower extremities, then moving to the shoulders and neck, and eventually involving the respiratory muscles. His age in the elementary school range indicates that he most likely has classic Becker muscular dystrophy (BMD). Patients with BMD typically have elevated creatine kinase concentrations (5 times that of normal), as also seen in Duchenne muscular dystrophy (DMD). However, patients with DMD have creatine kinase concentrations typically more than ten times those of normal. Patients with BMD present with weakness of the quadriceps femoris initially, proceeding to more diffuse proximal muscular weakness, activity-induced cramping, flexion contractures of the elbows, scoliosis, and wheelchair dependency after age 16 years. Patients with DMD become wheelchair dependent before age 13 years, with symptoms presenting between 3 and 5 years of age. These patients often have delayed motor milestones. Patients with both BMD and DMD will develop a Gower maneuver. This involves the patient's use of the arms to assist the weak pelvic girdle muscles when rising to an upright position. The neck flexor muscle strength is preserved in BMD, whereas in DMD the neck muscles are eventually affected. Dilated cardiomyopathy is the most common cause of morbidity and mortality in BMD, usually presenting in the second decade of life. The average age at death is in the mid-40s age range. The dystrophinopathies are caused by DMD gene deletions, duplications, or mutations affecting the dystrophin protein. In DMD, the protein function is absent. In BMD, the disease has a later onset because the protein is partially functional. The dystrophinopathies are X-linked recessive disorders with two-thirds of patients possessing a positive family history. One-third of cases are de novo or spontaneous new mutations. These are X-linked disorders, thereforeboys are typically most severely affected and female heterozygous carriers have an increased risk for dilated cardiomyopathy as they age. In this vignette, the patient's sister should be screened with serial echocardiography over time. Muscle biopsy findings are abnormal in affected boys. Treatment involves aggressive management of the dilated cardiomyopathy with medications and sometimes heart transplantation. Corticosteroids improve motor function and strength in patients with dystrophinopathies, thus enabling patients to prolong the ability to ambulate. Patients are closely followed by cardiology beginning around age 10 years (at least biannually) for the onset of the dilated cardiomyopathy. They also should be followed by neurology, orthopedics, and pulmonology for the progressive muscle weakness and eventual respiratory compromise. Patients should have regular pulmonary function tests and be evaluated for the need for chest physiotherapy, cough-assist devices, and nocturnal positive-airway pressure devices to help prevent heart failure. Orthopedic physicians follow the contractures and risk for scoliosis closely. Patients with dystrophinopathies should receive the pneumococcal vaccine, the annual influenza vaccination, and receive physical therapy to optimize their mobility and minimize contractures. Nutritional status should be followed closely. Heterozygous female carriers should have cardiac surveillance at least every 5 years, beginning in the second to third decade of life secondary to the elevated risk of dilated cardiomyopathy. This is often overlooked by primary care providers who do not realize that female carriers are at risk for cardiac involvement as well. Another well-known disorder of muscular weakness in childhood is spinal muscular atrophy (SMA). Spinal muscular atrophy is predominantly an autosomal recessive disorder caused by degeneration and loss of the anterior horn cells in the brain stem and the spinal cord leading to progressive muscle weakness and respiratory failure. Age at onset ranges from birth to young adulthood with 5 different subtypes: SMA type 0 (prenatal onset with joint contractures), SMA type I (onset < 6 months of age), SMA type II (onset 6-12 months of age), SMA type III (onset after age 12 months), and SMA type IV (adult onset). Patients with types III and IV can live normal lifespans but with evident disability. Common symptoms, especially in SMA I, SMA II, and SMA 0, include joint contractures, sleep problems, feeding difficulties, failure-to-thrive, scoliosis, respiratory difficulties progressing to failure, and eventual wheelchair dependency. One form of SMA is X-linked, whereas most forms are autosomal recessive. For the sister of a patient with BMD, we would not expect abnormal findings on electromyography or nerve conduction velocity testing because heterozygous females typically only manifest the dilated cardiomyopathy as they age, unless significant skewed X-inactivation (Lyonization), an Xp21.2 deletion, or an X-chromosome rearrangement is present, yielding a more global dystrophic picture. A skeletal muscle biopsy would be invasive and not necessary. Children affected with DMD or BMD and heterozygous female carriers do not manifest renal dysfunction or anatomic abnormalities, so renal ultrasonography would be unwarranted. PREP Pearls - Dilated cardiomyopathy, which usually presents in the second decade of life, is the most common cause of morbidity and mortality in Becker muscular dystrophy (BMD) and Duchenne muscular dystrophy (DMD). Heterozygous female carriers are at risk for dilated cardiomyopathy as well. - Duchenne muscular dystrophy and BMD are X-linked recessive disorders presenting with progressive proximal muscular weakness and calf hypertrophy in males. Duchenne muscular dystrophy presents in early childhood, typically between 3 and 5 years of age. Becker muscular dystrophy typically presents in later childhood or adolescence. They differ in age at presentation, rate of degeneration, and severity. - Laboratory abnormalities in DMD and BMD include elevated creatine kinase levels: five times normal in BMD and ten times normal in DMD. - Spinal muscular atrophy is predominantly an autosomal recessive disorder caused by degeneration and loss of the anterior horn cells in the brain stem and the spinal cord, leading to progressive muscle weakness and eventually respiratory failure in the more severe subtypes. The various subtypes vary in their age at presentation, severity, and clinical progression. ABP Content Specifications(s) - Understand the inheritance pattern in a patient who has a neuromuscular disorder (eg, muscular dystrophy, spinal muscular atrophy

A 7-year-old boy presents to your office with concerns of activity-induced muscular cramping and muscular weakness that have been progressively worsening and interfering with his ability to compete in sports. His family history is notable for a maternal male first cousin with similar symptoms who is currently 20 years old and wheelchair dependent. On physical examination, you note muscle weakness that is most prominent proximally with moderate calf hypertrophy. Neck flexor muscle strength is preserved. Neurologic examination findings are otherwise normal. The boy's serum creatine phosphokinase concentration is 5 times the normal range. Myoglobinuria is noted during a period of muscle cramping. Genetic testing is performed, and his mother and older sister are confirmed to be carriers for this disorder. Of the following studies, the MOST appropriate study for this patient's sister is A. echocardiogram B. electromyography C. nerve conduction velocity testing D. renal ultrasonography E. skeletal muscle biopsy

C. sexual abuse PREP Pearls - Inappropriate behavior and genital lesions are suggestive of sexual abuse as the etiology of dysuria in children. - Balanitis, the inflammation of the glans penis, is usually seen in association with diaper dermatitis in young children. - Vesicular lesions of the anogenital area may be caused by sexually transmitted infections (syphilis or herpes simplex virus infection), other viruses (Epstein-Barr virus), and other systemic conditions (Behçet disease, Crohn disease). ABP Content Specifications(s) - Recognize the etiology of dysuria in patients of various ages - Plan the appropriate diagnostic evaluation of dysuria in patients of various ages

A 7-year-old boy presents to your office with painful urination of 1 day duration. His mother mentions that for the last few weeks he has been frequently touching his genitalia, and for the last few months, he has occasionally requested her to touch his genitalia. She attributes this to age-appropriate curiosity. She denies any complaints of fever, trauma, blood in urine, increased frequency, or suspicion of child abuse. Physical examination is significant for small vesicular lesions on the glans. Of the following, the MOST likely cause of this patient's condition is A. local trauma B. nonspecific (chemical) urethritis C. sexual abuse D. urinary tract infection E. viral balanitis

A. bone age Precocious puberty is defined as the development of pubertal signs before 8 years of age in girls and before 9 years of age in boys. In 1999, the Pediatric Endocrine Society recommended lower age limits for girls: before 6 years of age for blacks and before 7 years of age for whites. However, considerable controversy still exists concerning how best to define precocious puberty. A significant risk associated with precocious puberty is loss of final adult height caused by early fusion of epiphyseal growth plates from sex steroid exposure. The best way to determine if the child in this vignette will have impaired final height is to make a height prediction based on bone age. A bone age radiograph (radiograph of the left hand and wrist) can be compared to that expected for the patient's current chronological age, and along with the patient's current height, can be used to make a height prediction. Numerous bone age atlases and standards exist to make this height prediction, which is usually done by a pediatric endocrinologist. When the bone age is delayed in an otherwise normal child, often there will be period of catch-up growth, and final height will be predicted to be higher than the current growth percentile. Similarly, when bone age is advanced, there is potentially less time for catch-up growth, and final height may be compromised. The bone age radiograph shows a bone age of 10 years when the child was only 7 years old, significantly advanced from the child's chronological age. This led to a height prediction that she would be much shorter than her midparental height and serves as the main impetus to consider further evaluation. It is important to note that an advanced or delayed bone age does not necessarily indicate an underlying pathological problem. For example, taller children with constitutionally advanced growth and short children with constitutionally delayed growth will have advanced and delayed bone ages, respectively, but will both ultimately grow to normal heights. The bone age study can be used by the consulting endocrinologist to help decide whether this patient simply has constitutionally advanced growth or may need additional workup. A brain magnetic resonance imaging scan is important to consider, especially if there are any neurologic symptoms or if the family ultimately opts to treat the child for precocious puberty. However, only 2% of girls with central precocious puberty starting between 6 and 8 years of age have abnormal brain imaging studies. The incidence of relevant findings is much higher for girls with central precocious puberty starting before 6 years of age. A better first test would be a bone age study. Laboratory results are also important in the evaluation of this child. A baseline luteinizing hormone (LH) level of greater than 0.3 IU/L, a gonadotropin-releasing hormone-stimulated LH level above 4 to 8 IU/L, and a random estradiol level in the pubertal range (> 20 pg/mL or > 73 pmol/L) may provide supportive evidence for progressive central precocious puberty. However, this child has evidence of puberty on physical examination and concern that breast development is progressing, so the question is not whether she has pubertal findings, but rather whether she would benefit from treatment. This is where the bone age study is most helpful. The compromised final height prediction should then prompt these additional laboratory tests, which can be sent by the consulting endocrinologist. A 17-hydroxyprogesterone level test is used to screen for 21-hydroxylase deficiency, a form of congenital adrenal hyperplasia. In this condition, signs of adrenarche (androgen exposure) such as pubic hair would be expected. As early breast development is a separate issue, it would not be appropriate to include a 17-hydroxyprogesterone level in the evaluation of a patient for central precocious puberty. PREP Pearls - Bone age can be used to predict final height, using one of many standard atlases and the patient's current age and height. - A compromised final height prediction from an advanced bone age should lead to consideration of additional workup for many conditions. ABP Content Specifications(s) - Understand the relationship between bone age and chronologic age

A 7-year-old girl presents to your office for concern for precocious puberty with progressive breast development. She is overweight but otherwise healthy. Her mother had her first period at age 15 years and is concerned that if her daughter is starting puberty now, she may be at risk for early menstrual periods. On physical examination, you are able to palpate what feels like a small amount of breast tissue but also an appreciable amount of adipose tissue around her breast area. She has no pubic hair. You are concerned for precocious puberty but also consider that she may simply be overweight, and not have true breast tissue present. You need to determine if this patient needs further workup for precocious puberty by an endocrinologist. Of the following, the BEST test to help make this decision is A. bone age B. brain magnetic resonance imaging C. estradiol level D. 17-hydroxyprogesterone level E. luteinizing hormone level

E. refer immediately to otolaryngology This case in the vignette is characteristic of an acquired cholesteatoma and the patient should be referred immediately to otolaryngology. A cholesteatoma is an abnormal accumulation of squamous epithelium in the middle ear and mastoid. It may be congenital or acquired. Fewer than 5% of all cholesteatomas are felt to be congenital. Acquired cholesteatomas are an uncommon result of chronic middle ear disease. The incidence in children ranges from 5 to 15 per 100,000. It is believed that the persistent presence of inflammatory mediators and chronic eustachian tube dysfunction may lead to atrophy of the tympanic membrane (TM) and formation of retraction pockets over time. As a retraction pocket is progressively drawn into the middle ear space because of longstanding negative middle ear pressure, it forms a pouch that traps desquamating cells, thus creating a cholesteatoma. Old tympanostomy tube sites, the posterosuperior portion of the TM, and the pars flaccida are the most common areas of occurrence. Warning signs for cholesteatoma include the following: (1) white round mass behind an intact TM, (2) deep retraction pocket with or without granulation tissue, (3) focal granulation on the surface of the TM, (4) persistently draining ear for more than 2 weeks despite treatment, and (5) new-onset hearing loss in a previously operated ear. As a cholesteatoma grows, it invades and destroys the local structures, including the ossicles and the temporal bone. Complications include sensorineural hearing loss, cranial nerve palsies (CN 6, 7), vertigo, venous thrombosis, and central nervous system infections (brain abscess, meningitis). Because of the invasive nature of cholesteatomas, whenever a cholesteatoma is suspected by the physical examination, prompt otolaryngology consultation is mandatory. Surgical excision is the mainstay of treatment. The risk of recurrence is significant, especially with larger, more infiltrative lesions, so close postoperative follow-up is important. Children with Down syndrome, cleft palate, or other craniofacial anomalies are at risk for recurrent middle ear disease, possibly because of mechanical or anatomic features that predispose to eustachian tube dysfunction. Indeed, children with Down syndrome may have an increased risk of immunodeficiency, so this evaluation should be considered; however, it is not the most important next step in evaluating the child in the vignette who is otherwise well appearing. The combination of persistent ear drainage despite appropriate topical therapy and the findings on physical examination should raise concern for cholesteatoma. Therefore, otolaryngology referral is more urgent. If the patient had more serious bacterial infections, then immune function should be evaluated. Simply continuing the same topical medication or adding a broad-spectrum oral antibiotic is not appropriate therapy, given the duration of drainage and the abnormality of the TM. Performing a culture and sensitivity of the drainage would be helpful to direct treatment of persistent purulent drainage, but this step alone is not sufficient given the concern for cholesteatoma on physical examination. PREP Pearls - Prompt otolaryngology consultation is mandatory whenever a cholesteatoma is suspected on physical examination. - Acquired cholesteatomas occur most commonly in areas of chronic retraction pockets, old perforations, or previous surgical sites of the tympanic membrane. - If left untreated, cholesteatomas will invade and destroy the local structures (ossicles and temporal bone). - The complications of untreated cholesteatomas are serious and include sensorineural hearing loss, cranial nerve palsies, vertigo, venous thrombosis, and central nervous system infections. ABP Content Specifications(s) - Recognize the clinical findings associated with a cholesteatoma and the consequences if left untreated

A 7-year-old girl with Down syndrome presents to your office for evaluation of nonbloody, nonpainful purulent ear drainage for the past 3 weeks. She had tympanostomy tubes placed 2 years ago, but the tubes fell out a few months ago. The mother has been using the topical antibiotic-steroid drops the otolaryngologist gave them previously to treat ear drainage, but there has been no improvement. The patient is afebrile and looks well. On physical examination, you find a deep retraction pocket of the tympanic membrane (Note, the tymphanic membrane is adherent to the stapes). Of the following, the MOST appropriate next step is to A. begin evaluation for immunodeficiency B. give a new prescription of the topical recommended by otolaryngology C. obtain culture and sensitivity of the drainage D. prescribe a broad spectrum oral antibiotic E. refer immediately to otolaryngology

E. serum ammonia Valproate can cause hyperramonemic encephalopathy without transaminitis. Of the choices listed, ammonia is the most likely to be abnormal and causing the sleepiness in the girl in the vignette. It is also possible that she is having subclinical seizures, also called nonconvulsive status epilepticus. If the parents have a medication to stop seizures at home, such as rectal diazepam, they should try this. If this does not cause her mental status to improve, she should be evaluated in the emergency department for other causes of encephalopathy such as infection or electrolyte abnormality. For hyperammonemia caused by valproate, the dose of valproate will need to be reduced or the medication may need to be discontinued altogether. The girl's neurologist should be consulted to help manage her seizures in this setting. Valproate can cause neutropenia and thrombocytopenia, but these do not typically present with sleepiness, so a complete blood count with differential is not the best choice. Renal dysfunction is not likely to occur in this patient with a normal valproate level. Checking levels of free and total carnitine would not be helpful to find a cause of sleepiness. Although valproate has been associated with low levels of free and total carnitine, it is unclear whether this causes any symptoms in children and whether giving carnitine is indicated. The valproate level was within the normal range, so it is unlikely that this is causing the girl's sleepiness; rechecking the level is unlikely to aid in diagnosis. PREP Pearls - Valproate can cause hyperammonemic encephalopathy without transaminitis. - Valproate can cause neutropenia and thrombocytopenia. ABP Content Specifications(s) - Recognize laboratory abnormalities associated with anticonvulsant drug therapy - Recognize side effects and toxicities associated with anticonvulsant drugs

A 7-year-old girl with a seizure disorder has been taking valproate for 1 year. She has not had any seizures during that time, nor any adverse effects from the medication. She is growing and developing well. Her mother brought the girl to your office because she has been unusually sleepy. There was no trauma and no known ingestions, and the parents have been supervising her medications closely. She has had no recent illness. You recommended checking a valproate level and liver enzymes, and now have the following results: - Valproate level, 61.00 µg/mL (423 µmol/L); reference range, 50 µg/mL to 120 µg/mL (347-832 µmol/L) - Alanine aminotransferase, 34 U/L - Aspartate aminotransferase, 41 U/L Of the following, the test MOST likely to reveal the cause of her sleepiness is A. blood urea nitrogen B. complete blood cell count C. free and total carnitine D. repeat valproate level E. serum ammonia

E. ventricular septal defect The patient in the vignette has significant findings on physical examination that suggest structural heart disease. The murmur is holosystolic and obscures S1. That indicates that the murmur begins in a short period when all the valves of the heart are momentarily closed and no blood is moving; this is called isovolumetric contraction. S1 is the heart sound made when the mitral and tricuspid valves close, so this type of murmur is often called regurgitant; it is also the classic murmur of a ventricular septal defect (VSD). This patient has a VSD. The other important findings on this patient's physical examination are the differences in blood pressure in the upper and lower extremities, the radial-femoral delay, and the decreased femoral pulses. All of these findings suggest a coarctation of the aorta, which is often seen with a VSD. S2 is the sound made when the pulmonary and aortic valves close. Usually a split in S2 is detectable, as the pulmonic valve closes after the aortic valve. A respiratory variation is seen in this timing difference. When the right ventricular volume increases (as in inspiration) a more obvious split of S2 into A2 and P2 is seen. If the right ventricular volume is already very large, as can be seen with a large atrioseptal defect (ASD), there will not be as much opportunity for decrease in the timing difference, which creates the "fixed split" of S2. A large ASD will also cause increased flow in the right ventricular outflow tract in systole and may cause a murmur at the left upper sternal border. It is important to listen for a "fixed split" in S2 when a murmur in this location is noted; this will help differentiate it from an innocent flow murmur. If there is a systolic click in this location (left upper sternal border) and a murmur, then pulmonary valve disease would be suspected. In the case of severe pulmonic stenosis, there extend far into systole but will not be holosystolic. The murmur expected from left ventricular hypertrophy and outflow tract obstruction would also be systolic, but not holosystolic: it would not obscure S1. It would begin along the left midsternal border and then radiate up to the right upper sternal border following the left ventricular outflow tract. Physical findings associated with total anomalous pulmonary venous drainage may vary depending on whether there is an associated ASD or venous obstruction. It would not cause a holosystolic murmur as described with a VSD. The physical findings of the patient in the vignette also suggest a coarctation of the aorta. Blood pressure is lower in the legs and the femoral pulses are decreased. Normally, the blood pressure is higher in the legs than the arms. Ideally, the right arm and leg are compared to lessen the chance of an incorrect measurement because of an aberrant right subclavian artery. Blood pressures are not routinely measured in pediatric patients until age 3 years and even then, not in both upper and lower extremities. If a murmur suggesting a VSD is heard, it is important to document femoral pulse quality and strength, and measure the blood pressure in the right arm and leg. PREP Pearls - The lower extremity blood pressures should be higher than the upper extremity blood pressures. - If a ventricular septal defect is suspected on physical examination, co-existing coarctation of the aorta should also be suspected. The femoral pulses and upper and lower extremity blood pressures should be documented. ABP Content Specifications(s) - Plan the initial management of hypertension in a patient with coarctation of the aorta - Recognize the major clinical findings associated with the various types of acyanotic congenital heart disease

A 9-year-old girl, recently adopted internationally, is seen for the first time in your office. She is currently taking no medications and has not had any surgeries. Her weight is at the 50th percentile for age, and her height is at the 75th percentile. She does not appear ill, but is somewhat anxious. Her oxygen saturation is 100% on room air, heart rate is 90 beats/min, and respiratory rate is 20 breaths/min. Her blood pressure in her right arm is 130/90 mm Hg. The blood pressure in her right leg is 90/50 mm Hg. Her neck does not show any jugular venous distention. The chest is clear with equal breath sounds, and her abdominal examination does not show any organomegaly. There is a 2/6 holosystolic murmur at the left midsternal border that obscures S1. Her femoral pulses are 1+ when compared to her radial pulses of 3+, and there is radial-femoral delay. You clinically suspect coarctation of the aorta. Of the following, the other cardiac lesion this child's history and physical findings BEST suggest is A. atrioventricular canal with a large atrial septal defect component B. left ventricular hypertrophy and outflow tract obstruction C. severe pulmonic stenosis D. total anomalous pulmonary venous return E. ventricular septal defect

E. obtain an abdominal radiograph of the abdomen Toxicity arising from iron exposure depends on the amount of elemental iron ingested. The minimum toxic dose and lethal doses of iron are not well established. The main pathophysiologic mechanism for iron toxicity is free radical production and lipid peroxidation, which results in damage to the body's tissues. Local toxicity (manifested as abdominal pain, vomiting, diarrhea, and gastrointestinal [GI] bleeding) results from damage to the GI mucosa; systemic toxicity results from injury to the cardiovascular system and liver. Although discharging the boy from the hospital with appropriate anticipatory guidance may be the ultimate resolution of this case if he remains asymptomatic and significant iron toxicity is ruled out, a thorough evaluation should be made, including laboratory and radiologic studies as described earlier. PREP Pearls - Key diagnostic tests to guide the treatment of children presenting after known or suspected iron ingestion include a serum iron level and abdominal radiography. - Although most children ingesting iron-containing products develop only mild symptoms or no symptoms at all, clinicians should consider them to be at risk for developing life-threatening toxicity until a thorough evaluation proves otherwise. - serum iron levels generally correlate with the likelihood of developing toxicity. Serum iron levels typically peak 2 to 6 hours after ingestion; therefore, most experts recommend 4 hours after ingestion as an optimal time to obtain this level - Management of acute iron poisoning cases begins with aggressive supportive care, with a focus on supporting the airway, breathing, and circulation. Fluid resuscitation to help correct hypovolemic shock is crucial in children with severe toxicity. ABP Content Specifications(s) - Plan the management of a patient who has ingested iron pills

A frantic mother brings her 3-year-old son to the emergency department after he ingested an unknown quantity of vitamin tablets containing iron. Approximately 45 min before arrival, his mother discovered him playing with tablets from a recently purchased bottle of prenatal vitamins. The bottle originally contained 50 tablets. The child's mother inadvertently left the bottle open on the bathroom counter while she ran into the nursery across the hall to check on the boy's newborn sister, who was crying. When questioned, the boy told his mother that he did eat some of the tablets before she discovered him playing with the bottle. She is not sure how many tablets he ingested, but she estimates that as many as 15 tablets could be missing from the bottle. His mother states that he has displayed no symptoms since she found him ingesting the prenatal vitamins and has acted normally. The boy has no significant past medical history, takes no medications, and has no allergies. As you are interviewing the boy's concerned mother, you note that he appears very well. He is happily pretending to read a storybook to his newborn sister. The boy's vital signs are normal for his age, and a complete physical examination yields no remarkable findings. Of the following, the BEST next step in management of this patient is to A. administer activated charcoal solution orally B. administer deferoxamine intravenously C. administer of syrup of ipecac orally D. discharge home with appropriate anticipatory guidance E. obtain an abdominal radiograph of the abdomen

C. injection of intramuscular vitamin K Classic Vitamin K Deficiency Bleeding (VKDB) typically presents with bleeding from the umbilicus, gastrointestinal tract, skin, and circumcision sites between 24 hours and 1 week after birth and has limited morbidity. Conversely, late-onset VKDB presents 2 weeks to 6 months after birth with intracranial hemorrhage in over 50% of cases and is associated with a high morbidity. The incidence of late-onset VKDB is low, with published numbers ranging from 1 to 20 cases per 100,000 births. A British study published in 1992 suggested a possible link between parenteral vitamin K and childhood cancer. Although these results were refuted by a subsequent study published in 1994, some health care providers and families choose to use oral vitamin K. Oral vitamin K requires repeated doses over the first 2 months after birth and can often be obtained only through a compounding pharmacy. Oral vitamin K has not been shown to be as efficacious in preventing late-onset VKDB. The testing of cord blood for blood type is required only if the mother is Rh negative. Neonates who are born to mothers with blood type O may be closely monitored clinically for the development of jaundice, with bilirubin screening performed between 24 and 48 hours after birth PREP Pearls - Parenteral vitamin K should be administered shortly after birth to prevent vitamin K deficiency bleeding. - The American Academy of Pediatrics policy statement on planned home birth can assist pediatricians in supporting women seeking to deliver at home. ABP Content Specifications(s) - Plan appropriate evaluation and management of an infant who was born at home - Recognize the signs, symptoms, and causes of vitamin K deficiency, and manage appropriately

A mother in your practice is planning to have her second baby delivered by home birth under the care of a certified midwife. She is meeting with you to review the plan for her baby following delivery. Her 3-year-old child was born vaginally at term with no complications and was breastfed for 2 years. Her pregnancy has been unremarkable. Her prenatal screen results include blood type, O+; antibody, negative; hepatitis B, negative; rapid plasma reagin, negative; HIV, negative; and group B Streptococcus, negative. The estimated fetal growth has been appropriate for gestational age. She requests no medical interventions, including shots or medications, be done to her baby after birth unless serious harm may occur as a result of their omission. Of the following, the intervention MOST appropriate to recommend after initial bonding of this baby is A. bathing with chlorhexidine gluconate B. immunization with hepatitis B vaccine C. injection of intramuscular vitamin K D. screening blood glucose for hypoglycemia E. testing of cord blood for blood type

E. urine specimen for GC-CT nucleic acid amplification testing and await results The symptom combination of dysuria and urethral discharge, as seen in the adolescent boy in the vignette, is diagnostic for urethritis. Occasionally, a male adolescent may have penile itching and tingling without a discharge; urinary frequency, urgency, or rarely hematuria may also be present. The organisms responsible for urethritis include those that cause urinary tract infections and those that may be sexually transmitted. Almost half of all cases are caused by sexual transmission of Chlamydia trachomatis, often referred to as nongonococcal urethritis (NGU). Usually, C trachomatis urethritis presents with a scant clear discharge; however, infection may vary from being asymptomatic to causing a profuse purulent discharge, as is seen with Neisseria gonorrhoeae infections. The frequency of C trachomatis urethritis reported cases is almost 3 times that of N gonorrhoeae urethritis cases. Both chlamydia and gonorrhea are reportable diseases, and the rate of partner notification and treatment may be improved with a specific diagnosis, therefore it is recommended that testing for these 2 organisms be performed in all symptomatic patients. This is best accomplished with a nucleic acid amplification test performed on a urine specimen, for both C trachomatis and N gonorrhoeae. This test has a high sensitivity and specificity, therefore there is no longer a need for urethral swabs. Other organisms implicated in urethritis include Ureaplasma urealyticum (20%-30% of NGU cases), Mycoplasma genitalium, Gardnerella vaginalis, herpes simplex virus, Trichomonas vaginalis, Staphylococcus saprophyticus, and Escherichia coli. Urinary tract infections are less common in the adolescent years and are not associated with a urethral discharge in either gender. Treatment goals for urethritis include symptom relief, preventing transmission to sexual partners and reducing complications. Untreated C trachomatis may lead to epididymitis, and rarely, prostatitis or Reiter syndrome. Unless the patient is not likely to return, treatment should await test results. Empiric treatment is with azithromycin 1 g orally as a single dose for C trachomatis and ceftriaxone 250 mg intramuscularly for N gonorrhoeae. Patients should be instructed to abstain from sexual intercourse for 1 week after single dose therapy (to prevent sexual transmission) and until all partners are treated (to prevent reinfection). The patient should be tested for other infections, including syphilis and HIV. Counseling should emphasize the proper use of condoms and the need for their consistent use. PREP Pearls - Dysuria and urethral discharge are the most common symptoms of urethritis in male adolescents. - Chlamydia trachomatis is the most common organism causing urethritis in male adolescents. - It is important to treat the sexual partners of patients with C trachomatis or Neisseria gonorrhoeae urethritis. ABP Content Specifications(s) - Recognize the clinical findings associated with urethritis in male adolescents - Plan the appropriate evaluation and management of urethritis in male adolescents

A previously healthy 17-year-old adolescent boy well known to your practice presents with the complaint of intermittent burning during urination for 1 week. He has no urgency, hematuria, or systemic complaints. He admits to intermittent use of condoms when sexually active. On physical examination, he is at sexual maturity rating 5. He is uncircumcised, with moistness and a clear discharge from the urethral meatus. There are no other genital lesions or abnormalities. Of the following, the MOST appropriate next step in diagnosis and management would be to order an A. urethral swab for chlamydia (CT) culture and await results B. urethral swab for gonorrhea (GC) culture and treat before results available C. urethral swab for GC-CT nucleic acid amplification testing and await results D. urine culture and treat him for a urinary tract infection E. urine specimen for GC-CT nucleic acid amplification testing and await results

E. the presence of stool reducing substances Bloody diarrhea can occur and a stool sample may reveal the presence of fecal leukocytes or red blood cells. Clostridium difficile infection after antibiotic exposure is more common in older children and adults. The diagnosis is made by detecting C difficile toxin or organisms in stool. Gastroenteritis caused by protozoa and helminths usually results in a protracted and intermittent disease, characterized by nonspecific abdominal pain and malabsorption (foul-smelling stools, flatulence), in addition to vomiting and diarrhea PREP Pearls - Viral gastroenteritis is generally a clinical diagnosis. It produces an osmotic diarrhea demonstrated by positive stool-reducing substances and a stool pH of less than 6. - Viruses are the most common cause of acute gastroenteritis, especially in young children, in both developed and developing countries. - Since the introduction of routine immunization, rotavirus is a less common cause of viral gastroenteritis than other pathogens. ABP Content Specifications(s) - Identify the pathogens commonly associated with infectious diarrhea in patients of various ages

A previously healthy 2-year-old boy comes to your office with a 3-day history of nonbilious vomiting and nonbloody diarrhea. The parents report that he has vomited 4 times and had approximately 10 watery stools in the last 24 hours. The boy's mother, father, and sibling also have been ill with similar symptoms. On physical examination, his temperature is 37.5°C, heart rate is 140 beats/min, respiratory rate is 30 breaths/min, and blood pressure is 98/72 mm Hg. The boy appears tired and ill but not toxic. His mucous membranes are tacky and capillary refill is 2 to 3 seconds. He is tachycardic with strong peripheral pulses. His abdomen is soft, nondistended, and mildly tender to palpation throughout without rebound or guarding. Bowel sounds are hyperactive. The remainder of his physical examination is unremarkable. Of the following, the finding MOST likely to be associated with this patient's illness is A. a positive stool for ova and parasites B. a positive stool toxin assay C. a stool pH greater than 6 D. the presence of fecal leukocytes E. the presence of stool reducing substances

D. nasal secretions for respiratory syncytial virus The infant in this vignette most likely has apnea from a respiratory tract infection, based on the history of being full term, previously healthy, with upper respiratory tract symptoms, decreased oral intake, and the presence of sick contacts. The duration of the respiratory pause and perioral cyanosis indicates a serious event of apnea. Although several different respiratory tract infections can cause apnea, respiratory syncytial virus (RSV) is one of the most common. Evaluation of nasal secretions for respiratory viruses is the most appropriate diagnostic test. Apnea is the clinical condition of the absence of respiratory air flow. The 3 general categories of apnea include central apnea from insufficient respiratory drive originating from the brainstem, obstructive apnea from upper airway obstruction, and a mixed etiology including both central and obstructive causes. Central apnea in neonates can be caused by a relatively immature brainstem respiratory center and is a very common problem in preterm infants. It can also be caused by other illnesses that can affect the respiratory center in both newborns and older children, such as brain hemorrhage, drugs, seizures, hypoxic injury, or increased intracranial pressure. A mixed etiology of central and obstructive apnea is also common in premature neonates, from the combination of an immature brainstem as well as an underdeveloped upper airway. This problem usually resolves by 37 to 40 weeks of gestational age. Obstructive apnea, which can be caused by airway obstruction at any point from the pharynx to the distal trachea, can occur both in infants and older children. In infants, common causes include macroglossia, Pierre-Robin sequence, subglottic stenosis, and laryngotracheomalacia, leading to intermittent collapse of the airway. In older children, common causes of obstructive sleep apnea include enlarged tonsils or adenoids, nasal polyps, obesity, and insufficient upper airway tone from trisomy 21 or neuromuscular conditions. Central apnea can be distinguished from periodic breathing, which can reflect short pauses in breathing, by the presence of apnea for periods of longer than 15 to 20 seconds, cyanosis, hypotonia, bradycardia, or pallor. Diagnostic workup and treatment depends on possible causes and the age group. In infants, neuroimaging may be warranted if the condition is severe, or if hemorrhage, hydrocephalus, or other central nervous system anatomic abnormalities that could lead to central apnea are suspected. However, central apnea in most premature and full-term infants generally improves over time as the respiratory center of the brainstem matures. Rigid or flexible endoscopy can be helpful to diagnose anatomic conditions of upper airway obstruction, such as subglottic stenosis or tracheobronchomalacia. Apnea of prematurity can be treated with theophylline or caffeine. Continuous positive airway pressure (CPAP) or high-flow nasal cannula can be used for mixed or obstructive conditions. For older children, it should be stressed that obstructive sleep apnea is not a benign condition. It can lead to impaired development from sleep disturbances, and if severe, pulmonary hypertension and death. More than 75% of children who have obstructive sleep apnea snore. Referral to pediatric pulmonology or otolaryngology may be warranted. Possible interventions include nighttime CPAP, tonsillectomy, adenoidectomy, and weight loss, where applicable. Respiratory syncytial virus is a single-stranded RNA virus that commonly causes bronchiolitis in children younger than 1 year of age, usually in yearly epidemics. Bronchiolitis generally causes small airway obstruction from necrosis of the bronchiolar epithelium, edema, and increased mucus production. Apnea is a common manifestation of RSV infection, especially in infants who are younger than 6 weeks of age, formerly premature, or have other pre-existing chronic illnesses. The cause of apnea in RSV is not completely understood, but could be at least partially obstructive in nature because of nasopharyngeal and upper airway obstruction from increased mucous production. Although neuroimaging, electroencephalography, and lumbar puncture can be important in the workup of central apnea in infants to rule out hemorrhage, seizure, or meningitis, these causes are unlikely in the setting of an acute upper respiratory infection. A sleep study would be more useful in older children or in cases of suspected obstructive sleep apnea. The evaluation of apnea depends on the age group and acuity of presentation. Apnea caused by respiratory infections in children generally resolves when the infection improves. Treatment of apnea is generally supportive, but could include medications in apnea of prematurity, acute or chronic positive pressure ventilatory support, or surgical correction of airway lesions. PREP Pearls - Causes of apnea in children can include central, obstructive, or mixed. - Obstructive sleep apnea is not a benign condition. It can lead to impaired development, pulmonary hypertension, cor pulmonale, and death. - The pathogenesis of apnea from respiratory infections is not well understood, but generally improves along with the infection. Evaluation of nasal secretions for respiratory viruses is the diagnostic study of choice. ABP Content Specifications(s) - Plan appropriate management for apnea of various etiologies - Plan the appropriate clinical and diagnostic evaluation of apnea of various etiologies

A previously healthy 5-week-old, full-term male patient is brought to the emergency department because he stopped breathing for approximately 20 seconds and turned blue around the lips. He has also had slight nasal congestion and poor oral intake today. His mother did not report any complications during pregnancy. He was born at term and discharged home in 2 days. He usually drinks four 6-oz bottles of formula per day, and today he is only finishing 2 oz of each bottle. His 2 older siblings have been ill with cough and rhinorrhea. Vital signs show a temperature of 38.0°C, pulse rate of 150 beats/min, respiratory rate of 60 breaths/min, blood pressure of 80/40 mm Hg, and oxygen saturation of 93% on room air. Physical examination shows a tired-appearing but well-developed, well-nourished baby. Anterior fontanelle is soft, open, and flat. Eyes and oral mucous membranes are erythematous. Pupils are 2 mm, equal, and reactive. His heart has a regular rate and rhythm, and extremities are warm and well-perfused, with capillary refill time of 2 s. Respiratory examination shows tachypnea with mild intercostal retractions, nasal flaring, and coarse breath sounds bilaterally. Abdomen is soft, nontender, nondistended, and with no organomegaly. Supplemental oxygen by nasal cannula is initiated with deep suctioning of the nasopharynx and results in improvement of oxygen saturation to 100%. The infant is observed to have 2 separate periods of apnea that last 15 seconds and are associated with a drop in oxygen saturation to 80% that resolves with gentle stimulation. Of the following, the MOST appropriate diagnostic test at this time is A. computed tomography of brain B. electroencephalogram C. lumbar puncture D. nasal secretions for respiratory syncytial virus E. sleep study

E. infection with Escherichia coli O157:H7 Abdominal pain is one of the most common complaints in childhood and one that frequently requires urgent evaluation in the office or emergency department. Although the cause is typically a self-limited illness (constipation, gastroenteritis), the clinician must identify those conditions that pose a potentially serious or life-threatening situation (most of which will require surgical intervention) such as appendicitis, intestinal volvulus, intussusception, or obstruction secondary to congenital (Ladd) bands or postsurgical adhesions. In this clinical setting, the presence of associated signs, including bilious vomiting (small intestinal obstruction), localized pain, (eg, in acute appendicitis), and colicky pain in an infant or toddler (intussusception, with or without evidence of lower gastrointestinal bleeding) will often signal the need for imaging studies and urgent surgical consultation. Many of these diagnoses are seen throughout the pediatric age range. However, because children younger than 5 years of age are often incapable of localizing the source of pain, reaching a definitive diagnosis is especially challenging in this age group. In addition to problems requiring surgical or radiologic intervention, specific infectious disorders must also be considered. The clinical history and available laboratory data described for the boy in the vignette indicate anemia, thrombocytopenia, and urinary abnormalities in association with the acute onset of abdominal pain and hematochezia. These findings strongly suggest a diagnosis of hemolytic-uremic syndrome (HUS) as a consequence of infection with the Shigatoxin-producing bacterium Escherichia coli O157:H7 (STEC). - Microangiopathic hemolytic anemia, - renal dysfunction - thrombocytopenia define the HUS triad. In the United States, HUS occurs in approximately 8% of children who have E coli O157:H7 diarrhea, an incidence that is significantly greater than observed with other Shigatoxin-producing E coli serotypes. Pathologic E coli species are transmitted most commonly via contaminated food products. Particular reservoirs of STEC include raw or undercooked ground beef, unpasteurized milk, and any food or beverage (eg, unpasteurized apple cider) that may be contaminated with bovine feces. Because all Shigatoxin-producing E coli are excreted in the feces of cattle, sheep, deer, and other ruminants, direct contact with these animals is another significant source of contamination. Several human outbreaks of E coli O157:H7 enteritis have been linked to animals in petting zoos. Shigatoxin-producing Escherichia coli O157:H7 is an important cause of acute renal injury and accounts for 70% to 90% of all HUS cases in countries where these bacteria are endemic. HUS typically develops 5 to 9 days (mean, 7 days) after the onset of gastrointestinal symptoms and 2 to 6 days (mean, 4 days) after the appearance of bloody stools. Accordingly, when acute enteritis is associated with the passage of blood, clinical assessment must include both a hematologic profile (including examination of a blood smear for fragmented cells) and an evaluation of renal function. The previous treatment with amoxicillin for otitis media reported for the boy in the vignette raises the question of a possible role for antibiotics in the development of HUS. Although earlier studies suggested that antibiotic usage in hemorrhagic colitis increased the risk of developing HUS, a metaanalysis failed to confirm these findings. Available evidence clearly indicates that antimicrobial agents do not alter the duration or outcome of illness, and the use of antibiotics for E coli O157:H7 infection is not recommended. Because most clinical microbiology laboratories can culture and identify this organism, as well as the other common bacterial pathogens that cause acute enterocolitis (Salmonella, Shigella dysenteriae, Campylobacter jejuni, Clostridium difficile), decisions about specific therapy generally should await bacteriologic identification. In the boy in this vignette, infection with any of the other organisms listed in the answer choices is unlikely because of the strong association between E coli O157:H7 enterocolitis and renal compromise associated with HUS. Intussusception would be less likely given the child's age (unless a specific lead point was present, eg, polyp in a hamartomatous polyp syndrome, Meckel diverticulum) and the positive urinary findings. Although idiopathic ulcerative colitis may occur in this age group (more common in the second decade), the acute onset of illness coupled with the findings of thrombocytopenia, hematuria, and proteinuria are not consistent with this diagnosis. When considering a diagnosis of ulcerative colitis, infectious causes of hematochezia must be ruled out. PREP Pearls - In the child with abdominal pain, bilious vomiting suggests small intestinal obstruction. - In the evaluation of acute abdominal pain and hematochezia, hemolytic-uremic syndrome must be ruled out. - Antibiotic therapy is not indicated for the treatment of most gastrointestinal infections, including infection with Escherichia coli O157:H7. ABP Content Specifications(s) - Formulate an age-appropriate differential diagnosis of acute abdominal pain - Plan the appropriate evaluation of acute abdominal pain

A previously healthy, 4-year-old boy presents to your office following 2 days of diffuse, crampy abdominal pain, tactile fevers, and watery diarrhea. For the past 24 hours, the diarrhea has been bloody. Symptoms began shortly after his family visited a local petting zoo. He recently completed a 10-day course of amoxicillin for otitis media. Physical examination demonstrates a well-developed, well-nourished child who is in moderate distress because of poorly localized direct abdominal tenderness without rebound. His pulse rate is 140 beats/min, blood pressure is 96/54 mm Hg, and other vital signs are normal. His mucous membranes are dry and capillary refill time is 2 seconds. Initial laboratory study results are as follows: - Hemoglobin, 10.5 g/dL (105 g/L) - White blood cells, 13,500/µL (13.5 × 109/L) - Platelet count, 70 × 103/µL (70 × 109/L) - Prothrombin time, 12 s (reference range, up to 12 s) - Partial thromboplastin time, 23 s (reference range, up to 26 s) - Urinalysis, 3+ positive for protein and blood, negative for glucose and ketones Of the following, the MOST likely cause of this child's illness is A. idiopathic ulcerative colitis B. ileocolonic intussusception C. infection with Campylobacter jejuni D. infection with Clostridium difficile E. infection with Escherichia coli O157:H7

C. discharge home with follow-up in 24 hour The risk of serious bacterial infection (SBI) in a well-appearing infant between 30 and 90 days of age with an unremarkable examination, normal complete blood cell count, and negative urinalysis, as described for the baby in this vignette, is low. When a baby meets these criteria and there is an expectation as noted in the vignette of good follow-up, the patient can be safely sent home with follow-up in 24 hours. In the absence of concerns regarding the baby's returning for serial evaluations, persistent fever, or the baby getting sicker, there is potential risk and no clear advantage in hospitalizing a baby for observation in this setting. Initiation of antibiotic therapy (intravenous or oral) without identifying a focus of infection, given the unremarkable examination and laboratory parameters described for the baby in this vignette, is not indicated. Although some studies suggest performing a lumbar puncture (LP) as part of the evaluation of all young infants younger than 90 days of age with fever, this is presently controversial. Regardless, just performing a LP in this setting would not be an indication to begin treatment with antibiotics in a well-appearing infant. In fully vaccinated, well-appearing children between 90 days to 24 months of age, clinical examination is more reliable than in younger infants. In these patients, the risk of invasive bacterial infection is low enough that observation alone or screening for infection with a complete blood cell count and urinalysis are appropriate options. These recommendations only apply to a well-appearing infant with fever. Lethargy, poor feeding, excessive irritability, or toxic appearance should prompt assessment and treatment for sepsis or other invasive bacterial infections. PREP Pearls - The risk of serious bacterial infection is low in febrile infants 30 to 90 days of age who are well appearing, with a normal physical examination, complete blood cell count, and urinalysis. Observation with close follow-up would be appropriate for these patients. - Lethargy, poor feeding, excessive irritability, or toxic appearance should prompt assessment and treatment for sepsis or other invasive bacterial infection. ABP Content Specifications(s) - Plan the appropriate evaluation and management of fever without source in patients of various ages

A previously well, 56-day-old female infant arrives at your office with a 1-day history of fever (up to 38.9°C). The parents deny cough, congestion, nausea, rash, skin lesions, vomiting, and diarrhea. The baby lives with her parents and 2 siblings who are 2 years of age and 5 years of age. No one else at home is ill. The baby is being fed formula 1.5 oz to 2 oz every 2 to 3 hours (down from her usual 2.5 oz to 3 oz per feed). You review the baby's birth history and see that her birth weight was 3.3 kg; her birth was a full-term, uncomplicated pregnancy and delivery; her nursery stay was without complications; and she went home in 2 days. She has no prior illnesses. She received hepatitis B vaccine in nursery. The family has been in your practice since the birth of the oldest child and has kept appointments reliably. The baby's vital signs show a temperature of 38.8°C, a heart rate of 130 beats/min, and a respiratory rate of 24 breaths/min. Physical examination shows a strong cry, but she is consolable by her mother. She is comfortable in her mother's arms and well appearing. The remainder of the physical examination findings are unremarkable. Laboratory results show the following: - White blood cell count, 12,800/µL (12.8 × 109/L), with 45% neutrophils, 3% bands; 49% lymphocytes; 3% monocytes - Hemoglobin, 13.4 g/dL (134 g/L) - Platelet count, 227 × 103/µL (227× 109/L) A urinalysis shows the following: - Specific gravity, 1.017 - pH, 7.0 - Dipstick, negative The blood and urine cultures are pending. Of the following, the BEST next step in the management of this infant's condition would be to A. admit and observe pending cultures B. admit and treat with ampicillin and cefotaxime pending culture results C. discharge home with follow-up in 24 hours D. perform a lumbar puncture and begin ampicillin and ceftriaxone pending culture results E. prescribe amoxicillin and follow-up in 24 hours

E. palpate for the presence of a testicular mass The adolescent in the vignette has a history of cryptorchidism. An undescended testicle is often abnormal to start with and is at increased risk for development of masses. A male adolescent begins the development of secondary sexual characteristics between 9 and 14 years of age. Testicular development is the first sign of puberty in a male adolescent and a teenager should enter sexual maturity rating 2 by 14.5 years of age at the latest. PREP Pearls - A history of cryptorchidism requires that a male adolescent be routinely screened for the development of testicular masses. - A number of lesions may be found on routine genital examination in a boy, despite a negative history of symptoms. ABP Content Specifications(s) - Understand the importance of testicular self-examination

A shy, 14-year-old adolescent boy presents to your office for a physical examination. He denies any concerns today and is not sexually active. His past medical history is significant only for cryptorchidism that was repaired at 1 year of age. The boy's last genital examination, at 10 years of age, was unremarkable for a prepubertal male. His height today is at the 5th percentile for his age, and weight is at the 10th percentile, and he has normal vital signs. He has no facial hair, axillary hair, or voice changes. He refuses a genital examination. The remainder of his physical examination findings are unremarkable. You discuss with your medical student the importance of performing a genital examination and how one would teach the boy about testicular self-examination. Of the following, the MOST important reason to do a genital examination in this case is to A. determine that puberty is progressing normally B. evaluate for the presence of an indirect hernia C. inspect and palpate for the presence of varicoceles D. inspect for signs of sexually transmitted infections E. palpate for the presence of a testicular mass

E. total cholesterol level is high, low-density lipoprotein cholesterol level is borderline, and high-density lipoprotein level is acceptable PREP Pearls - Total cholesterol over 200 mg/dL (5.18 mmol/L), low-density lipoprotein cholesterol over 130 mg/dL (3.37 mmol/L), or high-density lipoprotein cholesterol less than 35 mg/dL (0.91 mmol/L) are always considered abnormal at any age. - Hyperlipidemia is a risk factor for adult-onset cardiovascular disease, including stroke and ischemic heart disease. - New guidelines from the National Heart Lung and Blood Institute and the American Academy of Pediatrics recommend screening for hypercholesterolemia twice, once between the ages of 9 and 11 years and again between the ages of 17 and 21 years. ABP Content Specifications(s) - Recognize the clinical features associated with hypercholesterolemia/hyperlipidemia, and evaluate appropriately - Understand the importance of cholesterol/lipid screening examinations - Know the risk factors associated with hypercholesterolemia/hyperlipidemia

An 11-year-old girl is brought to your office for a routine health supervision visit. She is doing well academically, has good relations with her peers and parents, and has no complaints at this time. She has not yet begun menses. Her weight is 43 kg (75th percentile) and her height is 145 cm (50th percentile). Her physical examination shows sexual maturity rating of 2 with all other findings unremarkable. As part of her evaluation today, she has a screening total cholesterol result of 220 mg/dL (5.7 mmol/L). You send her for fasting laboratory tests that show a low-density lipoprotein cholesterol level of 120 mg/dL (3.11 mmol/L) and a high-density lipoprotein cholesterol level of 55 mg/dL (1.42 mmol/L). Of the following, the statement that MOST accurately describes her laboratory results is that A. total cholesterol level is acceptable, low-density lipoprotein cholesterol level is high, and high-density lipoprotein level is low B. total cholesterol level is borderline, low-density lipoprotein cholesterol level is acceptable, and high-density lipoprotein level is borderline C. total cholesterol level is borderline, low-density lipoprotein cholesterol level is borderline, and high-density lipoprotein level is high D. total cholesterol level is high, low-density lipoprotein cholesterol level is acceptable, and high-density lipoprotein level is low E. total cholesterol level is high, low-density lipoprotein cholesterol level is borderline, and high-density lipoprotein level is acceptable

E. the rapid pace of pubertal development and abnormal laboratory values suggest that she needs further workup for her symptoms The child in this vignette rapidly advanced through puberty, progressing from thelarche (breast development) to menarche (onset of menses) in only 5 months. In the United States, thelarche occurs at an average of 10.7 years of age ± 1 year, with menarche occurring at 12.7 years of age ± 1.0 year. More recently, it has been found that thelarche is occurring earlier, with 5% of girls showing breast development by 7 years of age and 9% by 8 years of age. This earlier development has been associated with obesity. Even so, the span of time between the onset of breast development and the age of menarche is still 2.3 ± 1.0 year. This child has very low (suppressed) luteinizing hormone (LH) and follicle-stimulating hormone (FSH) despite having rapidly progressed through puberty, and very high estradiol levels, suggesting an exogenous source or autonomous ovarian production of estrogen is suppressing LH and FSH levels. These laboratory findings, along with her rapid pace of pubertal development, suggest additional workup is warranted to find the source of estrogen secretion. This child was ultimately found to have an ovarian granulosa cell tumor. Although her height is within 2 inches of her midparental height, the concerning laboratory findings and rapid pace of pubertal development still warrant evaluation. Because of her physical examination findings, laboratory results do not suggest she is early in pubertal development, but rather that the LH and FSH are suppressed from very high estradiol levels. In addition, although it is true that irregular menses are common for the first 2 years after menarche, this child may be experiencing irregular menses from changes in estrogen levels secreted by an exogenous source or from autonomous ovarian production, and thus further evaluation is warranted. PREP Pearls - The span of time between the onset of breast development and the age of menarche averages 2.3 ± 1.0 year. - Early puberty or rapid pubertal progression warrant consideration for an underlying pathological diagnosis. ABP Content Specifications(s) - Recognize the stages of sexual development and the range of age of onset of each - Understand factors that influence the timing of puberty

An 11-year-old girl presents for concerns about early puberty and irregular menses. She had breast development beginning 6 months ago and then had her first menstrual period 3 months ago. Family history reveals that her mother had menarche at 12 years of age, and her midparental height is 160 cm (5 ft, 3 in). On physical examination, the girl is 155 cm (5ft, 1 in) tall, has sexual maturity rating 4 breasts with a secondary mound, and sexual maturity rating 3 pubic hair. Her family is concerned because her menstrual periods are irregular and heavy. Laboratory value results include the following: Laboratory value results include the following: - Luteinizing hormone, less than 0.2 mIU/mL (0.2 IU/L) - Follicle-stimulating hormone, less than 0.7 mIU/mL (0.7 IU/L) - Estradiol, 144 pg/mL (529 pmol/L); reference range for a sexual maturity rating 4 female, 21 pg/mL to 85 pg/mL (77 pmol/L-312 pmol/L) Of the following, you are MOST likely to tell the family that A. her height of 155 cm (5 ft, 1 in) is within 5 cm (2 in) of her midparental height and consistent with her genetic potential B. her laboratory results show a very low luteinizing hormone and follicle-stimulating hormone, suggesting that she is very early in pubertal development C. her progression through puberty is normal because the average age of menarche in North America is 12 ± 1 years of age D. irregular menses are common for the first 2 years after menarche, and follow-up should be scheduled in 6 months E. the rapid pace of pubertal development and abnormal laboratory values suggest that she needs further workup for her symptoms

E. validity The validity of a test refers to how well the test measures what it purports to measure. For the patient described in the vignette who has nasal congestion and an unremarkable urinalysis, the result of a urine culture obtained using a urine collection bag is likely not valid (accurate) because the test detects bacteria colonizing the genitourinary area, in addition to bacteria infecting the bladder. The appropriate test for a child who is not toilet-trained is a urine specimen for culture obtained by urethral catheterization. The internal validity of a test refers to its accuracy, while the external validity of a test relates to its generalizability. There cannot be external validity without internal validity; if a test is not accurate, it does not matter if it is generalizable. A bag urine specimen for culture can be obtained in any patient, so the test is generalizable, but the poor accuracy of the test in this case makes it not valid. The reliability of a test refers to the stability of its performance; if the test is repeatedly performed on the same individual, will it have the same result? For the patient described in the vignette, the culture result from a bag urine specimen may be reliable, but the result may not be accurate. The sensitivity of the test (bag urine culture) is its ability to correctly determine that a patient is infected, while the specificity is its ability to correctly classify an uninfected individual. The patient in the vignette is not infected but the urine culture was positive, so the test is sensitive, but not specific. The sensitivity and specificity of a test are not affected by the prevalence of disease. The negative predictive value of a test is the probability that a patient with a negative urine culture is not infected, while the positive predictive value is the probability that a patient with a positive urine culture actually is infected. For a urine culture obtained by bag collection, the negative predictive value is high, but the positive predictive value is low. Prevalence of disease affects the positive and negative predictive values. A higher disease prevalence results in a higher positive predictive value (fewer false positives) and lower negative predictive value (more false negatives). PREP Pearls - The validity of a test refers to how well the test measures what it purports to measure. - The reliability of a test refers to the stability of its performance over time. - Sensitivity is the probability that those with a disease will have a positive test result. - Specificity is the probability that those without disease will have a negative test result. ABP Content Specifications(s) - Understand validity and how it might be compromised - Understand reliability and how it might be compromised - Understand generalizability and how it relates to validity

An 18-month-old girl presented to the emergency department 2 days ago with nasal congestion and was diagnosed with a febrile urinary tract infection. A bag urine specimen was obtained; urinalysis was positive only for leukocyte esterase, and the urine culture shows bacterial growth. She was discharged to home with oral trimethoprim-sulfamethoxazole treatment. In your office, the patient is playful, afebrile, and has normal vital signs for age. Her physical examination findings are normal. You tell the mother to stop the trimethoprim-sulfamethoxazole. Of the following, the principle that best explains why this action is appropriate is its Errata: Content revised to fix typographical errors. 9/2015 A. generalizability B. negative predictive value C. reliability D. sensitivity E. validity

D. rickets The child in the vignette presents with clinical and radiologic findings consistent with rickets, an uncommon disorder that can occur in exclusively breastfed infants who are not receiving vitamin D supplementation or in dark-skinned children who are not exposed to adequate sunlight because of lifestyle or geographic location. Rickets can be diagnosed by characteristic changes on plain radiography, such as cupping and fraying of the costochondral junctions and epiphyses, demineralization, widened epiphyses, and cortical thinning. Children affected by rickets will have decreased serum concentrations of vitamin D metabolites, and elevated serum alkaline phosphatase concentration. The details in the vignette are supportive of a diagnosis of rickets, but differentiating the findings associated with fractures related to other musculoskeletal disorders from those associated with inflicted fractures can be challenging in many cases. Children with unexplained fractures can be victims of inflicted injury. A number of underlying musculoskeletal disorders are associated with increased bone fragility that can be subtle and difficult to diagnose. Pediatric health care providers should be aware of certain clinical features and fracture patterns that are associated with child abuse, which has major implications for the safety and well being of the children affected, as well as those that are associated with underlying musculoskeletal disorders. No fracture, when considered in isolation, can distinguish an abusive from a nonabusive cause. During the evaluation of individual bony abnormalities, the lesion site and type, age and developmental stage of the child, and associated reported history can help clinicians to determine the likelihood of abuse. A number of studies, including a recent systematic review of the child abuse literature, have found that abusive fractures are detected most commonly in infants younger than 1 year of age. When any infant or child presents with a fracture in the absence of a confirmed cause, inflicted injury should be considered as a potential cause. Fracture patterns that should further raise suspicion for child abuse include posterior rib fractures, metaphyseal fractures (also known as "bucket-handle" or corner fractures), humeral and femur fractures (especially those occurring in a nonambulatory child), and skull fractures (especially complex skull fractures). Multiple fractures are also associated with abusive trauma, though they can result from both accidental trauma or underlying musculoskeletal disorders. Repeat fractures occurring at an unusual location for repeat injury, fractures in radiographic stages of healing that do not correspond to the reported clinical history, or multiple fractures in various stages of healing should heighten a clinician's suspicion for abuse. When abuse is suspected, a radiographic skeletal survey should be obtained to evaluate the entire skeletal system for potential injuries. Whenever possible, clinicians should consult with providers possessing specific expertise and experience in the management of suspected child abuse cases. Physicians are also required to report suspected child abuse to the appropriate child protective service and/or law enforcement agency in their respective areas. Osteogenesis imperfecta (OI) is a musculoskeletal disorder resulting in fractures that are most frequently mistaken for abusive fractures. Osteogenesis imperfecta is a heterogeneous group of disorders, usually caused by mutations in the gene involved in the production of type I collagen (COL1A1 and COL1A2), which forms the structural framework of bone. Although OI is an inherited disorder, the presentation of the disease even within the same family can be quite variable. Osteogenesis imperfecta can usually be diagnosed by obtaining a thorough medical and family history, performing a physical examination, and interpreting the results of appropriate laboratory and radiographic studies. Many children—though not all children—with OI will have diagnostic signs such as osteopenia, bony deformities, and "wormian bones" of the skull on radiography. Findings such as blue sclera, discolored teeth, limb deformities, and a triangular-shaped face may be apparent on the physical examination of some children with OI, but these features may be subtle or absent in a number of patients with OI. The child described in the vignette has no reported family history of OI, no reported history of prior fractures, and no physical examination findings that would suggest the diagnosis of OI. Caffey disease, also called infantile cortical hyperostosis, is a rare musculoskeletal disorder that most often occurs in infants. A characteristic feature of Caffey disease is excessive new bone formation (hyperostosis), which mainly affects the mandibles, scapulae, clavicles, and long bone diaphyses. Infants affected by Caffey disease often develop swelling of the joints and soft tissues, with pain and erythema in the affected areas. Signs and symptoms of Caffey disease are generally apparent within the first few months after birth and often resolve by early childhood. The clinical presentation and radiographic findings for the child in the vignette are not consistent with that of Caffey disease. Scurvy, which arises from a deficiency of ascorbic acid (vitamin C), generally presents with hemorrhagic symptoms (petechiae, ecchymoses, and gum bleeding), hyperkeratosis, hypochondriasis, and hematologic abnormalities, in addition to nonspecific generalized symptoms such as malaise, fatigue, anorexia, and failure to thrive. Although tenderness and swelling over the long bones can occur in children with scurvy, fractures do not generally result from this disease process. Scurvy would be very unlikely in the girl in the vignette given her lack of the characteristic signs and symptoms. PREP Pearls - No fracture, when considered in isolation, can distinguish an abusive from a nonabusive cause. The fracture site and type, age and developmental stage of the child, and associated reported history can help clinicians to determine the likelihood of abuse. - When any infant presents with a fracture in the absence of a confirmed cause, inflicted injury should be considered as a potential cause. - Fracture patterns that are particularly suspicious for an abusive mechanism include posterior rib fractures, metaphyseal fractures, humeral and femur fractures (especially those occurring in nonambulatory children), and skull fractures (especially complex skull fractures). ABP Content Specifications(s) - Differentiate the findings associated with inflicted fractures from those of fractures related to other musculoskeletal disorders

An 18-month-old girl who is new to your practice presents for evaluation of right wrist swelling, which her mother initially noticed approximately 1 week ago. There is no reported history of trauma. The girl has had no recent fevers, and she displays no other symptoms of illness. The mother tells you that the girl has been healthy, takes no medications, and that she is adopted and came to live with her new family only 3 months ago. Few details were provided to the adoptive family regarding the child's birth history, past medical history, and family medical history. The girl reportedly began walking around 14 months of age, although the mother notes that "she always seems bow-legged when she walks." The girl's vital signs are within normal limits; her height and weight are both at the fifth percentiles for her age. She is playful and appears well. On physical examination, you note that the girl has a prominent forehead, but no facial dysmorphisms. Her right wrist seems slightly swollen when compared to the left wrist, but there is no overlying bruising, warmth, or erythema, and the girl is able to flex and extend both wrists without pain. You agree with the mother's observation that her legs seem "bowed," but both lower extremities physical examination is unremarkable. You order plain radiographs of the girl's right wrist, which reveal widening of the distal radius and ulna with cupping of the right radial metaphysis, in addition to generalized osteopenia (Item Q111). Of the following, the MOST likely cause of this child's wrist swelling is A. Caffey disease B. child abuse C. osteogenesis imperfecta D. rickets E. scurvy

D. glucose-6-phosphate dehydrogenase deficiency (G6PD) G6PD is an important enzyme in protecting the RBC from oxidative stress. It is the primary step in the pentose-phosphate pathway (PPP) that produces nicotinamide adenine dinucleotide phosphate (NADPH) and keeps glutathione in its reduced state. This is the only mechanism of producing NADPH in RBCs. Certain substances, such as sulfa drugs and fava beans, interact with oxygen and hemoglobin to produce a high level of toxic oxygen peroxides. Infection is the most common inciting factor for acute hemolysis in G6PD deficiency. In the presence of normal levels of NADPH and reduced glutathione, the oxygen peroxides are neutralized and converted to pentose sugars via the PPP, thus protecting the RBCs against damage. In G6PD deficiency, however, these oxygen peroxides persist, causing damage to the hemoglobin and cell membrane, and ultimately leading to hemolysis. Mutations in this gene are associated with resistance to Plasmodium falciparum malaria, therefore the prevalence of G6PD deficiency is higher in Mediterranean, African, Middle Eastern, and South Asian populations. The common clinical presentations for G6PD deficiency are as follows: (1) neonatal jaundice, (2) acute hemolytic anemia, and (3) chronic nonspherocytic hemolytic anemia (CNSHA) Neonatal jaundice: treatment with phototherapy or exchange transfusion is similar to neonatal hyperbilirubinemia from other causes. AHA: the common presenting signs and symptoms during an acute hemolytic episode are malaise, weakness, and abdominal and back pain. Within 2 to 3 days, patients often have jaundice and dark urine from hemoglobinuria, which can lead to renal failure. Treatment is supportive with packed RBC transfusion, fluids, and hemodialysis if necessary Children with CNSHA can present with significant splenomegaly. PREP Pearls - There is a high prevalence of G6PD deficiency in African, Mediterranean, and Asian populations. The African variant generally tends to be the less severe form. - Glucose-6-phosphate dehydrogenase (G6PD) deficiency is an X-linked disorder. - Higher levels of G6PD activity may be detected during acute hemolysis and reticulocytosis because reticulocytes have 5x G6PD activity compared to old RBCs, and may lead to false-negative results when testing for G6PD deficiency. ABP Content Specifications(s) - Recognize the clinical findings associated with G6PD deficiency in patients of differing ethnic backgrounds - Plan appropriate management of hemolysis in a patient with G6PD deficiency - Recognize the inheritance pattern associated with G6PD deficiency

An 18-month-old, African-American boy is brought to your clinic by his grandmother for yellowness of the eyes. The grandmother states that he has been irritable and sleeping more over the past 3 days. Last week, he had nasal congestion and a cough. Concerned that he might develop a fever, she gave him a few doses of antibiotics that were leftover from his sibling's recent treatment. He is pale and quiet. His oral temperature is 37.3°C, pulse rate is 125 beats/min, respiratory rate is 22 breaths/min, and blood pressure is 100/65 mm Hg. On physical examination, he has icteric sclerae and a 2/6 systolic ejection murmur. There is no hepatosplenomegaly. The remainder of the physical examination findings are unremarkable. The following laboratory tests are obtained: - White blood cell count, 10,500/µL (10.5 × 109/L), with 30% polymorphonuclear leukocytes, 61% lymphocytes, 6% monocytes, and 3% eosinophils - Hemoglobin, 6.0 g/dL (60 g/L) - Hematocrit, 18% - Mean corpuscular volume, 80 µm3 (80 fL) - Platelet count, 250 × 103/µL (250 × 109/L) - Reticulocyte count, 10% (0.10) - Direct bilirubin, 0.5 mg/dL (8.6 µmol/L) - Indirect bilirubin, 4.0 mg/dL (68.4 µmol/L) - Direct antiglobulin (Coombs) test, negative Of the following, the MOST likely cause of this patient's findings is A. autoimmune hemolytic anemia B. ß-thalassemia minor C. Gilbert syndrome (UGT1A1 mutation) D. glucose-6-phosphate dehydrogenase deficiency (G6PD) E. transient erythroblastopenia of childhood

A. angiotensin converting enzyme inhibitor use Chronic rhinitis can be due to many causes, and include those caused by allergic, nonallergic, mixed, hormonal, occupational, rhinitis medicamentosa, systemic-medication-induced, atrophic, systemic-disease related, and those associated with structural anomalies such as tumors, foreign bodies, and deviated nasal septum. The patient in the vignette has chronic rhinitis, most likely triggered by her use of enalapril, a long-acting angiotensin converting enzyme (ACE) inhibitor. Her symptoms of daily nasal congestion, rhinorrhea, sneezing, and postnasal drainage suggest a diagnosis of rhinitis. Her symptoms are of recent onset and do not have a seasonal pattern or identifiable triggers, making seasonal allergic rhinitis less likely. The timing of the start of her symptoms after the institution of the enalapril for control of the essential hypertension strongly implicates it as the most likely culprit. Antihypertensives, such as ACE inhibitors and angiotensin receptor blockers, can cause chronic rhinitis and cough as side effects. Other causes of systemic drug-induced rhinitis in general include cardiovascular agents such as α-1 receptor adrenergic antagonists, phosphodiesterase-5 selective inhibitors, gabapentin, and nonsteroidal anti-inflammatory drugs (NSAIDs). Aspirin and NSAIDs may produce rhinorrhea as an isolated symptom or as part of a symptom complex of rhinosinusitis, nasal polyposis, and asthma known as ''Samter's triad.'' The recent onset of the child's symptoms, absence of nasal polyps, and minimal use of aspirin make this a less likely etiology. In patients with nasal polyposis and aspirin associated respiratory disease (AERD), aspirin desensitization followed by long-term daily aspirin treatment may be considered. Long-term studies of patients with AERD suggest that maintenance aspirin therapy may reduce nasal symptoms, frequency of sinus infections, requirement for nasal polypectomies, and need for systemic corticosteroids. Causes of hormonal rhinitis include pregnancy and menstrual cycle-related rhinitis. Pregnancy rhinitis, when present, is associated with significant nasal congestion, starts after the second month of pregnancy, and usually disappears within 2 weeks after delivery. During the menstrual cycle, nasal congestion has been shown in 1 study to concur with ovulation and rise in serum estrogens, although additional evidence is needed. There is no direct evidence that the currently available combined oral contraceptive pills cause nasal symptoms. Nonallergic rhinitis with eosinophilia (NARES) is characterized by nasal eosinophils in patients who have perennial symptoms of sneezing paroxysms, profuse watery rhinorrhea, nasal pruritus, and occasionally reduced sense of smell. Nonallergic rhinitis with eosinophilia is characterized by large numbers (inconsistently defined as > 5% to > 20%) of eosinophils on nasal smear. These patients often lack evidence of allergic disease as demonstrated by absence of positive skin tests and specific immunoglobulin E antibodies in the serum. Patients with NARES are typically middle-aged and have a characteristic perennial course, but with paroxysmal episodes. Nonallergic rhinitis with eosinophilia occurs extremely infrequently in childhood and probably accounts for less than 2% of children with nasal eosinophilia. Nonallergic rhinitis with eosinophilia is therefore less likely in the patient in this vignette. Rhinitis medicamentosa is a condition of rebound nasal congestion that follows the overuse of intranasal α-adrenergic decongestants (such as oxymetazoline and phenylephrine) or recreational use of cocaine. Benzalkonium chloride in vasoconstrictor spray products, when used for 30 days or more, may augment local pathologic effects. Patients may develop rebound congestion, tachyphylaxis, reduced mucociliary clearance because of loss of ciliated epithelial cells, and on rare occasions, nasal septal perforation. Treatment of rhinitis medicamentosa consists of discontinuing the use of topical decongestants and administering intranasal corticosteroids to control symptoms. At times, a short course of oral corticosteroids may be needed to control the patients' symptoms while the effects of the nasal decongestant spray dissipate. Once the rhinitis medicamentosa is treated, the patient should be evaluated for underlying conditions such as allergic rhinitis. PREP Pearls - Chronic rhinitis can be caused by both allergic and nonallergic causes. - Use of antihypertensives, such as angiotensin converting enzyme inhibitors, a-1 receptor adrenergic blockers, and phosphodiesterase inhibitors, can cause chronic rhinitis. - Topical use of intranasal a-adrenergic decongestants (such as oxymetazoline and phenylephrine) or recreational use of cocaine can cause rhinitis medicamentosa and should be discouraged. ABP Content Specifications(s) - Formulate a differential diagnosis of chronic rhinitis

An 18-year-old female patient comes to your office for evaluation of a 6-month history of daily nasal congestion, cough, rhinorrhea, sneezing, and postnasal drainage. Her symptoms are year-round without any seasonal worsening. She has not been able to identify any triggers. She has tried different medications such as cetirizine, intranasal fluticasone spray, and intranasal azelastine with some improvement of symptoms. She denies use of over-the-counter nasal decongestants. She uses aspirin for an occasional migraine headache, perhaps once or twice a month. Apart from taking twice-daily enalapril for the past 9 months for control of essential hypertension, she denies use of any other medication. She denies being sexually active and has had regular menstrual cycles. She has not had any recent changes in weight, appetite, energy, or constitutional symptoms. On physical examination, her weight is at the 50th percentile and height at 60th percentile for age. Her blood pressure is 110/80 mm Hg. She has bilateral edematous turbinates, with mildly erythematous mucosa and clear secretions in her nares. The rest of her physical examination findings are unremarkable. Of the following, the MOST likely cause of this patient's symptoms is A. angiotensin converting enzyme inhibitor use B. aspirin sensitivity C. hormonal D. nonallergic rhinitis with eosinophilia E. rhinitis medicamentosa

E. seizure Adolescence is a time of exploration and most teenagers will try alcohol and put themselves at risk for the consequences of acute use. The young woman in the vignette probably had a seizure as a consequence of binge drinking. Although not common, acute alcohol ingestion by adolescents and young adults can result in a seizure that is usually generalized and tonic-clonic in nature, but occasionally can be a partial seizure. This may occur in someone with no history of epilepsy. Seizures are more likely to occur after acute withdrawal in chronic alcohol users. More common, and of greater concern, are the behavioral issues that occur after heavy alcohol use. In addition to central nervous system depression, adolescents may experience sleep disturbances, mood swings, agitation, anxiety, depression, memory loss, and academic failure. In addition, there may be aggressive behavior and violence. Accidents are the leading cause of death in adolescents and approximately half of the adolescents involved in an automobile crash are intoxicated; burns, falls, and drowning are also more likely. Dating violence is another major concern. Often, the victim is too intoxicated to resist and may suffer memory loss for the event. In addition to physical harm, other potential consequences include unwanted pregnancies and sexually transmitted infections. Chronic use of alcohol, which is less likely to be seen in adolescents, can affect every organ in the body. Liver effects start with steatosis, proceed to fibrosis, cirrhosis, and eventually liver failure with hepatic encephalopathy. Arrhythmias, hypertension, cardiac myopathy, and strokes may result from the effects of alcohol on the cardiovascular system. Other complications include pancreatitis, an increased incidence of a number of cancers, and suppression of the immune system. PREP Pearls - Alcohol ingestion can affect every organ system in the body. - In the pediatric population, the effects of acute alcohol ingestion are more commonly encountered than the effects of chronic alcohol ingestion. - Seizures may occur after acute alcohol ingestion even without a history of epilepsy. ABP Content Specifications(s) - Identify the major physiologic consequences associated with alcohol use/abuse - Recognize the major behavioral consequences of alcohol use/abuse

An 18-year-old young woman is brought to your office by her mother for follow-up after an emergency department visit for loss of consciousness while partying with her friends. She admits to binge drinking alcohol on weekends but denies the use of drugs. She states that no violence or trauma occurred. There is no known family history of epilepsy, syncope, or cardiac problems. She currently is well with an unremarkable physical examination. You review with the girl the potential consequences of binge drinking alcohol on her body. Of the following, the MOST likely acute medical complication of her behavior is A. atrial fibrillation B. hepatic encephalopathy C. hepatic steatosis D. pancreatitis E. seizure

B. initiate oral clindamycin after obtaining a culture of the fluid The infant in the vignette has characteristic findings of bullous impetigo and therefore should be treated with an anti-staphylococcal agent such as clindamycin. Bullous impetigo is the result of a superficial infection by exfoliative toxin-producing Staphylococcus aureus and is not usually associated with systemic symptoms; therefore, oral antibiotics are sufficient in most cases. In very young patients (younger than 1 month of age), some experts would recommend parenteral clindamycin because of the concern for the immunocompetency of that age group. The usual organism is from phage group II, and is most often type 71, which produces exfoliative toxin A or exfoliative toxin B. These toxins are responsible for the characteristic findings. Initially, bullous impetigo appears as vesicles, which rapidly progress to flaccid bullae that may contain clear yellow fluid. The fluid may progress to a cloudy appearance. The Nikolsky sign (top layer of skin coming off when lightly rubbed) is not present. The bullae rupture easily, revealing superficial, moist, red erosions surrounded by a collarette of scale. Eventually a light brown crust may cover the lesions, which ultimately heal without scars. The lesions are often present in the diaper area and over the buttocks, as is seen with the patient in the vignette. Bullous impetigo most commonly occurs in children younger than 5 years of age and very frequently affects newborns and young infants. It may appear as soon as a few days after birth, and nursery outbreaks have been documented. Treatment of bullous impetigo depends on the prevalence of methicillin-resistant Staphylococcus aureus (MRSA) in the community. Fluid from a bullous lesion should be cultured before treatment is started. Some experts recommend treatment with topical anti-staphylococcal agents such as mupirocin when there are few or localized lesions; topical bacitracin would not be considered sufficient treatment. For non-MRSA disease, amoxicillin-clavulanate or dicloxacillin may be used with erythromycin or clindamycin for penicillin-allergic patients. For MRSA-positive organisms, clindamycin or trimethoprim-sulfamethoxazole may be used; trimethoprim-sulfamethoxazole is contraindicated for infants younger than 2 months of age. Hospitalization and intravenous vancomycin should be reserved for patients with systemic disease, such as that which occurs with severe staphylococcal scalded skin syndrome. Severe staphylococcal scalded skin syndrome is differentiated from bullous impetigo by the presence of systemic symptoms, widespread erythema, perioral lesions, and a positive Nikolsky sign. Other vesicular conditions that may occur during the neonatal period include transient neonatal pustular melanosis, epidermolysis bullosa, mastocytomas, and herpetic infections. Transient neonatal pustular melanosis is often present at birth, and appears as small (2-3 mm) pustules that evolve into hyperpigmented macules that resolve in 3 to 4 months. For patients with this condition, reassurance to the family and observation are appropriate. Epidermolysis bullosa is a group of genetic disorders, some of which can present at birth or soon after. These are differentiated from bullous impetigo by a more widespread appearance, mucosal and nail involvement, and recurrence of lesions. Skin biopsy is often indicated to make the diagnosis. A mastocytoma is distinguished clinically from bullous impetigo by the presence of the characteristic peau d'orange skin appearance, Darier sign (blistering or urticating at the site of trauma), and recurrence. Neonatal mucocutaneous herpes simplex infection can present with vesicles, but they differ from impetigo with smaller size, tendency to cluster, associated mucous membrane involvement and, when present, systemic symptoms. Neonates for whom herpes simplex infection is considered should be admitted to the hospital for a sepsis evaluation and parenteral antiviral therapy. The presence of bullae is not characteristic of candida diaper dermatitis and therefore antifungal agents are not indicated. PREP Pearls - Bullous impetigo is caused by exfoliative toxin-producing Staphylococcus aureus and occurs commonly during the neonatal period and early infancy. - Bullous impetigo is characterized by bullae that rupture leaving a red, moist erosion with a collarette of scale; lesions heal without scarring. - Bullous impetigo is generally not associated with systemic symptoms and usually can be treated with oral or topical antistaphylococcal antibiotics. - Nursery outbreaks of bullous impetigo require contact investigation and isolation to prevent spread to other newborns. ABP Content Specifications(s) - Recognize the clinical findings associated with neonatal impetigo

An 8-week-old infant who was the product of an uneventful pregnancy and delivery is taken into your office for evaluation of a rash. Her mother reports that she had no infections during the pregnancy. The infant spent 5 days in the nursery for phototherapy. Since discharge, the infant has done well, but over the past day she has developed blisters in her diaper area. She has had no fever and continues to eat well. Physical examination shows a well-appearing infant who has several 1-cm bullae over the lower abdomen, buttocks, and round crusted erosions with a rim of scale. A photo of the lesions is shown. Of the following, the MOST appropriate management for this infant is to A. admit her to the hospital for acyclovir therapy after lumbar puncture B. initiate oral clindamycin after obtaining a culture of the fluid C. prescribe topical nystatin to be applied to the bullae D. reassure the parents that this will resolve without treatment E. refer her to a dermatologist for a skin biopsy

B. drug-induced neutropenia The child in this vignette, although well appearing and afebrile, presents with marked neutropenia (absolute neutrophil count [ANC] = 372/mm3). He has been on intravenous therapy with nafcillin for osteomyelitis caused by infection with methicillin-susceptible Staphylococcus aureus for 2.5 weeks. He had a normal ANC at the time of diagnosis. In this setting, drug-induced neutropenia secondary to nafcillin is the most likely diagnosis and stopping the drug (and continuing treatment with an alternative agent such as cefazolin) should lead to improvement in the ANC over the next 1 to 2 weeks. This is a relatively rare idiosyncratic reaction seen with nafcillin and penicillin G. Cyclic neutropenia is an inherited disorder that would be expected to present before 8 years of age and typically is characterized by cycles of neutropenia occurring approximately every 3 weeks. During the nadirs, fever, oral ulcerations, gingivitis, pharyngitis, skin infections, and invasive bacterial infections may be seen. People of African descent may normally have a mild neutropenia with ANC between 1,000 and 1,500/mm3. This type of ethnic neutropenia is not associated with an increased risk of infection. The normal ANC at time of presentation and the severity of the observed neutropenia in this case argue against the diagnosis of ethnic neutropenia for the boy in this vignette. Acute viral infections such as Epstein-Barr virus and influenza may be associated with neutropenia early in the course of infection. The child in this case has no stigmata of acute viral infection to suggest this as a consideration here. Sepsis or severe bacterial infection may also lead to neutropenia, but the child would have to be clinically ill for this to be a consideration. Given the timing of the neutropenia in relation to the nafcillin therapy and the other components of the complete blood cell count being normal, there is no reason to suspect a laboratory error as the explanation for the child's neutropenia. A repeat white blood cell count and differential would be expected to demonstrate the same finding. This adverse reaction to nafcillin is a relatively rare example of the types of drug reactions to antibiotics seen in children. Allergic or hypersensitivity reactions are the most common adverse events associated with exposure to antibiotics. Such reactions may be immediate, beginning within 1 hour of the first dose of the drug, or delayed, occurring after days of treatment. Immediate reactions are typically manifest by urticarial rash, pruritus, and potentially angioedema. These reactions are immunoglobulin E-mediated and carry the risk of anaphylaxis with re-exposure to the offending agent. Delayed drug reactions typically begin 7 to 10 days after onset of therapy with a given agent and are characterized by maculopapular eruptions. Amoxicillin is the most commonly implicated agent associated with these types of reactions. A severe form of delayed antibiotic reaction is drug rash with eosinophilia and systemic symptoms (DRESS) syndrome that can occur 1 to 12 weeks into a course of treatment. The β-lactam drugs (penicillins and cephalosporins) are the antibiotics most often associated with allergic reactions. Other examples of drug reactions unrelated to allergy may include gastrointestinal upset or diarrhea, nephrotoxicity (eg, aminoglycosides, vancomycin), or "red man" syndrome (too rapid infusion of vancomycin) may lead to excessive release of histamine, causing the "red man" appearance. This is not an allergic reaction and can be managed, when it occurs, by slowing the rate of infusion from 1 to 2 hours per dose. PREP Pearls - Marked neutropenia can occur in the course of treatment with nafcillin or penicillin G. - Drug allergies are the most common manifestation of adverse drug reactions. - Immediate drug reactions, beginning within 1 hour of drug intake, are associated with urticaria, angioedema, and potentially anaphylaxis, and are immunoglobulin E-mediated. - Delayed drug allergies typically occur 7 to 10 days after beginning a course of therapy and are associated with maculopapular eruptions, but anaphylaxis is not a risk in this setting. ABP Content Specifications(s) - Recognize the adverse effects associated with the use of various antibiotic drugs

An 8-year-old African-American boy in your practice is on home intravenous antibiotic therapy with nafcillin for treatment of acute osteomyelitis of the distal humerus of his left arm. An aspirate in the operating room at the time of diagnosis grew methicillin-sensitive Staphylococcus aureus (MSSA). He has responded well clinically with resolution of fever and decrease in pain at the initial site of infection. He has no complaints today and is now being seen for follow-up 2.5 weeks into therapy. His temperature is 37.1°C. His physical examination reveals a well-healing incision site over the distal humerus in his left arm. The examination findings are unremarkable. A complete blood cell count obtained today shows the following: - White blood cell count, 6,200/µL (6.2 × 109/L), with 6% neutrophils, 76% lymphocytes, 14% monocytes, 3% eosinophils, and 1% basophils - Hemoglobin, 13.4 g/dL (134 g/L) - Platelet count, 387 × 103/µL (387 × 109/L) - Erythrocyte sedimentation rate, 22 mm/h (reference range, < 25.00 mm/h) - C-reactive protein, 0.60 mg/L (5.7 nmol/L) (reference range, < 0.80 mg/L [< 7.6 nmol/L]) The complete blood cell count at the time of his hospital admission showed the following: - White blood cell count, 12,000/µL (12 × 109/L), with 45% neutrophils, 5% bands, 40% lymphocytes, and 10% monocytes - Hemoglobin, 13.8 g/dL (138 g/L) - Platelet count, 450 × 103/µL (450 × 109/L) Of the following, the MOST likely cause of this patient's neutropenia is A. cyclical neutropenia B. drug-induced neutropenia C. ethnic neutropenia D. intercurrent viral illness E. laboratory error

A. additional evaluation is required to determine if the boy has a learning disability A "learning disability" is a disorder in the psychological processing of language-based information (spoken or written) that can lead to a difficulty in reading, writing, spelling, speaking, listening, or performing mathematics. Because school-based instruction is highly reliant on both understanding and using language, impairment in this arena can lead to serious difficulties in many different areas of school performance quite separate from intelligence-based abilities. Therefore, a student may have adequate ability as measured by IQ, but poor performance in a formal learning setting. An example of a learning disability is the case of a very intelligent child with poor mathematical performance in school who makes rapid gains in performance once visualization strategies are used for his mathematical instruction. The boy in the vignette, who has somewhat lower-than-average IQ scores (average score = 100), could be struggling in school because of his intellectual abilities rather than a learning disability. Another interpretation is that this child's struggles in school may be unrelated to either IQ or learning disability, but rather he may have attention-deficit/hyperactivity disorder (ADHD) or he may be mentally preoccupied with thoughts about stressors at home. Therefore additional evaluation is needed to determine if the child has a learning disability. On the Weschler IQ test, a verbal IQ 6 points higher than the performance IQ would not raise a suspicion of a learning disability. It would be slightly more suspicious if the verbal IQ were significantly lower than the math score because one's language abilities are the primary pathway through which learning can become impaired. Also if the degree of difference between verbal and performance IQ was greater, such as greater than 15 points or 1 standard deviation, it would be more suggestive of a specific learning problem. However, even this would not be sufficient to diagnose a learning disability. One would still need corroborative information about functional difficulty processing new linguistic information that is out of proportion to overall intelligence. An example would be a child who has difficulty decoding written words but can understand the same information if spoken. A low math computation score on the academic achievement test (the Wide Range Achievement Test) indicates that current math knowledge is below average, but does not explain "why." It could be that he has had poor instruction or has had reasons to not pay attention like being bullied. PREP Pearls - Intelligence tests are assessments of the potential for learning, and cannot on their own indicate the presence of a learning disability. - Achievement tests assess level of current knowledge. Although not diagnostic of a learning disability, achievement test scores below that expected based on intelligence raises the possibility of a learning disability. - Learning disabilities are disorders of the psychological processing of language-based information (spoken or written) that can lead to a difficulty in reading, writing, spelling, speaking, listening, or performing mathematics. ABP Content Specifications(s) - Interpret the results of specialized and standardized achievement tests, with emphasis on understanding the significance of discrepancies between categories

An 8-year-old boy has been struggling in school over the past 2 years. He recently had testing performed by the school psychologist. His mother brought these test results in to discuss with you. On the Wechsler Intelligence Scale for Children, he had a performance score of 88, a verbal score of 94, and a full scale score of 92. He had a Wide Range Achievement Test composite reading score of 89, math computation score of 83, and spelling score of 85. Of the following, the BEST statement to make to this parent is that A. additional evaluation is required to determine if the boy has a learning disability B. the difference between his reading composite score and math computation score indicates he has a learning disability C. the difference between his verbal IQ and performance IQ scores indicates he has a learning disability D. the math computation score on its own indicates he has a learning disability E. the overall lower achievement test scores relative to his IQ scores indicate he has a learning disability

A. 125/75 mm Hg with a blood pressure cuff bladder that is 80% of the length and 40% of the circumference of the upper arm BP cuff bladder should be wide enough to cover 3 quarters of the upper arm length from the acromion to the olecranon. In its update in 1996, the working group recommended that the width of the cuff bladder should equal 40% of the mid-upper arm circumference. PREP Pearls - Norms exist for blood pressure (BP) by age and gender and should be referred to when evaluating BP in children. - Early referral to a pediatric nephrologist or cardiologist is important for evaluation and early treatment. - A proper BP cuff bladder that is 80% of the length and 40% of the circumference of the upper arm at the midpoint is key in accurate BP measurement. ABP Content Specifications(s) - Understand the appropriate technique, including appropriate cuff size, for measuring blood pressure - Understand when to screen for an increased blood pressure and how to interpret the results

An 8-year-old boy is seen for a health supervision visit. There is a family history of hypertension. You have been monitoring his blood pressure (BP) annually for the last 3 years. Last year, his BP was 90/65 mm Hg when taken in your office. He is active in sports without any symptoms of fatigue. He has been consistently in the 50th percentile for weight. His BP is recorded at 125/80 mm Hg today at check-in. You would like to assess whether his blood pressure was taken optimally and if it is abnormal. You ask the staff to take the BP with the optimal size cuff. Of the following, true hypertension in this child would MOST likely be present if readings are: A. 125/75 mm Hg with a blood pressure cuff bladder that is 80% of the length and 40% of the circumference of the upper arm B. 125/75 mm Hg with a blood pressure cuff bladder that is 100% the length of the upper arm and 50% of the circumference C. 140/80 mm Hg and the blood pressure cuff bladder is 50% of the length of the upper arm and 80% of the circumference D. 150/80 mm Hg and the blood pressure cuff bladder is equal in length and width E. 180/90 mm Hg with a blood pressure cuff bladder completely encircling the lower leg

C. microarray In patients with developmental or intellectual delays, microarray has replaced the standard karyotype and fluorescent in situ hybridization subtelomere tests as the first line test due to its much "higher resolution" and improved sensitivity in detecting genetic abnormalities. Its diagnostic rate is twice that of the standard karyotype. A child does not have to be dysmorphic to recommend a microarray. Close attention should be given to signs of a metabolic disorder including growth failure, recurrent unexplained illness, seizures, ataxia, regression, hypotonia, "coarse" appearance, eye abnormalities, recurrent somnolence, abnormal sexual differentiation, hepatosplenomegaly, metabolic acidosis, hyperammonemia, structural hair abnormalities, and bone and skin abnormalities. If any of these signs are present, a screening metabolic workup should be initiated or the child should be referred to a specialist. This can include an acylcarnitine panel, carnitine profile, urine organic acids, serum amino acids, ammonia level, lactate, or pyruvate Exome sequencing is an emerging technology in genetics that is being utilized after extensive genetic testing has not revealed a unifying diagnosis for a child's multiple congenital anomalies, neurodegenerative condition, or intellectual disability combined with other systemic anomalies PREP Pearls - In a patient presenting with unexplained global developmental delay or intellectual disability, the American Academy of Pediatrics and the American College of Medical Genetics recommend a chromosomal microarray and fragile X testing in all patients, both male and female, in addition to a detailed clinical history, 3-generation family history, dysmorphologic examination, and neurologic examination. - Trisomy X features include hypertelorism, epicanthal folds, hypotonia, tall stature, clinodactyly, motor and speech delays, and an increased risk of cognitive deficits or learning disabilities in association with attention-deficit disorder/mood disorder. - 47,XYY patients typically resemble those with Klinefelter syndrome (47,XXY) with respect to the tall stature, verbal learning disabilities, delayed speech, and attentional difficulties; however patients with 47,XYY have normal pubertal development and testosterone levels, whereas males with Klinefelter syndrome frequently experience childhood-onset testicular failure. ABP Content Specifications(s) - Recognize the clinical features associated with a 47,XYY chromosome abnormality - Recognize the clinical features associated with a 47,XXX chromosome abnormality

An 8-year-old girl is brought to your office because of the mother's concerns for her poor performance in school, attention difficulties, difficulty with speech and word pronunciation, and anxiety tendencies. The mother reports that her daughter had decreased muscle tone, slow development as an infant, and required therapy for motor and speech delay. She has a history of well-controlled absence seizures for which she takes an antiepileptic medication daily. She currently is in a mainstream classroom with an Individualized Education Plan in place for her learning difficulties in language processing and comprehension. Her full scale IQ is 75 with a verbal IQ of 70. Birth history is essentially unremarkable, with the exception of the mother's advanced maternal age of 45 years at the time of delivery. The family history is negative for Parkinson disease, premature ovarian failure, intellectual disability, early sudden death, recurrent miscarriages, and other birth defects. Her 2 brothers and parents have IQs within reference range, and both parents have normal stature. Physical examination shows her height to be at the 90th percentile for age. She has mild hypotonia, epicanthal folds, clinodactyly, and long limbs. The remainder of the examination is unremarkable. Of the following, the MOST appropriate initial diagnostic test is A. acylcarnitine profile B. exome sequencing C. microarray D. serum amino acids E. urine organic acids

A. examine him in the upright position with and without Valsalva maneuver While most are asymptomatic, 10% of boys with a varicocele will present with heaviness or discomfort in the scrotum as described by the boy in the vignette. A varicocele is an abnormal dilation of the pampiniform plexus in the scrotum as a result of valvular incompetence of the internal spermatic vein. It can decompress when the patient is in the supine position, therefore it is important to examine the patient standing, especially noting whether the mass enlarges with the Valsalva maneuver. A grade 1 varicocele is palpable only with Valsalva maneuver; a grade 2 is not visible but is palpable without Valsalva; and a grade 3 is found on visual inspection. The paratesticular mass is classically described as a "bag of worms." Typically, varicoceles occur on the left side, with bilateral involvement occurring in only 2% of cases and unilateral right side involvement even less frequent. In advanced cases the involved testicle may be smaller than the contralateral side, although this is not usually apparent on physical examination. The higher the varicocele grade, the greater the risk for poor outcome, notably decreased fertility. The diagnosis of varicocele is made clinically, and additional studies are rarely needed for confirmation. However, testing may be indicated to evaluate complications or to rule out other diagnoses. Ultrasonography with Doppler flow is used most often to evaluate for testicular torsion. The physical findings of that condition (acute pain, scrotal swelling, marked testicular tenderness, absent cremasteric reflex) readily differentiate it from a varicocele. However, a urologist may obtain nonemergent scrotal ultrasonography for a patient with varicocele to assess testicular growth. A urinalysis and urine culture should be obtained if there are complaints of dysuria or if there is evidence of epididymitis, but again the physical findings (acute scrotal pain, erythema, swelling) are distinct from those of varicocele. Nucleic acid amplification studies for gonorrhea and chlamydia are appropriate for sexually active adolescents, particularly if they have had multiple partners, but this infection is not related to the presence of a varicocele. A nonurgent urology consult is indicated for symptomatic patients such as the boy in the vignette, those with higher-grade varicoceles, or those with significant testicular size discrepancy. The treatment goal for varicoceles is to preserve fertility, although how best to do this is controversial. To date, it is unknown if aggressive surgical treatment during adolescence preserves fertility. Among men with primary infertility, 40% are noted to have a varicocele, but only 20% of men with varicoceles are infertile. Identifying teens with varicoceles who may be at increased risk for infertility is important. One possible marker is the presence of testicular hypotrophy on the involved side; however, a few studies show that even with a 20% difference in testicular size, some adolescents will undergo "catch-up" growth of the smaller testicle. Researchers are exploring the role of gonadotropin monitoring to predict outcome. Long-term follow-up studies are needed to best define which adolescents will benefit from varicocelectomy to preserve long-term fertility. PREP Pearls - Physical examination to detect varicoceles should be done with the patient standing, with and without the Valsalva maneuver. - Varicoceles typically involve the left side and are described as having a "bag of worms" appearance. - Referral to a urologist for a varicocele is indicated for patients who have a higher grade varicocele, testicular size discrepancy, or are symptomatic. - The goal of varicocele treatment is to preserve fertility. ABP Content Specifications(s) - Recognize the clinical findings associated with a varicocele, and manage appropriately

An otherwise healthy 14-year-old boy complains of occasional discomfort and heaviness in the left side of his scrotum. He denies sexual activity and any history of trauma and has had no dysuria or discharge. He is afebrile. On physical examination, he is at sexual maturity rating 4. Upon inspection, there is no sign of inflammation and no discoloration. Of the following, the BEST next step in evaluation of this patient is to A. examine him in the upright position with and without Valsalva maneuver B. order scrotal ultrasonography with Doppler flow studies C. order a urinalysis and urine culture D. order urine nucleic acid amplification testing for gonorrhea and chlamydia E. refer him for urgent evaluation by a urologist

E. refer the family to a behavior management specialist The origin of oppositional difficulties in this child is most likely a mismatch between parental expectations (expecting that their child would be easygoing and compliant), and their child's inherent temperament (strong willed, with irregular daily patterns and negative initial reactions to novel stimuli like a new food). Challenging-to-parent temperamental traits have been found to be lifelong characteristics of individuals. Research on behavior management techniques shows us that functional outcomes for children with a challenging-to-parent temperament are highly influenced by caregiver responses to their child. When strong-willed children are raised by compassionate and adaptable parents, there is every reason to expect they will become highly functional adults and can become future leaders in their environments. When interacting with a parent who is dealing with this challenge, it is helpful to acknowledge that some children are simply more difficult to parent than other more "easygoing" children. Doing this avoids blaming parents for their child's difficulties, while still making it clear that the solution will depend on them learning approaches to manage their child. A parent who feels blamed for the child's misbehavior is less likely to follow through with a referral to a behavior therapist, or to try implementing new parenting strategies that could be really helpful. After empathizing with their experiences, a referral to a behavior management specialist would be the most appropriate intervention in this case. Research-proven approaches for managing oppositional behaviors depend on changing how caregivers (ie, parents and teachers) interact with children, not on asking the child to independently reflect on his or her actions and come to an internal decision to make different behavioral choices in the future. When one-on-one counseling is the only care delivery model available, one should refer the parents to a counselor to learn behavior management methods themselves. Individual play therapy sessions with a counselor are unlikely to lead to behavior change for this child. It is also appropriate to coach the mother to make a single meal for the whole family at night, which the child is free to choose to eat or not eat, and to have a consistent plan for what will happen if the child refuses to eat (ie, no other options offered, or a single acceptable option, such as a fruit, would be offered). This dinnertime advice is a single example of a behavior management technique that a specialist might advise. However, correcting a mealtime problem would address only one of the many difficulties this child and parent are having. Although it is reasonable to offer the behavior management advice that the parent should avoid bringing a tantrum-prone child on any unnecessary shopping trips, it would not be appropriate to tell a parent to avoid bringing the child out in public at all. A gluten-free diet might be advisable for a child with symptoms suggestive of a gastrointestinal disorder like celiac disease; however, strong-willed behavior and tantrums are not pathognomonic of celiac disease. Elimination diets are commonly advised on talk shows and on the internet for all kinds of child behavior problems. Because a gluten-free diet is going to be experienced by the child as aversive, and it is difficult to maintain healthy nutrition in a child on a gluten-free diet, the principle of first do no harm means this intervention should be avoided unless clinically necessary. PREP Pearls - Effective discipline involves setting limits that are delivered in a calm, consistent, and caring fashion. - The caregivers of the child with recurrent or worsening discipline problems should be referred to a therapist who can provide behavior management advice. - One-on-one counseling with a young child is unlikely to resolve discipline problems. ABP Content Specifications(s) - Advise parents regarding appropriate discipline and limit-setting for children of various ages

During a health supervision visit, the mother of a 5-year-old boy states that she has been having difficulty with his behavior. She says he is much more oppositional than her daughter was at this same age and that his behavior is getting worse. He has several tantrums each day when he does not get his way, and these tantrums even occur at public places like a grocery store. He has a generally intense mood, rather than being an "easy-going" childlike his sister. At night, his parents let him fall asleep wherever he wants to and then carry him up to his bed, so as to avoid having conflicts about enforcing a bedtime. His mother also states that he is an extremely picky eater. For this reason, she always makes one of his preferred foods at each meal, despite what the rest of the family is eating. The boy has no history of developmental difficulties. Of the following, the MOST appropriate advice to offer would be to A. coach mom to make a single meal at night for the whole family B. coach mom to stop bringing him to public places like the grocery store C. recommend a trial of a gluten-elimination diet for 2 weeks to 4 weeks D. refer the boy for play therapy with a one-on-one counselor E. refer the family to a behavior management specialist

C. his gross hematuria is associated with an identifiable underlying etiology The differential diagnosis for red urine is quite extensive, but the patient's history and urinalysis are helpful in differentiating the possible causes. Hematuria described as bright red, as for the child in this vignette is usually indicative of lower urinary tract bleeding, whereas glomerular hematuria (as in nephritis) is usually described as cola, tea or brown colored. Presence of blood clots, with or without dysuria, is indicative of urinary tract bleeding. An underlying cause is more frequently identified in patients with gross hematuria compared with those presenting with asymptomatic microscopic hematuria. A detailed history, physical examination, and urinalysis usually provide clues to the underlying cause of gross hematuria. In patients with symptomatic hematuria, the presenting history can guide the choice of investigations for identifying the underlying cause. In patients with history of trauma, a computed tomography (CT) scan of the abdomen and pelvis is indicated. Patients with nephrolithiasis present with varying degrees of flank pain/discomfort with the severe colicky pain radiating to the flank more commonly seen with large and/or obstructing calculi. Such patients may have dysuria with associated infection or hypercalciuria. Renal ultrasonography is the preferred initial imaging modality for patients with suspected nephrolithiasis. Spiral CT is the most sensitive test for renal stones, but the risk of radiation exposure in young children/adolescents should be weighed against the benefit of detecting small stones missed on renal ultrasonography. Patients with a history of perineal and meatal irritation may present with gross hematuria and can be given supportive care and reassurance. Patients with glomerulonephritis may present with edema or history suggestive of an underlying autoimmune disorder (joint pain and swelling, fatigue, malaise, oral ulcers, or skin rash). Children presenting with asymptomatic gross hematuria would usually require a detailed evaluation for the underlying cause. The workup in such cases includes urinalysis (to differentiate between red blood cells [RBCs] and heme pigments), urine calcium-creatinine ratio (hypercalciuria), serum creatinine and C3 (nephritis), renal ultrasonography (congenital anomalies or bladder mass), and hemoglobin electrophoresis (sickle cell disease). The patient in the vignette has asymptomatic bright red hematuria of 1 day's duration, which suggests lower urinary tract bleeding. The likely cause for the patient's symptoms could be infection (cystitis), hypercalciuria, congenital renal anomalies, or bladder mass. Pigmenturia (hemoglobinuria and myoglobinuria) can result in hematuria with a positive result on dipstick test for blood, but both conditions lack the presence of RBCs on microscopy. Therefore, the urine in pigmenturias has a clear sediment (lack of RBC) whereas the supernatant is red because of heme or myoglobin pigment. In patients with glomerular or lower urinary tract bleeding, the sediment is red (presence of RBC in urine) and the supernatant is clear. Cystoscopy is rarely indicated for children presenting with hematuria (gross or microscopic). It is indicated in the case of a bladder mass or urethral trauma. PREP Pearls - An underlying cause is more frequently identified in patients with gross hematuria compared with those presenting with asymptomatic microscopic hematuria. - A detailed history, physical examination, and urinalysis usually provide clues to the underlying cause of gross hematuria. - Hematuria described as bright red is usually indicative of lower urinary tract bleeding, whereas glomerular hematuria (as in nephritis) is usually described as cola, tea or brown colored. - Pigmenturia (hemoglobinuria and myoglobinuria) results in hematuria with a positive finding on dipstick test for blood and absence of RBCs on microscopy. - Cystoscopy is rarely indicated for children presenting with hematuria. ABP Content Specifications(s) - Plan the appropriate clinical evaluation of gross hematuria

In your office, you are evaluating a 12-year-old boy who has had blood in his urine for the past 1 day. He describes the urine as bright red. He has no complaints of fever, frequency, painful urination, or flank pain. His vital signs are normal for his age. His physical examination findings are normal. His urinalysis demonstrates a specific gravity of 1.015, pH of 6.0, 4+ blood, 4+ leukocyte esterase, and no protein or nitrites. His urine microscopy shows more than 100 red blood cells per high-power field, 10 to 50 white blood cells per high-power field, and no crystals or bacteria. You discuss the evaluation of gross hematuria with the medical student assigned to your practice. Of the following, the MOST accurate statement regarding the evaluation of this child is A. cystoscopy is usually indicated in making the diagnosis B. depth of color in gross hematuria is indicative of the degree of blood loss C. his gross hematuria is associated with an identifiable underlying etiology D. if the urine sediment is clear and supernatant red, the etiology of hematuria is blood E. if the urine sediment is red and supernatant clear, the etiology of hematuria is not blood

A. audiologic evaluation The prevalence of hearing impairment among infants with preauricular skin tags or pits is higher. These findings may be the first sign identifying a syndrome associated with sensorineural hearing loss, so verifying that the result of the newborn hearing screen is normal is the most important first step. The incidence of renal anomalies in patients with isolated preauricular pits does not differ significantly from that in the general population. External ear malformations and renal anomalies occur in a number of multiple congenital anomaly syndromes. Patients with auricular anomalies should be examined carefully, specifically looking for facial asymmetry, colobomas, choanal atresia, cardiac murmurs, branchial cysts or sinuses, distal limb anomalies, and imperforate anus. Renal ultrasonography should be performed if the child has any other malformations or dysmorphic features, family history of deafness, auricular or renal malformations, or maternal history of gestational diabetes. PREP Pearls - Infants with preauricular pits or sinuses should undergo newborn hearing screening and receive a referral for formal audiologic evaluation if the screening result is abnormal. - The presence of a simple preauricular pit does not warrant further evaluation with renal ultrasonography or laboratory tests. ABP Content Specifications(s) - Know the significance of preauricular sinuses and pits

On examination of a healthy term Asian male newborn, you notice a pinpoint indentation located just anterior to the helix of his right ear. A thorough physical examination reveals no other abnormalities. The mother reports no medical problems or medications used during her pregnancy. Family history is negative for chronic kidney disease. Of the following, the MOST important screening test is A. audiologic evaluation B. genetic testing C. renal ultrasonography D. serum creatinine E. urinalysis

C. large tongue The newborn in the vignette has the clinical findings of Beckwith-Wiedemann syndrome (BWS), which is associated with macroglossia (large tongue). Beckwith-Wiedemann syndrome is related to alterations on chromosome 11p15.5, with a familial transmission rate of 15%. There is an increased incidence seen in pregnancies requiring assisted reproductive technology. Beckwith-Wiedemann syndrome is an overgrowth syndrome with affected infants often being large for gestational age at birth. Other manifestations of somatic overgrowth include macroglossia, visceromegaly, and hemihypertrophy. Islet cell hyperplasia leads to the hypoglycemia seen in up to 50% of affected infants. Abdominal-intestinal wall defects including omphalocele, umbilical hernia, and diastasis recti are commonly seen with BWS. An omphalocele is an abdominal-intestinal wall defect formed by the protrusion of bowel into the base of the umbilical cord, whereas a gastroschisis is formed by loops of bowel extending through a defect in the abdominal wall to the right of the umbilical cord. Both require immediate surgical evaluation and management in the neonatal period. The initial management in the delivery room includes covering the exposed bowel with saline-soaked sterile dressings, inserting a gastric tube to decompress the bowel, and placing the newborn in a bowel bag to the level of the axilla to minimize fluid and heat losses. Parenteral fluid resuscitation is often needed because of increased insensible losses associated with the defects. The type of abdominal-intestinal wall defect guides further evaluation. Gastroschisis is an isolated defect seen in infants with intrauterine growth restriction: approximately 10% will have coexisting intestinal atresia or stenosis. Omphalocele is associated with other anomalies in up to 70% of cases. Beckwith-Wiedemann syndrome may be seen in 10% of affected infants, whereas karyotype anomalies, including trisomy 13, 18, and 21, may be found in up to 30% of cases. Echocardiography should be performed in any infant with an omphalocele because of the 50% risk of congenital heart disease. Anorectal malformations, hypospadias, patent urachus, and single umbilical artery are not associated with BWS. They are unlikely to be found in the newborn in the vignette. PREP Pearls - An omphalocele is an abdominal wall defect formed by the protrusion of bowel into the base of the umbilical cord, whereas a gastroschisis is formed by loops of bowel extending through a defect in the abdominal wall to the right of the umbilical cord. - The initial management of an abdominal-intestinal wall defect in the delivery room includes covering the exposed bowel with saline-soaked sterile dressings, inserting a gastric tube to decompress the bowel, and placing the newborn in a bowel bag to the level of the axilla to minimize fluid and heat losses. ABP Content Specifications(s) - Plan the appropriate evaluation and management of a newborn infant who has abdominal-intestinal wall defect

You are assessing a newborn with hypoglycemia 8 hours after birth. She was born at 37 weeks' gestation to a 40-year-old primagravida woman whose pregnancy was notable only for the use of assisted reproductive technology. The newborn required admission to the special care nursery shortly after birth for intravenous glucose therapy to maintain her blood sugar values over 45 mg/dL (2.5 mmol/L). Your physical examination reveals a large for gestational age newborn with an abdominal wall defect (umbilical defect with bulging membrane covered contents). Of the following, the additional clinical finding this patient is MOST likely to have is A. anorectal malformation B. hypospadias C. large tongue D. patent urachus E. single umbilical artery

D. optic glioma The boy in the vignette most likely has neurofibromatosis type 1 (NF1). This diagnosis could be definitively made if there is an optic glioma. Neurofibromatosis type 1 is an autosomal, dominantly inherited neurocutaneous disorder characterized by café au lait macules, neurofibromas, and bony lesions. Often, children with NF1 have macrocephaly and learning disorders, as does the boy in the vignette. Diagnosis is based on the presence of 2 or more of the following clinical criteria: Six or more café au lait macules, greater than 5 mm in diameter in prepubertal children or 15 mm in diameter in postpubertal children - Two or more neurofibromas - One or more plexiform neurofibroma - Axillary or inguinal freckling - Optic glioma - Two or more iris hamartomas (Lisch nodules) - Osseous lesion such as sphenoid dysplasia or tibial pseudarthrosis - First-degree relative with NF1 The clinical presentation of macrocephaly, learning disorder, and more than 6 café au lait macules that are greater than 5 mm in diameter in a prepubertal boy suggests a diagnosis of NF1. Obtaining a family history and performing dermatologic and ophthalmologic examinations in the office can expedite a diagnosis for the boy and his family. In the absence of family history, the clinical diagnosis of NF1 can be difficult, because the findings are often absent in younger patients. Neurofibromas and optic gliomas are often slow growing, and the patient may be asymptomatic until late in childhood, adolescence, or even adulthood. Referral to an ophthalmologist may be necessary to diagnose and establish follow-up care for an optic glioma. Medical genetics referral would be helpful to establish a genetic diagnosis and estimate risk for future children. Cutaneous hemangioma and iris coloboma are not particularly associated with NF1 and are not part of the diagnostic criteria. Leptomeningeal angioma is associated with Sturge-Weber syndrome, which this boy does not have. Renal angiomyolipomas are seen in individuals with tuberous sclerosis. PREP Pearls - Diagnosis of neurofibromatosis type 1 (NF1) is based on clinical criteria. - Typical physical findings of NF1 may not be present until late childhood or adolescence. ABP Content Specifications(s) - Recognize the clinical findings associated with neurofibromatosis in patients of various ages

The parents of a 6-year-old boy bring him to your office for evaluation because he is struggling in first grade. The results of learning testing done at the school are pending, but the parents were told he most likely has a learning disorder, and the school wants you to evaluate him for a medical cause of his problems. The boy's parents report he pays attention well at school and at home and that he is trying hard to learn. He has no chronic medical problems. His vision and hearing are normal. His physical examination results show an occipital frontal head circumference at the 90th percentile, with a height and weight at the 50th percentile. His blood pressure is 96/56 mm Hg, heart rate is 92 beats/min, and respiratory rate is 32 breaths/min. On his back and legs, he has 8 flat, brown macules with regular borders, 6 mm to 10 mm wide. Of the following, the additional finding this boy is MOST likely to develop is A. cutaneous hemangioma B. iris coloboma C. leptomeningeal angioma D. optic glioma E. renal angiomyolipoma

B. insert an orogastric tube The newborn in the vignette should have an orogastric tube inserted to rule out esophageal atresia (EA). A fetus with EA can present in utero with polyhydramnios because of the inability to swallow amniotic fluid. When the membranes are ruptured before delivery, copious amniotic fluid may be seen. If not identified in the delivery room, newborns with EA will often have spitting, drooling, choking, and respiratory distress because of their continued inability to handle secretions or feedings. These findings should prompt a clinician to attempt to pass an orogastric tube. The inability to pass an orogastric tube into the stomach suggests an underlying diagnosis of EA. The diagnosis of EA may be confirmed with chest radiography, which demonstrates coiling of the catheter tube in the esophagus. Five types of congenital tracheoesophageal fistula (TEF) and EA have been described, with type C (EA with distal pouch fistula) occurring in approximately 85% of cases. Four types are associated with EA, which present in the newborn period. Occurring in only 4% of cases, type E (H-type fistula with a patent esophagus) may present later in infancy with refractory bronchospasm and recurrent pneumonias. More than 50% of infants with EA have other associated anomalies, with VACTERL (vertebral, anorectal, cardiac, tracheal, esophageal, renal, limb) syndrome most common. Prompt identification of EA in full-term newborns leads to a survival rate greater than 95% if no cardiac disease is present. The short- and long-term complications of TEF/EA are related both to the underlying structural abnormalities and the consequences of the surgical correction. A newborn with the diagnosis of EA requires transfer to a tertiary center with a pediatric surgical team. Stabilization at the referring center is focused on the maintenance of a patent airway and prevention of aspiration pneumonia. After inserting an orogastric tube in the newborn in the vignette, chest radiography should then be performed to demonstrate the location of the tube in the esophagus. Supplemental oxygen may be required if an infant with EA is hypoxic, but the newborn in the vignette has an acceptable oxygen saturation value of 95% in room air. Endotracheal intubation should be avoided, as severe gastric distention may arise if a distal pouch fistula is present. Echocardiography should be performed at the tertiary center because up to 35% of infants with EA are found to have structural heart disease. Chest physiotherapy has no role in the care of the newborn with EA. PREP Pearls - Esophageal atresia should be considered in newborns with spitting, drooling, choking, and respiratory distress because of their continued inability to handle secretions or feedings. - The inability to pass an orogastric tube into the stomach suggests an underlying diagnosis of esophageal atresia. ABP Content Specifications(s) - Recognize the clinical and laboratory features associated with tracheoesopheal fistula in a newborn infant

The normal nursery staff calls you to assess a term newborn who has turned dusky twice after breastfeeding. The newborn was delivered 4 hours ago by scheduled elective cesarean delivery at term following an uneventful pregnancy. Artificial rupture of the membranes occurred at the time of delivery and revealed copious clear amniotic fluid. The newborn emerged vigorous, and no resuscitation was needed. The nurses report that the newborn has spit clear mucus several times since delivery, requiring bulb suctioning. An examination reveals a pink, well-perfused newborn with a heart rate of 140 beats/min, a respiratory rate of 70 breaths/min, and an oxygen saturation of 95% on room air. The newborn has clear lung fields with mild subcostal retractions and no murmur. Of the following, the MOST appropriate next step in management is to A. initiate nasal cannula oxygen supplementation B. insert an orogastric tube C. obtain a chest radiograph D. order an echocardiogram E. perform chest physiotherapy

D. provide discipline based on the child's behavior without reference to the specific lie Lying behavior can cause a great amount of anxiety and concern for parents. Telling lies is a normal human behavior that appears long before children could reasonably be expected to adopt a personal moral code of truthfulness or honesty. As soon as children realize that their parents are not always aware of their actions, lying to obtain benefits or to avoid negative consequences becomes a possibility. Parents often engage in dysfunctional scenarios regarding lying interventions. For example, a parent who already knows that a child has done something that is not allowed, such as taking a sibling's toy, may initiate a behavior intervention with a question like, "Did you take her toy?" This approach essentially asks the child to practice lying, because the child knows a negative consequence will result if they say yes and a negative consequence will result if they say no but are still found out. The only desirable option left for the child would be to tell a lie convincingly enough to succeed in avoiding a negative consequence. In other words, working to catch a child lying and then punishing those lies can have the opposite effect, ultimately, of training the child to get better and better at telling lies over time. Behavior therapists see "intermittent reinforcement" in this sequence, in that a behavior that is occasionally rewarded is often repeated. The preferred approach with a child who is repeatedly lying to a parent to avoid punishment is to provide discipline/consequences based on what the parent actually knows transpired. In a sibling conflict where each child makes a case about the other being at "fault," a disciplinary choice is required which affects both individuals for involvement in the transgression, rather than applying discipline based on the most convincing story. Avoiding giving the child opportunities for and encouragement to lie demonstrates the parenting value that "actions speak louder than words." This is not to say that lying behavior should never be punished on its own. However, when lying is a recurrent issue, switching the parents' focus to delivering consequences for the specific misbehaviors, rather than the lies they attempt to cover up, helps to halt a worsening lying habit. Lying at 3 years of age is not unusual or pathological for a normally developing child. A typical lie a 3-year-old child might tell could involve eating unobserved a cookie that they were told not to eat, and then when asked if they ate the cookie the child answers "no." The fact that the child in the vignette was first observed to lie at age 3 years would not indicate a need for mental health treatment. Child mental health counseling, in general, is not an appropriate or effective intervention for isolated lying behavior. However, referral to a mental health professional who can provide parenting support and advice might be appropriate, if the lying and/or other areas of parenting difficulty are causing significant dysfunction in the household. Temporarily removing access to something the child enjoys as a consequence for a behavioral transgression is an appropriate behavior management strategy throughout most of childhood. However, the nature, timing, and duration of this consequence all greatly influence the effectiveness. For instance, a 5-year-old child is unlikely to remember the reason for receiving a punishment, such as not being allowed to watch television for 2 full weeks. Another risk with long-lasting consequences at young ages is that other undesired behaviors are quite likely to occur during that time frame, which leads the parent to enact additional restrictions that can no longer be differentiated by the child, such as having their no-television ban extended for multiple months. Once everything rewarding or fun has been removed as transgressions stack up, parents may find they have little leverage left with their child. Stealing can be a more problematic behavior than lying, and the parents should intervene by having the child undo the theft. The child should either return stolen articles or render their equivalent in earned money or reparation services. Children with recurrent stealing behaviors should be referred for professional assistance. PREP Pearls - Lying is not an abnormal behavior for children. For children who lie, parents should try to deliver discipline based on their child's known actions rather than in response to the child's story. - Trying to actively catch children telling lies is counterproductive, providing a reward for successful lying behavior. - For children who steal, part of the discipline response should be for parents to help the child make amends by having them return the stolen goods. ABP Content Specifications(s) - Plan the appropriate management of stealing behavior in school-age children - Plan the appropriate management of lying behavior in school-age children

The parents of a 10-year-old boy report being very concerned with his persistent lying both to them and to other adults. They report that he started lying to them when he was 3 years of age. Examples of lies he has told recently include denying he had broken a lamp in their house, saying that he had finished his homework when he had not, and denying having taken a toy from his sister when he had. His parents report that they regard lying as sinful and that his older sister almost never lied. The parents and the boy often get into significant arguments about a suspected lie. They report that he is frequently oppositional at other times as well. Of the following, the BEST next step for his parents to take is to A. ask him for more details that would falsely embellish the suspected lie, then confront him about the inconsistencies B. bring him to a mental health counselor since his lying began at 3 years of age C. implement a negative consequence, such as no television for 2 weeks, for every lie told D. provide discipline based on the child's behavior without reference to the specific lie E. tell their son that he will earn screen time after a set number of days without lying

C. prior warming up for 15 min prevents episodes The adolescent in the vignette has exercise-induced bronchospasm (EIB) that is most strongly supported by the historical observation that prior warming up for 15 min prevents episodes. Exercise-induced bronchoconstriction is defined as the transient narrowing of the lower airways that occurs after strenuous exercise. The symptoms of EIB typically occur 5 to 10 min after onset of strenuous physical activity and include symptoms of cough, wheeze, chest tightness, shortness of breath, or excessive mucous production. These symptoms occur even in patients who are in good physical condition and occur in response to heating and humidifying large volumes of inspired air during a short period. Symptom severity is linked to the water content of the inspired air or the level of ventilation sustained during exercise, which explains why the patient is able to swim without difficulty. Exercise-induced bronchospasm may be observed in patients who have or do not have chronic asthma. The EIB practice parameter recommends that the term Exercise-induced Asthma (EIA) no longer be used "because exercise does not induce asthma but rather is a trigger of bronchoconstriction." The diagnosis of EIB usually requires demonstration of a decrease in Forced Expiratory Volume in 1 second (FEV1) of 10% to 15% of the pre-exercise value after exercise. Exercise-induced bronchospasm is a manifestation of bronchial hyper-responsiveness and is often the first sign of asthma, although not all individuals with EIB develop asthma. β2-agonist by inhalation is the most widely used treatment. It is recommended immediately before exercise to prevent EIB. Pre-exercise warm-up done at 60% to 80% maximum heart rate provides partial attenuation of EIB secondary to the occurrence of a refractory period lasting from 1 to 3 hours. Symptoms accompanied by inspiratory stridor, occurring at the onset or anytime during the duration of physical activity, and responding to breathing (relaxation) techniques, suggest the possibility of exercise-induced laryngeal dysfunction (EILD). Exercise-induced laryngeal dysfunction is a condition that primarily includes vocal cord dysfunction (VCD) and other glottic abnormalities that can mimic EIB. Inspiratory stridor is a differentiating hallmark sign with EILD. The subjective perception of difficulty taking in air (rather than breathing out air) also suggests upper airway tightness. Lack of response to bronchodilators such as albuterol is another differentiating feature suggesting EILD. Inspiratory flattening or truncation of the flow-volume loop on spirometry and a decrease in FEV1 with no alteration in the FEV1/FVC ratio (when symptomatic) are consistent with, but not diagnostic of, vocal cord dysfunction. The diagnosis can be confirmed by demonstrating paradoxical vocal cord movement while performing direct laryngoscopic visualization during an acute attack. Vocal cord dysfunction has been described more commonly in individuals who are high achievers and can be triggered by stress and emotional conditions. The literature also reveals a high incidence of VCD in persons with psychiatric conditions; therefore, the history of undergoing counseling for an anxiety disorder might raise suspicion of these symptoms being attributable to VCD rather than EIB. Concomitant VCD has been reported in up to 40% of individuals with asthma and EIB, rendering the diagnosis and management a little more challenging. There is no pharmacological treatment for VCD (aside from treating co-existing asthma or EIB). Breathing exercises designed to promote abdominal or diaphragmatic breathing and to eliminate abnormal vocal movement are helpful in the management of this condition. PREP Pearls - Exercise-induced bronchospasm (EIB) should be suspected in any child who exhibits symptoms of shortness of breath, chest tightness and wheeze within 5 to 15 min of onset of strenuous physical activity. - The symptoms of EIB can be mitigated by pre-exercise warming-up at maximal heart rate of 60% to 80% or pretreatment with inhaled short acting ß2-agonist medication. - Clinical features such as history of "difficulty taking in air," inspiratory stridor, and lack of response to short actingßß2-agonist inhaler should raise the suspicion of exercise-induced laryngeal dysfunction or vocal cord dysfunction. ABP Content Specifications(s) - Recognize the clinical features associated with exercise-induced asthma

The parents of a 16-year-old male adolescent seek your help in dealing with the coach of the varsity soccer team. The adolescent has been having shortness of breath and noisy breathing that begins about 15 min into his game and requires him to be pulled out. The coach has berated him for being unmotivated, not conditioned enough, and has been less inclined to put him in to play. The parents are hoping for a sports scholarship in college and state that he practices intensely and is very committed. He is also an excellent swimmer and typically does not have these symptoms while swimming. His physical examination reveals a muscular and well-toned young man with clear lungs on auscultation, and an otherwise unremarkable examination. You make a tentative diagnosis of exercise-induced bronchoconstriction. Of the following, your diagnosis would be MOST strongly supported by the history that A. breathing exercises do not prevent episodes B. he is undergoing counseling for anxiety disorder C. prior warming up for 15 min prevents episodes D. symptoms are accompanied by inspiratory stridor E. symptoms occur at onset of strenuous exercise

B. begin a rabies vaccine series Although rabies virus infection is rare in the United States, the vast majority of indigenously acquired rabies in the United States is from bat exposure. A scratch or bite from a bat may not be readily apparent. In settings in which the contact is unknown, such as an unattended child in a room with a bat or, as in this vignette, sleeping in a room in which a bat is found, potential rabies exposure must be considered. If the animal is captured, it can be euthanized and its brain examined for rabies virus. If not (or if the brain examination is positive), institution of rabies prophylaxis is recommended. Present guidelines call for a combination of human rabies immune globulin (HRIG) and vaccine given on day 0 and subsequent doses of rabies vaccine on days 3, 7, and 14. If apparent, infiltration with as much of the HRIG as possible around the bite site is recommended. The remainder of the dose is given intramuscularly (IM). The vaccine is administered IM in the deltoid or anterior aspect of the thigh. Once signs or symptoms of rabies appear, HRIG and vaccine are not of benefit. All mammals are potentially at risk for acquiring rabies with bats, raccoons, skunks, foxes, coyotes, and bobcats being the primary potential sources for human and animal exposure in the United States. Lagamorphs (eg, rabbits, hares) and rodents (eg, squirrels, hamsters, guinea pigs, gerbils, rats, mice) rarely transmit rabies. Secondary bacterial infection may be a concern with larger animal bites, but is not a significant risk from a bat exposure, as reported in this vignette, making administration of antibiotics unnecessary. Similarly, assessing the need for tetanus prophylaxis may be indicated after a dirty wound from a larger animal, but not with a bat exposure with no visible lesion. Local reactions are not a hallmark of bat bites such that monitoring for skin lesions over time is of no clinical utility. Reassurance would not be appropriate here because, as noted previously, finding a bat in a room with someone who has been sleeping there is considered adequate exposure to rabies and prophylaxis as outlined above is indicated. PREP Pearls - Bat exposure is the leading cause of potential transmission of rabies in the United States. Bat bites are small and may not be felt in the setting of an unattended child or someone asleep in a room in which a bat is found. - Rabies prophylaxis consists of a dose of human rabies immune globulin and rabies vaccine on day 0 and additional doses of vaccine on days 3, 7, and 14. - All mammals may become infected with rabies with bats, raccoons, skunks, foxes, coyotes, and bobcats as the leading sources in the United States. Lagamorphs (eg, rabbits, hares) and rodents rarely transmit rabies. ABP Content Specifications(s) - Plan appropriate post-exposure rabies prophylaxis

The parents of a 17-year-old college student in your practice call to ask your advice. The student awoke this morning to find a bat flying around his dormitory room. He and his roommate opened the window and the bat flew out. He feels well and on self-examination noticed no skin lesions or bite marks. His immunizations are up to date, including receipt of diphtheria-tetanus-pertussis vaccine at 11 years of age. Of the following, the BEST course of action is to A. begin a 5-day course of amoxicillin-clavulanic acid B. begin a rabies vaccine series C. get a tetanus vaccine booster D. monitor his skin for signs of new skin lesions over the next 2 weeks E. provide reassurance

A. adenoidectomy is contraindicated Recurrent acute otitis media and chronic otitis media with effusion and hearing loss are indications to consider referral for tonsillectomy and adenoidectomy (T&A). It is important that pediatricians are aware of the contraindications and complications of these procedures. The boy in the vignette exhibits the characteristics of submucous cleft palate (SMCP), so adenoidectomy is contraindicated. The oropharynx should be examined for SMCP in all children for whom adenoidectomy is being considered. Submucous cleft palate should be suspected in patients with notching or palpation of a V shape along the junction of the hard and soft palates. The classic characteristics of SMCP includes the triad of bifid uvula, diastasis of the muscles in the midline of the soft palate with intact mucosa, and notching of the posterior border of the hard palate. Only a small percentage of cases of SMCP are symptomatic, but one of the most common consequences of SMCP is recurrent acute otitis media or chronic serous otitis media. Adenoidectomy is contraindicated in these patients because of the risk of acquired velopharyngeal insufficiency (VPI) after the procedure and worsening of an existing speech problem. Velopharyngeal insufficiency sometimes develops after adenoidectomy in children without predisposing factors. Additional contraindications to T&A include other velopharyngeal conditions that predispose to VPI (cleft palate, impaired palatal function because of neuromuscular causes), anemia (hemoglobin < 10 g/dL [100 g/L]), disorders of hemostasis, and acute local nonobstructive infection of the upper airway. Fortunately, the operative complications of T&A are rare, but they may be life threatening. These include anesthesia-related problems, such as difficulties with airway management, local damage caused by endotracheal intubation, aspiration, arrhythmias, adverse reactions to the anesthetic agent including malignant hyperthermia, and hemorrhage. The most common postoperative complications are throat pain, otalgia, and general anxiety or sleep disturbance. The incidence of postoperative hemorrhage is low, but it can be serious. Dehydration is a risk in children with persistent vomiting or refusal to swallow because of pain. Infection, upper airway obstruction, and VPI are less common complications. Children younger than 3 years of age, those with severe obstructive sleep apnea, coagulopathy, or other comorbid conditions (craniofacial or neuromuscular disorders, sickle cell disease, prematurity, morbid obesity) benefit from postoperative observation in the hospital. Pain relief is an important part of postoperative care, and acetaminophen or a nonsteroidal anti-inflammatory drug such as ibuprofen should be offered. Aspirin and ketorolac are contraindicated because of the increased risk of hemorrhage. In 2013, the US Food and Drug Administration added a black box warning that codeine should not be used for pain in children after tonsillectomy or adenoidectomy because its use may lead to rare, but life-threatening adverse events or death in certain children. The vignette highlights the importance of awareness of all these contraindications and complications. Although T&A may be considered in patients with recurrent otitis media, it is not necessary and is contraindicated in the boy in the vignette. Antibiotic prophylaxis or the use of nasal decongestant sprays is not clearly indicated from the existing evidence, especially in this asymptomatic boy with normal hearing. Hypernasal speech may be associated with VPI, but other articulation problems are not common and ongoing speech therapy may be helpful. A change in therapist is not warranted. PREP Pearls - Contraindications to tonsillectomy or adenoidectomy include velopharyngeal conditions that predispose to velopharyngeal insufficiency, anemia (hemoglobin < 10 g/dL [100 g/L]), disorders of hemostasis, and acute nonobstructive upper airway infection. - Life-threatening complications associated with tonsillectomy or adenoidectomy are rare. - Throat pain, otalgia, anxiety, or sleep disturbance are the most common postoperative complications. - Codeine should not be used for pain management in children after tonsillectomy or adenoidectomy. ABP Content Specifications(s) - Recognize complications associated with tonsillectomy and/or adenoidectomy, including those associated with velopharyngeal insufficiency

The parents of a 7-year-old boy come to your office for a referral to otolaryngology to discuss tonsillectomy and adenoidectomy. The patient has a history of recurrent otitis media and tympanostomy tubes were placed at 3 years of age. He has had fewer ear infections since, but his mother attributes his difficulty with speaking clearly to problems with hearing. On physical examination, you notice that the patient has hypernasal speech that is difficult to understand and notching of the posterior border of the hard palate. The patient is comfortable breathing through his nose and the remainder of his physical examination is unremarkable. A hearing screen done in your office today is within normal limits. Of the following, your BEST recommendation for the patient at this time is that A. adenoidectomy is contraindicated B. antibiotic prophylaxis is recommended until surgery C. recurrent otitis media necessitates tonsillectomy and adenoidectomy D. referral to a new speech therapist is needed E. a trial of a nasal decongestant spray is warranted

A. immediate referral to a pediatric surgeon for operative management Ingestion of multiple magnets carries a high risk of complications and preemptive removal is warranted. Magnets located in the esophagus or stomach should be removed promptly using endoscopy. Treatment of children with multiple ingested magnets located beyond the stomach depends on their symptoms and progression of the magnets through the gastrointestinal tract. Asymptomatic children may be followed closely with serial examinations and radiography every 4 to 6 hours. Symptomatic children, or those with multiple magnets that are not seen to progress on repeat radiography, should undergo operative removal of the magnets. PREP Pearls - When ingested, 2 or more strong magnets may attract across layers of bowel and lead to pressure necrosis, fistula, perforation, infection, or obstruction; this could result in severe consequences, including intestinal resection. - The need for more than 1 radiologic view to avoid misclassification of a multiple magnet ingestion as a single magnet ingestion should be emphasized because magnets may stick together and overlap on a single radiographic view. - Management of magnet ingestion depends on the number, location, and type of magnet ingested, as well as on the timing of the ingestion. ABP Content Specifications(s) Plan the management of a patient who has ingested a magnet

The parents of a healthy 12-year-old girl bring her to your office 4 hours after she inadvertently swallowed 2 small magnetic balls, which she was using to mimic a tongue piercing. The girl appears well. She reports having intermittent episodes of "my belly hurting just a little" over the past 2 hours, but she denies any episodes of vomiting. On physical examination, her vital signs are normal for age. Her abdomen is soft with normal bowel sounds and mild diffuse tenderness with palpation, but there are no peritoneal signs. The remainder of your physical examination findings are unremarkable. Plain radiography of the abdomen reveals that the 2 magnetic balls are lined up, "side-by-side," in the proximal portion of the girl's small intestine. Of the following, the BEST next step in management at this time is A. immediate referral to a pediatric surgeon for operative management B. reassurance with appropriate anticipatory guidance C. referral to a pediatric gastroenterologist if there is no improvement within 24 hours D. repeat abdominal radiograph within 1 week E. repeat abdominal radiograph within 24 hours

E. refer to surgery The young boy in the vignette has the classic history associated with a dermoid or dermoid cyst. A dermoid is a congenital lesion resulting from inclusion of ectodermal elements during closure of the neural tube adjacent to fetal suture lines. It is the most common orbital tumor seen in childhood. Complete surgical excision, with care not to rupture the cyst, is the standard of care, so referral to surgery is the correct answer. Dermoids will continue to grow slowly and there is a small risk of complications. Deep orbital dermoid cysts may not be detected until the adolescent years or later. These deep dermoids often present with proptosis. All patients with midline lesions or those who present with lesions in adulthood should be evaluated with imaging PREP Pearls - Complete surgical excision is the preferred treatment for a dermoid cyst. - Superficial dermoid cysts occur most commonly on or superior to the orbital ridge in children. - Dermoids that occur in the midline have a greater potential for deep extension and need evaluation for bony erosion and central nervous system communication. ABP Content Specifications(s) Recognize the clinical findings associated with a dermoid

The parents of a healthy 15-month-old boy arrive at your office inquiring about a small lump near his eyebrow that they have noticed for several weeks. They state this lump has never been red or tender. There is no history of trauma. The lump seems to be getting a little larger over time. On physical examination, you find a 1.5 cm mobile subcutaneous, rubbery nodule just above the lateral margin of the right eyebrow. The remainder of the examination is unremarkable. Of the following, the BEST initial plan for management is to A. begin child protective team investigation B. obtain plain radiographs of the orbit C. perform needle aspiration D. provide reassurance and observation E. refer to surgery

D. topical corticosteroid The female infant in the vignette has seborrheic dermatitis. It is characterized by greasy yellowish scales in the scalp and eyebrows, and as in this case, a fine erythematous papular eruption of the face, postauricular region and ears, nape of the neck, and intertriginous areas. Often cheesy debris is evident along with erythema in the skin folds of the axilla, anterior neckline, and groin. The location parallels the distribution and density of the sebaceous glands, but the pathogenesis is not known. Typical infantile seborrheic dermatitis usually improves after treatment with a low-potency topical corticosteroid; if not, a short course of medium potency topical corticosteroids may be implemented. If the condition does not resolve promptly, it is important to consider other causes, such as Langerhans cell histiocytosis, immunodeficiency, zinc deficiency, coexistent atopic dermatitis, secondary bacterial suprainfection, psoriasis, or superimposed candidiasis. PREP Pearls - Typically, infantile seborrheic dermatitis responds promptly to low-potency topical corticosteroids. - The common distribution for infantile seborrheic dermatitis involves the scalp, eyebrows, face, postauricular region, nape of the neck, and intertriginous areas. ABP Content Specifications(s) - Recognize the clinical findings associated with seborrheic dermatitis, and manage appropriately

The parents of a healthy 2-month-old female infant come to your office because of concerns about a red rash in her axilla and neckline. This rash has been present for 2 weeks and seems to be getting worse. She is primarily breastfed and she has no constipation or diarrhea. She has been otherwise well. They use baby skin care products only. On physical examination, you find a rash: Mild erythema with cheesy exudate of axilla, involvement in and around ear. The remainder of the physical examination is unremarkable. Of the following, the BEST initial plan for management is to recommend A. avoidance of milk in mother's diet B. evaluation by dermatology C. topical antifungal cream D. topical corticosteroid E. topical zinc oxide

C. reduced feeding volumes In the vignette, a 3-week-old newborn presents with an apparent life-threatening event. This is an observer-dependent phenomenon, characterized by a combination of apnea, abnormal muscle tone (limpness, rigidity), choking, and color change (pallor, cyanosis, plethora). Apparent life-threatening events typically are reported in infants at 1 to 2 months of age, and are rarely described after 8 months of age. Although specific etiologies are poorly understood, gastroesophageal reflux (GER) has been causally implicated. However, supportive data are conflicting, and the relationship between apnea and GER remains controversial. Accordingly, in this infant whose clinical history otherwise suggests a diagnosis of uncomplicated GER, which may be exacerbated by overfeeding, current best evidence indicates that the most appropriate approach to management would include a reduction in feeding volumes. Gastroesophageal reflux is defined by the passage of gastric contents into the esophagus, with or without regurgitation or vomiting. It is a normal physiologic process occurring several times per day in healthy infants, children, and adults. Most episodes are transient, lasting less than 3 minutes, occurring in the initial 2 hours postprandially, and followed by rapid esophageal clearance of the refluxate. These brief, recurring GER events are generally unassociated with clinical signs or symptoms. During the first 3 months after birth, regurgitation (defined as effortless passage of gastric contents into the mouth) or vomiting events (which may be projectile) are noted daily in 50% of infants. These GER episodes arise from several potential anatomic and physiologic mechanisms, the most prominent being repeated, transient relaxation of the lower esophageal sphincter (LES). Available reports show that 60% to 70% of infants experience more than 1 episode of emesis per day by 3 to 4 months of age. Resolution of GER symptoms occurs in most infants by 1 year of age. The emesis prevalence rate decreases to 10% by 7 to 9 months of age and to 5% by 10 to 12 months of age. As in the vignette, a diagnosis of GER in infants is based solely on a history of spitting-up gastric contents. Formal intraesophageal pH monitoring is of no value in routine evaluation. This study should be reserved for assessing equivocal reflux cases, or in an attempt to correlate symptoms with reflux episodes. A barium esophagram cannot quantify the extent or severity of GER, and is of use solely to document anatomic integrity. Symptoms and signs that have been associated with prolonged or increased GER include both respiratory and nonrespiratory events, as listed in Table. Although regurgitation or vomiting are not consistent findings in older patients with GER-related complications, spitting-up, with or without expulsion of gastric contents from the mouth, is considered a necessary condition for reaching a clinical diagnosis of reflux during infancy. Early studies suggest that a potential mechanism for reflux-induced apnea involved acid stimulation of pharyngeal and esophageal chemoreceptors, leading to laryngospasm. However, more recently, large case series have failed to demonstrate a consistent GER-apnea link. Where respiratory status was monitored along with both esophageal pH and bioelectrical impedance in infants presenting with a history of apnea, only 15% of apneic episodes were correlated with GER. Furthermore, these episodes were as likely to occur with nonacid as with acid GER. Additional data have failed to demonstrate any clinical efficacy of acid reduction therapy in preventing or ameliorating apnea events. Thus, acid blockade with either an H2-receptor antagonist or a proton pump inhibitor is not indicated in this clinical setting. Gastroesophageal reflux has also been implicated as a causative or exacerbating factor for other respiratory disorders. When studied using esophageal pH monitoring, 60% to 70% of children with reactive airway disease demonstrate pathological reflux. However, it is unclear whether this degree of GER is a primary problem or a secondary phenomenon caused by lung hyperinflation and downward movement of the diaphragm leading to upward displacement of the LES into the chest, accompanied by a reduction in LES pressure. Several studies have attempted to assess the role of acid reduction therapy in ameliorating asthma symptoms in both children and adults, and most have failed to demonstrate consistent treatment efficacy. However, a few reports have shown improvement in asthma symptoms following acid blockade for patients with poorly controlled asthma, particularly those subjects manifesting predominantly nocturnal symptoms. Gastroesophageal reflux has been implicated in the etiology of recurrent pneumonia and interstitial lung disease. Although these complications have been reported in otherwise healthy infants and children, they are common problems in children with significant neurological impairment. In these cases, lung disease is presumably the consequence of failure of normal airway mechanisms to protect the lungs from aspirated gastric contents. Several case series have presented conflicting results regarding the efficacy of either medical or surgical GER therapy in improving lung function and reducing the risk of pneumonia in affected patients. Gastroesophageal and respiratory function are closely linked, and therefore the finding of pathological GER in patients with chronic pulmonary disease is not surprising. In 1 report, 27% patients with cystic fibrosis (CF) reported symptoms of heartburn. However, when studied by esophageal pH monitoring, the prevalence of pathological GER in CF was even higher. Accordingly, GER may be considered an exacerbating factor for children and adults with a wide range of chronic respiratory disorders. Again, the effectiveness of acid blockade on respiratory symptoms in these patients has not been clearly demonstrated. PREP Pearls - During infancy, the presence of emesis or regurgitation is required to make a diagnosis of gastroesophageal reflux. - A clear relationship between gastroesophageal reflux and apnea has not been established, and acid suppression has not been shown to alter the clinical course related to apnea during infancy. - In patients with poorly controlled asthma, acid suppression therapy may be useful in the setting of predominantly nocturnal asthmatic symptoms. ABP Content Specifications(s) - Understand the prognosis for patients who have gastroesophageal reflux - Recognize the clinical features associated with gastroesophageal reflux, including symptoms associated with other organ systems (eg, respiratory)

You are asked to see a 3-week-old neonate who has just been hospitalized because of an apparent life-threatening event. He was delivered at term following an uncomplicated pregnancy. He weighed 2,800 g. He was started on a soy protein-based formula at birth because of a family history of allergy to cow-milk protein. At about 2 weeks of age, he developed postprandial emesis and was switched to a hydrolyzed protein-based formula. However, the vomiting episodes have persisted, and the baby spits up once or twice after each feeding. He currently consumes 6 oz of formula every 3 hours to 4 hours. Several hours ago, immediately after feeding, the baby appeared to be choking, stopped breathing, and developed perioral cyanosis. His mother slapped him on his back and formula was expelled from his nose and mouth. The parents called 911, and the infant was brought to the emergency department, where he appears active and alert. His weight is 3,700 g, temperature is 37.0°C, respiratory rate is 30 breaths/min, pulse rate is 130 beats/min, and oxygen saturation is 99% on room air. Physical examination findings are unremarkable. In the emergency department, blood, urine, and spinal fluid cultures are obtained, and he is started on intravenous antibiotics and admitted for observation and continued care. Of the following, the MOST appropriate next step is A. a barium upper gastrointestinal tract series B. intraesophageal pH monitoring C. reduced feeding volumes D. treatment with an H2-receptor antagonist E. treatment with a proton-pump inhibitor

B. observe for 48 hours with no additional evaluation The newborn in the vignette should be observed in the hospital for 48 hours with no additional evaluation unless clinical signs of sepsis develop. Mothers who are admitted in labor before 37 weeks and 0 days with unknown group B streptococcal (GBS) status should receive antibiotic prophylaxis. Late preterm newborns who appear well at birth may simply be observed for 48 hours or more in the hospital if their mother received adequate antibiotic therapy starting at 4 or more hours before delivery, had ruptured membranes less than 18 hours, and did not develop chorioamnionitis. Indications for diagnostic evaluation and antibiotic treatment of a newborn at risk for early-onset GBS, regardless of gestational age, include any clinical signs of neonatal sepsis or maternal chorioamnionitis. Chorioamnionitis has commonly been defined in the literature as the development of a maternal fever associated with 2 of the following clinical findings: uterine tenderness, maternal or fetal tachycardia, and foul smelling, purulent amniotic fluid. Maternal fever is seen in 95% to 100% of cases of chorioamnionitis. The US Centers for Disease Control and Prevention cite a maternal temperature of more than 100.4°F (38°C) as an intrapartum sepsis risk factor that warrants maternal evaluation and potential management for presumed chorioamnionitis as determined by the obstetric providers. Well-appearing newborns exposed to maternal chorioamnionitis should undergo a limited evaluation (blood culture and complete blood cell count with differential and platelets) and antibiotics should be initiated. Any newborn with signs of sepsis regardless of intrapartum risk factors should undergo a full diagnostic evaluation (blood culture, complete blood count with differential and platelets, chest radiography if respiratory symptoms are present, and lumbar puncture if cardiorespiratory status is stable) and immediately be given antibiotic therapy. The revised guidelines for the prevention of perinatal GBS disease were published in 2010, including an algorithm for neonatal management. This algorithm defines management and treatment strategies based on clinical presentation, maternal chorioamnionitis, gestational age, adequacy of antibiotic treatment, and duration of membrane rupture. Review of this algorithm supports observation of the late preterm newborn in the vignette for 48 hours. PREP Pearls - Indications for diagnostic evaluation and antibiotic treatment of a newborn at risk for early-onset group B streptococcal disease include any clinical signs of neonatal sepsis or maternal chorioamnionitis. - Late preterm newborns who appear well at birth may simply be observed for 48 hours or more in the hospital if their mother received adequate antibiotic therapy for 4 hours or more hours before delivery, had ruptured membranes less than 18 hours, and did not develop chorioamnionitis. ABP Content Specifications(s) - Plan the management of a neonate whose mother is febrile at the time of delivery

You are called by the nursery staff to inform you of a newborn admitted to your service. The mother presented in labor at 36 3/7 weeks of gestation after receiving good prenatal care that was complicated only by unknown group B Streptococcus screening status. She received 1 dose of cefazolin 4 hours prior to delivery. Spontaneous rupture of the membranes occurred 8 hours prior to delivery and the mother remained afebrile. The newborn appears clinically well 1 hour after delivery, with a glucose value of 51 mg/dL (2.8 mmol/L). The mother is requesting discharge 24 hours after delivery. Of the following, the MOST appropriate initial management is to A. observe for 24 hours with no additional evaluation B. observe for 48 hours with no additional evaluation C. obtain a blood culture and complete blood cell count with differential and observe for 24 hours D. obtain a blood culture and complete blood cell count with differential and observe for 48 hours E. obtain a blood culture and complete blood cell count with differential and treat with antibiotics

B. bolus normal saline The goal of maintaining the mean blood pressure greater than the gestational age in the first days after birth still remains the standard at many centers PREP Pearls - Poor tissue perfusion is associated with decreased tissue oxygenation, which leads to an increased serum lactate and metabolic acidosis. - The full-term neonate presenting with septic shock requires volume expansion to improve tissue perfusion. ABP Content Specifications(s) - Understand the metabolic consequences of continued poor perfusion in a newborn infant

You are called to assess a term newborn with tachypnea and temperature instability 2 hours after delivery. The maternal history is notable for 17 hours of ruptured membranes and unknown group B streptococcal status. Examination reveals a tachypneic infant with delayed capillary refill, mild grunting, and weak pulses. Vital signs include a heart rate of 180 beats/min, respiratory rate of 80 breaths/min, and a blood pressure of 50/20 mm Hg. The arterial blood sample values include a pH of 7.25, a PCO2 of 38 mm Hg, a PO2 of 70 mm Hg, bicarbonate of 16 mEq/L (16 mmol/L), lactate of 90.09 mg/dL (10 mmol/L), hemoglobin of 12 g/dL (120 g/L), and ionized calcium of 4 mg/dL (1 mmol/L). You initiate evaluation and treatment for possible sepsis. Of the following, the MOST appropriate next step in management is to A. administer hydrocortisone B. bolus normal saline C. infuse calcium gluconate D. infuse sodium bicarbonate E. transfuse packed red blood cells

D. Prader-Willi syndrome Prader-Willi syndrome presents in the neonatal period with hypotonia, poor suck, and characteristic facial features that include bitemporal narrowing of the head, almond-shaped eyes, elongated face, and thin upper lip. Hypotonia of just the lower extremities (sparing the trunk and upper extremities) suggests a spinal cord disorder such as myelomeningocele. Occult myelomeningocele may have no cutaneous findings, so if the clinical examination is suggestive, then imaging is necessary to make the diagnosis. Children with Angelman syndrome are typically asymptomatic at birth. During infancy they can develop hypotonia and poor feeding. The typical facial features are prognathia and wide-spaced teeth. PREP Pearls - In neonates with hypotonia, dysmorphic facial features suggest a central nervous system disorder. - The absence of reflexes in a hypotonic neonate, especially if accompanied by tongue fasciculations, suggests a peripheral nerve disorder such as spinal muscular atrophy. ABP Content Specifications(s) - Differentiate the findings associated with central nervous system causes of hypotonia from those of peripheral nervous system causes

You are called to evaluate a 10-hour-old full-term neonate with poor feeding. The chart documents a normal pregnancy, labor, and delivery. The mother did not take any medications during pregnancy and did not receive any medications during delivery. Physical examination of the neonate reveals a high forehead with narrowing at the temples and a thin upper lip. With crying, the facial movements are symmetric, and the palate and tongue are normal. The neonate's limbs move symmetrically when he is stimulated, but at rest, there is very low tone in the limbs and trunk. Deep tendon reflexes are present, but diminished. The remainder of the physical examination is unremarkable. Of the following, the MOST likely diagnosis is A. Angelman syndrome B. Duchenne muscular dystrophy C. infant botulism D. Prader-Willi syndrome E. spinal muscular atrophy

D. prolonged QTc The most common type of long QT syndrome (LQTS), type I, classically presents as drowning or near-drowning in cold water. Any child being evaluated after a drowning or near-drowning episode, especially if the event begins with diving into water, should be suspected of having LQTS type I. The patient in the vignette is alert and conscious after the automated external defibrillator (AED) is used on him, which suggests that he had a ventricular arrhythmia that responded to defibrillation. He also has a family history that includes early death, seizures, and hearing loss. Seizures can be secondary to poor brain perfusion in the context of hypotension. Congenital hearing loss is seen with the more severe form of LQTS type I. Stress and exercise can also be triggers for ventricular arrhythmias in LQTS; this would be consistent with the great grandfather who died during basic training. Given this combination of factors, a prolonged QTc would be the most likely finding to expect on the electrocardiogram (ECG). First-degree atrioventricular block would not respond to defibrillation, and by itself, unassociated with higher degree of atrioventricular block, would not be a cause of syncope. Generalized low voltages would be seen if myocarditis was suspected, but there is no history to suggest this diagnosis (viral illness or fever). Increased left ventricular voltages may occur as a normal variant or may be seen in cases of hypertrophic cardiomyopathy. Hypertrophic cardiomyopathy can cause sudden death or syncope with exertion, but it is not associated with hearing loss. Wolff-Parkinson-White (WPW) syndrome is also a rare cause of sudden death, especially if the patient has had syncope with atrial fibrillation and subsequent ventricular fibrillation. This rhythm would also respond to defibrillation. Wolff-Parkinson-White syndrome is not associated with hearing loss in the family, and the initiation of atrial fibrillation is not typically associated with diving into cold water. Further evaluation and treatment of a patient with suspected LQTS will depend on the presentation. In the case of cardiac arrest requiring defibrillation, the patient will need to be treated with β-blockers and an implantable cardiodefibrillator. In a less urgent situation, such as when LQTS is discovered on an ECG performed for other reasons such as chest pain, a more step-wise evaluation is possible. The ECG morphology can suggest the LQTS type, and although genetic testing can often confirm this, it is still not 100% sensitive. In patients with type I LQTS (without a history of syncope or cardiac arrest), treatment with a nonselective beta-blocker such as nadolol or propranolol is very effective. In patients with type II LQTS, the classic trigger is being awakened from sleep by a loud noise. β-blockers are used, but may not be as effective. In type III LQTS, the ventricular arrhythmias typically occur at night when the patients have bradycardia, and are not provoked by a trigger. Type I and II LQTS are defects in potassium channel activity with a decrease in function or too little potassium current. Type III LQTS is a defect in sodium channel activity with an increase in function or too much current; patients with these conditions are also often treated with a second medication, such as mexiletine, to block the sodium current. All patients with LQTS are cautioned against diving into cold water, and taking any medication that may prolong the QT interval. The current Bethesda guidelines for exercise restrictions in patients with LQTS limit older patients, especially high school children, fairly severely (please see Maron BJ, Zipes DP. Eligibility recommendations for competitive athletes with cardiovascular abnormalities. J Am Coll Cardiol. 2005;45(suppl). doi:10.1016/j.jacc.2005.02.002.). PREP Pearls - Long QT type I, the most common type, has a classic trigger of diving into cold water as a cause of ventricular arrhythmia such as Torsade de Pointe. - Triggers for events vary for the different types of long QT syndrome. - Family history is key in making the diagnosis. Children of individuals with long QT syndrome have a 50% chance of inheriting the disease. ABP Content Specifications(s) - Understand the clinical significance of a prolonged corrected QT interval

You are called to see a 9-year-old boy brought in by ambulance from the community swimming pool. He dove into the pool but did not come up; his 3 friends jumped in and tried to pull him out. The lifeguards arrived and performed cardiopulmonary resuscitation with use of an automated external defibrillator. When emergency medical services arrived 5 min later, he had regained consciousness and had a heart rate of 120 beats/min. When you see him 30 min later in the emergency department, the patient is awake and talking. His mother is with him. The mother tells you that there are distant relatives with seizure disorder on her mother's side, one of whom wore hearing aids and one great grandfather who died at age of 23 years during basic training in the army. You admit the patient overnight for observation. While on telemetry, he has rare premature ventricular contractions. Of the following, the electrocardiogram pattern you would MOST likely see in this patient is A. first-degree atrioventricular block B. generalized low voltages C. increased left ventricular voltages D. prolonged QTc E. Wolff-Parkinson-White syndrome

A. ampicillin and gentamicin Inadequate maternal antibiotic therapy prior to delivery (only 1 dose of a nonpenicillin agent.) Although group B Streptococcus is the leading cause of infection in this setting, Escherichia coli and other gram-negative organisms must be considered as well. Although less common, Listeria monocytogenes is another potential pathogen in early-onset sepsis of the newborn. Ampicillin and cefotaxime would be another acceptable regimen in this setting, especially if ampicillin-resistant E coli are prevalent. In view of concerns over development of resistant bacteria and increased risk of candidiasis with prolonged use of cephalosporins, cefotaxime might be reserved for those with meningitis because of its excellent central nervous system penetration. PREP Pearls - Ampicillin is effective against Listeria. No cephalosporin is effective against this organism. - Early onset sepsis in the neonate is typically caused by group B Streptococcus or enteric gram-negative rods such as Escherichia coli. Listeria monocytogenes is a less common cause of this syndrome. ABP Content Specifications(s) - Plan appropriate antimicrobial therapy for suspected sepsis in the immediate newborn period

You are called to the delivery room to evaluate a newborn who is grunting, floppy, and mottled. The mother came to the emergency department in active labor with ruptured membranes. She received her prenatal care elsewhere; her group B Streptococcus status is unknown. She denies any history of sexually transmitted infections or genital lesions during pregnancy, and she has a history of anaphylaxis to penicillin. She received 1 dose of vancomycin prior to delivery. The baby's Apgar scores were 4 at one min and 6 at five min. Gestational age is estimated at 36 weeks. On physical examination, the vital signs show a temperature of 35.5°C, heart rate of 170 beats/min, and respiratory rate of 60 breaths/min with grunting and retractions. Respiratory effort is shallow with coarse breath sounds throughout. The heart has a regular rhythm without murmur, rub, or gallop; the capillary refill is about 3 seconds; and the skin contains scattered petechiae. A complete blood cell count includes the following results: - White blood cell count, 2,700/µL (2.7 × 109/L), with 26% neutrophils, 17% bands, 46% lymphocytes, 8% monocytes, and 3% metamyelocytes - Hemoglobin, 15.8 g/dL (158 g/L) - Hematocrit, 49% (0.49) - Platelets, 66 × 103/µL (66 × 109/L) Blood and urine culture specimens are obtained. The newborn is deemed too unstable for a lumbar puncture at this time. Of the following, the BEST choice for initial antimicrobial therapy is A. ampicillin and gentamicin B. ampicillin and ceftriaxone C. clindamycin and cefotaxime D. clindamycin and gentamicin E. vancomycin and gentamicin

A. Citrobacter koseri Presence of multiple brain abscesses is uniquely associated with sepsis and meningitis caused by the enteric gram-negative organism Citrobacter koseri (previously Citrobacter diversus) and some other gram-negative organisms (eg, Cronobacter, Serratia marcescens). Salmonella species have also been associated with brain abscesses in neonates. PREP Pearls - Brain abscesses in infants and children are very rare. Brain abscesses can arise through hematogenous spread (eg, in endocarditis or sepsis and meningitis) or direct extension (eg, complicating sinusitis, mastoiditis, or odontogenic infection). - Citrobacter koseri is relative uniquely associated with multiple brain abscesses in neonates. ABP Content Specifications(s) - Recognize the clinical findings associated with brain abscess - Plan the appropriate diagnostic evaluation of a brain abscess - Plan the appropriate management of a brain abscess

You are called to the emergency department to evaluate a 5-day-old male newborn who was brought in by ambulance because of a prolonged episode of twitching and perioral cyanosis at home. Over the course of several hours before this episode, he was noted to be fussy, did not feed well, and seemed less active. The parents report he acted well the day before, and they did not note any fever. His birth weight was 3 kg, and he had an Apgar score 8 at 1 min and 9 at 5 min. The prenatal course and delivery are said to be unremarkable. The mother was negative for group B Streptococcus, human immunodeficiency virus, and rapid plasma reagin. He was discharged home at 2 days of age. On physical examination, the baby's temperature is 35.9°C, heart rate is 160 beats/min, blood pressure is 52/32 mm Hg, and respiratory rate is 56 breaths/min. The newborn is floppy with little spontaneous activity. His anterior fontanelle is full, and his skin is noted to be without lesions. A head, ears, eyes, nose, and throat examination shows positive red reflex bilaterally. His neck is supple. His lungs have shallow respirations. His heart has a regular rhythm without murmur, rub, or gallop. His abdomen is distended and soft, and his liver edge is palpable. Findings on genitourinary examination are unremarkable. His extremities are without lesions. Laboratory test results include the following: White blood cell count, 1,800/µL (1.8 ×109/L), with 37% neutrophils, 22% bands, 7% metamyelocytes, 32% lymphocytes, and 2% basophils Hemoglobin, 14.2 g/dL (142 g/L) Platelets, 122 × 103/µL (122 × 109/L) Analysis of his cerebrospinal fluid shows the following: White blood cell count, 1,932/µL (1.9 × 109/L), with 78% neutrophils and 22% lymphocytes Glucose, 10 mg/dL (0.6 mmol/L) Protein, 0.25 g/dL (2.5 g/L) A Gram stain shows white blood cells and few gram-negative rods. Results are shown from a computed tomography scan of the head (Item Q43 - brain abcesses). Of the following, the MOST likely infectious agent causing this baby's illness is A. Citrobacter koseri B. group B Streptococcus C. herpes simplex virus D. Listeria monocytogenes E. Pseudomonas aeruginosa

A. Ages and Stages Questionnaire The ASQ has been well researched and validated as a screening tool to identify developmental difficulties for children up to 5 years of age. Another appropriate developmental screening instrument to use for children up to age 8 years would be the Parents' Evaluation of Developmental Status (PEDS) for children up to 8 years of age. The Child Behavior Checklist (CBCL) is a well-validated mental health screening tool commonly used by mental health clinics and screens for mental health issues, not developmental issues. The Patient Health Questionnaire (PHQ-9) is a brief, free, and validated depression rating scale for adults and adolescents. The Pediatric Symptom Checklist (PSC) is a general mental health disorder screening tool, available in 17- and 35-item versions. It is free to use, and has been validated for use in children in primary care settings between the ages of 6 and 16 years. The Wechsler Preschool and Primary Scale of Intelligence™ (WPPSI) is an IQ assessment tool that can be administered by a psychologist trained in its use. It is an appropriate tool for use in children ages 2 1/2 to 7 years. PREP Pearls - Parent-completed developmental screening tools help practitioners to identify the presence of a developmental disability and to know when to refer a child for intervention. - Providers should know the valid age ranges for the different rating scale assessments. - The Modified Checklist for Autism in Toddlers is designed and validated for use between 16 and 30 months of age. Screening is recommended at the 18- and 24-month health supervision visits. ABP Content Specifications(s) - Understand the factors that can cause variations in the normal developmental sequence and recognize their associated features - Understand the uses and limitations of various developmental screening tool

You are seeing a 3-year-old girl for a health supervision assessment. Her mother says that she does not have concerns to discuss today. The mother reports that her daughter, who is an only child, started walking at 1 year of age and uses mostly single words when she talks. She does not attend any day care or preschool. In the office, you notice that the girl avoids making eye contact and spends most of her time focused on a doll that she brought with her, rather than on either you or her mother. When she talks, she clearly enunciates words, such as "doll," "boy," and "door," but does not interact with you in a meaningful way. She is at the 60th percentile for weight and the 75th percentile for height. Her physical examination findings are unremarkable. Of the following, the MOST helpful test to administer today would be the A. Ages and Stages Questionnaire B. Child Behavior Checklist C. Patient Health Questionnaire D. Pediatric Symptom Checklist E. Wechsler Preschool and Primary Scale of Intelligence

E. insufficient renal water excretion Fractional Excretion of Sodium: (FENa)% = 100 × (SCr × UNa ) / (SNa × UCr) The child in the vignette has the syndrome of inappropriate antidiuretic hormone (SIADH) evidenced by hyponatremia, decreased urine output, a state of euvolemia or hypervolemia, a fractional excretion of sodium (FENa) of 2%, and history of traumatic brain injury. Syndrome of inappropriate antidiuretic hormone causes increased water reabsorption from the renal collecting duct, and therefore decreased water excretion. Several medications (ie, some sedatives and neuroleptics) can stimulate ADH release, and others (ie, oxytocin and vasopressin analogs) can potentiate AVP action Irreversible neurologic damage can occur with rapid correction of hyponatremia; the rate of correction should not exceed 10 to 12 mEq/L in a 24-hour period. Adrenal insufficiency: can occur after traumatic brain injury and can cause hyponatremia. Cases that are significant enough to cause hyponatremia are often associated with hyperkalemia, non-anion gap metabolic acidosis, hemodynamic instability, or hypoglycemia, none of which was seen in the patient in the vignette. Excessive renal sodium loss: can occur in CNS insults in the form of cerebral salt wasting. However, urine sodium levels would be higher than seen in the vignette. Excessive atrial natriuretic peptide secretion: does not typically occur in head trauma, and will also be associated with increased urinary losses of both sodium and water PREP Pearls - Disorders of sodium are usually from imbalances between intake and output of free water. - Syndrome of inappropriate antidiuretic hormone (SIADH) can be commonly seen in central nervous system disorders (trauma, infection, and tumor) and pulmonary disorders (pneumonia, asthma, pleural effusion). - The mainstay of treatment of SIADH is fluid restriction. ABP Content Specifications(s) - Recognize the role of head trauma in the development of SIADH

You are caring for a 10-year-old boy who suffered a severe traumatic brain injury from a motor vehicle accident 3 days ago. He does not have trauma to any other organ system. His elevated intracranial pressure is currently being managed with mild hyperventilation, mannitol, sedation, and continuous cerebrospinal fluid drainage. Medications include fentanyl continuous infusion, cefazolin, ranitidine, and mannitol. He is receiving 30 kcal/oz formula at 50 mL/h by nasogastric tube. He has a temperature of 37°C, heart rate of 80 beats/min, blood pressure of 120/70 mm Hg, respiratory rate of 20 breaths/min, and oxygen saturation of 98% on the ventilator. His weight is 30 kg. On physical examination, he is intubated and sedated. He does not open his eyes and withdraws all extremities equally upon painful stimuli. Pupils are 4 mm, equal, and sluggishly reactive. Mucous membranes are moist, and he has periorbital and soft tissue edema. His pulmonary and cardiovascular examinations are unremarkable. Over the last 24 hours, fluid intake was 1,200 mL of feeds and 300 mL of medications and other fluids to maintain catheter patency. Fluid output includes 500 mL of urine and 100 mL of cerebrospinal fluid drainage. Serum chemistry results include: - Sodium, 125 mEq/L (125 mmol/L) - Potassium, 4 mEq/L (4 mmol/L) - Chloride, 90 mEq/L (90 mmol/L) - Carbon dioxide, 24 mEq/L (24 mmol/L) - Blood urea nitrogen, 20 mg/dL (7.1 mmol/L) - Creatinine, 0.6 mg/dL (53 µmol/L) - Osmolality, 270 mOsm/kg (270 mmol/kg) - Urine specific gravity, 1.020 Urine chemistry results include: - Sodium, 50 mEq/L (50 mmol/L) - Creatinine, 12 mg/dL (1061 µmol/L) - Urine urea nitrogen, 6 mg/dL (2.1 mmol/L) Of the following, the MOST likely physiologic mechanism for his hyponatremia is A. adrenal insufficiency B. excessive atrial natriuretic peptide secretion C. excessive renal sodium loss D. inadequate sodium intake E. insufficient renal water excretion

B. discontinue feeds and place gastrostomy tube to dependent drainage Nonobstructive ileus causing vomiting, abdominal distention, and discomfort can be treated with gastric drainage, and usually resolves within 24 to 72 hours. PREP Pearls - Nonobstructive ileus can occur after surgery, and is more common in abdominal surgery or cases requiring large doses of pain medication or sedation. - Abdominal distention and discomfort due to functional ileus can be treated with gastric drainage. - A surgical consultation should be pursued if a mechanical bowel obstruction is suspected. ABP Content Specifications(s) - Plan appropriate management for a patient with intestinal obstruction

You are caring for a 30-kg, 10-year-old boy with a history of severe spastic quadriplegia from a hypoxic-ischemic event at 3 years of age. Other medical problems include subglottic stenosis from prolonged intubation requiring tracheostomy, seizure disorder, hydrocephalus, scoliosis, and inability to feed that required a gastrostomy tube. He was electively admitted to the hospital for airway reconstructive surgery. Postoperative management required deep sedation with a morphine drip and continuous enteral feeds. Now, on postoperative day 3, he develops abdominal distention and bilious vomiting. Vital signs show a temperature of 37°C, heart rate of 80 beats/min, blood pressure of 100/60 mm Hg, respiratory rate of 20 breaths/min, and oxygen saturation of 98% on the ventilator. On physical examination, he is ventilated and sedated. He has moist mucous membranes, and his lungs are clear to auscultation bilaterally. Cardiovascular examination shows a regular rate and rhythm, with warm and well-perfused extremities. His abdomen is soft, distended, and tympanitic to percussion. There is no organomegaly. Bowel sounds are hypoactive. Over the last 24 hours, fluid intake was 1,200 mL of feeds and 100 mL of medications and other fluids to maintain catheter patency. Fluid output includes 600 mL of urine (1.5 mL/kg per hour) and approximately 500 mL of emesis. Frontal and cross table lateral abdominal radiographs are shown (Item Q42A)(Item Q42B). Of the following, the MOST appropriate initial step in management is to A. consult surgery for lysis of adhesions B. discontinue feeds and place gastrostomy tube to dependent drainage C. discontinue sedation and administer naloxone D. place transpyloric feeding tube and administer metoclopramide E. place urinary catheter and measure intra-abdominal pressure

C. penicillin is administered to the patient at the wrong dose For the patient described in the vignette, the most likely medication error to occur is that penicillin is administered to the patient at the wrong dose. The abbreviation "U" for "units" is on the official "Do Not Use List" published by The Joint Commission (Item C129) because the "U" can easily be mistaken for a zero and result in a ten-fold overdose of medication. In order to avoid this error, "units" should be spelled out completely. For the patient described in the vignette, common hospital measures to prevent medication errors, such as using contrasting labeling for look-alike drugs and confirming a patient's identity before administering a medication, make administering the wrong medication and giving a medication to the wrong patient less likely than giving the wrong dose. In addition, giving penicillin at the wrong time would not be an expected error with the medication prescription provided. PREP Pearls - The avoidance of "Do Not Use" Abbreviations published by The Joint Commission can prevent medication errors. Strategies to prevent medication errors include: -- ensuring an adequate number of pharmacy and nursing staff to prepare and administer medications -- providing appropriate equipment and systems for medication administration -- providing appropriate equipment and systems for medication administration -- standardizing measurement systems (ie, kilograms instead of pounds) -- standardizing order sheets or utilizing computerized physician order entry with alerts -- employing evidence-based clinical pathways with order sets -- developing error tracking systems, providing educational programs -- adopting family-centered care with discussion of medication regimens -- fostering a nonpunitive culture for reporting errors -- championing quality improvement and patient safety initiatives ABP Content Specifications(s) - Understand and apply methodologies to prevent medication errors

You are caring for a neonate with congenital syphilis in a busy intensive care unit and prescribe penicillin 50,000 U/kg per day divided every 8 hours. Of the following, the MOST likely medication error to occur in this scenario is A. a "look-alike" drug is administered to the patient B. a "sound-alike" drug is administered to the patient C. penicillin is administered to the patient at the wrong dose D. penicillin is administered to the patient at the wrong time E. penicillin is administered to the wrong patient

C. meta-analysis of published studies A systematic review of a clinical question entails a comprehensive review of the existing literature on the topic. In performing a systematic review of the literature on a treatment regimen, a meta-analysis offers the advantage of combining the results of multiple studies meeting specified criteria on the subject using statistical methods that then provides a quantified estimate of the benefits or adverse effects of the therapeutic intervention. A high quality meta-analysis requires a comprehensive search of the literature and a critical evaluation of studies to be included. A meta-analysis may be limited by the quality and number of studies analyzed. Case control studies are evaluations of patients with a given condition (in this case pneumonia) who received a specific treatment and comparing their outcomes to those receiving another regimen. Such studies are limited by being retrospective in nature and generally involve only a single specific population. A double-blind, placebo-controlled trial, if randomized, is a study design in which a group of patients are randomly assigned to receive the drug being studied or a control drug (placebo or present standard treatment) and then assessed for prespecified outcomes. Limitations may include the nature of the study population and the inability to compare multiple treatment regimens. A published editorial likely gives expert opinion based on existing evidence, but is limited by the review and opinions of the authors. The product insert of a drug describes the US Food and Drug Administration (FDA) approved features of a given agent and includes studies submitted to the FDA, but may not provide a comparison of alternative agents. PREP Pearls - A systematic review includes a comprehensive review of the literature on a clinical question. - A meta-analysis uses statistical methods to combine results of multiple studies on a given topic. ABP Content Specifications(s) - Understand the uses and limitations of systematic review and meta-analysis

You are charged with helping your hospital's pharmacy and therapeutic committee to formulate a policy for recommending drug choices to treat pediatric pneumonia. In preparing for the meeting, you review the literature on the topic. Of the following, the MOST reliable of available analyses would be a A. case-control study B. double-blind, placebo-controlled trial C. meta-analysis of published studies D. published editorial on the subject E. review of the product insert of relevant drugs

B. 3 weeks ARF leads to rheumatic heart disease in approximately 60% of patients not treated for streptococcal pharyngitis. The peak age at presentation is between 5 and 14 years. In 60% to 80% of patients, the first clinical manifestation is arthritis, described as a painful migratory polyarticular arthritis that affects medium and large joints. Approximately half of patients with ARF develop carditis that can manifest as valvulitis, with or without pericarditis and myocarditis. PREP Pearls - Acute rheumatic fever presents approximately 3 weeks after a group A Streptococcus infection. - Diagnosis of acute rheumatic fever (ARF) is based on the Jones criteria, which require evidence of a group A streptococcal infection and either 2 major criteria or 1 major and 2 minor criteria. - Major Jones criteria for ARF are polyarthritis, carditis, Sydenham chorea, subcutaneous nodules, and erythema marginatum. - Minor Jones criteria for ARF are fever, arthralgia, prolonged PR interval on electrocardiograph, and elevated acute-phase reactants. ABP Content Specifications(s) -Understand the natural history of rheumatic fever Recognize the clinical findings associated with rheumatic fever, including major and minor criteria Confirmation of preceding Group A Streptococcus by positive throat culture, positive rapid antigen test, elevated or risingantistreptolysin O titer AND 2 major manifestations OR 1 major and 2 minor manifestations

You are discussing a clinical case with a group of medical students. The patient has a history of arthritis that started in the ankles, moved to the knees, and then shifted to the wrists. On physical examination, there is a fever (40°C) and a friction rub. Echocardiography reveals mitral valve prolapse and left ventricular dysfunction consistent with carditis. The laboratory results show an elevated erythrocyte sedimentation rate. The medical students ask how long before onset of symptoms does the infection associated with this diagnosis typically occur. Of the following, the MOST accurate response to the students' question is A. less than 1 week B. 3 weeks C. 6 weeks D. 9 weeks E. 12 weeks

D. more adolescents are using contraceptive methods effectively 7 in 10 teenagers will have had vaginal intercourse by 19 years of age. However, currently more teenagers are waiting longer to start (until after 15 years of age) than before, and those who are sexually active are more likely to use condoms and other forms of contraception more effectively. Early onset of coital activity is often a red flag for other issues such as abuse, substance abuse, and emotional adjustment problems, and is especially a concern when there is a 3 year or greater age difference between the early adolescent and the partner. PREP Pearls - Early and repetitive discussions around sexuality by parents have been shown to decrease the early onset of sexual activity and reduce high-risk sexual behaviors. - School-based comprehensive sexuality education has been successful at getting adolescents to use contraception more often and effectively, and has resulted in a falling pregnancy, birth, and abortion rate in the United States. ABP Content Specifications(s) - Recognize the common patterns of sexual behavior and experimentation in adolescents of various ages - Plan appropriate parental counseling regarding adolescent sexuality - Identify common sources of information sought by adolescents regarding sexuality

You are discussing sexuality in your adolescent patients with a medical student. You are asked for an explanation for the dramatic drop in birthrates for adolescents over the last 2 decades. Of the following, the MOST accurate reason for this phenomenon is that A. the abortion rate has increased over the same period B. abstinence education is commonly available in schools C. more accurate sex-related information is available on the internet D. more adolescents are using contraceptive methods effectively E. oral sex rates have increased, whereas vaginal intercourse rates have decreased

B. computed tomography scan of head Mannitol can be an effective agent to reduce ICP, but it is not routinely used in pseudotumor cerebri, and it is more important to first establish the nature of the elevated ICP. Morphine and other medications that could potentially depress respiratory drive should be avoided at all costs. In elevated ICP, hyperventilation is a natural compensatory mechanism to decrease the blood component of the cranial vault because hypocapnia decreases cerebral blood flow. Even a slight increase in Pco2 from slowed respirations because of narcotics can lead to herniation and death. PREP Pearls - Computed tomography of the head is the first step when evaluating a patient with suspected increased intracranial pressure (ICP). - Narcotics and sedatives should be avoided in cases of elevated ICP. - Nighttime or early morning vomiting can be a sign of elevated ICP. - Lumbar puncture with measurement of opening and closing pressure can be helpful in elevated ICP, especially in pseudotumor cerebri or infections, but only after obstructive hydrocephalus has been excluded. ABP Content Specifications(s) - Recognize the clinical findings associated with increased intracranial pressure in patients of various ages - Understand the indications and contraindications for examination of the cerebrospinal fluid in a patient who has increased intracranial pressure

You are evaluating a 12-year-old girl who was brought to the emergency department with complaints of severe headache that started 3 weeks ago and has been worsening. The headaches are not preceded by any specific symptoms, are described as throbbing in nature, and are in the back of the head and behind the eyes. They are worse at night and in the morning, and are associated with vomiting. She also has sensitivity to light and sound. Vital signs show a temperature of 37°C, heart rate of 60 beats/min, blood pressure of 140/90 mm Hg, respiratory rate of 15 breaths/min, and SpO2 is 98% on room air. Physical examination shows a generally well-developed and well-nourished girl who is awake, oriented, and lying still. She is complaining of a "10 out of 10" headache. Pupils are 6 mm, equal, and reactive to light. Her right eye cannot move laterally, but the rest of her extraocular movements and cranial nerve functions are intact. Funduscopic examination shows bilateral papilledema. She does not have any other focal neurologic deficits. She is breathing comfortably, and lungs are clear to auscultation bilaterally. Cardiovascular examination is unremarkable. Abdomen is soft, nontender, nondistended, and with no organomegaly. Extremities are warm and well perfused. Of the following, the MOST appropriate next step in management is A. acetazolamide B. computed tomography scan of head C. lumbar puncture with opening and closing pressure D. mannitol E. morphine

D. prednisone The girl in the vignette has Bell palsy, or idiopathic unilateral facial nerve palsy. If started within 3 days of symptom onset, a course of prednisone improves the chances of complete recovery. Most cases of Bell palsy do not have an identifiable underlying cause, and are presumed to be a postviral inflammation of cranial nerve VII, the facial nerve. History and physical examination should assess for secondary causes of facial nerve palsy, such as viral reactivation of herpes simplex or varicella zoster virus, acute otitis media, meningitis (including Lyme disease), leukemia, or lymphoma. In the absence of signs and symptoms of secondary causes, the diagnosis of Bell palsy can be made clinically; imaging is not needed. Most cases of Bell palsy have a good recovery, especially if the weakness is mild or the symptoms improve rapidly. In Bell palsy, oral steroids, typically prednisone, at a dosage of 2 mg/kg per day for 5 days, followed by a 5-day taper, can increase the chance of complete recovery. Until eye closure strength returns, the cornea needs to be protected, which can be done with artificial tears during the day and topical lubricant at night or taping the eyelid shut. Ophthalmology referral to monitor the cornea can be helpful. Studies have not shown additional benefit of antiviral agents in the absence of clinical signs of viral infection such as vesicles, and even with clinical signs, acyclovir is not recommended as the sole treatment for Bell palsy. It is critical to distinguish Bell palsy from a stroke on clinical examination. The key finding in Bell palsy is forehead weakness, which manifests as decreased forehead wrinkling on the affected side, as is present in the girl in the vignette. A pediatric patient who presents with unilateral facial weakness, with sparing of the forehead, should be referred to the nearest emergency department immediately for evaluation for stroke. The optimal treatment for stroke in pediatric patients is currently under investigation. Aspirin, intravenous tissue plasminogen activator, and mechanical thrombolysis have all been used. Intravenous heparin is not used as an initial treatment for stroke, unless a thrombus or critical arterial stenosis is identified. PREP Pearls - Unilateral facial weakness involving the forehead is suggestive of facial nerve palsy (not of stroke). - Oral prednisone, started within 3 days of onset of symptoms, can improve the chance of complete recovery in Bell palsy. ABP Content Specifications(s) - Recognize the clinical findings associated with Bell palsy, and manage appropriately

You are evaluating a 15-year-old girl in your clinic for new-onset right facial weakness. She first noticed it this morning when she was putting on makeup and could not pucker her lips or close her right eye all the way. She denies limb weakness, facial pain, sensory loss, and changes in hearing or taste. Her only medication is oral contraceptives. She occasionally smokes cigarettes. The girl has not had any recent immunizations. Her general physical examination is unremarkable. Neurologic examination shows normal pupillary reflexes, eye movements, and facial sensation. When raising her eyebrows, the right side of her forehead is weak and has fewer wrinkles than the left side of her forehead. She has right ptosis and weak right eye closure, and the right side of her mouth droops. Her tongue is midline and her extremities have normal strength and tone. It is now afternoon and her symptoms have not improved. Of the following, the MOST appropriate treatment for her condition is A. acyclovir B. aspirin C. heparin D. prednisone E. tissue plasminogen activator

D. urology referral The patient in the vignette has distal hypospadias. In patients with hypospadias, the urethral meatus opens on the ventral surface of the penis proximal to the normal glanular location. Hypospadias is the most common malformation of the male genitalia, occurring in 1 in 300 births. Hypospadias is classified by the anatomic location of the meatus and the presence or absence of chordee (ventral curvature of the penis). Hypospadias can be classified as first degree (on the glans), second degree (on the penile shaft), and third degree (penoscrotal to perineal). Another classification approach is distal (glandular, coronal, subcoronal), middle (distal shaft, midshaft, posterior penile), or proximal (penoscrotal, scrotal, perineal). The most common form of hypospadias is distal (subcoronal)/first degree. Proximal hypospadias is more frequently associated with other malformations than is distal hypospadias. Genital malformations (such as cryptorchidism and inguinal hernia) are more common (8%-15%) than urinary tract abnormalities (such as vesicoureteral reflux, ureteropelvic junction, ectopic kidneys, and renal agenesis) in patients with hypospadias. The association of distal forms of hypospadias with renal anomalies (1% in patients with distal and 5% in patients with proximal hypospadias) is no greater than that of the general population, making it unnecessary to perform renal ultrasonography in patients such as the child in this vignette. The penile foreskin is used in hypospadias repair; therefore, circumcision is contraindicated in these patients. Patients with hypospadias should be referred to a urologist within the first few weeks after birth. This initial evaluation addresses parental concerns and provides them with information about the required surgical intervention. The optimal time for hypospadias repair is at 6 months of age. A referral to genetics should be considered in patients with third-degree or proximal hypospadias. Urine culture and reassurance are not the next indicated steps in evaluating the patient in the vignette. PREP Pearls - In hypospadias, the urethral meatus opens on the ventral surface of the penis, proximal to the normal glanular location. - Proximal hypospadias is more frequently associated with other malformations than is distal hypospadias. - The penile foreskin is used in hypospadias repair; therefore, circumcision is contraindicated in these patients. - Patients with hypospadias should be referred to a urologist within the first few weeks after birth. ABP Content Specifications(s) - Recognize disorders associated with hypospadias - Plan the appropriate management of hypospadias

You are evaluating a 2-day-old neonate in the hospital. The mother mentions that the newborn is feeding every 3 to 4 hours and has had 7 wet diapers in the last 24 hours. She is concerned about the funny urinary stream she noticed in the morning. His physical examination is significant only for an uncircumcised male infant with a ventral opening of the penile urethra (hypospadias). Of the following, the MOST appropriate next step in the management of this patient's condition is A. circumcision B. reassurance C. renal ultrasonography D. urology referral E. urine culture

D. thiazide Thiazide diuretics decrease calcium excretion in the urine. Thiazides decrease sodium/chloride absorption in the distal tubule. The ensuing diuresis leads to increased calcium absorption in the proximal tubule resulting in a significant fall in calcium excretion in the urine. Therefore, for the child in the vignette, thiazide diuretics would decrease the risk of kidney stones. Although multiple factors have been associated with nephrocalcinosis in neonates, prescription of loop diuretics (furosemide, bumetanide) for managing bronchopulmonary dysplasia is a major risk factor. Loop diuretics increase the excretion of calcium in the urine, leading to an increased risk for calcium deposits in the renal parenchyma. Other factors contributing to neonatal nephrocalcinosis include: (1) increased urinary phosphate excretion, (2) hypercalcemia between 32 and 40 weeks of gestation, (3) immature renal tubular function in preterm neonates associated with alkaline urine pH and low urinary citrate concentration (risk factors for calcium phosphate precipitation), and (4) renal injury secondary to ischemia or nephrotoxic medications leading to tubular injury, which promotes calcium deposition. Acetazolamide is a proximal tubule carbonic anhydrase inhibitor. Its use leads to an elevation in urinary pH, which is a risk factor for calcium stones. Increased risk for hypercalciuria and nephrocalcinosis has been reported in neonates receiving loop diuretics and acetazolamide concomitantly. Triamterene is a potassium-sparing diuretic used for treating hypertension in combination with thiazide diuretics. Potential nephrotoxicity has been reported with triamterene including an increased risk for crystalluria, kidney stones, cast formation, or reversible acute kidney injury. PREP Pearls - Thiazide diuretics decrease urinary calcium excretion by increasing absorption of calcium in the proximal renal tubule. - The use of loop diuretics (furosemide, bumetanide) is a risk factor for nephrocalcinosis. - Loop diuretics increase the excretion of calcium in urine, leading to increased risk for calcium deposits in the renal parenchyma. - Acetazolamide, a proximal tubule carbonic anhydrase inhibitor, elevates urinary pH, which is a risk factor for calcium stones. - Triamterene increases the risk of crystalluria, kidney stones, cast formation, or reversible acute kidney injury. ABP Content Specifications(s) - Differentiate the effects of various diuretic drugs on calcium excretion

You are evaluating a 4-month-old during a health supervision visit. She was born at 29 weeks' gestation with a weight of 1,250 g. She has chronic lung disease of prematurity. Her mother says that she is feeding every 3 to 4 hours without breathing problems and has 8 to 10 wet diapers a day. Her current medications include furosemide and thiazide. Her evaluation reveals an afebrile infant, heart rate of 110 beats/min, respiratory rate of 38 breaths/min, blood pressure of 80/40 mm Hg, with normal physical examination findings and growth parameters for her gestational age. The mother is concerned because she read on the Internet that prolonged diuretic use will increase her child's risk of kidney stones. Of the following, the BEST medication choice to address the mother's concern is A. acetazolamide B. bumetanide C. furosemide D. thiazide E. triamterene

D. maintaining consumption of traditional foods while adding new food types PREP Pearls - Dietary acculturation is the process by which a minority group adopts the dietary practices of the host or dominant group. - Dietary acculturation typically involves retaining some traditional foods, adapting new ones to traditional dishes, and adopting foods of the host country outright. - Evidence suggests that the nutritional transition from low- to high-calorie dense foods combined with a more sedentary lifestyle increases chronic disease risk among immigrant populations. ABP Content Specifications(s) - Understand the family and cultural determinants that influence dietary practices and nutrition

You are examining a healthy 4-year-old child whose parents immigrated to the United States from Nepal just before he was born. You note that his height and weight are at the 25th to 50th percentile, while his parents are at approximately the third percentile. You compare this family's nutrition and exercise patterns with those of the traditional Nepalese lifestyle. Of the following, this family's nutrition and exercise patterns are MOST likely to be characterized by A. adherence to the traditional diet of the parents' country of origin B. an increase in physical activity levels C. increased consumption of lower caloric density foods such as fruits and vegetables D. maintaining consumption of traditional foods while adding new food types E. rapid assimilation of the typical dietary pattern of the United States

A. round, raised. yellowish peripherally, brown centrally Cutaneous anthrax begins as a pruritic papule or vesicle that progresses over 2 to 6 days to form the depressed eschar with surrounding edema. Antibiotic therapy of anthrax varies with the clinical presentation. For bioterrorism-associated cutaneous anthrax, treatment with ciprofloxacin or doxycycline is recommended until antimicrobial sensitivity results are available. Anthrax vaccine may be used in conjunction with antimicrobial therapy. Treatment should be continued for 60 days, as spores may remain dormant in mediastinal lymph nodes for prolonged periods of time. B: cellulitis due to GAS C: erythema migrans due to Lyme D: abscess likely due to CA-MRSA E: tinea corporis PREP Pearls - Cutaneous anthrax presents as a pruritic papule or vesicle that progresses over 2 to 6 days to form a typical necrotic eschar with surrounding erythema. - Bacillus anthracis is considered a leading potential agent for a bioterrorist attack. - Anthrax can present with cutaneous, respiratory, gastrointestinal, or neurologic symptoms based on the route of acquisition and dissemination. ABP Content Specifications(s) - Recognize the characteristic skin lesions of anthrax

You are precepting a group of medical students. There have been recent news reports of ricin-laced mailings to public officials. In discussing the events, one of the students asks about potential infectious agents of bioterrorism. In reviewing this topic, you present them a number of pictures of lesions. Of the following, the image MOST consistent with potential transmission in a bioterrorism event is A. round, raised. yellowish peripherally, brown centrally B. confluent erythema of forearm sparing the hand/wrist C. round, erythematous, perhaps central clearing vs targetoid D. abscess E. round patch, erythematous, desquamating, overlaying scale

E. should trigger a thorough dietary and exposure history Studies show that with an increase in BLL from 1 µg/dL (0.05 µmol/L) to 4 µg/dL (0.19 µmol/L), IQ can show a decline of 2 to 5 points. In addition, there are behavioral consequences of increased BLL, including decreased attention, increased impulsivity, and reduced academic achievement. Chelation therapy for children with BLLs less than 45 µg/dL (2.17 µmol/L) has not shown any benefit. PREP Pearls - The reference standard for blood lead levels is currently 5 µg/dL (0.24 µmol/L) and levels at or above this should trigger further investigation and follow-up. - There is no biological threshold below which lead is without effects. - The primary effects of lead intoxication are neurocognitive and behavioral, but other organ systems including the cardiovascular, endocrine, gastrointestinal, and immunologic may be affected. - The neurocognitive effects of lead intoxication appear to be irreversible. ABP Content Specifications(s) - Understand the outcomes associated with lead poisoning Recognize the multiple sources of exposure to lead

You are reviewing the laboratory results of a healthy 2-year-old child seen for a health supervision visit. His hemoglobin level is 11.2 g/dL (112 g/L), hematocrit is 33.5% (0.34), mean corpuscular volume is 74 µm3 (74 fL), and blood lead level is 6.1 µg/dL (0.3 µmol/L). Of the following, the MOST appropriate statement regarding this child's lead level is that it A. falls below the current reference value for lead poisoning B. is too low to cause neurocognitive effects C. requires chelation therapy to reverse central nervous effects D. should be treated with a low calcium diet E. should trigger a thorough dietary and exposure history

E. spirometry The next step to assess asthma control in the child in this vignette is to perform a spirometry in order to objectively assess lung function. The 2007 National Heart, Lung, and Blood Institute guidelines (Expert Panel Report 3, "EPR3") recommend periodic assessment of asthma control in all individuals with asthma. The parameters of control assessment in the EPR3 include the use of validated tools such as the Asthma Control Test (ACT) (> 12 years of age), Childhood Asthma Control Test (C-ACT) (4-11 years of age), Asthma Therapy Assessment Questionnaires, and Asthma Control Questionnaires. These tools, while validated screening measures, are based on self-report, with the ACT and C-ACT measuring recall of asthma control over the past 4 weeks. A score of greater than 20 indicates good control. A score of less than 19 indicates lack of control, and a score of less than 14 indicates very poor control. The aspects of asthma control assessed by the ACT and C-ACT include nocturnal symptoms, ability to perform regular activities, and subjective assessment of control. Part of the C-ACT is filled out by the parent and part by the child (using pictorial cues if needed). The additional advantages of these measures are that they promote dialogue between the caretaker and child, and between the provider, the caretaker, and child. Individual parameters in these questionnaires can also be used to set goals and strategies to achieve asthma control. For example, if the child is doing well in the daytime, but wakes up often because of asthma, then emphasizing the importance of the pre-bedtime dose and trying to control comorbid conditions, such as postnasal drainage or reflux, may help alleviate the limitations in achieving optimal asthma control. Other validated tools to measure asthma control such as the Asthma Therapy Assessment Questionnaire (ATAQ) have been used both in research studies and in clinical care. These instruments are easily interpretable and have evidence to support their use in clinical decision making. The ATAQ can be self-administered by adults or used in interviewer-administered versions for younger patients. The instrument consists of questions relating to the patient's past month of asthma control and is scored on a scale of 0 to 4, with a higher score meaning more control issues. While these instruments cover most of the guideline characteristics of control, practical differences, such as in ease of administration, may determine preferential adoption in clinical practice. Since the ATAQ measures domains of asthma control similar to those assessed in C-ACT, administration of the ATAQ will not provide additional discriminatory information in this case. The Asthma Quality of Life Questionnaire (AQLQ) is a disease-specific, health-related quality of life instrument developed by Elizabeth Juniper and colleagues that measures the functional problems (physical, emotional, social and occupational) that are most troublesome to adults (17-70 years) with asthma. There are 32 questions in the AQLQ and they are in 4 domains (symptoms, activity limitation, emotional function, and environmental stimuli). A shorter version for clinical use with 15 questions has been developed (called MiniAQLQ). The AQLQ does not assess domains of asthma control and would not contribute to the clinical assessment of asthma control. Review of school nurse records is helpful to assess asthma control, or explore reasons for failure to achieve control in the school setting. It may also help evaluate for associated conditions such as vocal cord dysfunction, exercise-induced bronchospasm, or malingering. While this would provide useful adjunct historical information, further determination of her current asthma control at this clinic visit will require the use of an objective measure of lung function such as spirometry. Peak expiratory flow rate can be an objective method to evaluate airway obstruction. However, the EPR3 states that "for diagnostic purposes, spirometry is generally recommended over measurements by a peak flow meter in the clinician's office because there is wide variability even in the published predicted peak expiratory flow reference values." Reference values may also vary with the brand of meter used. Interpretation of peak flow values would depend on background knowledge of the child's personal best, which in turn would be dependent on the child performing peak flows twice a day for a period of time. Peak flow meters therefore are better utilized as home-based monitoring tools and not as diagnostic tools in the office. The EPR3 recommends spirometry as one of the measures to assess asthma control (apart from diagnosing asthma). The spirometry is objective and can be used in children 5 years of age and older. The spirometric parameters that are helpful include the forced expiratory volume at 1 second (FEV1) and FEV1/FVC (forced vital capacity) ratio greater than 80%, indicating well-controlled asthma. The other parameters in conjunction with the spirometric indices can help the provider gauge the degree of asthma control. PREP Pearls - Assessment of lung function indices using spirometry, on an annual basis after the age of 5 years, is recommended in the Expert Panel guidelines to obtain objective measures of asthma control. - Asthma control assessment tools, such as the Asthma Control Test, Childhood Asthma Control Test, Asthma Control Questionnaires, or Asthma Therapy Assessment Questionnaires, should be routinely used in the ambulatory management of children with asthma. - Peak flow measurements are helpful for home-based assessment of asthma control, but are not helpful in diagnosing asthma in the office because reference values are not well-established and are device-dependent. ABP Content Specifications(s) - Plan appropriate outpatient management of a patient with asthma (eg, self-assessment, education, pulmonary function testing, drug therapy, asthma action plans)

You are seeing a 10-year-old girl for follow-up of her asthma. The girl and her mother complete the Childhood Asthma Control Test handed to them by the nurse. You notice a discrepancy in the scoring between the child and the mother. As you go over the questions with them, they start arguing. The child reports more symptoms than the mother is aware of. The mother feels the child is prone to exaggeration. The physical examination is unremarkable. Of the following, the BEST next step to assess control in this child is A. the Asthma Quality of Life Questionnaire B. the Asthma Therapy Assessment Questionnaire C. peak flow measurement D. review of school nurse records E. spirometry

A. creatine phosphokinase level Although elevation of creatine phosphokinase is a nonspecific marker, elevation suggests a primary muscle problem and can guide further workup. The girl in the vignette has rash, muscle pain, and evidence of muscle weakness. These physical findings are consistent with juvenile dermatomyositis. PREP Pearls - Proximal muscle weakness is usually a sign of a primary muscular disorder. - Muscle enlargement, muscle wasting, and fasciculation point toward a lower motor neuron problem. - Pain as a symptom of muscle disease suggests an inflammatory or infectious process. - A creatine phosphokinase level should be measured in patients with muscle pain and weakness. ABP Content Specifications(s) - Identify the etiology of myositis - Plan the appropriate evaluation and management of myositis

You are seeing a 12-year-old girl in your office for complaints of fatigue and arm pain that has lasted 2 weeks. The girl states that she feels fatigued after activity and that certain activities are difficult because of pain and weakness. She has the most difficulty with fixing her hair and walking up stairs. On physical examination, she has decreased strength. She is able to overcome gravity, but not resistance in her shoulder girdle and hip flexors. She has erythematous patches over her metacarpophalangeal joints and proximal interphalangeal joints bilaterally (Item Q32). You also note a mild violaceous color over her eyelids bilaterally. Of the following, the BEST next step in the evaluation of this patient is A. creatine phosphokinase level B. electromyography C. magnetic resonance image of the spine D. neurology referral E. reassurance and symptomatic pain treatment

E. radiography of the lumbar spine The girl in the vignette is a gymnast with chronic back pain exacerbated by activity and relieved with rest. She has no neurologic signs or symptoms. On physical examination, she has point tenderness and pain with spinal extension. This clinical picture should raise concern for spondylolysis, a bony defect in the pars interarticularis. Radiography would be the next best step to evaluate for this diagnosis. Although computed tomography and magnetic resonance imaging of the lumbar spin would give the diagnosis of spondylolysis, plain radiography may yield the diagnosis and are less invasive and more cost effective. Bone scintigraphy would identify an area of metabolic activity at the site of spondylolysis but it is nonspecific and further imaging is required for diagnosis. The girl in the vignette is a gymnast, but shows no signs or symptoms of the classic "female athlete triad" (disordered eating, amenorrhea, and osteoporosis), therefore a dual-energy X-ray absorptiometry scan is not indicated. Although uncommon in young children, spondylolysis is a common cause of back pain in athletes older than 10 years during periods of rapid growth. Spondylolysis usually occurs with repetitive trauma and is frequently seen in athletes participating in ballet and other forms of dancing, football, gymnastics, high jump activities, and weight lifting. Most often presenting with mild-to-moderate lumbar pain on physical examination, children typically have localized spinal tenderness with muscle spasm and loss of the normal lordosis. Lumbar flexion and extension may be limited, and the pain is usually worsened with hyperextension of the spine. Treatment is conservative, including rest and avoidance of aggravating activities. The approach to a patient with back pain should consist of a thorough history and physical examination that includes assessment of the timing and location of the pain. The clinician should ask whether the pain is associated with activity, whether the patient participates in any sports, whether there is any history of trauma, and whether there was acute versus insidious onset. If nothing in the history is suggestive and the physical examination is normal, then a diagnosis of nonspecific back pain can be considered and treated conservatively with physical therapy and nonsteroidal anti-inflammatory medication. If no improvement is noted after 1 month of conservative management, the patient should be re-evaluated. If the child's history raises concern for a disease process or injury, then anteroposterior and lateral plain radiographs should be obtained, with or without oblique views depending on the suspected diagnosis. Historically oblique views have been recommended for diagnosis of spondylolysis, but anteropoterior and lateral views may be sufficient for this diagnosis. Laboratory tests should be considered to rule out infection, inflammation, or an oncologic process. If the laboratory tests and radiography do not yield a diagnosis, then further imaging should be considered. Magnetic resonance imaging (MRI) is recommended for patients in whom infection, a herniated disc, or a tumor are suspected. Computed tomography can be helpful for diagnostic confirmation if spondylolysis is suspected, but not identified on plain radiography. A bone scan is an acceptable alternative to MRI in young children in cases in which infection or tumor is suspected, but the results can be nonspecific, and require further testing. Bone scan can also be used to localize metabolic bone activity in the spine to identify areas for further imaging in cases of suspected stress fracture or spondylolysis. If all test results are negative and there is no response to conservative treatment, then a referral to a pain management service should be considered. PREP Pearls - Initial assessment of back pain in children with an abnormal physical examination or concerning history should begin with plain radiography and laboratory studies to rule out infection, inflammation, and tumor. - An oblique view radiograph is the best first imaging method for diagnosing spondylolysis. - Magnetic resonance imaging is the best choice for evaluating infection, herniated disc, or tumor in children with back pain. - Computed tomography is preferred in cases of suspected spondylolysis with normal plain radiographs. ABP Content Specifications(s) - Plan the appropriate management of back pain - Plan the appropriate evaluation of back pain

You are seeing a 14-year-old female gymnast who complains of back pain. The pain has been increasing over the last several months. She has no history of injury, but has been complaining of pain more frequently with bending backwards. She denies numbness, tingling, or urinary or stool incontinence. She states that the pain is worse with activity and is relieved with rest. Occasionally, ibuprofen decreases the pain. On physical examination, she is a well-appearing girl with pain with extension of her spine. She has point tenderness over the left lower lumbar paraspinal muscles. Of the following, the next BEST step in evaluation of this patient is A. bone scintigraphy B. computed tomography of the lumbar spine C. dual-energy X-ray absorptiometry scan D. magnetic resonance imaging E. radiography of the lumbar spine

C. abdominal ultrasonography The rash and constellation of symptoms in the girl in this vignette is consistent with Henoch-Schönlein purpura (HSP). HSP is a leukocytoclastic vasculitis primarily affecting small blood vessels. The European League Against Rheumatism and Paediatric Rheumatology Europe Society criteria for HSP include purpura or petechiae with lower limb predominance and at least 1 of the following: arthritis or arthralgia, abdominal pain, histopathology demonstrating immunoglobulin A deposition, or renal involvement demonstrated by hematuria or proteinuria. Occasionally, the purpura are preceded by an urticarial or maculopapular rash that fades. The vasculitic rash follows, usually within 24 hours, with a purpuric, necrotic, or deep bruising appearance. The purpuric rash is most often present on dependent areas, but can be seen on the arms, face, and ears. Abdominal pain occurs in over half of HSP patients and can precede the pathognomonic rash by as much as 2 weeks. Intestinal bleeding may occur in up to 33% of cases, and may require evaluation by a subspecialist. The patient in this vignette has intussusception, which only occurs in 1% to 5% of children with HSP, but is an important complication to recognize. Patients who are suspected of having intussusception should be evaluated with abdominal ultrasonography and a surgical consultation. Computed tomography would expose the child to a significant amount of radiation unnecessarily. Abdominal radiography would not be diagnostic, though abnormalities such as air fluid levels or fluid levels may be observed. A barium enema would be unlikely to show the intussusception, because most cases in this age group are small bowel to small bowel, and a barium swallow would not be indicated in a case of suspected intussusception. Arthritis affects about 75% of children with HSP. Most often, the knees and ankles are affected. The arthritis of HSP is usually oligoarticular, nondeforming, and self-limited. Approximately 15% of patients have arthritis as the presenting feature of HSP. The most common renal manifestation of HSP is microscopic hematuria. Renal involvement is relatively common, occurring in 20% to 60% of patients, with chronic renal impairment occurring in 2% to 15% and end-stage renal disease in only 1% of HSP patients. Most renal involvement presents within the first 6 weeks of disease, and 97% occur within 6 months. Rare manifestations seen in HSP include scrotal edema, periorbital edema, hand swelling, pulmonary hemorrhage, seizures, stroke, and mental status changes. The clinical symptoms of HSP can last up to a month, and can recur in one-third of patients. Management of HSP is primarily supportive, including analgesics and nonsteroidal anti-inflammatory drugs (NSAIDs). Early use of corticosteroids in the hospital setting should be considered in the case of gastrointestinal manifestations, particularly abdominal pain. Arthritis should be managed with NSAIDs; however, corticosteroids may be used in severe or nonresponsive cases. The use of corticosteroids in mild cases of HSP remains controversial. The rare, life-threatening, or severe manifestations of HSP, including acute renal failure, intussusception, and severe arthritis, often require management by a subspecialist. PREP Pearls - Common manifestations of Henoch-Schönlein purpura (HSP) include a vasculitic rash, abdominal pain, and arthritis. - Uncommon manifestations of HSP include acute renal failure, intussusception, scrotal edema, periorbital edema, hand swelling, pulmonary hemorrhage, seizures, stroke, and mental status changes. - Abdominal pain occurs in 50% of patients with HSP, gastrointestinal bleeding in up to 33%, and intussusception in 1% to 5%. - Corticosteroid therapy should be considered in patients with HSP and abdominal pain. ABP Content Specifications(s) - Recognize the typical and atypical clinical findings associated with Henoch-Schönlein purpura, and manage appropriately

You are seeing a 6-year-old girl in the emergency department for acute onset of abdominal pain that has been worsening throughout the day. The pain is severe, but comes and goes according to the parents. The patient has been vomiting for the last 2 hours. She had a stool that the mother describes contains blood and mucus. On physical examination, the patient has a tender abdomen with a palpable mass in the right upper quadrant and several plaques and papules on her legs (Item Q70). Of the following, the BEST test to establish the diagnosis for this patient is A. abdominal computed tomography B. abdominal radiograph C. abdominal ultrasonography D. barium enema E. barium swallow

C. protein hydrolysate formula In children with moderate to severe atopic dermatitis, the atopic dermatitis can be triggered or exacerbated by food allergies. This occurs more commonly in those younger than 5 years of age. In this sub-group of children, evaluation of allergy to milk, egg, peanut, wheat, and soy could be considered if the child has persistent AD in spite of optimized management and topical therapy, has a reliable history of an immediate reaction after ingestion of a specific food, or both. Avoidance is key. False-positive specific IgE tests (either on skin or blood testing) often occur, making it important to correlate the test results with history, food diary, and elimination and reintroduction challenges, as indicated. Almond milk: there is a possibility the child has a concomitant tree nut allergy (the child is allergic to peanut, therefore there is a 30%-40% chance of cross-reactivity.) Goat's milk: has more than 90% cross-reactivity to cow's milk. PREP Pearls - In protein hydrolysate formula proteins are broken down into fragments too small to be recognized as allergens. - Children with food allergies and on a restricted diet need to be monitored to prevent nutritional imbalances. - Children younger than 5 years of age with atopic dermatitis and food allergy triggers may be at risk for nutritional inadequacies secondary to dietary restrictions. Phytates found in soy and rice may impair absorption of zinc. - Children with multiple food allergies need to be co-managed with a nutrition expert. ABP Content Specifications(s) - Recognize the effects of a restricted diet for multiple food allergies on the nutritional adequacy of a patient's diet

You are seeing a 15-month-old girl for the first time in your clinic. She was breastfed until 10 months of age and was given soy formula between 10 months of age and 12 months of age and tolerated it without any reactions. She has atopic dermatitis and a concomitant zinc deficiency recently diagnosed by an allergist at 1 year of age, when she also tested positive for milk, egg, peanut, and wheat allergies. At that time, the parents switched her to fortified orange juice. The girl is scratching herself and is irritable on examination. Weight and height are at the 10th percentiles (previously at 50th percentile at her 1-year checkup). On physical examination, she has sparse, thinning hair; generalized erythematous, dry papules, and patches with excoriations; and lichenification on her chest, and antecubital and popliteal fossae. She also has a perioral, erythematous, desquamating, scaly rash that looks different from the rash on the rest of her body (Item Q3: erythematous well demarcated rash with overlaying yellow crust concentrated at the corners of the mouth). The remainder of her examination is unremarkable. The mother is frustrated that her current diet is highly restricted and seeks your help. You set her up for an appointment at the nutrition clinic. Of the following, the BEST recommendation for this patient is a trial of A. almond milk B. goat milk C. protein hydrolysate formula D. rice milk E. toddler soy formula

B. concrete thinking Abstract thinking is the next stage of thought development, in which one uses hypothetical ideas more readily in reasoning. With abstract thinking one is able to consider several possibilities and outcomes at once, and can understand and manipulate concepts as relative rather than absolute. These are important skills in learning to solve problems in novel situations. In working with a patient who is still using concrete thinking, healthcare providers should avoid using strategies that require the patient to manipulate abstract concepts, such as weighing risks and benefits of behavior change, in the manner that an adult would. PREP Pearls - Adolescent psychological development is marked by a transition from concrete thinking to abstract thinking. - Changes in academic demands requiring more abstract manipulation of concepts can lead some adolescents to suddenly start to struggle in their academics. ABP Content Specifications(s) - Understand the timing of and factors influencing the development of concrete thinking and abstract reasoning in adolescents, and provide health advice accordingly

You are seeing a 15-year-old adolescent boy who has recently developed some academic difficulty. He is getting good grades in math and biology at school, but has just started to have significant struggles with English and social studies. The school difficulties started when expectations in those classes moved toward reflection and creative thinking, rather than just memorizing and following rules. When he is given something truly novel to work on, he has difficulty coming up with strategies to address the new problem. You note from his history that he appears to have a personal sense of invincibility, in that he does not hesitate to take significant physical risks while playing sports or skateboarding. Of the following, the predominant thought process that BEST describes what he is exhibiting is A. abstract reasoning B. concrete thinking C. feelings of guilt D. inferiority thoughts E. unconscious drives

A. she can receive the second dose today without restarting the series (2015) Currently, the recommendation is to begin performing Pap tests at 21 years of age. Therefore, the girl in the vignette neither requires a Pap test today, nor should she wait to get the vaccine until results are available, if it is performed. The recommendations are based on the finding that in 91% of those with low-grade lesions, the lesions will clear in 3 years, while only 3% of high-grade lesions show progression with time. HPV types 16 and 18, which are responsible for 70% of cases of cervical, 87% of anal, 60% of oropharyngeal, and 31% of penile cancers. HPV types 6 and 11, which are responsible for 90% of anogenital warts and almost all cases of juvenile recurrent respiratory papillomatosis. PREP Pearls - The human papillomavirus (HPV) vaccine series can be resumed at any time after being interrupted. - The HPV vaccine series should be administered irrespective of the history of previous exposure to the virus. ABP Content Specifications(s) - Know the recommendations, limitations, and schedule for the human papillomavirus vaccine

You are seeing a 17-year-old adolescent girl in your office for a lesion on her vulva. She is sexually active and you diagnose her with genital warts. According to your records, she has never received the human papillomavirus (HPV) vaccine. Her mother had not given consent, stating that her daughter was not sexually active and did not need the vaccine. The girl reports that she received 1 dose of the vaccine at a local clinic 1 year ago, and asks if she will need additional doses or if she will need to restart the vaccine series. Of the following, the BEST advice to the girl is that A. she can receive the second dose today without restarting the series B. the results of a Papanicolaou (Pap) test with HPV DNA are required before restarting the series C. the series can be restarted today after performing a Pap test with HPV DNA D. the vaccine can be given as treatment for the genital warts E. the vaccine will not be protective, as she is already infected with HPV

E. set her cellphone alarm to medication use times Nonadherence to a therapeutic plan is the commonest reason for failure of therapy. This is an especially big issue with an adolescent patient. They may misunderstand the instructions, forget, or choose to ignore advice and the therapeutic plan. Adherence is influenced by a number of factors including the following: attitude and beliefs about the condition being treated (eg, the severity of the condition, benefits, and adverse effects of treatment), the complexity of the regimen, perception of the ability to follow through on the plan, availability of social support, the cultural context, and emotional status, particularly if the patient is depressed. In addition, the relationship with the physician is very important, because it enables the development of a mutually agreed upon therapeutic plan. Collaborating on a plan will work better than dictating the plan and using aids that the patient is comfortable with are crucial. Most adolescents are comfortable with technology. The use of the alarm on her cellphone, for the girl in the vignette, as a cue to take medications would enhance her self-efficacy and thus her motivation to adhere with the plan. The use of a calendar is less helpful, as most adolescents do better with an active reminder. Religiosity and church attendance are a reflection of a conservative outlook and indicate that a teenager is less likely to be rebellious, but this alone does not indicate she will adhere to a regimen. No single intervention will work with all patients. Social support can also be helpful. This includes support from a group or a parent. The involvement of a parent is very important, but an adolescent who will soon be on her own needs help in achieving her independence, rather than remaining dependent on a parent as the only means to enhance adherence. PREP Pearls - The commonest reason for failure of therapy is lack of adherence. - An understanding of the many factors that influence adherence is crucial. - Using technology to create reminders has been shown to be effective in improving adherence. ABP Content Specifications(s) - Understand factors that can affect adherence to health maintenance activities by adolescents - Understand how to improve adherence to medical regimens by adolescent patients, including those with chronic illness, and the barriers to such adherence

You are seeing a 17-year-old adolescent girl in your office for follow-up of her asthma that now requires daily medications. She has not been adhering to her treatment regimen. She has maintained her school grades and has a clear understanding of her illness. She says she is not depressed, but does admit to being stressed with homework and college applications. You consider ways to help her comply with the treatment plans. Of the following, the intervention MOST likely to be successful for this patient is to A. encourage religiosity and church attendance B. enroll her in a support group C. have her mother supervise her treatment plan D. mark a calendar after each dose taken E. set her cellphone alarm to medication use times

C. recheck occipital frontal head circumference in 1 week The infant in the vignette is at risk for hydrocephalus because of his underlying diagnosis of myelomeningocele. His occipital frontal head circumference and his physical examination should be monitored weekly until his head growth is stable, which may take weeks or months. If the head circumference is increasing rapidly and there are signs of hydrocephalus, such as difficulty feeding, difficulty breathing, or encephalopathy, then he would require neurosurgical intervention. This infant is asymptomatic, so his head circumference can be rechecked in 3 to 10 days. An ultrasonogram of the spine does not show hydrocephalus, so that study would not be helpful in this case. The infant's flaccid lower extremities are a result of the myelomeningocele, not of hydrocephalus; it may be prudent to confirm with the parents that his current lower extremity movement is the same as it was on discharge from the hospital. Magnetic resonance imaging of the head would show signs of hydrocephalus, but is a lengthy and expensive study that would require sedation. For urgent clinical concerns, ultrasonography is the preferred imaging modality. This infant has asymptomatic increasing head size, and so does not require referral to either an emergency department or to the pediatric neurosurgery clinic at this time. Abnormally increasing occipital frontal head circumference is suggestive of an acute intracranial process, such as brain tumor or hydrocephalus. Brain tumors often cause additional symptoms referable to the region of brain involved. For instance, posterior fossa tumors can cause ataxia, in addition to increasing occipital frontal head circumference and intracranial pressure. Hydrocephalus can cause increasing occipital frontal head circumference. Risk factors for hydrocephalus include intraventricular hemorrhage, meningitis, or brainstem malformation such as aqueductal stenosis. Rare genetic disorders such as X-linked hydrocephalus can also cause hydrocephalus. Close monitoring of the occipital frontal head circumference and clinical examination in infants with risk factors is the best screening tool to detect early signs of hydrocephalus. PREP Pearls - Myelomeningocele is a risk factor for hydrocephalus. - In infants with repaired myelomeningocele, asymptomatic head enlargement can be monitored clinically. ABP Content Specifications(s) - Understand the risk factors for hydrocephalus

You are seeing a 2-week-old, full-term male newborn for a routine health supervision visit. He had a prenatal diagnosis of myelomeningocele, and at birth, was noted to have lower extremity flaccid paralysis. He underwent surgical repair of the defect on the second day after birth, and he was discharged home 7 days after birth. The discharge paperwork reports an occipital frontal head circumference of 36 cm and bilateral lower extremity flaccid paralysis. At home, the infant has been afebrile, alert, breathing, and eating normally. On physical examination, he has an occipital frontal head circumference of 37.5 cm, a temperature of 37°C, a blood pressure of 80/54 mm Hg, a heart rate of 110 beats/min, and a respiratory rate of 42 breaths/min. The lumbosacral wound site is nonerythematous and without exudate. The remainder of his examination is unchanged from the hospital discharge examination. Of the following, the BEST next step in management of this infant is to A. obtain magnetic resonance imaging scans of the head B. obtain ultrasonograms of the spine C. recheck occipital frontal head circumference in 1 week D. refer to the nearest emergency department E. refer to the pediatric neurosurgery clinic

E. survival motor neuron gene (SMN1) testing The girl in the vignette has spinal muscular atrophy (SMA). SMA is inherited in an autosomal recessive pattern, caused by mutations in both copies of the survival motor neuron 1 (SMN1) gene, located on chromosome 5. SMN1 gene testing establishes the diagnosis. The severity of the clinical manifestations, however, depends at least somewhat on another gene, survival motor neuron 2 (SMN2), also located on chromosome 5. The SMN2 gene codes small amounts of the same protein as the SMN1 gene. The more copies of the SMN2 gene a patient with SMA has, the milder their symptoms. SMA can present from the neonatal period to adulthood. Symptoms of SMA in all ages include limb weakness, hypotonia, and minimal or absent reflexes. Many, but not all patients, have tongue fasciculations. Finger trembling is almost always present after infancy. Other findings can include arthrogryposis in neonates, bell-shaped chest with paradoxical breathing in infants, and pronounced lower extremity weakness in older children. Cognition and facial strength is preserved at all ages. The girl described in the vignette has gross motor delay and normal language. Her physical examination results show limb weakness, hypotonia, and areflexia. These findings suggest a problem of the nerves or muscles. Magnetic resonance imaging of the brain, electroencephalogram, or cerebrospinal fluid analysis would not help make the correct diagnosis. Nerve conduction studies with repetitive stimulation evaluate the neuromuscular junction. This girl does not have symptoms of a neuromuscular junction disorder, such as myasthenia gravis or botulism, therefore this test would not be the most helpful test. PREP Pearls - Spinal muscular atrophy (SMA) can present at all ages, from the neonatal period to adulthood. - Symptoms of SMA in all ages include limb weakness, hypotonia, and minimal or absent reflexes. In children with SMA, finger trembling is almost always present, while tongue fasciculations are often but not always seen. ABP Content Specifications(s) - Recognize the clinical findings associated with different forms of spinal muscular atrophy

You are seeing a 25-month-old girl for a health supervision visit. She missed several previous appointments. She has no chronic medical problems. She has had several hospitalizations for pneumonia, but never required intubation. At her 12-month visit, she had 2 words and was pulling to stand and cruising. In the office today, her parents report that she has more than 50 words, puts 2 words together, and can walk independently and run. They are concerned because she is not able to go up or down stairs and that she is clumsy, falling often. The girl's general physical examination is unremarkable. Her neurologic examination shows normal eye movements, normal facial strength, and no tongue fasciculations. She is weak and hypotonic in all 4 extremities. Her legs are weaker than her arms, she has finger trembling with outstretched fingers, and you are unable to elicit any reflexes. Of the following, the test MOST likely to yield the correct diagnosis is A. cerebrospinal fluid analysis B. electroencephalogram C. magnetic resonance imaging of the brain D. nerve conduction study with repetitive stimulation E. survival motor neuron gene (SMN1) testing

D. last year's teacher found him to be consistently inattentive and impulsive The boy in the vignette is exhibiting symptoms of attention-deficit/hyperactivity disorder (ADHD). The steps required to make the diagnosis include confirming that symptoms have been persistent since an early age, reviewing the current edition of the Diagnostic and Statistical Manual of Mental Disorders diagnostic criteria to ensure an adequate number of inattentive and/or hyperactive symptoms are present, and considering whether other diagnoses (eg, anxiety or learning disability) might be responsible for the inattentive or hyperactive symptoms. If symptoms of ADHD are present in only 1 setting (only at home or only at school) the diagnosis of ADHD should not be made. Children may be hyperactive and inattentive through learned behavior patterns that only appear in one setting, such as in a home that does not provide any consistent limits. True ADHD creates pervasive dysfunctions across settings. Among the response choices listed, the fact that last year's school teacher also found him to be inattentive and impulsive is most supportive of an ADHD diagnosis because this implies both the presence of symptoms in different settings and symptom persistence over time. The presence of early cognitive milestones in the normal range decreases the likelihood that the boy's behaviors are the result of a developmental impairment, but does not otherwise support the ADHD diagnosis. Most children who have ADHD have intact early cognitive milestones, as is the case for most mental health conditions. Although multiple members of the same family may have ADHD, the presence of a parent's past diagnosis of ADHD is a less potent predictor of their child having ADHD than the actual symptoms their child demonstrates. Behavioral and attentional response to methylphenidate does not confirm a diagnosis of ADHD. This is why treatment guidelines do not recommend a therapeutic trial of stimulant to help confirm the presence of ADHD. Stimulants have temporary attention-focusing effects on all people who take them, an outcome used in the past to help non-ADHD military pilots on long duration missions. Alcohol exposure in utero puts a child at risk of developing fetal alcohol syndrome (FAS), which does include inattention and hyperactivity among the possible clinical manifestations. The "comorbid" diagnosis of ADHD in children with FAS has been reported in 50% to 94% of those with heavy alcohol exposure. However, ADHD symptoms in a child with FAS are often of a different pattern than typical ADHD symptoms, including very early onset inattention. Also children with ADHD have difficulty with focusing and sustaining attention, whereas children with FAS have trouble with encoding and attentional shift (an expected transition of attention). Children with FAS tend not to respond as well to treatment with stimulant medication as children with pure ADHD; it has been suggested that a unique differential response unlike typical ADHD, would favor dextroamphetamine over methylphenidate for FAS treatment. PREP Pearls - The evaluation for ADHD should include: -- a full symptom history -- assessment of functioning in multiple settings (through a rating scale often completed by the school and parent) -- review of the inattention/hyperactivity symptom checklist from the DSM -- consideration of other disorders that might also present with inattention and hyperactivity. - Response to a stimulant medication trial is not an appropriate method for diagnosing ADHD. ABP Content Specifications(s) - Plan the appropriate diagnostic evaluation of ADHD - Plan the appropriate management of ADHD

You are seeing a 7-year-old boy who is having school difficulties. His teacher informed the mother that he is not learning as well or as easily as the other children in his class. At home, when she is trying to help him with his schoolwork, he is often off task and does not seem to grasp things very quickly. His schoolwork is sloppy, and he often fails to turn his assignments in to his teacher. You decide to evaluate him for attention-deficit/hyperactivity disorder. Of the following, the statement that would MOST strongly support your suspected diagnosis is A. his early cognitive milestones were in the normal range B. his father and paternal uncle carried this diagnosis as children C. his mother gave him 3 doses of a sibling's methylphenidate, which improved his attention D. last year's teacher found him to be consistently inattentive and impulsive E. this child has a history of significant alcohol exposure in utero

A. caregiver stress specific to the management of diabetes mellitus is associated with better diabetes outcomes There are different types of life stresses that vary by type and degree. Some kinds of stress, such as multiple catastrophic events happening over a short period, can overwhelm a person's emotional well-being and ability to function (at least over the short term). Other kinds of stress, such as managing a single high-risk chronic medical illness, may actually generate increased functioning and focus on solving or managing that particular problem. Research on chronic medical conditions like type I diabetes, as seen in the child in the vignette, demonstrates that parents who experience the management of the diabetes as an isolated life stressor are likely to have children with better clinical outcomes. This is because they have greater involvement in the management of the condition, as opposed to parents who are so stressed overall in their lives that they can spare no additional focus and attention on their child's condition. Children whose parents do not find their child's illness to be stressful may have poorer clinical outcomes, presumably through less parental engagement in the day-to-day management that is often required. This phenomenon can also be understood in terms of what is well known about anxiety and learning—that a little anxiety increases one's focus and ability to perform, but too much anxiety becomes overwhelming and decreases functioning. Thus, a little daily stress about a child's illness can improve the child's outcomes, but too much stress risks a breakdown in parental functioning. A parent experiencing stress about a child's type I diabetes commonly has poorer mental health, including depression and decreased life satisfaction. All of that daily focus on keeping a child healthy does in fact take a toll. Parents of children with chronic illnesses like cancer, asthma, cystic fibrosis, and diabetes report having greater overall parenting stress than their peers. Parents of children with asthma and type 1 diabetes typically experience equal levels of generalized stress. PREP Pearls - Disease-specific parental stress about their child's chronic medical condition is likely to improve management of the disorder, as long as that stress is not overwhelming. - Excessive parental stress about their child's chronic medical condition can impair their ability to manage the disorder. - Chronic medical conditions of children often have an overall negative impact on parental psychological functioning. ABP Content Specifications(s) - Understand the effect of socioeconomic stressors on family dynamics

You are seeing a 9-year-old girl in your office who was diagnosed with type 1 diabetes mellitus 1 month ago. She is currently doing well both physically and emotionally. At the end of the appointment, her mother tells you that she herself is feeling overwhelmed with the stress of managing this new medical problem. The parents are divorced, with the girl's father living across the country. The mother has few others to help her with parenting. She is employed, but the family is struggling financially. The mother is wondering how household and life stress will affect her daughter's diabetes. You want to respond to the mother's concerns. Of the following, the MOST accurate statement would be that A. caregiver stress specific to the management of diabetes mellitus is associated with better diabetes outcomes B. general family stress and caregiver stress specific to diabetes mellitus are unlikely to influence her child's clinical outcomes C. nonspecific family stress is associated with improved diabetes outcomes D. over time, parents of children who have diabetes mellitus report experiencing less stress than do parents of children with asthma E. parent stress from managing a child who has diabetes mellitus usually does not affect parental mental health

A. acknowledge their concerns and inform them that current evidence does not support such a link PREP Pearls - More than half of parents have concerns about vaccine safety, even while acknowledging immunizations' important role in disease prevention. - Pediatricians should be prepared to present clear, evidence-based recommendations about vaccines in language free of scientific jargon. - The American Academy of Pediatrics (AAP) bioethics committee does not support the practice of dismissing patients and families from a practice because of vaccine refusal. - The AAP does not support the use of alternative vaccine schedules because they are not science-based and do not provide timely protection for the youngest, most vulnerable population. ABP Content Specifications(s) - Plan an appropriate approach to addressing the needs of the vaccine-hesitant family

You are seeing a healthy 1 month old for a health supervision visit. As you review the plans for the next visit, the parents state they are considering not immunizing their child because a relative was diagnosed with autism 2 months after receiving a vaccine. Of the following, the MOST appropriate response to these parents is to A. acknowledge their concerns and inform them that current evidence does not support such a link B. ask them to sign the American Academy of Pediatrics vaccine refusal form at this visit C. provide them with a copy of an epidemiologic study showing the absence of correlation between vaccines and autism D. recommend an alternative vaccine schedule that would not overwhelm the child's immune system E. tell them you can no longer care for their child in your practice

E. using computerized physician order entry The physician described in the vignette is behind schedule and late for an appointment, while trying to provide care to the last clinic patient of the day who needs prescriptions and a subspecialty referral. In this rushed situation, a medication prescribing error is a significant risk. Among the response choices, computerized physician order entry (CPOE) would be most likely to prevent an error. CPOE reduces error by: - the elimination of handwriting errors - avoidance of errors caused by sound-alike drugs - avoidance of errors caused by trailing or leading zeros - decision support regarding drug formulation and dosing - links to clinical pathways for correct drug prescribing - decision support - links to drug-drug interaction warnings Computerized physician order entry is one strategy to prevent medical errors in the setting of stress and distraction. Ensuring appropriate staffing (nurse to patient ratio), standardized operating systems, reasonable workload (ie, time to see patients), family-centered care with education (ie, patient handouts), and reducing physician fatigue have all been proven to reduce medical errors. However, for the situation described in the vignette, we cannot know if staffing is inadequate, the workload is imbalanced, patient education is lacking, or the physician is fatigued. We can determine that the physician is rushed and that CPOE may reduce medication prescription errors. PREP Pearls - Stress, distraction, and fatigue increase the risk of medical errors. - Computerized physician order entry may reduce medical errors. ABP Content Specifications(s) - Recognize what interventions can reduce error in situations (eg, stress, fatigue, distraction) at high risk for medical error

You are seeing the last patient in your clinic and realize that you are 15 minutes late for an important meeting. You need to give the patient 2 prescriptions and referral information for a subspecialty clinic visit. Of the following, the intervention MOST likely to prevent a medical error in the scenario described in the vignette is A. increasing the nurse-to-patient ratio B. increasing the time allotted for each patient encounter C. providing a patient handout about subspecialty referrals D. reducing physician fatigue E. using computerized physician order entry

D. knee swelling associated with fever For the child in this vignette, knee swelling associated with fever is the most concerning presentation. Any patient with swelling in a single joint associated with fever should be referred immediately for orthopedic evaluation, because septic arthritis is a medical emergency. The specific symptoms and patterns of pain associated with any case of joint pain can lead the clinician to the appropriate disease process and referral. The history should address several specific domains that can help guide the diagnosis. The location of the pain is important. The clinician should assess whether the pain is localized to the joint or exists in other areas such as the muscle, tendon, or bone, as well as the time of day and pattern of the pain. Morning pain that lasts 1 hour or more, improving throughout the day, points toward an inflammatory cause. If the pain is constant, an orthopedic cause should be considered. Pain that is poorly localized suggests a pain syndrome. Pain that is worsened by a specific activity or is worse on more active days points to an overuse syndrome. Nighttime pain can be caused by benign nocturnal limb pain of childhood ("growing pains") or benign joint hypermobility; however, severe nighttime pain should always make one consider leukemia and lymphoma. If swelling is reported, the clinician should discern whether the swelling is persistent or transient. Swelling caused by arthritis will persist for days at a time, whereas overuse syndromes can have transient swelling that comes and goes, lasting a few hours to a day. Limping associated with inflammation is usually more pronounced in the morning and improves with activity, whereas limping associated with injury or overuse is usually worsened with activity. The clinician should also inquire about systemic symptoms such as fever, rash, weight loss, history of recent illness, and infectious exposures. Fever and weight loss associated with bone or joint pain and swelling should raise concern for leukemia and lymphoma. Several infections can be associated with arthritis such as Lyme disease, parvovirus B19, and mycoplasma. These patients will typically have other symptoms and may have a history of exposure. Patients with transient synovitis will have a history of recent illness. Joint pain in the absence of arthritis is a common complaint in the pediatric clinic. The most common causes of joint pain in children include benign nocturnal limb pain of childhood, benign joint hypermobility, overuse syndromes, and pain amplification syndromes. Decreased range of motion and morning stiffness of the knee is consistent with an inflammatory arthritis, but the differential diagnosis of arthritis in a single joint should also include septic arthritis. Knee pain and a grinding sensation when walking upstairs is the presenting pattern for patellofemoral syndrome, an overuse syndrome that is common in teenagers. Knee pain with activity is likely caused by trauma or overuse. Overuse syndromes may cause mild swelling and should be considered in a patient who performs repetitive movements that cause well-localized pain. The pain occurs during the repetitive activity, but can also occur at other times when the specific joint is used. These syndromes result from increased stress over areas of growth during skeletal maturation. Overuse pain should be managed with modification of the repetitive stress-causing activity and nonsteroidal anti-inflammatory drugs. If bone fragments or skeletal defects are found on imaging studies, the patient should be referred to orthopedics. Most overuse syndromes respond well to physical therapy. PREP Pearls - Benign nocturnal limb pain of childhood ("growing pains") is characterized by nighttime chronic intermittent bilateral shin pain without daytime symptoms or limitation. - Benign joint hypermobility is characterized by joint pain occurring at the end of the day, worsened with activity, and improved with rest in a child with joint hyperextension. - Overuse syndromes should be considered in a patient performing repetitive movements that cause well-localized pain. ABP Content Specifications(s) - Recognize the typical presentation of a patient with functional joint complaints

You are supervising an intern in the outpatient pediatric clinic. She is scheduled to see a 6-year-old girl with a chief complaint of leg pain. You review the patient's chief complaint and the signs and symptoms that will guide your differential diagnosis. You discuss the causes of joint pain and the more and less concerning symptoms. The resident asks you what symptoms would require an immediate workup. Of the following, the MOST accurate response to the resident's question would be A. decreased range of motion and morning stiffness of the knee B. knee pain and grinding sensation when walking upstairs C. knee swelling and pain with activity D. knee swelling associated with fever E. nighttime knee and leg pain associated with activity

E. ventilator associated pneumonia The child in this vignette received endotracheal intubation and was placed on mechanical ventilation because of refractory status epilepticus. Over the course of a week, she developed fever, tachypnea, and increased ventilator support, and chest radiography showed right middle lobe consolidation, all of which suggest ventilator-associated pneumonia (VAP). Respiratory failure is defined as inadequate function of the respiratory system to provide oxygenation or carbon dioxide removal. Ventilatory assistance should be planned to support the respiratory system to achieve both of those functions. In some conditions, this could be achieved with noninvasive treatments such as supplemental oxygen, helium-oxygen mixture, high-flow nasal cannula, or noninvasive positive pressure ventilation. If those measures are inadequate or not feasible, endotracheal intubation and mechanical ventilation may be necessary. The child in the vignette has decreased respiratory drive or apnea primarily from status epilepticus or secondary to anti-epileptic medication. Patients given pentobarbital infusions universally require intubation and mechanical ventilation. In the absence of a primary pulmonary cause of respiratory failure, ventilator settings can include a respiratory rate normal for the child's age, and a fraction of inspired oxygen (FiO2) of 0.21. If volume control ventilation is the chosen mode, a tidal volume of 8 mL/kg can be prescribed, and in pressure control ventilation, enough pressure to achieve that tidal volume is adequate. Settings can be titrated according to arterial, venous, or capillary blood gas analysis and pulse oximetry. Ventilator-associated pneumonia can be diagnosed by new onset fever, leukocytosis, positive respiratory culture, or new infiltrate on chest radiography. It can cause worsened gas exchange, increased ventilator requirement, increased hospital and intensive care unit (ICU) stay, and even death. Other potential complications of endotracheal intubation and mechanical ventilation include ventilator-induced lung injury, neuromuscular weakness from sedatives required to maintain tube placement, post-extubation upper airway obstruction, subglottic stenosis, and VAP. Measures to attenuate some of this risk include avoidance of high inflation pressures, high FiO2, oversedation, and agitation. Weaning and early extubation should be attempted as soon as the patient is able. Also, appropriate selection of endotracheal tube size is important, and should follow the general rule that diameter (mm) = 4 + age/4. Protocols also exist to prevent VAP, including elevation of the head of the bed to 30 degrees and measures for oral decontamination, including chlorhexidine oral care. Initial treatment usually includes empiric broad-spectrum antimicrobials to cover for gram-positive and gram-negative nosocomial infections, with subsequent tailoring according to culture results. Atelectasis can occur during mechanical ventilation because of generally impaired airway clearance and mucous plugging. However, it is not usually associated with fever, and the chest radiograph shown is more indicative of an infiltrative process rather than a discrete segment of atelectasis. Similarly, pulmonary edema does not cause fever. Given the constellation of findings, pulmonary embolism is very unlikely. Acute respiratory distress syndrome can occur as a consequence of pneumonia, shock, or systemic inflammation, but 4 criteria must be satisfied, including acute onset, noncardiogenic pulmonary edema, bilateral pulmonary infiltrates, and PaO2/FiO2 ratio of less than 200. Ventilator-associated pneumonia is a potential complication of endotracheal intubation and mechanical ventilation, and is usually marked by fever, impaired gas exchange, increased requirement of ventilator settings, and new infiltrate on chest radiograph. Contemporary ICU practices include protocols to prevent VAP. Daily assessment of extubation readiness to limit ventilator days is the most effective strategy. PREP Pearls - Ventilator-associated pneumonia is a potential complication of intubation and mechanical ventilation, and can be prevented by minimizing ventilator days. - In respiratory failure without primary lung pathology, initial ventilator settings can be chosen to mimic a normal breathing pattern and gas exchange. - Post-extubation upper airway obstruction is a potential complication of endotracheal intubation and mechanical ventilation, and can be prevented by appropriate sizing of endotracheal tube and minimizing agitation when intubated. ABP Content Specifications(s) - Understand the potential complications associated with endotracheal intubation - Plan the appropriate ventilatory support for patients with various conditions

You are taking care of a 10-year-old girl who is in the intensive care unit because of refractory status epilepticus. Her seizures have been difficult to control on traditional anti-epileptic therapies, so she has been placed on a pentobarbital infusion for the past week. She is intubated and on mechanical ventilation because of her coma. Over the past 48 hours, she has developed a fever as high as 38.5°C, appears to be working harder to breathe, and requires increased ventilator support to maintain her oxygen saturation and blood gases. Vital signs show a temperature of 38.5°C, heart rate of 120 beats/min, respiratory rate of 20 breaths/min, and blood pressure of 100/60 mm Hg. Physical examination shows a generally sedated and intubated patient. Pupils are 4 mm equal and sluggishly reactive. Gag reflex is not present. She withdraws minimally to painful stimuli. Cardiovascular examination shows tachycardia with regular rhythm and warm, well-perfused extremities. Lung examination reveals tachypnea, bronchial breath sounds, and crackles in the right lung fields. The left lung is clear to auscultation, with good air movement and no wheezing. Abdomen is soft, nontender, and nondistended with no organomegaly. A chest radiograph is shown in Item Q99 (RML opacity) Of the following, the MOST likely cause of her worsened respiratory status is A. acute respiratory distress syndrome B. atelectasis C. pulmonary edema D. pulmonary embolism E. ventilator associated pneumonia

C. decreased total lung capacity The newborn in this vignette has Jeune syndrome, which is a form of asphyxiating thoracic dystrophy. As with many forms of thoracic deformity, this syndrome causes restrictive lung disease and decreased total lung capacity. Jeune syndrome is a rare autosomal recessive genetic disorder characterized by skeletal dysplasia that is evident at birth, manifesting with deformities and respiratory distress. Skeletal deformities include shortened ribs, small thoracic cage, reduction in limb bone length, and a characteristic trident-shaped appearance of the pelvis on radiographic evaluation. Most affected children die of respiratory failure because of pulmonary hypoplasia in infancy or early childhood. Microcystic renal disease can also occur, leading to chronic renal insufficiency. Restrictive lung disease is the failure of lung expansion from parenchymal, pleural, or chest wall excursion disorders. Parenchymal causes can be further subdivided into alveolar disorders, such as atelectasis, hemorrhage, hypoplasia, and infection, and extra-alveolar causes such as fibrosis and interstitial edema. Pleural causes include infection, pleural effusion, and pneumothorax. Chest wall excursion causes include diaphragmatic hernia, skeletal malformations, abdominal distention, thoracic dystrophy, flail chest, and kyphoscoliosis. In older children, the impact of chest wall deformities on lung function can be assessed by pulmonary function tests (PFTs). Pulmonary function tests can be helpful in differentiating obstructive causes of respiratory insufficiency (ie, asthma, cystic fibrosis, and other airway abnormalities) from restrictive causes. Severity of disease and response to treatment modalities can also be assessed. The most useful measurements in PFTs are forced vital capacity (FVC), forced expiratory volume in 1 second (FEV1), and forced expiratory flow (FEF) between 25% and 75% FVC (FEF25%-75%). In obstructive diseases, FVC, FEV1, and FEF are all decreased. In restrictive diseases, FVC and FEV1 are either normal or decreased. The ratio of FEV1 to FVC is increased because FVC is decreased to such a large extent. In the newborn in this vignette, assessing total lung capacity using PFTs would not be feasible because of her age. Pulmonary function tests require effort and the ability to follow commands, and generally cannot be done until the patient can cooperate. Total lung capacity in this newborn would be decreased because of decreased thoracic volume. Lung compliance, which is the reciprocal of elastance, is the change in volume for a given change in pressure. Restrictive lung diseases, such as pneumonia, pulmonary edema, and atelectasis, as well as chest wall abnormalities, manifest as decreased compliance and increased elastance. Functional residual capacity is increased in obstructive lung disease, but decreased in restrictive lung disease. Item C175D depicts a classic spirogram showing tidal breathing and the divisions of total lung capacity. Two broad classifications of respiratory failure include obstructive and restrictive diseases. Obstructive diseases generally cause resistance to airflow at any point from the pharynx to the terminal bronchioles, and can lead to air trapping and increased functional residual capacity. Restrictive diseases are marked by insufficient lung expansion and therefore decreased lung volume. PREP Pearls - Two broad classifications of respiratory disease include obstructive and restrictive. - Pulmonary function testing can differentiate restrictive lung disease from obstructive lung disease and assess disease severity, but requires patient effort. - Skeletal deformities involving the thoracic cage can cause restrictive lung disease. ABP Content Specifications(s) - Recognize the association of thoracic deformities with restrictive pulmonary disease

You are taking care of a 3-day-old newborn with a long, narrow thorax with short ribs and short limbs. She is diagnosed with asphyxiating thoracic dystrophy (Jeune syndrome) by a genetics consultant. Vital signs show a temperature of 37°C, heart rate of 160 beats/min, respiratory rate of 70 breaths/min, blood pressure of 80/50 mm Hg, and oxygen saturation of 95% on room air. Her breathing is fast and shallow. During inspiration, there is exaggerated paradoxical breathing in which her abdomen protrudes out while her chest moves in. There is good air exchange bilaterally with no wheezing or crackles. Of the following, the BEST characterization of her respiratory pathophysiology is A. decreased forced expiratory volume in 1 s B. decreased lung elastance C. decreased total lung capacity D. increased dynamic lung compliance E. increased functional residual capacity

B. edema that improves throughout the day In nephrotic syndrome, the edema is usually periorbital and more prominent early in the morning, with minimal systemic symptoms on presentation and clinical examination. In the initial stages, the periorbital and peripheral edema of nephrotic syndrome improves during the course of the day as the interstitial fluid is mobilized and excreted by the normal functioning heart and kidneys. With chronic proteinuria, persistent peripheral edema and anasarca develop in these patients. PREP Pearls - Nephrotic syndrome usually presents with edema that is periorbital and more prominent early in the morning and with minimal systemic symptoms. - Edema is the most easily identified clinical feature associated with hypoproteinemia. - Impaired cardiac function leads to worsening peripheral edema as the day progresses and is associated with exertion and increased activity. -Pitting edema is not pathognomonic for any specific underlying etiology. Scrotal edema can also be seen with generalized anasarca from any cause. ABP Content Specifications(s) - Recognize the clinical features associated with hypoproteinemia Etiology of Edema in Children Hypoproteinemia—decreased capillary oncotic pressure Proteinuria (nephrotic syndrome) Malnutrition Liver failure Increased intestinal protein losses Increased capillary hydrostatic pressure Heart failure Acute renal failure (acute glomerulonephritis) Chronic renal failure Vasodilators Venous obstruction (thrombosis, localized edema) Increased capillary permeability Sepsis Allergic reaction Lymphatic obstruction (edema may be nonpitting)

You are teaching a group of medical students the differential diagnosis of edema in children. You take them to the bedside of a patient who has edema associated with hypoproteinemia secondary to proteinuria. Of the following, the feature you are MOST likely to observe in this patient is A. cola-colored urine, oliguria, and hypertension B. edema that improves throughout the day C. edema that worsens throughout the day D. genital edema E. pitting pedal edema

D. initiate positive-pressure ventilation Positive-pressure ventilation should be initiated if the assessment of a newborn at 30 seconds after delivery reveals apnea, gasping, or a heart rate less than 100 beats/min. PREP Pearls - The establishment of air exchange by effective ventilation of the lungs is the most important action in neonatal resuscitation. - It is difficult to determine whether a newborn has primary or secondary apnea at birth, and delays in initiating effective resuscitation may cause further compromise. ABP Content Specifications(s) Recognize the indications for immediate positive-pressure ventilation in a neonate

You attend the cesarean delivery of a term neonate due to breech presentation. The mother received good prenatal care with no history of substance abuse. General anesthesia is used due to maternal scoliosis with rod placement. Artificial rupture of the membranes occurs at the time of delivery revealing clear amniotic fluid. After breech extraction, the neonate emerges limp with a weak cry. The neonate is placed on the warmer, dried, stimulated, and the airway cleared. Your assessment at 30 seconds after birth reveals a hypotonic neonate with a heart rate of 90 beats/min, intermittent apnea, and central cyanosis. Of the following, the BEST next management step for the neonate is to A. administer naloxone B. deliver blow-by oxygen C. flick the soles of the feet D. initiate positive-pressure ventilation E. start chest compressions


Related study sets

Chem Exam 2 Intext questions (ch 4-6)

View Set

Introduction to Health: Chapter 1, sec. 1-4

View Set

Body Mechanics and Mobility Aides

View Set

Environmental Science study Guide

View Set